Download as pdf or txt
Download as pdf or txt
You are on page 1of 470

Department of Medicine

Dunedin School of Medicine

Dunedin R.A.C.P. Part One


Written Examination Revision Course
14 - 25 November, 2016

Multi-choice Questions and Answers


Index

Specialty Page
Cardiology – Medical Sciences 1
Cardiology – Clinical Applications 14
Endocrinology – Medical Sciences 39
Endocrinology – Clinical Applications 51
Gastroenterology – Medical Sciences 80
Gastroenterology – Clinical Applications 90
Genetics – Medial Sciences 118
Genetics – Clinical Applications 134
Geriatrics – Medical Sciences 147
Geriatrics – Clinical Applications 153
Haematology – Medical Sciences 163
Haematology – Clinical Applications 176
Infectious Diseases – Medical Sciences 203
Infectious Diseases – Clinical Applications 213
Immunology – Medical Sciences 238
Immunology – Clinical Applications 249
Neurology – Medical Sciences 258
Neurology – Clinical Applications 270
Oncology – Medical Sciences 298
Oncology – Clinical Applications 304
Pharmacology – Medical Sciences 323
Pharmacology – Clinical Applications 344
Renal Medicine – Medical Sciences 357
Renal Medicine – Clinical Applications 363
Respiratory Medicine – Medical Sciences 384
Respiratory Medicine – Clinical Applications 401
Rheumatology – Medical Sciences 429
Rheumatology – Clinical Applications 440
1

Medical Sciences

Cardiology

Question 1
A 47-year-old woman with overt thyrotoxicosis is found to have aortic stenosis.

Which one of the following features, if present in this woman, is most likely to indicate the need for
aortic valve replacement?

A. A peak systolic left ventricular aortic pressure gradient of 60 mmHg.


B. The presence of New York Heart Association (NYHA) class III angina.
C. A resting heart rate greater than 120/minute.
D. An aortic valve orifice area of 0.9 cm2.
E. The presence of concomitant moderate aortic regurgitation.

Question 2
A 35-year-old woman with hypertrophic cardiomyopathy presents with chest pain and palpitations.
Her heart rate is 155/minute and irregular, and her blood pressure is 130/80 mmHg.

What is the most appropriate initial therapy?

A. Beta-adrenergic blocker.
B. Nitroglycerine.
C. Amiodarone.
D. Frusemide.
E. Digoxin.
2

Question 3
The picture below shows a computed tomography (CT) scan of the heart.

Which one of the following is the most likely diagnosis?

A. Metastatic tumour in the pericardium.


B. Left atrial myxoma.
C. Ruptured myocardial papilla.
D. Cor triatriatum.
E. St. Jude mitral valve prosthesis.

Question 4
A young man presents with breathlessness and ankle swelling. He had a ventricular septal defect
(VSD) repaired at the age of three years. He is centrally cyanosed and has a prominent right
ventricular heave on palpation of the chest.

The most likely finding is:

A. Pulmonary hypertension.
B. Blood oxygen saturation of 95%.
C. Left ventricular hypertrophy.
D. Left to right shunt.
E. Normal haemoglobin.
3

Question 5
Which one of the following findings during a stress exercise ECG test is most predictive of
angiographically significant multivessel or left main coronary artery disease?

A. Late-onset of ST-segment depression.


B. Return to normal of ST-segment changes early in recovery phase.
C. ST-segment elevation in lead aVR.
D. Multifocal premature ventricular contractions.
E. Failure to increase systolic blood pressure by at least 10 mmHg.

Question 6
Which one of the following physical findings would be least expected in association with the chest
X-ray shown below?

A. A soft pulmonary component of the second heart sound (P2).


B. Prominent "v" wave in the jugular venous pressure.
C. Right ventricular lift.
D. Ejection click.
E. Early diastolic murmur.
4

Question 7
Which one of the following echocardiographic findings is most specific for hypertrophic
cardiomyopathy? The presence of:

A. Systolic anterior motion of the mitral valve.


B. Late systolic closure of the aortic valve.
C. A ratio of septal to posterior wall thickness of greater than 1.3:1.0.
D. Left ventricular outflow tract obstruction.
E. Abnormal Doppler mitral inflow pattern.

Question 8
In pregnant patients, which clinical feature is most likely to be associated with chronic hypertension
as opposed to pre-eclampsia?

A. Young age (less than 20 years).


B. Proteinuria.
C. Primigravida status.
D. Systolic blood pressure less than 160 mmHg.
E. Elevated diastolic blood pressure at week 15 of pregnancy.

Question 9
The most frequent complication of cardiac catheterisation is:

A. Anaphylactic contrast reaction.


B. Myocardial infarction.
C. Cerebral emboli.
D. Arterial false aneurysm.
E. Ventricular fibrillation

Question 10
Which one of the following interventions is least likely to lower blood pressure?

A. Weight reduction.
B. Reduction in dietary sodium.
C. A daily exercise program.
D. Smoking cessation.
E. Reduction of ethanol consumption to less than 10 g/day.
5

Question 11
During pregnancy, many cardiovascular conditions are associated with an increased risk of maternal
mortality.

Which one of the following cardiovascular conditions is associated with the highest rate of maternal
mortality?

Moderate to severe:

A. Pulmonary hypertension.
B. Hypertrophic cardiomyopathy.
C. Coarctation.
D. Aortic regurgitation.
E. Peripartum cardiomyopathy.

Question 12
The force of contraction of normal cardiac myocytes is determined primarily by the passage of
calcium ions (Ca2+) via:

A. The sodium-calcium exchanger.


B. Plasma membrane Ca2+ channels.
C. The plasma membrane calcium pump.
D. Cisternal Ca2+ channels in the sarcoplasmic reticulum.
E. The Ca2+ pump of the sarcotubular network.

Question 13
A patient with aortic regurgitation has the following haemodynamic measurements:

cardiac output (CO) 7.5 L/minute


heart rate (HR) 75/minute
left ventricular end-diastolic volume (LVEDV) 200 mL
left ventricular end-systolic volume (LVESV) 50 mL

The regurgitant fraction is defined as the ratio of the regurgitant volume to the total volume flowing
through the valve with each beat.

The regurgitant fraction in this patient is:

A. 25%.
B. 33%.
C. 50%.
D. 67%.
E. 75%.
6

Question 14
An athlete has the following physiological measurements performed at rest and at maximal exercise.
Which of these is most likely to remain unchanged from rest to maximal exercise?

A. Coronary sinus oxygen concentration.


B. Ratio of Vo2 (rate of consumption of O2) to Vco2 (rate of production of CO2).
C. Coronary sinus lactate concentration.
D. Mixed venous oxygen concentration.
E. Systolic blood pressure.

Question 15
Maximal oxygen consumption (Vo2 maximum) is the best measure of aerobic capacity or
cardiovascular fitness and declines with age. In healthy ageing, the most important physiological
change which contributes to this decline is a reduction in:

A. Maximum heart rate.


B. Stroke volume.
C. Arterial PO2.
D. Forced expiratory volume in 1 second (FEV1).
E. Total lung capacity.

Question 16
A patient with aortic regurgitation has the following haemodynamic measurements:
cardiac output (CO) 7.5 L/minute
heart rate (HR) 75/minute
left ventricular end-diastolic volume (LVEDV) 200 mL
left ventricular end-systolic volume (LVESV) 50 mL

The regurgitant fraction is defined as the ratio of the regurgitant volume to the total volume flowing
through the valve with each beat.

The regurgitant fraction in this patient is:

A. 25%.
B. 33%.
C. 50%.
D. 67%.
E. 75%.
7

Question 17
A 36-year-old woman has a very strong family history of premature coronary disease. Physical
examination is normal. Fasting lipid profile reveals:
total cholesterol 9.5 mmol/L [2.5-5.2]
high-density lipoprotein (HDL) cholesterol 1.5 mmol/L [0.9-1.8]
triglycerides 2.00 mmol/L [0.10-1.85]
Which abnormality is most likely to be present?

A. apoB 3500 mutation.


B. Cholesteryl ester transfer protein (CETP) deficiency.
C. Hypoalphalipoproteinaemia.
D. Low-density lipoprotein (LDL) receptor defect.
E. Lecithin:cholesterol acetyltransferase (LCAT) deficiency.

Question 18
A 40-year-old woman with a 20-year history of systemic lupus erythamatosus (SLE) presents with
severe anterior chest pain, dyspnoea and nausea.
Her SLE has manifested previously as arthritis, rash, mucosal ulceration and focal proliferative
glomerulonephritis. She has had two miscarriages and delivered two normal children. Treatment has
included hydroxychloroquine, prednis(ol)one in doses of 7.5 mg to 100 mg daily, intermittent
cyclophosphamide and azathioprine. At presentation she is taking prednis(ol)one 7.5 mg daily and
hydroxychloroquine.
Physical examination reveals obesity, pulse rate of 105/minute, blood pressure of 105/75 mmHg,
normal heart sounds, and clear lung fields. The jugular venous pulse and pressure are obscured by
fat.
Laboratory results show:
haemoglobin 124 g/L [115-165]
9
white cell count 3.4 x10 /L [3.5-11.0]
9
lymphocytes 0.9 x10 /L [1.5 – 4.0]
erythrocyte sedimentation rate (ESR) 35 mm/hr [0-20]
ECG shows ST-segment elevation in the anterior chest leads and sinus tachycardia. During the
assessment, the patient develops ventricular fibrillation and dies. Which of the following is the most
likely cause of her death?

A. Vasculitis involving the coronary arteries.


B. Libman-Sacks endocarditis with coronary artery embolisation.
C. Viral myocarditis.
D. Coronary atherosclerosis.
E. Pericardial effusion with tamponade.
8

Question 19
In a patient with rheumatic mixed mitral valve disease, for which of the following procedures is
there the strongest indication for antibiotic prophylaxis?

A. Flexible bronchoscopy.
B. Cardiac catheterisation.
C. Colonoscopy without biopsy.
D. Oesophageal dilatation.
E. Gastroscopy.

Question 20
Which of the following best explains why the jugular venous pressure (JVP) is raised in patients with
congestive heart failure?

A. Pulmonary arterial pressure is increased.


B. Blood pools in the right heart because left heart output is reduced.
C. The failing right ventricle requires a higher distending pressure.
D. The enlarged left atrium compresses the right atrium.
E. The enlarged left ventricle compresses the right ventricle.

Question 21
A 36-year-old woman has a very strong family history of premature coronary disease. Physical
examination is normal. Fasting lipid profile reveals:

total cholesterol 9.5 mmol/L [2.5-5.2]


high-density lipoprotein (HDL) cholesterol 1.5 mmol/L [0.9-1.8]
triglycerides 2.00 mmol/L [0.10-1.85]

Which abnormality is most likely to be present?

A. apoB 3500 mutation.


B. Cholesteryl ester transfer protein (CETP) deficiency.
C. Hypoalphalipoproteinaemia.
D. Low-density lipoprotein (LDL) receptor defect.
E. Lecithin:cholesterol acetyltransferase (LCAT) deficiency.

Question 22
Which of the following is most likely to affect interpretation of the pulmonary artery occlusion
pressure trace?

A. Right ventricular outflow tract obstruction.


B. Tricuspid regurgitation.
C. Mitral regurgitation.
D. Pulmonary vein obstruction.
E. Aortic stenosis.
9

Question 23
The diagram below shows the time-course of a typical ventricular myocyte action potential (Figure
1). Figure 1b shows the principal cation currents between the intra and extra-cellular space
concurrent with phase 0-3 of the action potential shown in Figure 1a. The direction and size of the
arrows in Figure 1b indicate whether the current is inward-directed or outward-directed and the
relative current strengths of the ion identified by a letter at the base of the arrow.

From the table below, select the cations that BEST represent X, Y, and Z in Figure 1b.
A. X=Na+ Y=Ca2+ Z=K+
B. X=K+ Y=Na+ Z=Ca2+
C. X=Ca2+ Y=K+ Z=Na+
D. X=Na+ Y=K+ Z=Ca2+
E. X=Ca2+ Y=Na+ Z=K+

Question 24
A patient presents with an anterior non-ST elevation myocardial infarction with transient T-wave
inversion in V3-6. There is a typical rise in CK-MB (creatine kinase myocardial tissue) (2 x URL at
12 hr and 5 x URL at 24 hrs) and Troponin T (2 x URL at 12 hr and 20 x URL at 24 hrs). Levels are
expressed as a multiple of the upper reference limit (URL). 72 hours after the onset of the first
episode, the patient experiences recurrent chest pain. If present, which of the following BEST
indicates that the patient has had a recurrent myocardial
infarct?

A. The left anterior artery is occluded at angiography.


B. CK-MB (creatine kinase myocardial tissue) = 5 x URL 12 hrs after recurrent pain.
C. Troponin T = 10 x URL 12 hrs after recurrent pain.
D. Pain duration and intensity is similar to the first event.
E. The ECG now shows deep T-wave inversion in V3-6.
10

Question 25
Symptom-limited treadmill exercise testing is often done in the weeks after ST elevation myocardial
infarction to identify those at low risk of death. Which of the following parameters at exercise testing
most strongly indicates a good prognosis with medical treatment?

A. Absence of ST depression.
B. % predicted maximal heart rate achieved.
C. Absence of chest pain.
D. Absence of ventricular arrythmias.
E. Above average exercise capacity.

Question 26
Which of the following factors is most strongly associated with risk of sudden death in the first six
months after myocardial infarction?

A. Persistent occlusion of the infarct-related artery.


B. Cigarette smoking.
C. 3-vessel coronary disease at angiography.
D. Low left ventricular ejection fraction.
E. High LDL (low density lipoprotein) cholesterol

Question 27
In a patient with rheumatic mixed mitral valve disease, for which of the following procedures is there
the strongest indication for antibiotic prophylaxis?

A. Flexible bronchoscopy.
B. Cardiac catheterisation.
C. Colonoscopy without biopsy.
D. Oesophageal dilatation.
E. Gastroscopy.

Question 28
Differentiation between cardiogenic and non-cardiogenic pulmonary edema is most reliably
performed by measurement of:

A. left ventricular end diastolic pressure.


B. left ventricular peak systolic pressure.
C. pulmonary artery peak systolic pressure.
D. pulmonary artery diastolic pressure.
E. pulmonary artery mean pressure.
11

Question 29
A 70-year-old man is admitted with general malaise, nausea and vomiting. He had recently suffered
from gastroenteritis with three days of diarrhoea. He has longstanding hypertension and steoarthritis
of his knees. He is taking Bendrofluazide, Enalapril, Aspirin, Codeine and Ibuprofen.
Serum biochemistry shows:

Sodium 132 mmol/L [137-147]


Potassium 8.4 mmol/L [3.5-5.0]
Creatinine 0.32 mmol/L [0.5-1.0]
Urea 17.8 mmol/L [3.5-6.5]
Bicarbonate 14.5 mmol/L [22.0-28.0]

His ECG is shown below.

Which is the most appropriate initial treatment?

A. Calcium gluconate.
B. Magnesium sulphate.
C. Sodium bicarbonate.
D. Amiodarone.
E. Lignocaine.
12

Question 30
Which of the following findings most favours a diagnosis of constrictive pericarditis over cardiac
tamponade?

A. Pulsus paradoxus.
B. Third heart sound.
C. Kussmaul’s sign.
D. Prominent y-descent.
E. Low voltage on ECG.

Question 31
Which of the following maternal conditions is associated with the greatest risk of cardiac
complications during pregnancy?

A. Atrial septal defect.


B. Ventricular septal defect.
C. Eisenmenger syndrome.
D. Patent ductus arteriosus.
E. Mitral valve prolapse.

Question 32
A 25-year-old female indigenous patient has a history of rheumatic fever. She presents with
increasing shortness of breath. She describes paroxysmal nocturnal dyspnoea and pedal oedema. On
examination her blood pressure is 95/60 mmHg. The jugular venous pressure is elevated with a
prominent a wave. The S1 is loud, S2 is normal. There is an early diastolic sound after S2 with a
diastolic rumble at the apex. There is also an early diastolic murmur which is accentuated with
expiration. There is hepatomegaly and pitting oedema to the mid shin.

What is the most likely diagnosis?

A. Mitral stenosis.
B. Mitral stenosis and tricuspid regurgitation.
C. Mitral stenosis and tricuspid stenosis.
D. Mitral stenosis and aortic regurgitation.
E. Mitral stenosis and pulmonary stenosis.
13

Answers

1. D
2. A
3. B
4. A
5. E
6. A
7. C
8. E
9. D
10. D
11. A
12. D
13. B
14. A
15. A
16. B
17. D
18. D
19. D
20. C
21. D
22. D
23. A
24. B
25. E
26. D
27. D
28. A
29. A
30. D
31. C
32. D
14

Clinical Applications

Cardiology

Question 1
A 60-year-old man is to undergo elective cholecystectomy. He has no significant medical history,
and has no symptoms on specific enquiry. Physical examination is normal.
His ECG is shown below.

Which one of the following is most appropriate before proceeding to surgery?

A. Exercise stress test.


B. Dobutamine stress echocardiography.
C. Dipyridamole thallium scan.
D. Coronary angiography.
E. No further cardiac investigation.
15

Question 2
A 35-year-old woman has increasing breathlessness on exertion. Her cardiac silhouette is slightly
enlarged on a chest X-ray and an ECG demonstrates sinus rhythm.

The continuous wave Doppler flow signal through the mitral inflow tract (shown above) is most
consistent with which one of the following?

A. Severe pulmonary hypertension (cor pulmonale).


B. Aortic stenosis.
C. Mitral regurgitation.
D. Mitral stenosis.
E. Aortic regurgitation.
16

Question 3
A 48-year-old woman presents to the emergency department of a large rural hospital with severe
chest pain. An ECG shows 3-4 mm ST elevation in the anteroseptal leads.

The risk of major haemorrhage complicating thrombolysis therapy is greatest in a patient who:

A. Has polycystic kidneys.


B. Had an open cholecystectomy two weeks ago.
C. Has recurrent heartburn.
D. Is currently menstruating.
E. Had an admission blood pressure of 160/95 mmHg.

Question 4
The major cause of death in patients more than six months following cardiac transplantation is:

A. Graft-versus-host reaction.
B. Opportunistic infection.
C. Rejection.
D. Malignancy.
E. Coronary artery disease.

Question 5
Digoxin is indicated for use in all of the following except:

A. Prevention of paroxysmal atrial fibrillation.


B. Systolic heart failure unresponsive to angiotensin-converting enzyme (ACE) inhibitors and loop
diuretics.
C. Systolic heart failure with co-existing atrial fibrillation.
D. Acute atrial fibrillation with rapid ventricular rate.
E. Diastolic dysfunction with co-existing rapid atrial fibrillation.
17

Question 6
A general practitioner refers a 55-year-old woman to a cardiologist for assessment of atypical chest
pain unrelated to exercise. She smokes 25 cigarettes/day, has hypertension controlled with enalapril
10 mg/day and hydrochlorothiazide 50 mg/day. Plasma cholesterol and glucose levels have not been
measured in the last five years. Her father had a heart attack at 65 years of age and her mother has
hypertension and diabetes controlled by diet.

The resting ECG is shown

Which one of the following is the most appropriate test to determine her risk of future major
coronary events?

A. Stress electrocardiography.
B. Computed tomography (CT) scanning for coronary calcification.
C. Dipyridamole radionuclide perfusion scanning.
D. Exercise radionuclide perfusion scanning.
E. Contrast echocardiography
18

Question 7
A 65-year-old woman complains of exertional dyspnoea which has increased over several weeks.
She takes hydrochlorothiazide for longstanding hypertension. Her father died suddenly at 72 years of
age. Her height is 162 cm and her weight is 80 kg. Blood pressure is 165/90 mmHg and her pulse is
60/minute. Breath sounds are reduced. There are no wheezes or crackles. An echocardiogram reveals
normal systolic function and increased left ventricular wall thickness. Her ECG is shown (see
below).

The most likely cause of her dyspnoea is:

A. Myocardial ischaemia.
B. Diastolic dysfunction.
C. Asthma.
D. Obesity.
E. Pulmonary embolism.
19

Question 8
A 48-year-old woman with primary pulmonary hypertension and New York Heart Association
(NYHA) class III symptoms undergoes right heart catheterisation. The table below shows the results
of haemodynamic variables at baseline and following peak response to 40% inspired oxygen,
prostacyclin, amlodipine and ramipril.

Mean
Mean Pulmonary Pulmonary Systemic
Pulmonary
Capillary Wedge Vascular Vascular
Arterial
Pressure Resistance Resistance
Pressure
(mmHg) (Wood units) (Wood units)
(mmHg)
Baseline 54 12 12 22
40% Inspired
51 7 11 19
Oxygen
Prostacyclin 50 15 7 14
Amlodipine 58 8 10 13
Ramipril 52 10 11 14

Which one of the following treatments is likely to be most beneficial for her primary pulmonary
hypertension?

A. Home oxygen.
B. Prostacyclin.
C. Amlodipine.
D. Ramipril.
E. Atrial septostomy.

Question 9
A 64-year-old man presents to hospital seven hours after onset of severe chest pain. His ECG shows
4-5 mm ST elevation in leads II, III and aVF, and complete heart block with a ventricular rate of
50/minute. His blood pressure is 115/60 mmHg which falls transiently to 90/50 mmHg with the
administration of streptokinase, before returning to 120/55 mmHg.

Four hours later, he is reviewed in the coronary care unit. His heart rate is 50/minute and his blood
pressure is 85/60 mmHg. His jugular venous pressure is 4 cm with cannon waves. A third sound is
audible but there are no murmurs. Breath sounds are normal. The patient complains of mild chest
pain which is relieved by sitting up. The ECG remains unchanged. Urine output has declined to 5
mL/h for the last two hours.

What is the most appropriate next step in management?

A. Insert a temporary pacing wire to increase the heart rate to about 80/minute.
B. Increase intravenous fluid administration.
C. Organise immediate rescue percutaneous transluminal coronary angioplasty (PTCA).
D. Commence an intravenous dobutamine infusion.
E. Insert a Swan-Ganz catheter for haemodynamic monitoring.
20

Question 10
A 36-year-old man presents with increasing central chest pain which is aggravated by Inspiration.
His temperature is 37.6°C. The blood pressure is 150/90 mmHg. The jugular venous pressure is not
elevated, and the cardiac impulse is not displaced. Heart sounds are normal, and there are no
murmurs or rubs. There are no signs of cardiac failure. The respiratory rate is increased and the chest
is clear.

His ECG is shown (see below).

Which one of the following is the most likely diagnosis?

A. Pulmonary embolism.
B. Unstable angina.
C. Left ventricular hypertrophy.
D. Pericarditis.
E. Acute myocardial infarction.
21

Question 11
A 79-year-old male is brought to the Emergency Department shortly after a fall in a shopping centre.
His medical history includes hypertension, hyperlipidaemia and a current infective exacerbation of
his chronic obstructive pulmonary disease. Medications are simvastatin, hydrochlorothiazide,
captopril and roxithromycin. On examination his temperature is 37.8oC and his blood pressure is
160/85 mmHg. He has not had any acute injury. Blood tests reveal an elevated creatine kinase of
1325 U/L [50 - 180 U/L]. Initial troponin is normal. His ECG is shown below.

The most likely explanation for the raised creatine kinase is:

A. Acute ST-elevation myocardial infarction.


B. Infective myositis.
C. Statin-induced myopathy.
D. Muscle injury.
E. Non ST-elevation myocardial infarction (STEMI).

Question 12
Which one of the following clinical features is least characteristic of acute dissection of the proximal
thoracic aorta?

A. ST-T wave abnormalities of the ECG.


B. Severe pain.
C. Hypotension.
D. Aortic regurgitation.
E. Brachial artery blood pressure difference.
22

Question 13
Which one of the following conditions is most likely to be associated with the continuous wave
Doppler flow trace shown (see below) of the left ventricular outflow tract?

A. Mitral stenosis and mitral incompetence.


B. Aortic stenosis.
C. Aortic stenosis and aortic incompetence.
D. Aortic stenosis and mitral incompetence.
E. Aortic incompetence.
23

Question 14
A 60-year-old man is to undergo elective cholecystectomy. He has no significant medical history,
and has no symptoms on specific enquiry. Physical examination is normal. His ECG is shown
below.

Which one of the following is most appropriate before proceeding to surgery?

A. Exercise stress test.


B. Dobutamine stress echocardiography.
C. Dipyridamole thallium scan.
D. Coronary angiography.
E. No further cardiac investigation.
24

Question 15
Public access to defibrillators has been provided in airports, casinos and aircraft for bystander
management of cardiac arrest. Ventricular fibrillation is documented in the majority of cases. When
this arrhythmia is detected and treated by the device, what is the most likely survival profile?

Survival to hospital
Survival at one year
admission
A. <10% <10%
B. 10-25% <10%
C. 10-25% 10-25%
D. >25% 10-25%
E. >25% >25%

Question 16
A 67-year-old woman with congestive cardiac failure remains breathless on moderate exertion
despite treatment with 40 mg frusemide and 20 mg enalapril daily. On examination she has a pulse
rate of 80/minute, blood pressure of 125/70 mmHg and a jugular venous pressure (JVP) of +1 cm.
She has a soft systolic murmur with no added sounds, her chest is clear and she has no oedema. An
ECG shows sinus rhythm. A chest X-ray shows cardiomegaly with a cardiothoracic ratio of 15.5/28
but no pulmonary congestion. Echocardiography demonstrates systolic dysfunction with fractional
shortening of 18% and mild mitral regurgitation. Her serum creatinine level is normal.

Which of the following is the most appropriate next step in treatment?

A. Increase the frusemide dose.


B. Add digoxin.
C. Add an aldosterone antagonist.
D. Add an angiotensin II receptor antagonist.
E. Add a beta blocker.

Question 17
A 70-year-old man with type 2 (non-insulin-dependent) diabetes mellitus and obesity is due to have a
colon cancer resection. His current medication is twice daily insulin. His exercise tolerance is good
and his baseline ECG is normal.

Which of the following would be the most appropriate treatment to add in order to reduce the risk of
perioperative cardiac events?

A. Metoprolol.
B. Aspirin.
C. Amiodarone.
D. Magnesium.
E. Nitrates.
25

Question 18
A 68-year-old man presents with an altered conscious state. Three days prior to presentation he had
an episode of ataxia and vertigo, with residual imbalance. On the day prior to presentation he
developed an acute horizontal diplopia, which persists. He is brought to hospital after a sudden
deterioration in his level of consciousness.

The most likely diagnosis is:

A. pontine haemorrhage.
B. paraneoplastic syndrome.
C. acute disseminated encephalomyelitis (ADEM).
D. multiple cardioembolic strokes.
E. basilar artery thrombosis.

Question 19
Which one of the following features is most suggestive of autosomal dominant familial
hypercholesterolaemia rather than other causes of hypercholesterolaemia?

A. Tendon xanthomas.
B. Corneal opacities.
C. Lipaemia retinalis.
D. Eruptive xanthomas.
E. Recurrent pancreatitis.

Question 20
A 28-year-old woman presents to the emergency department with a four-day history of headache and
blurring of vision. She has previously been well. She is on no medication.

On examination she is very confused and disorientated. Her blood pressure is 230/140 mmHg. Her
jugular venous pressure (JVP) is not elevated, heart sounds are normal and lung bases clear. Her
reflexes are generally brisk. Fundoscopy shows bilateral haemorrhages, exudates and papilloedema.

Which of the following is the most appropriate treatment?

A. Intravenous sodium nitroprusside.


B. Intravenous frusemide.
C. Oral nifedipine.
D. Intramuscular hydralazine.
E. Oral enalapril.
26

Question 21
A 65-year-old woman presents with a scalp laceration sustained after her second syncopal episode in
seven days. She takes metoprolol, simvastatin and amitryptyline for hypertension,
hypercholesterolaemia and depression respectively. Cardiovascular examination and all non-invasive
cardiac investigations are unremarkable.

For which of the following ECG findings is electrophysiology testing least useful for management?

A. Polymorphic ventricular extrasystoles.


B. Paroxysmal atrial fibrillation (rate 110/minute).
C. Significant QT prolongation.
D. Second degree heart block during sleep.
E. Sinus bradycardia.

Question 22
The major cause of death in patients more than six months following cardiac transplantation is:

A. Graft-versus-host reaction.
B. Opportunistic infection.
C. Rejection.
D. Malignancy.
E. Coronary artery disease.

Question 23
A 65-year-old man presents to the emergency department with an anterior ST elevation myocardial
infarction. He has a past history of a q-wave inferior myocardial infarction 10 years earlier. He
receives thrombolysis, appropriate heparin, anti-platelet and beta-blocker therapy. His ECG shows
resolution of the ST elevation at six hours, but with evolution of antero-septal q waves. The peak
creatine kinase is 3,000 U/L [<180] and peak troponin T is 2.5 µg/L [< 0.01].

What is the most appropriate investigation to guide management in this patient?

A. Coronary angiography.
B. An imaging study to assess viability of the anterior wall.
C. An exercise ECG.
D. Transthoracic echocardiogram.
E. Stress echocardiograph.
27

Question 24
A 49-year-old man sees you six weeks after inferior Q-wave myocardial infarction treated at a
country hospital. Current drug treatment is aspirin 100mg, ramipril 5mg, and atorvastatin 20mg
2
daily. He is obese (body mass index 31 kg/m [18-25]). Glycosylated haemoglobin is 6.0%.

Fasting lipid profile reveals:

total cholesterol 4.8 mmol/L [2.5-5.2]


high-density lipoprotein (HDL) cholesterol 0.5 mmol/L [0.9-1.8]
triglycerides 9.80 mmol/L [0.10-1.85]

Which one of the following is the most effective drug treatment to lower the triglyceride level?

A. Add a bile acid binding resin.


B. Increase the dose of atorvastatin.
C. Add metformin.
D. Change atorvastatin to an alternative statin.
E. Add a fibrate.

Question 25
A 67-year-old woman with congestive cardiac failure remains breathless on moderate exertion
despite treatment with 40 mg frusemide and 20 mg enalapril daily. On examination she has a pulse
rate of 80/minute, blood pressure of 125/70 mmHg and a jugular venous pressure (JVP) of +1 cm.
She has a soft systolic murmur with no added sounds, her chest is clear and she has no oedema. An
ECG shows sinus rhythm. A chest X-ray shows cardiomegaly with a cardiothoracic ratio of 15.5/28
but no pulmonary congestion. Echocardiography demonstrates systolic dysfunction with fractional
shortening of 18% and mild mitral regurgitation. Her serum creatinine level is normal.

Which of the following is the most appropriate next step in treatment?

A. Increase the frusemide dose.


B. Add digoxin.
C. Add an aldosterone antagonist.
D. Add an angiotensin II receptor antagonist.
E. Add a beta blocker.
28

Question 26
A 70-year-old man with type 2 (non-insulin-dependent) diabetes mellitus and obesity is due to have a
colon cancer resection. His current medication is twice daily insulin. His exercise tolerance is good
and his baseline ECG is normal.

Which of the following would be the most appropriate treatment to add in order to reduce the risk of
perioperative cardiac events?

A. Metoprolol.
B. Aspirin.
C. Amiodarone.
D. Magnesium.
E. Nitrates.

Question 27
An 81-year-old man presents to the emergency department after collapsing while running for a bus.
He has a history of hypertension treated with felodipine but no other major illnesses. He has noted
mild breathlessness on exertion, worsening over the past few years. Examination reveals blood
pressure is 125/90 mmHg. He has a soft systolic murmur but no signs of heart failure. His ECG is
shown below. Serum creatine kinase (CK) is 205 U/L [<180]. Troponin level is pending.

Which one of the following is the most appropriate initial management step?

A. Observation.
B. Intravenous heparin.
C. Intravenous amiodarone.
D. Thrombolysis.
E. Primary angioplasty.
29

Question 28
A 56-year-old man is referred for assessment of resistant hypertension. Blood pressure readings at
his general practitioner’s surgery have been between 145 and 172 systolic and 86 and 98 diastolic
despite various combinations of up to two anti-hypertensive agents at a time. He has no other
vascular risk factors. He has no family history of hypertension. Examination does not reveal any
clinical findings suggesting a secondary cause of hypertension. He has normal creatinine and
electrolytes. An ECG and trans thoracic echocardiogram are normal

What is the most appropriate next investigation?

A. Aldosterone : renin ratio.


B. Magnetic resonance angiography of renal arteries.
C. Ambulatory blood pressure recording.
D. Renal ultrasound.
E. Urine testing for drug compliance.

Question 29
A 65-year-old man collapses during a brisk walk, losing consciousness for 30 seconds. He had no
warning symptoms and he recovered rapidly. Six years earlier he suffered an uncomplicated
myocardial infarction without further chest pain and is taking aspirin, atenolol and simvastatin. On
examination he has an apex rate of 60, and blood pressure 118/84 mmHg. There is an ejection
systolic murmur and a left carotid bruit. ECG shows Q waves in the inferior leads with no acute
changes. Troponins are normal. Overnight monitoring shows sinus rhythm with ventricular ectopics
and runs of up to five beats of non-sustained ventricular tachycardia.

Which is the most appropriate investigation for the patient’s syncope?

A. Treadmill exercise ECG.


B. Transthoracic echocardiogram.
C. Carotid ultrasound.
D. Electrophysiologic study.
E. Tilt table testing.

Question 30
Several randomised controlled studies have compared anticoagulation with placebo in patients with
non-rheumatic atrial fibrillation. These trials suggest that the annual risk of stroke in untreated
patients with atrial fibrillation would be nearest to:

A. 2%
B. 5%
C. 8%
D. 12%
E. 15%
30

Question 31
A 58-year-old male presented with an acute inferior ST elevation myocardial infarction to a rural
centre and received thrombolysis. There was no resolution of ST elevation but he became pain free
after 18 hours of chest pain. The patient had no further chest pain. A week later coronary
angiography was performed. This showed a proximal occlusion of the right coronary artery with no
significant disease elsewhere. Cardiac systolic function was only mildly impaired. In addition to
optimal medical therapy and risk factor modification, which of the following is the most appropriate
next step?

A. Coronary artery bypass grafting of the occluded vessel.


B. Percutaneous angioplasty and stenting of the occluded vessel.
C. Percutaneous angioplasty and stenting of the occluded vessel with a drug-eluting stent.
D. No additional intervention.
E. Implantation of a cardiac .

Question 32
The ECG shown below is taken from a 25-year-old man after an episode of syncope. His mother
died suddenly at age 38; his younger sister has recently survived an out-of-hospital cardiac arrest.
The most appropriate management would be:

A. Dual chamber pacemaker.


B. Sotalol.
C. Amiodarone.
D. Flecainide.
E. Implantable defibrillator.

Question 33
A 56-year-old man suffers a community cardiac arrest. Cardiopulmonary resuscitation (CPR) is
started by a bystander and continued by ambulance paramedics who arrive five minutes later. He is
intubated and given adrenaline. His initial rhythm is ventricular fibrillation (VF). On arrival in the
emergency department he remains in VF despite three direct current shocks en route. Giving which
of the following agents is most likely to result in a successful defibrillation?

A. Metoprolol.
B. Bretyllium.
C. Amiodarone.
D. Bicarbonate.
E. Magnesium.
31

Question 34
A 62-year-old female presents with severe central chest pain unrelated to exertion. She is given
sublingual nitrates in the ambulance and the pain resolves 15 minutes later. The ECG is normal. The
first troponin is normal. She is observed in a chest pain unit. The second troponin is normal. What is
the next most appropriate investigation?

A. Upper gastrointestinal (GI) endoscopy.


B. Myocardial perfusion study.
C. Coronary angiography.
D. Exercise stress test.
E. CT coronary angiography.

Question 35
A 70-year-old woman is admitted to a rural hospital with chest pain of three hours duration. ECG
shows anterior ST elevation with Q waves. She is given oral aspirin and intravenous plasminogen
activator. One hour later she still has pain despite nitrates and ST elevation persists. Her heart rate is
102/min, blood pressure 150/110, there are bibasal crackles with oxygen saturation 98%.

What should be the next step in her management?

A. Intravenous heparin.
B. Intravenous beta-blocker.
C. Oral clopidogrel.
D. Intravenous frusemide.
E. Oral diltiazem.

Question 36
A 54 year-old man undergoes primary angioplasty and stent of a total right coronary artery occlusion
after presenting with ST elevation inferior infarction. A 70% stenosis is noted in the mid left anterior
descending (LAD) artery; no other stenosis is evident. He is treated with antiplatelet therapy, a beta
blocker and a statin and undergoes a limited exercise test two days later. He completes a submaximal
exercise test (approximately 9 METS [Metabolic equivalent units]) without symptoms; heart rate
reaches 120/min (75% predicted maximal) and blood pressure 170 systolic. Planar ST depression
first appears at 7 METS and is maximal at 1.5mm in leads V3-V6 at peak.

What should be the next step in his management?

A. LAD stent.
B. Coronary bypass surgery.
C. Dipyridamole sestamibi scan.
D. Exercise echo.
E. Medical treatment.
32

Question 37
An 80-year-old female has a routine ECG which is shown below. She is living independently in her
own home and has no symptoms of dyspnoea or chest pain. Her BP is 165/90 mmHg. Which of the
following management strategies is most appropriate for her arrhythmia?

A. Amiodarone.
B. Sotalol.
C. Direct current cardioversion.
D. Metoprolol.
E. Flecainide.

Question 38
A 51-year-old man presents with unstable angina. His mother died at age 58 years as a result of a
myocardial infarction and his 48-year-old brother, who smoked heavily and was overweight,
survived a heart attack two years previously. After discharge he consults the internet and, when
reviewed, expresses concern that he may have familial hypercholesterolemia and that there could be
a risk that his son, aged 11 years, may also be at risk of heart attack at an early age.What finding
would give you greatest concern that the patient is correct and that is a need to consider further the
possibility that other members of his family are at risk of early onset coronary vascular disease?

A. Arcus cornealis.
B. Tendinous xanthomata.
C. LDL (lowdensity lipoprotein) cholesterol of 7.8 mmol/litre [<3.0 mmol/L].
D. Xanthelasma.
E. His son’s LDL cholesterol level is 1.5 standard deviations above the mean for 11 year olds.
33

Question 39
A previously well 25-year-old man is admitted unconscious following a suspected overdose. He
withdraws to pain and has dilated pupils that react sluggishly. His heart rate is 120/minute and he has
decreased bowel sounds. His ECG is shown above. The presentation is most consistent with
overdose of which of the following drugs?

A. Potassium.
B. Amitryptyline.
C. Lithium.
D. Cocaine.
E. Dextropropoxyphene.
34

Question 40
A 56-year-old male presents with chest pain and the ECG shown below. The symptoms have been
present for 45 minutes. A facility with acute angioplasty capability is located 180 minutes travel
time. There is no contraindication to thrombolysis.

The most appropriate management of this patient is:

A. Aspirin, heparin and immediate transfer for acute angioplasty.


B. Aspirin, heparin and tirofiban and transfer for acute angioplasty.
C. Immediate thrombolysis.
D. Immediate thrombolysis and transfer for acute angioplasty.
E. Ibuprofen.

Question 41
A 65-year-old male has chest pain when he walks up hills. His pattern of angina has been unchanged
over the last five years. He is taking optimal medical therapy and follows a healthy lifestyle. He asks
about his prognosis with angioplasty and stenting. Compared with optimal medical therapy, stenting
would be most likely to reduce:

A. His risk of myocardial infarction.


B. His risk of cardiac death.
C. The frequency of angina.
D. The need for subsequent coronary artery bypass grafting.
E. The risk of arrhythmia.
35

Question 42
A patient in the coronary care unit suffers a cardiac arrest due to ventricular fibrillation. The initial
two direct current (DC) shocks are unsuccessful. Cardiopulmonary resuscitation (CPR) is initiated.
The Advanced Cardiac Life Support Guidelines now recommend what rate of chest compressions,
over what period of time after the second defibrillation before re-examining cardiac rhythm?

Question 43
A 70-year-old male presents with an irregular heart beat and the ECG confirms atrial fibrillation. In
terms of stroke prevention, which of the following is the strongest indication for recommending long
term oral anticoagulation in this patient?

A. The patient’s age.


B. Hypertension.
C. Diabetes.
D. Dyslipidaemia.
E. Mitral stenosis.
36

Question 44
An 87-year-old male has New York Heart Association (NYHA) Stage IV heart failure.
Echocardiography has shown systolic dysfunction with an estimated ejection fraction of 15% and his
chest X-ray is shown below. He has been admitted to high level residential care (nursing home) and
is bedbound with dyspnoea and fatigue secondary to heart failure. Current cardiac medications are
frusemide and enalapril.

The most effective treatment to relieve his dyspnoea is:

A. Carvedilol.
B. Morphine.
C. Candesartan.
D. Spironolactone.
E. Oxygen
37

Question 45
A 55-year-old male diabetic patient has a history of acute onset of severe mid-sternal chest
discomfort during sleep. The patient is diaphoretic. The heart rate is 58 beats/minutes. The blood
pressure is 88/62 mmHg in the right arm and 92/60 mmHg in the left arm. The respiratory rate is 20
breaths/minute and there is marked jugular venous pressure elevation. Auscultation reveals no
murmurs but an S4 gallop is heard. There are clear lung fields. There is no peripheral oedema.

His ECG is shown below:

What is the most likely diagnosis?

A. Acute pericarditis.
B. Aortic dissection.
C. Acute pulmonary embolism.
D. Right ventricular infarction.
E. Acute pericardial tamponade.
38

Answers
1. E
2 D
3. B
4. E
5. A
6 C
7. B
8 B
9. B
10. D
11. C
12. C
13. C
14. E
15. E
16. E
17. A
18. C
19. A
20. A
21. C
22. E
23. A
24. E
25. E
26. A
27. A
28. C
29. B
30. B
31. D
32. E
33. C
34. D
35. B
36. E
37. D
38. B
39. B
40. C
41. C
42. A
43. E
44. B
45. D
39

Medical Sciences

Endocrinology

Question 1
Which one of the following is not characteristically elevated in the third trimester of normal
pregnancy?

A. Serum prolactin.
B. Serum free thyroxine (free T4).
C. Serum cortisol.
D. Serum alkaline phosphatase (ALP).
E. Serum iron binding capacity.

Question 2
Which one of the following drugs, when used continuously in the treatment of Paget's disease, is
most likely to cause defective mineralisation of newly formed bone matrix?

A. Pamidronate.
B. Calcitonin.
C. Mithramycin.
D. Alendronate.
E. Etidronate.

Question 3
Which one of the following best describes the association between hyperinsulinaemia (insulin
resistance) and coronary heart disease?

The relationship is:

A. Independent of total cholesterol.


B. Confined to subjects with low HDL (high density lipoprotein) cholesterol.
C. Confined to obese individuals.
D. Confined to hypertensive subjects only.
E. Confined to diabetic subjects only.
40

Question 4
Significant intercurrent illness is commonly associated with disturbances of thyroid function (sick
euthyroid syndrome).

Which one of the following is least likely to be present in a euthyroid patient with a severe illness?

A. Raised free thyroxine (free T4).


B. Raised free triiodothyronine (free T3).
C. Low free T3.
D. Low thyroid-stimulating hormone (TSH).
E. Raised TSH

Question 5
Which one of the following is the most common biochemical association of obesity?

A. Mutation of the insulin receptor.


B. Reduced serum leptin.
C. Mutation of the •3-adrenergic receptor.
D. Raised serum C-peptide.
E. Raised blood glucose.

Question 6
Which one of the following is least likely to alter parathyroid hormone secretion by direct effect on
the parathyroid gland?

A. Parathyroid hormone.
B. Ionized calcium.
C. Phosphate.
D. Calcium sensing receptor.
E. 1, 25 (OH)2 vitamin D.
41

Question 7
A 28-year-old woman presents with galactorrhoea. She has been treated for hypothyroidism for five
years with thyroxine 0.1 mg/day. She is also receiving lithium carbonate for depression and her
serum lithium levels are in the mid-therapeutic range. She takes diazepam 5 mg nocte for insomnia
and metoclopramide 10 mg as required for nausea. Her serum prolactin concentration is 1800 mU/L
[<600].

From this history, which one of the following factors is most likely to be responsible for her
hyperprolactinaemia and galactorrhoea?

A. Use of metoclopramide.
B. History of hypothyroidism.
C. Treatment with thyroxine.
D. Use of lithium carbonate.
E. Use of diazepam.

Question 8
A 68-year-old man with rheumatoid arthritis for 15 years has the following bone densitometry results
on dual energy X-ray absorptiometry scan:

T score Z score
(related to young normal, (related to age,
sex specific mean value) sex specific mean value)
Lumbar spine (L2-4) +1.3 +2.0
Femoral neck -2.4 -1.7

The most likely explanation for the discrepancy between the femoral neck and lumbar spine readings
is:

A. Technical error.
B. Paget's disease in lumbar spine.
C. Lumbar spondylosis.
D. Secondary osteoarthritis in the hip.
E. Synovitis in the hip.
42

Question 9
The most likely cause of thyrotoxicosis producing the appearance shown in this thyroid scintiscan is:

A. Activating mutation of the TSH receptor.


B. Abnormal Gs protein.
α

C. Thyroid-stimulating immunoglobulin.
D. Ret proto-oncogene mutation.
E. Coxsackievirus infection.

Question 10
Which of the following best explains the lipid-modifying effect of fibric acids (such as gemfibrozil)?

A. Increasing low density lipoprotein (LDL) receptor expression.


B. Stimulating lipoprotein lipase activity.
C. Interfering with bile acid reabsorption in the small intestine.
D. Inhibiting hepatic cholesterol synthesis.
E. Inhibiting LDL peroxidation.
43

Question 11
A 63-year-old woman presents with weight gain, reduced energy and depression, five years after
pituitary surgery and radiotherapy for a non-functioning pituitary adenoma. Current medications are
thyroxine 0.15 mg/day, hydrocortisone 20 mg/day (equivalent to cortisone acetate 25 mg/day) in
divided doses, conjugated oestrogen 0.625 mg/day, medroxyprogesterone 2.5 mg/day, and fluoxetine
20 mg/day.

Examination is unremarkable except for truncal obesity. Her body mass index (BMI) is 30 kg/m2
[20-25] and her blood pressure is 145/85 mmHg.

Which one of the following is the most likely cause of her weight gain?

A. Inadequate thyroxine replacement dose.


B. Excess hydrocortisone replacement dose.
C. Excess oestrogen replacement dose.
D. Growth hormone deficiency.
E. Fluoxetine therapy.

Question 12
Which of the following mechanisms best describes the mode of action of the sulphonylurea class of
drugs?

A. Stimulates insulin synthesis.


B. Enhances insulin binding to the beta cell insulin receptor.
C. Stimulates insulin secretion.
D. Stimulates conversion of proinsulin to insulin and C-peptide.
E. Stimulates beta cell proliferation.

Question 13
In addition to decreased gastrointestinal calcium absorption, by which of the following mechanisms
is prednis(ol)one most likely to cause net bone loss in a 65-year-old man?

A. Increased renal calcium loss.


B. Reduced 1,25 (OH)2 vitamin D formation.
C. Reduced testosterone production.
D. Direct suppression of osteoblast function.
E. Reduced adrenal androgen production.
44

Question 14
A 60-year-old man with a five-year history of type 2 (non-insulin-dependent) diabetes mellitus
presents for a routine checkup. The most appropriate screening test for the presence of diabetic
nephropathy is:

A. Serum creatinine level.


B. Spot urine albumin concentration.
C. 24 hour urine protein excretion.
D. Spot urine albumin/creatinine ratio.
E. 12 hour urinary albumin excretion rate.

Question 15
The deterioration in glycaemic control over time seen in patients with type 2 (non-insulin-dependent)
diabetes mellitus is primarily due to:

A. Medication non-compliance.
B. Increasing insulin resistance.
C. Increasing obesity.
D. Decreasing insulin secretion.
E. Reduced energy expenditure.

Question 16
Carbimazole acts predominantly by inhibiting which of the following mechanisms?

A. Sodium/iodide (Na/I) symporter.


B. Thyroid peroxidase activity.
C. Binding of thyroxine to thyroid binding globulin.
D. Thyroglobulin synthesis.
E. 5’-deiodinase activity.

Question 17
Which of the following parameters is most likely to decrease in the first trimester of pregnancy?

A. Total T4.
B. Free T4.
C. Total T3.
D. Free T3.
E. Thyroid stimulating hormone.
45

Question 18
A 42-year-old man with schizo-affective disorder and long-standing use of lithium presents with
polyuria. Which of the following parameters is most suggestive of psychogenic polydipsia as the
cause of his polyuria?

A. Serum osmolarity 287 mOsm/kg [285-295].


B. Serum creatinine 80 umol/l [< 120].
C. Serum sodium 130 mmol/l [135 – 145].
D. Urine osmolality 100mOsm/kg [700-1400].
E. Lack of response to Desamino-D-arginine-8 vasopressin (DDAVP).

Question 19
Which of the following drugs used in diabetes treatment is most likely to cause hypoglycemia?

A. Repaglinide.
B. Metformin.
C. Acarbose.
D. Rosiglitazone.
E. Orlistat.

Question 20
A 35-year-old man presents with several weeks of generalised unwellness, epigastric pain, vomiting
and headaches. He has no past medical history of note. He has a smoking history of 15 pack years
and uses daily cannabis. He has been holidaying at the beach for the summer and is tanned. He is
mildly dehydrated. The rest of the physical examination is unremarkable.

Blood tests show:


Sodium 139 mmol/L [138-145]
Potassium 5.4 mmol/L [3.5-5.2]
corrected calcium 3.10 mmol/L [2.15-2.57]
phosphate .50 mmol/L [0.87-1.45]
urea 10.4 mmol/L [3.5-8.0]
creatinine 0.32 mmol/L [0.05-0.12]
parathyroid hormone 0.6 pmol/L [1.6-6.2]

What mechanism best explains the hypercalcaemia?

A. Secretion of parathyroid hormone-related peptide (PTHrP).


B. Increased bone resorption.
C. Increased 1,25(OH)2-vitamin D synthesis.
D. Reduced renal clearance of calcium.
E. Mineralocorticoid deficiency.
46

Question 21
Which of the following is the most appropriate tumour marker for follow-up of a person with
medullary thyroid carcinoma?

A. Thyroglobulin.
B. Carcinoembryonic antigen.
C. Calcitonin.
D. Thyroid binding globulin.
E. Chromogranin.

Question 22
The coupling of osteoblast and osteoclast function is mainly related to the action of which of the
following factors?

A. Tumour Necrosis Factor (TNF).


B. Parathyroid hormone (PTH).
C. Receptor activator of NFKB (RANK) ligand.
D. 1,25 (OH)2 cholecalciferol.
E. Oestradiol.

Question 23
Which of the following counter-regulatory factors is released earliest in response to hypoglycaemia?

A. Adrenaline.
B. Cortisol.
C. Noradrenaline.
D. Growth hormone.
E. Glucagon.

Question 24
Which of the following has the greatest effect on peak bone mass?

A. Pubertal age.
B. Childhood calcium intake.
C. Genetic factors.
D. Childhood physical activity.
E. Body weight.
47

Question 25
A 74-year-old female presents with severe jaw pain following tooth extraction. The jaw is swollen
and disfigured. The X-ray shows extensive changes around the site of extraction consistent with
osteonecrosis. The medication most likely to cause this condition is:

A. Ergocalciferol.
B. Zolendronate.
C. Calcitriol.
D. Fluoride.
E. Strontium.

Question 26
A 16-year-old male presents with short stature. Which of the following findings is most suggestive
of growth hormone deficiency?

A. Reduced growth velocity.


B. Decreased bone age.
C: Height below the 3rd centile.
D. Delayed puberty.
E. Increased body weight.

Question 27
In a normal person, which of the following is most likely to cause a decrease in circulating growth
hormone concentration?

A. Slow wave sleep.


B. Hyperglycaemia.
C. Protein meal.
D. Exercise.
E. Sepsis.

Question 28
A 68-year-old diabetic has chest pain and T wave inversion in the precordial chest leads. In this
patient, elevation of which biomarker is most predictive of an adverse outcome?

A. B natriuretic peptide.
B. Myeloperoxidase.
C. Blood glucose.
D. High-sensitity C-reactive protein.
E. Troponin.
48

Question 29
Which of the following electrolyte disturbances is most suggestive of refeeding syndrome?

A. Hypokalaemia.
B. Hyponatraemia.
C. Hypocalcaemia.
D. Hypophosphataemia.
E. Hypomagnasaemia.

Question 30
In a normal person, which of the following is most likely to cause a decrease in circulating growth
hormone concentration?

A. Slow wave sleep.


B. Hyperglycaemia.
C. Protein meal.
D. Exercise.
E. Sepsis.

Question 31
Glucagon-like peptide–1, secreted postprandially by the L-cells in the intestine, increases which of
the following?

A. Gastric emptying.
B. Body weight.
C. Insulin secretion.
D. Hepatic glucose output.
E. Insulin sensitivity.

Question 32
Which of the following hormones is likely to be at its highest level during the luteal phase compared
with any other time of the menstrual cycle?

A. Luteinising hormone (LH).


B. Follicle stimulating hormone (FSH).
C. Progesterone.
D. Oestradiol.
E. Testosterone.
49

Question 33
In the average person, which of the following accounts for the greatest amount of daily energy
expenditure?

A. Fat mass metabolism.


B. Basal metabolic processes.
C. Spontaneous physical activity (fidgeting).
D. Physical exercise.
E. Thermic effect of food.

Question 34
When interpreting the results of bone mineral density scans, which of the following parameters
correlates best with the risk of future fractures?

A. Absolute bone density.


B. T-score.
C. Z-score.
D. Soft tissue density.
E. Change in bone density.
50

Answers

1. B
2. E
3. A
4. B
5. D
6. A
7. A
8. C
9. A
10. A
11. D
12. C
13. D
14. D
15. D
16. B
17. E
18. C
19. A
20. C
21. C
22. C
23. E
24. C
25. B
26. A
27. B
28. E
29. D
30. B
31. C
32. C
33. B
34. B
51

Clinical Applications

Endocrinology

Question 1
A 50-year-old man presenting for an insurance medical examination is found to be hypertensive with
a resting blood pressure of 170/110 mmHg. He takes no medication. The following laboratory
investigations were obtained while on a normal diet:

serum potassium 2.8 mmol/L [3.5-5.0]


serum sodium 145 mmol/L [135-145]
serum creatinine 0.095 mmol/L [0.080-0.120]
urinary potassium 65 mmol/L
urinary sodium 80 mmol/L
plasma renin 100 mU/L [8-70]
plasma aldosterone 800 pmol/L [ambulant 350-700]

Which one of the following diagnoses best fits the clinical picture?

A. Conn's syndrome.
B. Renovascular hypertension.
C. Essential hypertension.
D. Cushing's syndrome.
E. Licorice-induced hypertension.
52

Question 2
A 68-year-old man presents with a history of three months of lethargy and two weeks of declining
vision. He appears pale but is not clinically anaemic. There are clinical signs of hypothyroidism and
hypogonadism. Visual acuity is 6/12 in both eyes and there is a bitemporal hemianopia.

Investigations show:

thyroid-stimulating hormone (TSH) 1.2 mU/L [0.3-4.3]


free thyroxine (free T4) 3 pmol/L [10-19]
serum cortisol 60 nmol/L [150-600]
plasma adrenocorticotrophic hormone (ACTH) 3 pmol/L [2-12]
plasma testosterone 1.2 nmol/L [9.0-30.0]
serum luteinising hormone (LH) 2.1 U/L [2.0-10.0]
serum prolactin 210,000 mU/L [<500]

A magnetic resonance imaging (MRI) scan of the pituitary region is shown below.

After commencing treatment with cortisone acetate and thyroxine, which one of the following is the
most appropriate next step in management?

A. Reassess visual fields in two weeks.


B. Bromocriptine therapy.
C. Dexamethasone therapy.
D. Transfrontal hypophysectomy.
E. Trans-sphenoidal hypophysectomy.
53

Question 3
A 63-year-old man presents with streaky haemoptysis, fatigue and weight loss of 6 kg. He has had no
previous serious illness but admits to having been a heavy smoker for over 38 years. Physical
examination reveals normal mental status and no evidence of organ failure. He is clinically
euvolaemic and is on no medication.

Investigations are as follows.

Serum:
sodium 113 mmol/L [136-144]
potassium 3.8 mmol/L [3.4-4.8]
chloride 83 mmol/L [103-110]
bicarbonate 23 mmol/L [24-30]
creatinine 0.06 mmol/L [0.06-0.10]

Urinary:
specific gravity 1.032 [1.026-1.044]
sodium 60 mmol/L

The most appropriate initial therapy is:

A. Intravenous hypertonic saline.


B. Oral demeclocycline.
C. Oral frusemide.
D. Water restriction.
E. Intravenous isotonic saline.

Question 4
A 45-year-old man is referred with hypercalcaemia which is known to have been present for at least
six months, during which time it has been stable. He is asymptomatic and has no past medical
history.

Recent investigations are as follows:


serum calcium 2.7 mmol/L [2.2-2.5]
serum albumin 41 g/L [38-44]
serum intact parathyroid hormone 4.5 pmol/L [1.0-5.0]
24-hour urine calcium excretion 2.1 mmol/L [3.0-5.0]
24-hour urine creatinine excretion 16 mmol/L [8-18]

Which one of the following investigations is most likely to be diagnostically useful?

A. Measurement of serum 1, 25(OH)2 vitamin D.


B. Measurement of serum parathyroid hormone related peptide (PTHrP).
C. Bone scintigraphy with radiographs of areas of increased uptake.
D. Serum calcium measurements in first degree relatives.
E. Ultrasound of the neck.
54

Question 5
An 82-year-old woman presents with a swelling in the neck which has been present for two years
and has recently increased in size. There are no other symptoms.
The patient is clinically euthyroid and has a regular pulse of 80/minute. A firm, non-tender 4 cm
nodule is palpable in the upper portion of the right lobe of the thyroid gland. The left lobe is slightly
enlarged. There is no lymphadenopathy. The heart, chest and abdomen are normal.

Serum free thyroxine (free T4) is 23 pmol/L [10-19] and thyroid-stimulating hormone
(TSH) is 0.02 mU/L [0.30-4.30].

The technetium-labelled thyroid isotope scan is shown below.

Which one of the following is the most likely diagnosis?


A. Painless thyroiditis.
B. Thyroid cyst.
C. Hyperfunctioning (‘toxic’) adenoma.
D. Toxic multinodular goitre.
E. Thyroid carcinoma.
55

Question 6
A 63-year-old woman presents with a three-week history of malaise, fatigue, tremor, palpitations,
insomnia and weight loss of 2 kg. There has been no exposure to iodine. She is thin and tremulous
with a small non-tender goitre. Her eyes are normal. Her pulse is regular at 80/minute and blood
pressure is 150/85 mmHg. Examination of her heart, chest and abdomen reveals no abnormality. She
has proximal muscle weakness with normal reflexes and no other neurological abnormalities.

Investigations show:
haemoglobin 131 g/L [09/L [150-400]
white cell count 3.5 x 109/L [4.0-10.0]
erythrocyte sedimentation rate (ESR) 93 mm/h [4-15]
free thyroxine (free T4) 31 pmol/L [9-19]
free triiodothyronine (free T3) 5.9 pmol/L [3.5-6.5]
thyroid-stimulating hormone (TSH) <0.03 mU/L [0.30-4.30]

Antithyroid peroxidase and TSH receptor antibodies are not detected.

The isotope thyroid scan is shown below.

Which one of the following is the most appropriate initial therapy?

A. Propranolol.
B. Carbimazole.
C. Radio-iodine.
D. Perchlorate.
E. Prednis(ol)one.
56

Question 7
A 63-year-old man presents with recent-onset of exertional chest pain during moderate activities.
Past medical history includes hypertension, gout and osteoarthritis. Current medications include
enalapril and ibuprofen. He admits that he does not follow dietary recommendations for his gout and
hypertension.

Examination findings include:


body mass index 30 kg/m2 [20-25]
waist-to-hip ratio (WHR) 1.01 [<0.90]

Fasting blood analysis reveals:


triglycerides 2.0 mmol/L [0.0-2.5]
total cholesterol 6.0 mmol/L [0.0-5.2]
high density lipoprotein (HDL) 0.7 mmol/L [1.0-2.5]
low density lipoprotein (LDL) 4.0 mmol/L [0.0-3.5]
glucose 6.5 mmol/L [3.6-6.6]
insulin 85 pmol/L [15-60]

The most appropriate management plan related to his lifestyle at this point would be:

A. Low cholesterol diet and low intensity aerobic exercise regimen.


B. Hypocaloric diet with a weight loss goal of 2.0 kg/week.
C. Diet free of sucrose and cholesterol.
D. Hypocaloric diet with a weight loss goal of 0.5 kg/week.
E. Low cholesterol, hypocaloric diet with resistance training exercises.
57

Question 8
A 53-year-old woman has had severe interscapular pain since carrying two buckets of sand three
weeks ago. There is a lower thoracic kyphosis with pain, tenderness and restricted movement. An X-
ray of the thoracic spine is shown below.

There is 1+ proteinuria. Full blood examination, serum electrolytes and creatinine levels as well as
liver function tests are normal. Serum calcium is 2.37 mmol/L [2.25-2.65]. Serum protein
electrophoresis shows reduced gamma globulins but no paraprotein.

Mean lumbar spine bone density is 4.0 standard deviations (SD) below the normal age-related mean,
and 4.8 SD below the young adult mean. Femoral neck values are 1.8 and 2.2 SD below the age-
related and young adult means respectively.

Which one of the following is the most appropriate next investigation?

A. Isotope bone scan.


B. Bone turnover markers.
C. Urine protein electrophoresis.
D. Vertebral biopsy.
E. Serum follicle-stimulating hormone (FSH).

Question 9
A 35-year-old man with sarcoidosis is receiving treatment with prednis(ol)one. He is hypertensive,
and bone densitometry shows him to be osteoporotic. He had a hernia operation three years ago and a
renal calculus one-year later.

The most appropriate treatment for his osteoporosis is:

A. Alendronate.
B. Calcium.
C. Calcitriol (1,25 (OH)2 vitamin D).
D. Sodium fluoride.
E. Ergocalciferol.
58

Question 10
A 42-year-old man with acromegaly undergoes transphenoidal pituitary surgery for a growth
hormone-secreting macroadenoma. Six months post-operatively he has an elevated insulin-like
growth factor type I (IGF-I) concentration. A repeat magnetic resonance imaging (MRI) scan reveals
a residual and inoperable tumour confined to the right cavernous sinus only.

Which of the following is the most appropriate management?

A. Expectant management.
B. Radiotherapy.
C. Bromocriptine.
D. Octreotide.
E. Cabergoline.

Question 11
A 65-year-old man presents with sudden onset of severe headache. He appears generally unwell and
has a blood pressure of 90/60 mmHg. There are no focal neurological signs and visual field testing is
normal. An urgent non-contrast T1 weighted magnetic resonance imaging (MRI) scan is shown
below (coronal view).

Which of the following is the most appropriate first step in management?

A. Glucocorticoid administration.
B. Emergency transphenoidal surgery.
C. Pituitary radiotherapy.
D. Dopamine agonist therapy.
E. Somatostatin analogue therapy.
59

Question 12
An 18-year-old woman with type 1 (insulin-dependent) diabetes mellitus is at most risk of
developing which of the following conditions?

A. Addison’s disease.
B. Pernicious anaemia.
C. Hashimoto’s thyroiditis.
D. Primary ovarian failure.
E. Systemic lupus erythematosus (SLE).

Question 13
An 84-year-old female nursing home resident is ambulant but rarely goes outside. She is thin, eats
little and has reflux oesophagitis. Medical history includes mastectomy and radiation therapy for
breast cancer at age 60 years. There is a past history of deep venous thrombosis (DVT). Dual-energy
X-ray absorptiometry (DEXA) scan reveals a t score of -2.2 at the spine and -3.0 at the femoral neck.
Serum calcium is normal.

In addition to calcium supplementation, which of the following is the most appropriate initial therapy
for her osteoporosis?

A. Alendronate.
B. Calcitonin.
C. Vitamin D.
D. Oestrogen.
E. Raloxifene.
60

Question 14
A 35-year-old man presents with bilateral gynaecomastia. He has a past history of viral orchitis. He
has hypertension treated with spironolactone and admits to binge drinking of alcohol. Examination
reveals normal body hair distribution and 20 mL testicles bilaterally. He has moderate bilateral
gynaecomastia.

Investigations show:

serum:
testosterone 12 nmol/L [10-25]
luteinising hormone <1 IU/L [1-9]
oestradiol 250 pmol/L [40-110]
prolactin 350 mIU/L [<400]
dehydroepiandrosterone sulphate (DHEAS) 10 µmol/L [2-12]

Which of the following is the most important next step in management of his gynaecomastia?

A. Cease alcohol intake.


B. Magnetic resonance imaging (MRI) scan of pituitary gland.
C. Testicular ultrasound.
D. Karyotype analysis.
E. Cease treatment with spironolactone.

Question 15
A 45-year-old asymptomatic man returns for follow-up. He was diagnosed 10 years ago with aortic
regurgitation due to a congenital bicuspid aortic valve. He has never had endocarditis.

Which one of the following echocardiographic profiles most strongly indicates the need for aortic
valve replacement?

LVEDD(mm) FS LA size (mm)


(35-55) (0.30-0.40) (<40)
A. 70 0.30 50
B. 75 0.40 40
C. 70 0.25 45
D. 65 0.45 50
E. 75 0.35 55

Key:
LVEED Left ventricular end-diastolic diameter
LVESD Lef ventricular end-systolic diameter
FS Frational shortening – (LVEDD – LVESD)/LVEDO
LA Left atrial
61

Question 16
A 30-year-old woman presents with a 4 cm solitary thyroid nodule. Which of the following is the
most appropriate initial investigation?

A. Serum thyroid-stimulating hormone (TSH).


B. Fine needle aspiration biopsy.
C. Thyroid ultrasound.
D. Technetium scan.
E. Serum calcitonin.

Question 17
A 32-year-old woman is found to have high blood pressure (180/105 mmHg) at an insurance medical
examination. She is asymptomatic. Clinical examination is normal. Similar blood pressure readings
are recorded on two follow-up examinations.

Serum biochemistry shows:

sodium 146 mmol/L [134-146]


potassium 2.5 mmol/L [3.4-5.0]
creatinine 0.08 mmol/L [0.06-0.12]

Which of the following investigations is the most appropriate next step?

A. 24-hour urinary electrolytes.


B. Upright plasma aldosterone to renin ratio.
C. Captopril renogram.
D. Adrenal computed tomography (CT) scanning.
E. 24-hour urinary aldosterone.

Question 18
Which of the following diabetic neurological complications is most likely to improve with time?

A. Diabetic amyotrophy.
B. Orthostatic hypotension.
C. Gustatory sweating.
D. Loss of deep tendon reflexes.
E. Painful peripheral neuropathy.
62

Question 19
A 40-year-old man presents with weight loss, heat intolerance and tremor. On examination he has a
small non-tender diffuse goiter and is clinically thyrotoxic. The following results are obtained:
Free T4 36 pmol/L [10-23]
Free T3 12.5 pmol/L [2.8-6.8]
thyroid-stimulating hormone (TSH) <0.01 mIU/L [0.3-5.0]
anti-microsomal antibodies 1:1600 [<1:400]

A technetium scan is as shown below.

Arrow points to sternal notch marker

The most likely diagnosis is:

A. Graves’ disease.
B. Toxic multinodular goiter.
C. Subacute thyroiditis.
D. Silent thyroiditis.
E. Hashimoto’s thyroiditis.
63

Question 20
One week after a 47-year-old male has transphenoidal pituitary surgery for a histologically
confirmed non-functioning pituitary tumour, the following endocrine results are obtained:

thyroid-stimulating hormone (TSH) <0.05 mIU/L [0.30-5.00]


free thyroxine (FT4) 16 pmol/L [10-23]

Synacthen Test:
0800 plasma cortisol 90 nmol/L [100-700]
0900 plasma cortisol 680 nmol/L [>550]
luteinising hormone (LH) <1 IU/L [2-8]
follicle-stimulating hormone (FSH) <1 IU/L [2-8]
testosterone 8 nmol/L [10-30]
growth hormone (GH) <0.03 ng/ml [ <10]

What is the next most appropriate management step at this point in time?

A. Observe and repeat testing in six months.


B. Commence thyroxine.
C. Commence hydrocortisone.
D. Commence testosterone.
E. Commence growth hormone.

Question 21
Which one of the following medications is most likely to result in an increase in circulating insulin
concentrations?

A. Metformin.
B. Acarbose.
C. Rosiglitazone.
D. Repaglinide.
E. Orlistat (Xenical).

Question 22
A 35-year-old woman with amenorrhoea is diagnosed with a microprolactinoma and is treated with
bromocriptine 2.5 mg/day. Her prolactin level normalises, and her periods become regular. A year
later she presents six weeks pregnant. What would be the most appropriate next step in management?

A. Stop bromocriptine and observe.


B. Continue bromocriptine and observe.
C. Increase bromocriptine dosage.
D. Change to cabergoline.
E. Advise termination of pregnancy.
64

Question 23
A 25-year-old woman with no significant past medical history presents with weight gain, depression,
easy bruising and proximal muscle weakness. On examination she has a plethoric complexion,
central obesity with striae and a proximal myopathy. Investigations reveal low plasma concentrations
of cortisol and adrenocorticotrophic hormone (ACTH). Which of the following is the most likely
diagnosis?

A. Cushing’s disease.
B. Pseudo-Cushing’s syndrome.
C. Factitious Cushing’s syndrome.
D. Ectopic ACTH syndrome.
E. Cortisol-producing adrenal adenoma.

Question 24
A 54-year-old Indian woman complains of a constant pain in her hips which is worse on standing
and walking. She has mild weakness of hip flexion and walks with difficulty. She is a vegetarian and
avoids dairy products. She immigrated to New Zealand three years ago. A pelvic X-ray is normal.

The following blood tests are obtained:


alkaline phosphatase (ALP) 457 IU/L [30-115]
corrected calcium 2.13 mmol/L [2.15-2.57]
phosphate 0.79 mmol/L [0.90-1.55]
parathyroid hormone (PTH) 16.0 pmol/L [1.3-7.6]

The most appropriate next investigation is:

A. Dual-energy X-ray absorptiometry (DEXA) scan.


B. Bone biopsy.
C. Isotope bone scan.
D. Vitamin D levels.
E. 24-hour urinary calcium.
65

Question 25
A 45-year-old woman is admitted after being found unconscious. Her plasma glucose is 1.5
mmol/L.

The following results are obtained:


Plasma insulin 105 pmol/L [<80 fasting]
Plasma C-peptide 650 pmol/L [250-700]
CT (computed tomography) pancreas Normal

Which of the following best explains these findings?

A. Addison’s disease.
B. Insulin administration.
C. Alcohol ingestion.
D. Insulinoma.
E. Growth hormone deficiency.

Question 26
A patient with known cancer presents with increasing thirst. Initial biochemistry reveals normal
glucose, plasma sodium of 147 mmol/L [135-145], plasma osmolality of 300 [280-296], urinary
sodium of 31 and urinary osmolality of 77. These findings are most consistent with:

A. Psychogenic polydipsia.
B. Extrarenal fluid loss.
C. Primary hyperaldosteronism.
D. Diabetes insipidus.
E. SIADH (syndrome of inappropriate anti-diuretic hormone).

Question 27
A patient has the following abnormal findings:

sodium 130 mmol/L [134-145]


potassium 2.9 mmol/L [3.5-5.0]
phosphate 0.22 mmol/L [0.6-1.3]
magnesium 0.6 mmol/L [0.72-0.92]
creatinine 120 µmol/L [70-110]
ionised calcium 1.1 mmol/L [1.14 - 1.27]
glucose 5.1 mmol/L [4.1- 6.1]

Which of the following is the most likely cause?

A. Tumour lysis syndrome.


B. Diabetic ketoacidosis.
C. Hyperparathyroidism.
D. Refeeding syndrome.
E. Vitamin D deficiency.
66

Question 28
In a 50-year-old man undergoing a computed tomography (CT) scan for investigation of abdominal
pain, which of the following is the most likely cause of an incidentally discovered 3cm adrenal mass
lesion?

A. Adrenal metastasis.
B. Cortisol secreting adrenal adenoma.
C. Phaeochromocytoma.
D. Non-functioning adrenal adenoma.
E. Adrenal carcinoma.

Question 29
A 25-Year-old woman with no significant past medical history presents with weight gain,
depression, easy bruising and proximal muscle weakness. On examination she has a plethoric
complexion, central obesity with striae and a proximal myopathy. Investigations reveal low plasma
concentrations of cortisol and adrenocorticotrophic hormone (ACTH). Which of the following is
the most likely diagnosis?

A. Cushing’s disease.
B. Pseudo-Cushing’s syndrome.
C. Factitious Cushing’s syndrome.
D. Ectopic ACTH syndrome.
E. Cortisol-producing adrenal adenoma.

Question 30
A 27-year-old woman presents with weight loss, tremor, palpitations and pain and tenderness over
her thyroid. The following results are obtained:

Free T4 75 pmol/L [10-24]


Free T3 25 pmol/L [2.5-6.5]
thyroid-stimulating hormone (TSH) <0.03 mIU/L [0.4-4.0]
erythrocyte sedimentation rate (ESR) 105 mm/hour [<10]

A thyroid scintiscan is awaited. In the meantime, in addition to beta blockers, which of the following
is the most appropriate first line treatment for her condition?

A. Prednisone.
B. Carbimazole.
C. Radioiodine.
D. Propylthiouracil.
E. Diclofenac.
67

Question 31
In a woman with Graves disease, which of the following is associated with an increased relative risk
of developing Graves ophthalmopathy?

A. Pregnancy.
B. Young age.
C. High titre microsomal antibodies.
D. Smoking.
E. Previous thyroid surgery.

Question 32
An 18-year-old woman presents with hirsutism. The following results are obtained:

Testosterone 4.5 nmol/L [<2.5]


Luteinising hormone(LH) 9 IU/L [2-15]
Follicle stimulating hormone (FSH) 4 IU/L [2-15]
Oestradiol 500 pmol/L [130-1500]
Dehydroepiandrosterone (DHEA) 10 umol/L [0.5-12]
17OH Progesterone 62 pmol/L [<6]
24 hour urinary free cortisol 280 nmol/24hours [100-300]
Adrenocorticotrophic hormone (ACTH) 3 pmol/L [1-5]

Which of the following is the most likely explanation for her hirsutism?

A. Polycystic ovarian syndrome.


B. Androgen secreting ovarian tumour.
C. Cushing’s syndrome.
D. Late onset congenital adrenal hyperplasia.
E. Adrenocortical carcinoma.

Question 33
In patients with type 1 diabetes mellitus the greatest benefit in using a long-acting insulin analogue is
in reducing which of the following?

A: Diabetic retinopathy.
B: Nocturnal hypoglycaemia.
C: Insulin hypersensitivity.
D: Preprandial hyperglycaemia.
E Microalbuminuria.
68

Question 34
A patient with long standing type 1 diabetes presents has the appearances on fundoscopy shown
below.

Which of the following therapeutic strategies is most likely to preserve his long-term vision?

A. Tight blood pressure control.


B. Aspirin.
C. Tight blood glucose control.
D. Vitrectomy.
E. Pan-retinal photocoagulation.

Question 35
Which of the following is the most common long-term side effect of radiation treatment of pituitary
tumours?

A. Stroke.
B. Cranial nerve palsy.
C. Optic apparatus damage.
D. Second intracranial tumour.
E. Hypopituitarism.
69

Question 36
A 58-year-old woman has been unwell for several weeks with back pains. She has no past medical
history of note. She is a smoker of 25 pack years. On examination she is tender over the mid thoracic
spine and over several ribs. The rest of the examination is unremarkable.

Blood tests show:


Hb 105 g/l [115-170]
Erythrocyte sedimentation rate 80 mm/hr [< 25]
Corrected Calcium 3.5 mmol/l [2.15-2.57]
Creatinine 0.11 mmol/l [0.4-0.10]
Alkaline Phosphatase 125 IU/l [40-130]
Parathormone 1.1 pmol/l [1.6-6.2]

A CXR shows lytic lesions in several thoracic vertebrae with clear lung fields. Spinal X-rays confirm
several lytic vertebral lesions. Immunoglobulins are normal. Serum and urine electrophoresis and
immunoelectrophoresis are negative. Mammography is normal. CT scanning of chest and abdomen
shows the vertebral lesions but no other masses, no organomegaly and no lymphadenopathy. Thyroid
and pelvic ultrasound are normal. A bone scan shows no increased uptake.

Which of the following tests should next be performed?

A. 24 hour urine calcium.


B. Bone marrow examination.
C. Biopsy of a bone lesion.
D. Vitamin D levels.
E. Parathyroid hormone related peptide levels

Question 37
A 56-year-old man is referred for assessment of resistant hypertension. Blood pressure readings at
his general practitioner’s surgery have been between 145 and 172 mmHg systolic and 86 and 98
mmHgdiastolic despite various combinations of up to two anti-hypertensive agents at a time. He has
no other vascular risk factors. He has no family history of hypertension. Examination does not reveal
anyclinical findings suggesting a secondary cause of hypertension. He has normal creatinine and
electrolytes. An ECG and trans thoracic echocardiogram are normal

What is the most appropriate next investigation?

A. Aldosterone : renin ratio.


B. Magnetic resonance angiography of renal arteries.
C. Ambulatory blood pressure recording.
D. Renal ultrasound.
E. Urine testing for drug compliance.
70

Question 38
A 65-year-old man with a history of paroxysmal atrial tachycardia who has been treated with
amiodarone for five years presents with worsening palpitations. Thyroid function tests show the
following results:

FT4 45pmol/l [10-24]


FT3 10pmol/l [ 2.5-6]
TSH <0.03mIU/l [ 0.4-4]

He is commenced on Carbimazole 10mg tds but fails to improve and three weeks later his thyroid
function tests are as follows:

FT4 100pmol/l
FT3 14pmol/l
TSH <0.03mIU/l

Which of the following is the most likely cause of this patient’s thyrotoxicosis?

A. Graves Disease.
B. Type 1 (iodine-induced) amiodarone induced thyrotoxicosis.
C. Type 2 (inflammatory) amiodarone induced thyrotoxicosis.
D. Subacute viral thyroiditis.
E. Toxic multinodular goitre.

Question 39
In the Western population, body mass index (BMI) has significantly increased over the past 40 years
in all age groups. Most of this is accounted for by:

A. Increased caloric intake.


B. Increased working hours.
C. Increased maternal participation in the workforce.
D. Increased use of passive entertainment.
E. Decreased exercise.
71

Question 40
A 49-year-old woman with a 25 year history of type 1 diabetes mellitus presents with progressive
midfoot pain and swelling over six months. An X-ray of the affected foot is shown below.

The most likely diagnosis is:

A. Osteoarthrititis.
B. Neuropathic arthropathy.
C. Osteomyelitis.
D. Gout.
E. Rheumatoid arthritis

Question 41
In a woman with Graves disease, which of the following is associated with an increased relative risk
of developing Graves ophthalmopathy?

A. Pregnancy.
B. Young age.
C. High titre microsomal antibodies.
D. Smoking.
E. Previous thyroid surgery.
72

Question 42
A 58-year-old smoker presents with a new diagnosis of hypertension. His BP has been difficult to
control with levels of 190/110 despite amlodipine 10 mg/day and prazosin 5 mg bd. On examination
he has a loud abdominal bruit. His CT scan reveals a 1.5 cm left adrenal mass. Laboratory
investigations reveal:

Sodium 138 mmol/l [135-145]


Potassium 4.8 mmol/l [3.5 – 5.5]
Bicarbonate 23 mmol/l [22 – 28]
Creat 105 umol/l [< 120]

The most likely cause of his hypertension is:

A. Conns syndrome.
B. Phaeochromocytoma.
C. Essential hypertension.
D. Renal artery stenosis.
E. Cushings syndrome.

Question 43
A patient with known lung cancer presents with nausea and lethargy. His serum sodium is 119
mmol/L. Which of the following sets of results is most consistent with a diagnosis of the syndrome
of inappropriate antidiuretic hormone (ADH) secretion?

Serum Urine osmolality Urine sodium


osmolality (mosmol/kg) (mmol/L)
(mosmol/kg)
N [275-290]
A 275 50 60
B 275 80 10
C 275 180 6
D 298 50 10
E 298 180 60
73

Question 44
A 31-year-old woman presents with an absence of menstrual periods since stopping the oral
contraceptive pill five months ago. She has the following investigations:

Weight 80.4 kg
Height 1.70 m
Body Mass Index 27.8 kg/m2, P
rolactin 2566 mIU/L [60 – 550 mIU/L]
Luteinising hormone (LH) <1.0 IU/L [3.0 – 12.0 IU/L]
Follicle-stimulating hormone (FSH) <1.0 IU/L [2.0 – 10.0 IU/L]
Oestrogen 35 821 pmol/L [100 – 2400 pmol/L]
Testosterone 3.1 nmol/L [0.5 – 2.5 nmol/L]

Which of the following is the most likely explanation?

A. Prolactinoma.
B. Polycystic ovarian syndrome (PCOS).
C. Pregnancy.
D. Congenital adrenal hyperplasia.
E. Hydatidiform mole.

Question 45
Which one of the following most frequently causes hypoglycaemia in patients with diabetes?

A. Pioglitazone.
B. Glibenclamide.
C. Metformin.
D. Acarbose.
E. Sitagliptin.
74

Question 46
A 52-year-old man with type 2 diabetes has been non-specifically unwell for a week. He underwent
resection of a craniopharyngioma nine months ago. He is maintained on replacement therapy with
desmopressin nasal spray, testosterone decanoate injections, oral hydrocortisone and thyroxine. He
presents to hospital with a reduced level of consciousness. Systolic blood pressure is 70 mmHg and
axillary temperature is 35oC. Investigations reveal the following results:

Serum sodium 136 mmol/L [136 – 148 mmol/L]


Serum potassium 5.2 mmol/L [3.6 - 5.5 mmol/L]
Serum bicarbonate 30 mmol/L [22 – 32 mmol/L]
Serum urea 8 mmol/L [3 - 8 mmol/L]
Serum creatinine 120 µmol/L [30 – 120 µmol/L]
Random glucose 18 mmol/L [3.5 - 7.8 mmol/L]
Free L-thyroxine 9.8 pmol/L [10.0 – 26.0 pmol/L]

What is the most likely cause of his acute presentation?

A. Raised intracranial pressure due to tumour recurrence.


B. Myxoedema coma.
C. Glucocorticoid deficiency.
D. Diabetes insipidus.
E. Hyperosmolar non-ketotic coma.

Question 47
A 60-Year-Old Male Presents With Central Weight Gain, Hypertension, And Diabetes. He has a
history of asthma treated with inhaled corticosteroids. He drinks two standard drinks of alcohol/day.
On examination he has central obesity, thin skin, and proximal muscle weakness.

The following test results are obtained:


24 hour urinary free cortisol 1400 nmol/day [<400 nmol/day]
0900 plasma adrenocorticotrophic hormone (ACTH) 15 pmol/L [3 – 20 pmol/L]
Magnetic resonance imaging (MRI) pituitary normal

Which of the following is the most likely cause of this man’s presentation?
A. Exogenous glucocorticoid administration.
B. Pseudo-Cushing’s syndrome.
C. Cushing’s disease.
D. Ectopic Cushing’s syndrome.
E. Cortisol producing adrenal adenoma.
75

Question 48
The presence of which of the following is the best reason not to commence alendronate for a woman
with postmenopausal osteoporosis?

A. Oesophageal stricture.
B. Dental caries.
C. Hyperparathyroidism.
D. Serum creatinine 140 µmol/L.
E. Recent fracture.

Question 49
A 35-year-old woman with type 1 diabetes since age 14 years is reviewed. She reports three episodes
of hypoglycaemia with levels <3.0 mmol/L during the night in the last month. There have also been a
number of hypoglycaemic episodes in the late morning.

Recent results show:


Weight 55 kg,
Fasting plasma glucose 12.3 mmol/L [4.0 – 6.0 mmol/L],
Hb A1c 6.9% [<6%]
Urine microalbumin:creatinine ratio 0.1 [<3.5]

Current insulin regimen: Neutral Protamine Hagedorn (NPH) insulin 14 units before breakfast and
12 units before bed.

Insulin aspart 4 - 6 units before breakfast and lunch and 8 - 10 units, before dinner

Which one of the following modifications to her insulin regimen would be most effective in
optimising her glycaemic control?

A. Omit evening NPH insulin.


B. Change time of evening NPH insulin to before evening meal.
C. Reduction of evening aspart insulin by 10%.
D. Introduction of insulin glargine at a dose of 12 units in place of evening NPH insulin.
E. Substitution of both doses of NPH insulin with one pre-breakfast dose of 20 units of insulin
76

Question 50
A 30-year-old woman presents three months postpartum with weight loss, tremor, palpitations and
heat intolerance. On clinical examination she is tachycardic, has a fine finger tremor, lid retraction,
lid lag but no goitre.

Thyroid function test results are:


Free T4 35 pmol/L [10 – 24 pmol/L]
Free T3 12.0 pmol/L [3.8 - 6.8 pmol/L]
Thyroid-stimulating hormone (TSH) <0.01 mIU/L [0.40 – 4.00 mIU/L]
Anti thyroid peroxidase antibodies Positive

Thyroid scintiscan is shown below.

Which of the following is the most likely cause of this woman’s thyrotoxicosis?

A. Factitious thyrotoxicosis.
B. Postpartum thyroiditis.
C. Graves disease.
D. Subacute thyroiditis.
E. Iodine-induced thyrotoxicosis.
77

Question 51
Which of the following agents is most likely to induce weight loss when used to treat type 2 diabetes
mellitus?

A. Acarbose.
B. Glargine insulin.
C. Glimepiride.
D. Metformin.
E. Pioglitazone.

Question 52
Which of the following secondary causes of hyperlipidaemia is most likely to cause a predominant
increase in LDL-cholesterol?

A. Protease inhibitor therapy.


B. Type 2 diabetes.
C. Obesity.
D. Alcohol excess.
E. Hypothyroidism.

Question 53
A 65-year-old man with a history of paroxysmal atrial tachycardia who has been treated with
amiodarone for five years presents with worsening palpitations. Thyroid function tests show the
following results:
FT4 45 pmol/L [10 – 24 pmol/L]
FT3 10.0 pmol/L [2.5 – 6.0 pmol/L]
Thyroid-stimulating hormone (TSH) <0.03 mIU/L [0.4 – 4.0 mIU/L]

He is commenced on carbimazole 10 mg tds but fails to improve and three weeks later his thyroid
function tests are as follows:
FT4 100 pmol/L
FT3 14 pmol/L
TSH <0.03 mIU/L

Which of the following is the most likely cause of this patient’s thyrotoxicosis?

A. Graves Disease.
B. Type 1 (iodine-induced) amiodarone induced thyrotoxicosis.
C. Type 2 (inflammatory) amiodarone induced thyrotoxicosis.
D. Subacute viral thyroiditis.
E. Toxic multinodular goitre.
78

Question 54
A 43-year-old woman presents with a six month history of hypertension which has been difficult to
control. She is now taking three agents for her blood pressure control. She has a serum potassium of
2.7 mmol/L [3.6 - 5.2 mmol/L] and you are concerned about primary hyperaldosteronism (Conn’s
syndrome).

Which of the following medications is most likely to interfere with the interpretation of an
aldosterone:renin ratio result?

A. Amlodipine.
B. Enalapril.
C. Spironolactone.
D. Metoprolol.
E. Alpha-methyl-dopa.

Question 55
A 68-year-old female develops pulmonary oedema and myocardial infarction during general
anesthesia for parathyroidectomy. Her coronary arteries are normal at coronary angiography. While
ventilated in the Intensive Care Unit (ICU), significant problems occur due to hypertension. Her
abdominal computed tomography (CT) scan is shown below.

Which of the following is the most appropriate treatment for her hypertension?

A. Beta-blockers.
B. Angiotensin II converting enzyme inhibitors.
C. Nitrates.
D. Alpha-blockers.
E. Calcium channel blockers.
79

Answers

1. B 49. E
2. B 50. B
3. D 51. D
4. D 52. E
5. D 53. C
6. A 54. C
7. D 55. D
8. C
9. A
10. D
11. A
12. C
13. C
14. C
15. D
16. A
17. B
18. A
19. D
20. C
21. D
22. A
23. C
24. D
25. D
26. D
27. D
28. D
29. C
30. E
31. D
32. D
33. B
34. E
35. E
36. B
37. C
38. C
39. A
40. B
41. D
42. C
43. C
44. D
45. B
46. C
47. C
48. A
80

Medical Sciences

Gastroenterology

Question 1
A 45-year-old man develops stage C cancer of the caecum. There is a strong family history of bowel
cancer in the absence of polyps and there is no history of colitis.

The most likely underlying inherited genetic abnormality is in:

A. The ras gene.


B. The DNA mismatch repair (MMR) gene.
C. The p-glycoprotein (MDR1) gene.
D. The adenomatous polyposis coli (APC) gene.
E. The deleted in colon cancer (DCC) gene.

Question 2
Which one of the following is associated with the highest risk of NSAID (non-steroidal anti-
inflammatory drug)-induced gastric ulceration?

A. Concomitant use of steroids.


B. Infection with Helicobacter pylori.
C. Higher therapeutic dose of NSAID.
D. Advanced age.
E. Concomitant use of anticoagulants.

Question 3
In treating constipation, which one of the following agents administered over an extended period of
time is most likely to lose its effectiveness?

A. Docusate sodium (Coloxyl).


B. Lactulose.
C. Senna.
D. Psyllium husk powder (Metamucil).
E. Epsom salts.
81

Question 4
Which one of the following is the best predictor of outcome in a patient with colorectal cancer?

A. Tumour stage.
B. Degree of microsatellite instability.
C. Ethnic background.
D. Strong family history.
E. Sporadic occurrence.

Question 5
The manometric feature most characteristic of achalasia is:

A. Lack of peristalsis in the oesophageal body.


B. High resting tone of the lower oesophageal sphincter.
C. Failure of lower oesophageal sphincter relaxation with swallowing.
D. Poor propagation of oesophageal contraction waves.
E. Diffuse spasm in the oesophageal body.

Question 6
The most important application of carcinoembryonic antigen (CEA) measurements in patients with
colon cancer is:

A. Testing of relatives.
B. Detection of resectable relapse.
C. Determining prognosis of stage III disease.
D. Identification of patients who require adjuvant chemotherapy.
E. Monitoring during adjuvant treatment.

Question 7
A 45-year-old man develops stage C cancer of the caecum. There is a strong family history of bowel
cancer in the absence of polyps and there is no history of colitis.

The most likely underlying inherited genetic abnormality is in:

A. The ras gene.


B. The DNA mismatch repair (MMR) gene.
C. The p-glycoprotein (MDRI) gene.
D. The adenomatous polyposis coli (APC) gene.
E. The deleted in colon cancer (DCC) gene.
82

Question 8
Which one of the following is the most efficacious in the prevention of gastro-duodenal ulceration
from non-steroidal anti-inflammatory drugs?

A. Misoprostol.
B. Enprostol.
C. Omeprazole.
D. Sucralfate.
E. Ranitidine.

Question 9
Which one of the following is the most important pathogenic factor in gastro-oesophageal reflux
disease?

A. A sliding hiatus hernia.


B. Smoking more than 20 cigarettes/day.
C. An increased frequency of transient lower oesophageal sphincter relaxation.
D. Impaired oesophageal peristalsis.
E. A low resting lower oesophageal sphincter pressure.

Question 10
In the treatment of hepatitis C (HCV) with interferon •, which one of the following is the most
predictive of a poor treatment response (as defined as failure to normalise alanine transaminase
(ALT) levels)?

A. Viral genotype.
B. Pre-treatment ALT level.
C. Presence of cirrhosis.
D. Age of the patient.
E. Plasma HCV RNA concentration (viral load).

Question 11
The best serological marker of an acute exacerbation of chronic hepatitis B is:

A. Hepatitis B surface antigen.


B. Hepatitis B e antigen.
C. Hepatitis B e antibody.
D. Hepatitis B surface antibody.
E. Hepatitis B core IgM.
83

Question 12
In patients with untreated post-transfusion hepatitis C, the lifetime risk of progression to cirrhosis
best approximates:

A. 1%.
B. 5%.
C. 20%.
D. 50%.
E. 80%.

Question 13
In the acute management of oesophageal variceal bleeding, which of the following is least likely to
control bleeding?

A. Intravenous omeprazole.
B. Intravenous octreotide.
C. Endoscopic band ligation.
D. Endoscopic injection therapy.
E. Intravenous vasopressin.

Question 14
Cirrhosis of the liver in genetic alpha-1-antitrypsin deficiency is most likely to be the result of which
of the following?

A. Loss of inhibition of serum elastase.


B. Concomitant infection with hepatitis C.
C. Intracellular accumulation of alpha-1-antitrypsin.
D. Associated alcohol-induced fibrosis.
E. Hypoxia secondary to emphysema.

Question 15
Which one of the following colonic polyps has the highest potential for malignant transformation?

A. Hyperplastic.
B. Tubular.
C. Villous.
D. Hamartomatous.
E. Peutz-Jegher.
84

Question 16
In a patient with a distal gastrectomy, which of the following deficiencies is most likely?

A. Vitamin B12.
B. Folate.
C. Iron.
D. Calcium.
E. Magnesium.

Question 17
Which one of the following is the predominant stimulus to gastric acid secretion in a healthy
individual?

A. Acetylcholine.
B. Gastrin.
C. Histamine.
D. Calcium.
E. Somatostatin.

Question 18
A 60-year-old male presents with worsening dysphagia for solid and liquid food. Manometry is
shown above (reference normal on left, patient tracing on right).

The most likely diagnosis is:

A. Hiatus hernia.
B. Gastroesophageal reflux.
C. Achalasia.
D. Distal oesophageal malignancy.
E. Presbyoesophagus.
85

Question 19
A 22-year-old male presents with his fourth episode of pancreatitis. A computed tomography (CT)
scan of the pancreas is shown below. He has two first degree relatives with chronic pancreatitis.
There is no other significant family history.

The most likely explanation for his episodes is:

A. Cystic fibrosis.
B. Alcoholic pancreatitis.
C. Hyperparathyroidism.
D. Hereditary pancreatitis.
E. Haemochromatosis.

Question 20
The most common mechanism for malignant ascites is:

A. Cardiac failure.
B. Hypoalbuminaemia.
C. Obstruction of abdominal lymphatics.
D. Portal hypertension.
E. Portal vein thrombosis.

Question 21
Which of the following is most likely to cause predominantly zone 1 (periportal) rather than zone 3
(centrilobular) liver pathology?

A. Alcohol.
B. Paracetamol.
C. Morbid obesity.
D. Budd-Chiari syndrome.
E. Primary biliary cirrhosis.
86

Question 22
Liberation of dietary cobalamin from R-proteins would be most likely to be adversely affected by:

A. Ileal resection.
B. pancreatic insufficiency.
C. Coeliac disease.
D. Crohn’s disease.
E. Helicobacter pylori chronic gastritis.

Question 23
Which of the following is most predictive of variceal haemorrhage in a patient with oesophageal
varices?

A. Portal vein pressure.


B. Child-Pugh score.
C. Variceal size.
D. Prolonged prothrombin time.
E. Thrombocytopenia.

Question 24
The primary factor leading to the accumulation of ascites in patients with chronic liver disease is?

A. Excessive abdominal lymph formation.


B. Splanchnic vasodilatation.
C. Hypoalbuminaemia.
D. Increased aldosterone secretion.
E. Expansion of plasma volume.

Question 25
In the general community, which of the following is the major pathogenic factor for reflux
oesophagitis?

A. Excessive acid secretion.


B. Excessive transient lower oesophageal sphincter relaxations.
C. Presence of a hiatus hernia.
D. Impaired oesophageal motility.
E. Impaired gastric emptying.
87

Question 26
In which of the following conditions is ursodeoxycholic acid most likely to relieve symptoms?

A. Primary sclerosing cholangitis.


B. Massive choledocholithiasis.
C. Primary biliary cirrhosis.
D. Microlithiasis associated pancreatitis.
E. Cholecystolithiasis in a poor surgical candidate.

Question 27
The risk factor with the greatest impact on development of esophageal cancer is:

A. Achalasia.
B. Barrett’s oesophagus.
C. Low fibre diet.
D. Smoking.
E. Reflux.

Question 28
Absorption of which of the following is most likely to be impaired in untreated coeliac disease?

A. Vitamin K.
B. Iron.
C. Vitamin B12.
D. Folate.
E. Calcium.

Question 29
The primary factor leading to the accumulation of ascites in patients with chronic liver disease is?

A. Excessive abdominal lymph formation.


B. Splanchnic vasodilatation.
C. Hypoalbuminaemia.
D. Increased aldosterone secretion.
E. Expansion of plasma volume.
88

Question 30
Which of the following is the most appropriate initial intervention that will result in sustained
improvement in liver histology in an obese patient with non-alcoholic steato-hepatitis?

A. Vitamin E.
B. Metformin.
C. Weight loss.
D. Rosiglitazone.
E. Smoking cessation.

Question 31
Following partial gastrectomy, a patient complains of recurrent light-headedness, sweating and
palpitations two hours after eating. These symptoms are most likely due to:

A. Hypoglycaemia.
B. Hypovolaemia.
C. Vagal stimulation.
D. Serotonin (5-HT) release.
E. Adrenergic stimulation.

Question 32
A 65-year-old man with cirrhosis has significant (grade 2 of 3) oesophageal varices detected on
screening endoscopy. Which of the following is the most appropriate initial management?

A. Transjugular intrahepatic portosystemic shunt (TIPS).


B. Non-selective beta-blocker.
C. Isosorbide mononitrate.
D. Lienorenal shunt.
E. Endoscopic variceal ligation.

Question 33
What is the best interpretation of the following serological results?

A. Previous Hepatitis B vaccination.


B. Past Hepatitis B infection.
C. Chronic Hepatitis B infection.
D. Acute Hepatitis B infection.
E. Pre-core mutant infection.
89

ANSWERS

1. B
2. D
3. C
4. A
5. C
6. B
7. B
8. C
9. C
10. C
11. E
12. C
13. A
14. C
15. C
16. C
17. B
18. C
19. D
20. C
21. E
22. B
23. C
24. B
25. B
26. C
27. B
28. B
29. B
30. C
31. A
32. B
33. B
90

Clinical Applications

Gastroenterology

Question 1
A 40-year-old man is seen for follow-up at the outpatients’ department. Eight weeks prior, he had
presented with melaena and was found to have a duodenal ulcer and Helicobacter pylori gastritis. He
was treated with a combination pack containing omeprazole, amoxycillin and metronidazole for two
weeks and took all the prescribed tablets according to the instructions.

A urea breath test performed six weeks following the end of the eradication regimen was positive.

The most likely explanation for the failure of the eradication of Helicobacter pylori in this patient is:

A. Metronidazole resistance.
B. Amoxycillin resistance.
C. The short duration of therapy.
D. The low sensitivity and specificity of the urea breath test following eradication therapy.
E. An insufficient period between the end of the eradication therapy and the urea breath test.

Question 2
In which one of the following disorders is the highest proportion of patients with anti-mitochondrial
antibodies found?

A. Drug-induced cholestasis.
B. Extrahepatic biliary obstruction.
C. Primary biliary cirrhosis.
D. Cryptogenic cirrhosis.
E. Autoimmune active chronic hepatitis.
91

Question 3
A 36-year-old woman presents with a 12-month history of lassitude, intermittent diarrhoea
(described as watery, occurring up to six times a day), colicky abdominal pain, bloating and
involuntary weight loss of 12 kg.

Blood tests reveal normal electrolytes, urea, creatinine and liver function tests. Her full blood count
reveals a mild anaemia (haemoglobin 110 g/L [120-160]), but other indices are normal. Further
blood tests show that she is iron and folate deficient.

Upper and lower gastrointestinal endoscopies are performed and the mucosa appears normal
macroscopically. Shown below are sections of small bowel mucosa taken from the duodenum (A),
distal to the major papilla, and from the terminal ileum (B).

The most likely diagnosis is:

A. Crohn’s disease.
B. Coeliac disease.
C. Irritable bowel syndrome.
D. Giardiasis.
E. Yersiniosis.

Question 4
A critically ill 50-year-old patient in the intensive care unit develops abdominal distension. A supine
abdominal X-ray is obtained and is shown below.

The most likely diagnosis is:

A. Obstructing carcinoma at the splenic flexure.


B. Colonic pseudo-obstruction.
C. Small bowel paralytic ileus.
D. Faecal loading.
E. Sigmoid volvulus.
92

Question 5
A 34-week pregnant patient presents to emergency with epigastric and right upper quadrant pain and
nausea. Blood pressure is 140/85 mmHg. Relevant blood work shows:

Aspartate transaminase (AST) 150 IU/L [15 – 45 IU/L]


Prothrombin Time 14 secs [11 – 15 secs]
Bilirubin 25 µmol/L [14 – 22 µmol/L]
Platelets 75 x 109/L [150 – 400 x 109/L]
Haemoglobin (Hb) 90 g/L [110 – 130 g/L]

Transabdominal ultrasound reveals sludge in the gall bladder.

The most appropriate management is:

A. Urgent endoscopic retrograde cholangiopancreatography (ERCP) with biliary sphincterotomy.


B. Antibiotics and elective cholecystectomy.
C. Urgent delivery.
D. Plasma exchange.
E. Magnesium sulphate.
93

Question 6
A 54-year-old man with colonic Crohn’s disease is in remission following an eight-week course of
prednis(ol)one.

Which one of the following medications is most likely to maintain his remission?

A. Prednis(ol)one.
B. Sulphasalazine.
C. Azathioprine.
D. Metronidazole.
E. Methotrexate.

Question 7
Which one of the following is most likely to slow progression of primary biliary cirrhosis?

A. Prednis(ol)one.
B. Colchicine.
C. Methotrexate.
D. Cyclosporin.
E. Ursodeoxycholic acid.
94

Question 8
A 56-year-old man presents with a long history of symptoms suggestive of gastro-oesophageal
reflux. He undergoes upper endoscopy which reveals severe oesophagitis, but without evidence of a
stricture. Proton pump inhibitor therapy is commenced with complete relief of his symptoms.

Six weeks later, he undergoes a further upper endoscopy. The area just above the cardio-oesophageal
junction is shown below. Biopsies of this area are taken which show intestinal metaplasia with no
dysplasia.

The next most appropriate action is to:


A. Double the dose of proton pump inhibitor.
B. Refer for anti-reflux surgery.
C. Repeat endoscopy in 12 months.
D. Check autoimmune markers.
E. Continue current therapy.
95

Question 9
The criteria for a diagnosis of dependence on alcohol include all of the following features except:

A. Giving up important activities in order to drink alcohol.


B. Drinking larger amounts of alcohol than intended over longer periods of time.
C. Developing tolerance to alcohol so that higher doses are required for the same effect.
D. Drinking six or more standard drinks per day on most days for at least three months.
E. Continued use of alcohol despite physical or psychological difficulties.

Question 10
A 28-year-old man was found apnoeic and pulseless. There were a syringe and needle nearby and he
had well defined "track marks" (venous puncture marks) on his arms. He was successfully
resuscitated at the scene and brought to the Accident and Emergency Department. On arrival, he was
breathing spontaneously and was normotensive. Following routine investigations, he was admitted
for observation.

The next day you are called because of the results of the liver function tests below. The patient is
clinically unchanged.

Bilirubin 25 µmol/L [3-21]


Alkaline phosphatase (ALP) 45 U/L [30-115]
Gamma glutamyltranspeptidase (GGT) 95 U/L [<65]
Aspartate transaminase (AST) 9,867 U/L [5-40]
Alanine transaminase (ALT) 7,345 U/L [5-40]
Total protein 76g/L [60-85]
Albumin 41 g/L [40-52]
Prothrombin time-international normalised ratio (PT-INR) 1.0 [0.9-1.1]

The most likely cause for these liver function tests is:

A. Acute hepatitis C.
B. Post-CPR (cardiopulmonary resuscitation) liver trauma.
C. Acute hepatitis A.
D. Drug-induced hepatitis.
E. Ischaemic hepatitis.
96

Question 11
A 52-year-old man with known chronic liver disease, due to alcohol, is admitted to hospital because of
increasing ascites, peripheral oedema and confusion. He has been abstinent from alcohol for over two
years. His condition has been stable prior to this deterioration which has developed over the previous four
weeks. On examination, he is afebrile. His vital signs are normal. He has a flapping tremor, with
moderate ascites and peripheral oedema to the knees. He is drowsy with disorientation. The liver span is
decreased (10 cm by percussion) and the spleen is palpable 2 cm below the costal margin. No malignant
cells are detected in ascitic fluid following ascitic fluid drainage. The fluid is a transudate.

The results of his initial blood tests are as follows:

white cell count 4.2 x 109/L [4.0-11.0]


haemoglobin 112 g/L [120-160]
platelet count 56 x 109/L [150-400]
sodium 130 mmol/L [135-150]
potassium 4.5 mmol/L [3.5-5.0]
urea 6.5 mmol/L [3.6-9.3]
creatinine 0.09 mmol/L [0.06-0.12]
bilirubin 35 µmol/L [3-21]
alkaline phosphatase (ALP) 450 U/L [30-115]
gamma glutamyltranspeptidase (GGT) 320 U/L [<65]
aspartate transaminase (AST) 98 U/L [5-40]
alanine transaminase (ALT) 67 U/L [5-40]
albumin 25 g/L [40-52]
Prothrombin time-international normalised ratio (PT-INR) 1.5 [0.9-1.1]

A representative slice of an abdominal computed tomography (CT) scan is shown below.

The most likely cause for his deterioration is:

A. Hepatocellular carcinoma.
B. Multiple liver metastases.
C. Gallstone disease.
D. Portal vein thrombosis.
E. Liver abscess.
97

Question 12
A 50-year-old woman undergoes an upper abdominal ultrasound because of long standing
intermittent epigastric pain. She has had no weight loss or fever. A representative slice of her
ultrasound is shown below. Her liver function tests are normal. An endoscopy performed soon after
shows a chronic duodenal ulcer with Helicobacter pylori gastritis. Following successful H. pylori
eradication therapy, her pain resolves. An abdominal computed tomography (CT) scan is performed
in view of the ultrasound appearance; a representative slice is also shown below.

The most likely diagnosis is:

A. Liver metastases.
B. Liver haemangiomata.
C. Benign liver cysts.
D. Liver abscesses.
E. Hydatid disease

Question 13
A 35-year-old man presents to your office four months after resection of 50 cm of terminal ileum for
Crohn’s disease. He has been well, but complains of excessive diarrhoea, especially in the morning.
The motion has been watery, but without blood or mucus. A recent colonoscopy failed to show any
evidence of colitis or enteritis and the anastomosis appeared healthy. He is currently on no
medications. Physical examination reveals a well-healed scar, with no tenderness in the abdomen and
normal bowel sounds.

Stool cultures are negative.

The most appropriate treatment is:

A. Prednis(ol)one.
B. Sulfasalazine.
C. Azathioprine.
D. Cholestyramine.
E. Metronidazole.

Question 14
Which one of the following has the greatest positive impact on outcome in patients with acutely
bleeding peptic ulcers?

A. Nil by mouth for 24 hours.


B. Endoscopic injection therapy.
C. Admission to an intensive care unit.
D. Intravenous ranitidine.
E. Intravenous octreotide.
98

Question 15
Which one of the following is most likely to slow progression of primary biliary cirrhosis?

A. Prednis(ol)one.
B. Colchicine.
C. Methotrexate.
D. Cyclosporin.
E. Ursodeoxycholic acid.

Question 16
A 50-year-old woman is referred for investigation of longstanding bloating and watery diarrhoea.
Her history includes treated vitamin B12 deficiency. She denies laxative abuse. Her blood pressure is
110/65 mmHg with a 10 mmHg postural fall. Examination of her heart, chest and abdomen reveals
no abnormality.

Serum potassium is 3.1 mmol/L [3.5-5.0]. Serum sodium and creatinine are normal. Full blood
examination, erythrocyte sedimentation rate (ESR), thyroid function tests and stool cultures are also
normal. Urinary sodium is 2 mmol/L and urinary potassium is 50 mmol/L.

The appearance shown below is seen throughout the colon.

The most appropriate next investigation is:

A. Upper gastrointestinal endoscopy and duodenal biopsy.


B. 24-hour urinary 5-hydroxyindoleacetic acid (5-HIAA) excretion.
C. Small bowel enema.
D. 14C-xylose absorption test.
E. Stool and urine testing for laxatives.
99

Question 17
A 24-year-old woman was investigated for right upper quadrant pain. The computed tomography
(CT) scan of the upper abdomen is shown (below) before and following injection of contrast.

Which one of the following is the next most appropriate investigation?

A. Abdominal ultrasonography.
B. Endoscopic retrograde cholangiopancreatography.
C. CT-guided biopsy.
D. Pooled labelled red blood cell nuclear scan.
E. Repeat abdominal CT scan in six months.
100

Question 18
A 43-year-old man with a long history of ileal Crohn’s disease presents with rigors, a temperature of
39.8°C, lower abdominal pain and marked right iliac fossa tenderness. He is taking prednis(ol)one 10
mg/day and sulfasalazine 1 g twice/day. Blood cultures are obtained. An urgent computed
tomography (CT) scan of the abdomen is shown below.

Which one of the following is the most appropriate next step in this patient’s management?

A. Intravenous hydrocortisone.
B. Intravenous antibiotics.
C. Azathioprine.
D. Intravenous hydrocortisone and intravenous antibiotics.
E. Laparotomy.
101

Question 19
A 63-year-old man presents to his general practitioner with a four-month history of crampy lower
abdominal pain, increasing constipation and bright red rectal bleeding. There is mild left iliac fossa
tenderness. He undergoes barium enema examination which is shown below

Which one of the following is the most appropriate next step in the management of this patient?

A. Abdominal ultrasonography.
B. Abdominal computed tomography (CT).
C. Colonoscopy.
D. Institution of a high fibre diet and therapy with antispasmodic agents.
E. Sigmoid colectomy.

Question 20
A 52-year-old patient presents with indigestion. Upper endoscopy reveals extensive gastritis and a
biopsy shows both mucosal-associated lymphoid tissue (MALT) lymphoma and Helicobacter pylori
infection. Physical examination, computed tomography (CT) scan of the chest, abdomen and pelvis,
gallium scan and bone marrow examination are all normal.

The most appropriate initial therapy is:

A. Combination cytotoxic chemotherapy.


B. Helicobacter pylori eradication therapy.
C. Single agent chemotherapy.
D. Gastric irradiation.
E. Gastrectomy.
102

Question 21
Which of the following is the most appropriate medication to maintain remission in ileo-colonic
Crohn’s disease?

A. Nicotine.
B. Azathioprine.
C. Mesalazine.
D. Budesonide.
E. Cyclosporin.

Question 22
A 45-year-old man presents with a one-week history of anorexia, vomiting and abdominal pain. He
denies any fever, but has noted his sclerae to be icteric in the last three days. There is a history of
intravenous drug use to the age of 25 years. There are no recent prescribed or over-the-counter
medications. He drinks the equivalent of 100g of alcohol per day, with frequent heavy intake on
weekends.

Examination confirms icterus, but there are no peripheral stigmata of chronic liver disease. There is
moderate to severe tenderness in the right upper quadrant. The liver is percussed at 18 cm in the mid-
clavicular line. There is no splenomegaly.
Cardiovascular, respiratory and peripheral nervous system examinations are normal.

Routine blood tests show:

The most likely explanation for this clinical scenario is:

A. Advanced cirrhosis.
B. Alcoholic hepatitis.
C. Acute cholecystitis.
D. Acute viral hepatitis.
E. Hepatoma.
103

Question 23
A 55-year-old woman with a long history of constipation-predominant irritable bowel syndrome
presents for review. She complains of worsening constipation and increased bloating. She had a
normal colonoscopy at the age of 40 years.

She presented to the emergency department six days ago with abdominal bloating which subsided
following an enema. An abdominal X-ray is shown below.

The most appropriate next step is to:

A. Increase fibre intake.


B. Add a laxative to her regimen.
C. Perform a colonoscopy.
D. Perform an abdominal computed tomography (CT) scan.
E. Perform an upper abdominal ultrasound.
104

Question 24
A 68-year-old man is sent for a second opinion regarding recurrent iron deficiency anaemia requiring
blood transfusion.

Over the last three years, he has required an average of 4 units of packed cells every three months
despite oral iron supplements. There is no history of melaena. Bone marrow examination is normal
except for absent iron stores.

Gastroscopy and colonoscopy have been negative on three separate occasions, the last being two
months ago. Small bowel biopsy is normal. The patient had a barium meal and follow-through six
months ago which was normal.

General health is good and he is on no regular medications.

Which of the following investigations is most likely to find the cause of his iron deficiency?

A. Small bowel enema.


B. Repeat gastroscopy and colonoscopy.
C. Enteroscopy.
D. Abdominal angiography.
E. Labelled red cell scan.

Question 25
A 75-year-old man presents with progressive dysphagia. He is currently only able to ‘keep down’
liquids. He has lost 7 kg in weight over the last six weeks. Endoscopy shows a mass lesion in the
distal oesophagus which does not allow passage of the endoscope into the stomach. Biopsies show
adenocarcinoma. A computed tomography (CT) scan reveals thickening in the distal oesophagus as
well as two liver lesions which prove to be metastatic on biopsy.

Which of the following is most appropriate to give him immediate and long lasting (>6 months)
relief from dysphagia?

A. Oesophageal dilation.
B. External beam radiotherapy.
C. Percutaneous endoscopic gastrostomy (PEG) tube.
D. Self-expanding metal stent placement.
E. Ivor-Lewis procedure (distal oesophagectomy).
105

Question 26
A 45-year-old woman has a history of terminal ileal Crohn’s disease for which she had a terminal
ileal resection eight years ago. Surgery was complicated by small bowel leak post operatively,
requiring laparotomy and prolonged drainage with bowel rest (total parenteral nutrition). Following
recovery she remained well until two weeks ago, when she presented with two episodes of
abdominal pain, bloating and vomiting which lasted two to three hours each and then resolved. There
has been no change in bowel habit and no fever. She is on no medication.

Examination reveals mild tenderness in the lower abdomen. Bowel sounds are normal.

A computed tomography (CT) scan is performed and representative slices are shown below.

Which of the following is the most likely cause for her current symptoms?

A. Adhesions.
B. Recurrence of Crohn’s disease.
C. Psoas abscess.
D. Colonic carcinoma.
E. Irritable bowel syndrome.
106

Question 27
A 25-year-old presents with acute pancreatitis. There is no significant past medical history and
minimal alcohol intake. Ultrasound demonstrates stones in the gall bladder. Which of the following
most strongly indicates the need for urgent endoscopic retrograde cholangiopancreatography
(ERCP)?

A. Age <30 years.


B. Bilirubin 80 µmol/L [14 – 22 µmol/L].
C. Alanine transaminase (ALT) >310 IU/L [15 – 45 IU/L].
D. Lipase 10 000.
E. Extensive interstitial (oedematous) pancreatitis in computed tomography (CT) scan.

Question 28
For pre-operative staging of oesophageal cancer, which one of the following provides the most
accurate information on the T stage?

A. Endoscopy.
B. Computed tomography (CT) scanning.
C. Magnetic resonance imaging (MRI).
D. Endoscopic ultrasonography (EUS).
E. Positron emission tomography (PET) scanning.

Question 29
Prior to which of the following procedures is antibiotic prophylaxis most appropriate?

A. Colonoscopy with polypectomy.


B. Colonoscopy in a patient with a knee prosthesis.
C. Percutaneous endoscopic gastrostomy placement.
D. Endoscopic retrograde cholangiopancreatography (ERCP).
E. Band ligation of oesophageal varices.
107

Question 30
A 55-year-old woman presents with a 10-year history of rheumatoid arthritis and is on longstanding,
stable treatment with methotrexate 10 mg weekly, folic acid 10 mg weekly, diclofenac 50 mg three
times daily and prednis(ol)one 7.5 mg daily. She also has hypertension controlled with amlodipine 5
mg daily and mild type 2 (non-insulin-dependent) diabetes mellitus treated with diet and metformin
500 mg three times daily. She denies drinking alcohol.
2
On examination, she is overweight (body mass index (BMI) 28 kg/m [18-25]), with rheumatoid
nodules, rheumatoid hand deformity and synovitis in the wrists and metatarsophalangeal (MTP)
joints. Her blood pressure is 150/85 mmHg. There are no other abnormal signs.
Blood tests (which have been stable over 6 months) reveal:
haemoglobin 105 g/L [120-160]
erythrocyte sedimentation rate (ESR) 55 mm/h [<20]
rheumatoid factor (RF) 140 IU/mL [<20]
antinuclear antibody (ANA) positive speckled pattern titre 320 (negative titre <40)
anti-smooth muscle antibodies positive (low titre)
bilirubin normal
aspartate transaminase (AST) 65 U/L [15-45]
alanine transaminase (ALT) 90 U/L [15-50]
alkaline phosphatase (ALP) 115 U/L [38-126]
albumin 32 g/L [33-48]
A liver biopsy is planned.
Which of the following is most likely to be found on liver biopsy?
A. Methotrexate hepatotoxicity.
B. Amyloidosis.
C. Non-alcoholic fatty liver disease.
D. Chronic active hepatitis.
E. Cirrhosis.
108

Question 31
A 35-year-old male has the following hepatitis B virus (HBV) profile with normal liver
biochemistry:
hepatitis B surface antigen (HBsAg) +
hepatitis B e antigen (HBeAg) -
HBVDNA -

He develops lymphoma requiring chemotherapy (CHOP: cyclophosphamide, doxorubicin,


vincristine and prednisone). With regard to his HBV status, which of the following is the most
appropriate strategy?

A. Monitor liver enzymes during and after treatment.


B. Prophylactic anti-viral therapy.
C. Monitor e-antigen status during therapy.
D. No specific management necessary.
E. Modify chemotherapy regimen.

Question 32
In a 23-year-old female with Crohn’s disease who becomes pregnant, which of the following
medications is most likely to result in an adverse foetal outcome?

A. Prednis(ol)one.
B. Azathioprine.
C. Methotrexate.
D. Sulfasalazine.
E. Mesalazine.

Question 33
A 45-year-old male with known chronic hepatitis B virus (HBV) infection and multiple flares over
five years presents with mild jaundice and lethargy. There is no history of intravenous drug use.
Blood tests reveal the following:

aspartate transaminase (AST) 350 U/L [0-40]


alanine transaminase (ALT) 400 U/L [0-40]
bilirubin 55 µmol/L [0-20]
gamma glutamyltranspeptidase (GGT) 350 U/L [0-45]
alkaline phosphatase (ALP) 450 U/L [30-115]
hepatitis B surface antigen (HBsAg) +
hepatitis B e antigen (HBeAg) -

Which of the following is the most likely explanation for this presentation?

A. Development of hepatoma.
B. Choledocholithiasis.
C. Emergence of an HBV pre-core mutant.
D. Delta super-infection.
E. Emergence of a YMDD mutant.
109

Question 34
An otherwise well 55-year-old female undergoes upper endoscopy for investigation of mild reflux
symptoms. An endoscopic image of the lower oesophagus is shown above. Biopsies demonstrate
intestinal metaplasia with high grade dysplasia and no inflammation

The most appropriate management is:

A. Repeat endoscopy in six months.


B. Surgical resection.
C. Photodynamic therapy.
D. High dose proton-pump inhibitor (PPI) therapy.
E. Thermal ablative therapy.

Question 35
A 45-year-old alcoholic presents with his first oesophageal variceal bleed. His acute bleeding is
controlled with endoscopic band ligation. Which one of the following is the most appropriate
management strategy to prevent recurrent bleeding?

A. Refer for liver transplantation.


B. Propranolol.
C. Isosorbide mononitrate.
D. Transjugular intrahepatic portosystemic shunt (TIPS).
E. Repeated endoscopies and band ligation.
110

Question 36
A patient is referred for investigation of diarrhoea and weight loss. The following results are
obtained:

haemoglobin 100 g/L [130-180]


mean corpuscular volume (MCV) 72 fL [82-98]
anti-gliadin IgG 43 units [0-25]
anti-gliadin IgA 0 units [0-25]
IgA-endomysial antibody <5 [<5]

Which of the following is the next most appropriate test?

A. Serum IgA level.


B. Haemoglobin electrophoresis.
C. 3-day faecal fat.
D. Tissue transglutaminase antibodies.
E. Faecal microscopy and culture.

Question 37
A 62-year-old woman with rheumatoid arthritis for eight years presents for review. Her arthritis is
well controlled with 15mg of methotrexate weekly, folic acid supplements and ketoprofen sustained
release 200mg daily, all of which she has been on for the past five years. She is overweight with a
body mass index of 35kg/m2 [18-26] and has mild, diet-treated type 2 diabetes mellitus. She is on
no other medications. You review her routine blood tests which are normal apart from her liver
function tests which have been progressively worsening over the past six months.

The most recent test is shown below.


protein 88 g/L [63-82]
albumin 41 g/L [35-50]
bilirubin 8 µmol/L [<10]
alanine transaminase (ALT) 150 U/L [<52]
aspartate transaminase (AST) 120 U/L [<36]
alkaline phosphatase (ALP) 120 U/L [<126]
gamma glutamyltranspeptidase (GGT) 150 U/L [<43]

The most likely explanation for this woman's liver function test abnormalities is:

A. Methotrexate induced hepatitis.


B. Ketoprofen-induced hepatitis.
C. Non-alcoholic steatohepatitis.
D. Auto-immune hepatitis.
E. Viral hepatitis.
111

Question 38
A 50-year-old male has chronic liver disease secondary to Hepatitis B infection. Previous endoscopy
has revealed large oesophageal varices. Blood results are:

Albumin 32 g/L [38-55]


Prothrombin Time 14 seconds [8-12]
Platelet count 70 x 109/L [150-450]

On routine review, his alpha fetoprotein is 300 ng/mL [0-15]. His abdominal computed tomography
(CT) scan is shown below.

The optimal treatment for this condition is:

A. Chemoembolisation.
B. Liver transplantation.
C. Cryotherapy.
D. Local resection.
E. Radiofrequency ablation.
112

Question 39
A patient is referred for investigation of diarrhoea and weight loss. The following results are
obtained:

Haemoglobin 100 g/L [130-180]


mean corpuscular volume (MCV) 72 fL [82-98]
anti-gliadin IgG 43 units [0-25]
anti-gliadin IgA 0 units 0-25]
IgA-endomysial antibody <5 [<5]

Which of the following is the next most appropriate test?

A. Serum IgA level.


B. Haemoglobin electrophoresis.
C. 3-day faecal fat.
D. Tissue transglutaminase antibodies.
E. Faecal microscopy and culture.

Question 40
Which of the following is the most appropriate indication for treatment with ursodeoxycholic acid?

A. Primary sclerosing cholangitis.


B. Massive choledocholithiasis.
C. Primary biliary cirrhosis.
D. Microlithiasis associated pancreatitis.
F. Cholecystolithiasis in a poor surgical candidate

Question 41
A previously well 64-year-old man presents with symptoms of gastro-oesophageal reflux disease.
He is on no current medications. Gastroscopy demonstrates Barrett’s oesophagus with erosive
oesophagitis above the squamocolumnar junction. Random biopsies are reported as demonstrating
dysplastic epithelium with features of active inflammation. The most appropriate next step in
management is:

A. Endoscopic ultrasound.
B. CT (computed tomography) chest/abdomen.
C. Repeat biopsies after two months of treatment with proton pump inhibitors.
D. Surgical resection.
E. Surveillance endoscopy in one year.
113

Question 42
Which of the following features in a patient with colon cancer is most suggestive of hereditary non-
polyposis colon cancer (HNPCC)?

A. Right-sided lesion.
B. Microsatellite instability in the tumour tissue.
C. Multiple synchronous polyps.
D. Brother with colon cancer at age 55.
E. Uncle with a germline MLH1 mutation.

Question 43

A 55-year-old male presents for antihypertensive medication. During the interview, you are told that
his 82-year-old father has recently been diagnosed with colon cancer. Which of the following is the
most appropriate colon cancer screening for this patient?

A. One-off colonoscopy.
B. Five yearly colonoscopy.
C. Annual faecal occult blood testing.
D. Barium enema.
E. Computed tomography (CT) colonography.

Question 44
Which of the following factors is the strongest contraindication to the use of interferon in the
treatment of viral hepatitis:

A. Disease acquisition during childhood.


B. Child-Pugh category C status.
C. Serum alanine transaminase levels <2 times normal.
D. Hepatitis e antigen negative infection.
E. Immunocompromised status.

Question 45
A 75-year-old presents with progressive dysphagia for solids and liquids over two years. There has
been no episodes of bolus impaction. There has been a two kilogram weight loss over six months.
Oesophageal manometry demonstrates increased teritiary wave activity and decreased amplitude of
contractions. The most likely diagnosis is:

A. Ulcerative reflux oesophagitis.


B. Presbyoesophagus.
C. Achalasia.
D. Scleroderma.
E. Diffuse oesophageal spasm .
114

Question 46
For which of the following presentations of inflammatory bowel disease is infliximab of most
benefit?

A. Ulcerative colitis refractory to treatment with azathiaprine.


B. Crohn’s disease with perianal fistulae.
C. Pouchitis refractory to antibiotic treatment.
D. Fulminant ulcerative colitis.
E. Small bowel Crohn’s disease with previous stricturoplasty.

Question 47
An 65-year-old man is on chronic haemodialysis. He complains of crampy lower abdominal pain and
passes blood per rectum three times over two hours. Colonoscopy demonstrates normal rectal
mucosa and inflammation from the proximal sigmoid to transverse colon. The most likely cause of
the
inflammation is:

A. Salmonella infection.
B. Ischaemic colitis.
C. Clostridium difficile colitis.
D. Diverticulitis.
E. Crohn's colitis.

Question 48
A 26-year-old female has been found to be homozygous for the HFE gene C282Y polymorphism,
when screened for haemochromatosis. Her serum ferritin is 18 micrograms/L [15-200 micrograms/L]
and her transferrin saturation is 82% [< 45%]. Her haemoglobin is 120 g/L [120-160 g/L] Which of
the following is the most appropriate next step in her management?

A. Venesection.
B. Observation.
C. Liver biopsy.
D. Imaging of liver for iron loading.
E. Desferrioxamine.
115

Question 49
A 60-year-old male with no significant past or family history presents for anti-hypertensive
medication. With regard to his risk of colon cancer, which of the following is the most appropriate
strategy?

A. Dietary advice.
B. CT colography.
C. Annual digital rectal examination.
D. Annual faecal occult blood test.
E. Five yearly colonoscopy.

Question 50
A 28-year-old male has the following biochemical findings 24 hours after suffering a thoracic gun
shot wound resulting in hypovolemic shock, requiring massive blood transfusion.

AST 3800 U/L [5-55 U/L]


Alkaline Phosphatase 69 U/L [0-130 U/L]
ALT 6400 U/L [5-55 U/L]
Gamma GT 90 U/L [0-60 U/L]
Bilirubin 20 micromole/L [0-18 micromole/L]
LDH 1800 U/L [<220 U/L]
Creatinine 180 micromole/L [70-110 micromol/L]

The most likely explanation for these findings is:

A. Halothane hepatitis.
B. Hepatic vein transection.
C. Transfusion reaction.
D. Hepatic congestion.
E. Ischaemic hepatitis.

Question 51
For which of the following presentations of inflammatory bowel disease is infliximab of most
benefit?

A. Ulcerative colitis refractory to treatment with azathioprine.


B. Crohn’s disease with perianal fistulae.
C. Pouchitis refractory to antibiotic treatment.
D. Fulminant ulcerative colitis.
E. Small bowel Crohn’s disease with previous stricturoplasty.
116

Question 52
Which of the following factors is the strongest contraindication to the use of interferon in the
treatment of viral hepatitis?

A. Disease acquisition during childhood.


B. Child-Pugh category C status.
C. Serum alanine transaminase levels <2 times normal.
D. Hepatitis e antigen negative infection.
E. Immunocompromised status.
117

Answers

1. A 28. D
2 C 29. C
3. B 30. C
4. B 31. B
5. C 32. C
6. C 33. C
7. E 34. B
8. C 35. E
9. D 36. A
10. E 37. C
11. A 38. B
12. C 39. A
13. D 40. C
14. B 41. C
15. E 42. E
16. E 43. C
17. D 44. B
18. D 45. B
19. C 46. B
20. B 47. B
21. B 48. B
22. B 49. D
23. C 50. E
24. C 51. B
25. D 52. B
26. B
27. B
118

Medical Sciences

Genetics

Question 1
Which one of the following best describes the G2 phase of the cell cycle?

Microtubule Other protein


DNA replication
production production
A. low low low
B. low low high
C. low high low
D. high high low
E. high high high

Question 2
Which one of the following most accurately reflects the clinical value of DNA testing for
Huntington’s disease?

A. It allows the age of onset to be determined.


B. It allows investigation of an individual presenting with tremor.
C. It allows young children to be tested.
D. It allows an at risk individual to be tested before clinical features develop.
E. It requires only a sample of blood.

Question 3
Assume that in a certain homogeneous population the carrier frequency for an autosomal recessive
disorder is 1 in 10. What is the risk of a child born in this population being homozygous-affected?

A. 1 in 20.
B. 1 in 100.
C. 1 in 200.
D. 1 in 400.
E. 1 in 800.

Question 4
Which one of the following mutations in a gene is most likely to disrupt the production of a protein?

A. Splicing defect.
B. Missense change.
C. Nucleotide transversion.
D. Nonsense change.
E. A three base pair deletion.
119

Question 5
A mutation is located in the middle portion of a double-stranded DNA molecule 200 nucleotides
long. At each end of the DNA molecule are sites at which short DNA primers for the polymerase
chain reaction (PCR) bind to each strand.

In the figures shown below, five possible combinations of primer location and orientation relative to
the mutation are shown. In each figure, the long pair of arrows represents the anti-parallel strands of
the long DNA molecule and the short arrows represent each short single-stranded DNA primer
binding to the closer of the two long strands. The mutation is located at position (II), the primers are
located at positions (I) and (III), and the arrowhead on each DNA strand indicates the direction of
DNA polymerisation (5’ to 3’).

Which one of the following combinations of primer location and orientation is required to amplify
the mutation by PCR?
120

Question 6
Which one of the following mutations in a gene is most likely to disrupt the production of a protein?

A. Splicing defect.
B. Missense change.
C. Nucleotide transversion.
D. Nonsense change.
E. A three base pair deletion.

Question 7
A non-consanguineous family with a rare genetic disease has been identified. The pedigree is shown
below.

What is the most likely mode of inheritance?

A. Mitochondrial.
B. X-linked.
C. Autosomal dominant.
D. Autosomal recessive.
E. Polygenic.
121

Question 8
In the pedigree shown below, the man indicated by an arrow has been shown to have an autosomal
recessive biochemical disorder with complete penetrance. The causative gene has not been identified.
His parents are obligate carriers and do not exhibit any biochemical abnormalities. His sister also has
normal biochemical studies. The carrier frequency in this population is 10%.

In the absence of consanguinity, what is the risk of the sister having a child with the biochemical
abnormality?

A. 1 in 40.
B. 1 in 60.
C. 1 in 80.
D. 1 in 100.
E. 1 in 120.
122

Question 9
A typical Southern blot for DNA studies can be described as four steps. These steps are listed below
in random order.

1. Genomic DNA is transferred to a synthetic membrane.


2. Genomic DNA is separated according to length by gel electrophoresis
3. Genomic DNA is probed with a labeled DNA molecule.
4. Genomic DNA is cut at specific sites by enzymatic digestion.

What is the correct order?

A. 4 2 1 3

B. 2 4 3 1

C. 4 3 2 1

D. 4 3 1 2

E. 1 4 2 3

Question 10
Which of the following options is the best description of “maternal uniparental disomy”?

Number of Maternal Alleles Number of Paternal Alleles


A. 1 0
B. 1 1
C. 1 2
D. 2 0
E. 2 1

Question 11
The gene responsible for an x-linked recessive disorder consists of 27 exons. The following DNA
sequence is obtained from the gene of an affected man. The nucleotides at positions 37 –51 in the
first exon of the gene are shown below. The DNA sequence from a normal control sample is shown
below for comparison. No other sequence variant was identified in the gene.

Patient sample gcagtgtgctgtgat


Normal control gcagtgctgctgtga

What is the correct genetic interpretation of the DNA sequence data from this patient?

A. This is a pathogenic mutation.


B. The interpretation is determined by the flanking DNA sequence.
C. The interpretation is determined by the reading frame in which it occurs.
D. The interpretation is determined by the codon usage in the gene.
E. This is a benign polymorphism.
123

Question 12
The polymerase chain reaction (PCR) requires the use of a DNA polymerase. The action of the
DNA polymerase is best described as:

A. Causing double-stranded DNA to become single-stranded.


B. Allowing primers to affix to the single strands.
C. Extending the primers once they arer affixed to the single strands.
D. Splitting the single strand and the extended primer.
E. Inducing fluorescence of DNA

Question 13
Many familial disorders are due to mutations in genes for transcription factors. Which of the
following statements is the best description of the role of a transcription factor? It:

A. Generates an RNA template.


B. Is a component of the ribosome.
C. Regulates the binding of DNA polymerase.
D. Regulates the polyadenylation of messenger RNA (mRNA).
E. Regulates the binding of RNA polymerase.

Question 14
The mechanism by which specific information encoded by viral DNA is transferred to messenger
RNA (mRNA) is:

A. Translation.
B. Transcription.
C. Reverse transcription.
D. Transduction.
E. Conjugation.
124

Question 15
In the pedigree shown below, the affected man has an X-linked recessive disorder. A polymorphic
DNA marker has been identified close to the mutant gene responsible for this disorder. The
genotypes at this marker are given below each symbol in the pedigree. The recombination fraction
between the gene and the DNA marker is 10%.

What is the best estimate of the risk of the woman indicated by the arrow being a carrier of her
grandfather's disorder?

A. 100%.
B. 90%.
C. 81%.
D. 66%.
E. 10%.

Question 16
A patient has haemophilia A due to a frameshift mutation in the F8C gene. At the conclusion of the
S-phase in a cell of this patient, how many copies of the mutant gene would be present in the cell?

A. 0.
B. 1.
C. 2.
D. 3.
E. 4.
125

Question 17

In the pedigree shown above, the affected male has a rare autosomal recessive disorder. His niece
and nephew have a newborn son (indicated by the arrow).

What is the chance that the baby will have the same disorder?

A. 1 in 18.
B. 1 in 32.
C. 1 in 36.
D. 1 in 64.
E. 1 in 128.

Question 18
The mechanism by which specific information encoded by viral DNA is transferred to messenger
RNA (mRNA) is:

A. Translation.
B. Transcription.
C. Reverse transcription.
D. Transduction.
E. Conjugation.
126

Question 19
In patients with a certain genetic disorder, more than 95% of the abnormal genes have the same
mutation. Analysis of a highly polymorphic locus in an intron of the gene shows similar allele
frequencies in patients as in the general population.

Which one of the following genetic mechanisms is least likely to account for the predominance of
one mutation in patients with this disorder?

A. Gene conversion involving a related pseudogene.


B. Founder effect (i.e. a historical population “bottleneck”).
C. Selective advantage for carriers with the mutation.
D. Gene rearrangements due to repetitive sequences flanking an exon.
E. Highly mutable DNA sequence at one point in the gene.

Question 20
Consider the portion of an exon shown below. The DNA sequence is presented as nucleotide triplets,
reflecting the reading frame of the gene. A numbering system for the nucleotides is shown below the
sequence.

Which one of the following mutations is least likely to have an effect on the amino acid sequence of
the protein encoded by this gene?

A. Substitution of the nucleotide at position 4.


B. Substitution of the nucleotide at position 5.
C. Substitution of the nucleotide at position 6.
D. Substitution of the nucleotides at positions 5, 6, and 7.
E. Substitution of the nucleotides at positions 6, 7, and 8.
127

Question 21
Many familial disorders are due to mutations in genes for transcription factors. Which of the
following statements is the best description of the role of a transcription factor? It:

A. Generates an RNA template.


B. Is a component of the ribosome.
C. Regulates the binding of DNA polymerase.
D. Regulates the polyadenylation of messenger RNA (mRNA).
E. Regulates the binding of RNA polymerase.

Question 22
In a study of polymorphisms in the gene encoding the â2-adrenergic receptor, 20% of people were
heterozygous for a polymorphism at codon 16 and 22% were heterozygous for a polymorphism at
codon 27.

18% of the population were heterozygous at both loci.

Which one of the following statements is the best explanation for this observation?

A. Linkage disequilibrium between the two polymorphisms.


B. Low frequency of recombination between the two polymorphisms.
C. Co-inheritance of the two polymorphisms providing a selective advantage.
D. Intragenic cross-over between the two codons.
E. Variable triplet repeat expansion between the two codons.

Question 23
Which one of the following statements is the best description of the molecular basis of a chimaeric
protein (eg Bcr-Abl)?

A. Formation of disulphide bonds between cytosine residues on different peptides.


B. Post-transcription ligation of non-homologous messenger RNA (mRNA) molecules.
C. Hybrid messenger RNA (mRNA) synthesis from a bi-directional promoter.
D. Deletion of an exon from one gene.
E. In-frame ligation of the 5`-end of one gene to the 3`-end of another.
128

Question 24
Which of the following observations regarding a rare disorder would provide the best evidence of
autosomal dominant inheritance?

A. Father and son affected.


B. Father and daughter affected.
C. Mother and son affected.
D. Mother and daughter affected.
E. Male and female cousins affected.

Question 25
What is the most distinguishing feature of loss of imprinting of a growth-promoting imprinted gene?

A. Restoration of normal methylation patterns.


B. Inhibition of cell growth.
C. Reduced expression of the imprinted gene product.
D. Activation of the normally silent allele.
E. Ablation of biallelic gene expression.

Question 26
A man is diagnosed with a rare autosomal recessive disorder. His pedigree is shown below.

What is the best estimate of the risk of his cousin (indicated on the diagram by an arrow) developing
the same disorder?

A. 1/2.
B. 1/4.
C. 1/8.
D. 1/16.
E. 1/32.
129

Question 27
Which one of the following statements is the best description of the molecular basis of a chimaeric
protein (eg Bcr-Abl)?

A. Formation of disulphide bonds between cytosine residues on different peptides.


B. Post-transcription ligation of non-homologous messenger RNA (mRNA) molecules.
C. Hybrid messenger RNA (mRNA) synthesis from a bi-directional promoter.
D. Deletion of an exon from one gene.
E. In-frame ligation of the 5`-end of one gene to the 3`-end of another.

Question 28
A boy and his maternal uncle have haemophilia a. Polymerase chain reaction (pcr) analyses of three
exons of the gene responsible for haemophilia A, F8C, are shown in the figure below. The analyses
included DNA from a normal male (lane 1), the affected boy (lane 2), his affected uncle (lane 3), and
the boy’s mother (lane 4).

What is the most likely explanation for the presence of a band corresponding to exon 4 in the
mother?
The mother:

A. Has a deletion encompassing exon 4.


B. Is a gonadal mosaic for a mutation in exon 4.
C. Is a somatic mosaic for a mutation in exon 4.
D. Inherited a normal F8C gene from her father.
E. Has a polymorphism at a PCR primer site in exon 4.

Question 29
X-linked dominant inheritance is distinguished by:

A. Females often being more variably and mildly affected than males.
B. Affected males having affected daughters and healthy sons.
C. Affected females having affected daughters and healthy sons.
D. Males often being more variably and mildly affected than females.
E. Healthy females having affected sons and healthy daughters.
130

Question 30
What is the most likely consequence of inheriting a mutated, maternally imprinted gene?

A. 100% of the children of a carrier mother will be affected.


B. 50% of the children of a carrier mother will be affected.
C. None of the children of a carrier mother will be affected.
D. 100% of the children of a carrier father will be affected.
E. None of the children of a carrier father will be affected.

Question 31
Small interfering RNAs (also called “micro-RNAs”) influence gene expression. The mechanism of
action is best explained by their effect on:

A. Gene promoter activation.


B. Gene promoter repression.
C. RNA synthesis rate.
D. RNA degradation rate.
E. Chromatin compaction.

Question 32
A certain autosomal recessive disorder affects 1 in 1600 people; the carrier frequency is 5%. A DNA
assay can identify the mutation in 80% of carriers; the false-positive rate of this assay is zero.

What is the best estimate of the positive predictive value (PPV) and negative predictive value (NPV)
of this assay in screening the population for carriers?

PPV NPV
A 20% 100%
B 80% 80%
C 100% 80%
D 100% 99%
E 100% 100%

Question 33
What is the most distinguishing feature of loss of imprinting of a growth-promoting imprinted gene?

A. Restoration of normal methylation patterns.


B. Inhibition of cell growth.
C. Reduced expression of the imprinted gene product.
D. Activation of the normally silent allele.
E. Ablation of biallelic gene expression.
131

Question 34
In the pedigree shown below, the person (II-l) in generation II has Wilson disease. What is the
probability that his unaffected brother (II-2) is a carrier of a Wilson disease-causing mutation?

A. 1/4
B. 1/3
C. 1/2
D. 2/3
E. 1/1

Question 35
Some autosomal recessive diseases have high prevalence in particular populations, even though they
are often fatal (e.g. Alpha thalassaemia in South East Asia; G6PD deficiency in the Mediterranean).

Which of the following is the most likely explanation?

A. Consanguinity.
B. High mutation rates in specific populations.
C. Survival advantage in heterozygous carriers.
D. Survival advantage in wild type homozygotes.
E. Founder effect.

Question 36
A certain autosomal recessive disorder affects 1 in 1600 people; the carrier frequency is 5%. A DNA
assay can identify the mutation in 80% of carriers; the false-positive rate of this assay is zero. What
is the best estimate of the positive predictive value (PPV) and negative predictive value (NPV) of
this assay in screening the population for carriers?
132

Question 37
The normal phenotype of individual II:3 in the family below is best explained by:

A. Maternal imprinting.
B. Non-paternity.
C. Non-penetrance.
D. Multigenic inheritance.
E. X-linked recessive inheritance.

Question 38
Which one of the following most specifically describes the use of DNA linkage analysis in the search
for a disease-causing gene in a family?

A. The correlation of a DNA polymorphism with a family-specific mutation.


B. The finding of an association between a DNA polymorphism and a specific clinical phenotype.
C. The drawing of a pedigree and identifying who is normal or affected.
D. Having access to a large pedigree and being able to obtain DNA from most family members.
E. The drawing of a pedigree and excluding the potential for non-paternity.
133

Answers

1. C
2. D
3. D
4. D
5 E
6. D
7. C
8. B
9. A
10. D
11. A
12. C
13. E
14. B
15. B
16. C
17. C
18. B
19. B
20. C
21. E
22. A
23. E
24. A
25. D
26. D
27. E
28. D
29. A
30. C
31. D
32. D
33. D
34. D
35. C
36. D
37. C
38. B
134

Clinical Applications

Genetics

Question 1
A 40-year-old man had profuse colonic polyposis diagnosed 15 years ago. A clinical diagnosis of
familial adenomatous polyposis (FAP) had been made. He had a total colectomy. There is no family
history of polyposis or colorectal cancer.

Mutation studies fail to identify a pathogenic mutation in the adenomatous polyposis coli (APC) gene
in a blood sample.

The normal DNA result is best explained by:

A. The correct clinical diagnosis being juvenile polyposis.


B. The mutation occurring in a non-coding region of the APC gene.
C. Gonadal mosaicism of the APC gene mutation.
D. The causative mutation being in another gene.
E. The APC gene mutation occurring only in cells derived from adenomatous polyps.

Question 2
A couple seek your advice regarding the recurrence risk of the NARP (neuropathy, ataxia, retinitis
pigmentosa) syndrome, a disorder with mitochondrial inheritance. They are first cousins and have a
family history of the condition.

What is the impact of their consanguinity on the risk of having an affected child?

A. Approximately 10% reduction in risk.


B. No alteration to risk.
C. Approximately 10% increase in risk.
D. Approximately 25% increase in risk.
E. The alteration in risk can only be determined by detailed examination of the pedigree.

Question 3
Which one of the following is most likely to involve a defect in the CFTR (cystic fibrosis
transmembrane conductance regulator) gene?

A. Congenital bilateral absence of the vas deferens.


B. Bronchiectasis.
C. Malabsorption.
D. Chronic respiratory infection with Pseudomonas aeruginosa.
E. Biliary cirrhosis.
135

Question 4
The following mutations (Cys282Tyr and His63Asp) are associated with hereditary
haemochromatosis. Which one of the following genotypes provides the greatest risk for the
development of clinical disease?

A. Heterozygous Cys282Tyr.
B. Heterozygous His63Asp.
C. Double-heterozygote for Cys282Tyr and His63Asp.
D. Homozygous Cys282Tyr.
E. Homozygous His63Asp.

Question 5
The DNA approach to genetic disorders means that individuals who are at risk, but clinically
asymptomatic, can be tested to determine if they will develop the disease.
In this circumstance, which one of the following is the most appropriate reason for testing a nine-
year-old child?

A. Informed consent can be obtained from the parents.


B. Parental anxieties can be relieved.
C. In the case of an autosomal recessive disorder, the child's carrier status can be determined.
D. Medical intervention can alter the disorder's natural history.
E. The child wants to be tested.

Question 6
A 35 year old mother with breast cancer reports that her aunt had developed breast cancer at 48 years
of age. She is keen to clarify the risk of her young daughter developing breast cancer. A blood
sample from the mother is submitted for mutation analysis if the BRCA1 and BRCA2 genes, but no
mutation is found.

What impact does the mutation analysis have on the estimate of the daughter’s risk of developing
breast cancer?

A. The maternal studies place the daughter at low risk of developing breast cancer.
B. The maternal studies do not clarify the daughter’s risk.
C. The maternal studies place the daughter at high risk of developing breast cancer.
D. The daughter’s risk cannot be clarified without studies of her father’s BRCA1 and BRCA2 genes.
E. The daughter’s risk cannot be clarified without studies of her own BRCA1 and BRCA2 genes.
136

Question 7
The carrier frequency for cystic fibrosis in the Caucasian population in Australia is approximately
4%. The ∆F508 mutation accounts for 75% of all mutations among carriers in this population.

A girl is diagnosed with cystic fibrosis. DNA studies identify one ∆F508 mutation but fail to
identify a mutation in the other allele. DNA studies of her unaffected brother are normal.

What is the risk of the brother being a carrier?

A. <1%
B. 4%
C. 25%
D. 50%
E. 66%

Question 8
A patient with a mitochondrial cytopathy is found to have a frameshift mutation in the ND4 gene, a
complex I gene in the mitochondrial genome.

Which of the following statements represents the most likely outcome of this mutation?

A. Polyplasmy with expression limited to females.


B. Homoplasmy with autosomal recessive inheritance.
C. Heteroplasmy with autosomal recessive inheritance.
D. Homoplasmy with mitochondrial inheritance.
E. Heteroplasmy with mitochondrial inheritance.
137

Question 9
A young man has Duchenne muscular dystrophy (DMD). There is no known family history of DMD.
His sister has persistent mild elevation of serum creatine kinase concentration. She also has four
young sons whose clinical status is unknown. The family has been genotyped at two polymorphic
marker loci (M-1 and M-2) which flank the DMD gene. The genotypes of the family are shown in
the table below with the various alleles at the two loci presented as arbitrary codes. Each marker
locus has a recombination fraction of 5% with the DMD gene.

Person M-1 genotype M-2 genotype


Affected man A D
His sister A,B C,D
Sister’s son (1) A C
Sister’s son (2) B C
Sister’s son (3) A D
Sister’s son (4) B D

Which of the sons is/are least likely to develop DMD?

A. Son 1.
B. Son 2.
C. Son 3.
D. Son 4.
E. Son 1 and son 4.

Question 10
In patients with familial Creutzfeldt-Jakob disease, which of the following types of mutation is most
likely to be identified in the prion protein gene?

A. Promoter methylation.
B. Deletion with frameshift.
C. Splice-junction mutation.
D. Missense mutation.
E. Nonsense mutation.
138

Question 11
Some polymorphic loci on the Y chromosome are homologous with polymorphic loci on the X
chromosome. DNA samples from six members of a family (parents and four children) have been
analysed at one of these loci. These family members have DNA fragments of the following lengths
visible on a gel after appropriate polymerase chain reaction (PCR) amplification of this locus.

Fragment length
Person
(in base pairs)
Father 127, 162
Mother 127, 140
Daughter 127, 140
Child 1 140, 162
Child 2 127
Child 3 140

Child 1 Child 2 Child 3


A Female Male Male
B Female Female Male
C Female Male Female
D Male Female Female
E Male Female Male
139

Question 12
A 52-year-old man with no personal or family history of colon cancer, colonic polyps, or
inflammatory bowel disease underwent a colonoscopy for rectal bleeding that showed haemorrhoids
and a 1.5 cm pedunculated polyp at the hepatic flexure that was removed by means of a snare with
cautery. The polyp was a tubulovillous adenoma without high-grade dysplasia. What advice should
be conveyed bout risks to close relatives?

A. He should encourage his first-degree relatives to discuss their family history and screening with
their clinicians.
B. He should encourage his first and second-degree relatives to discuss their family history and
screening with their clinicians.
C. First-degree relatives should begin colonoscopic screening at 40 years of age.
D. First and second-degree relatives should begin colonoscopic screening at 50 years of age.
E. There is no increased risk of colorectal cancer among relatives, and they should undergo standard
population screens.

Question 13
A doctor makes a diagnosis of hypertrophic cardiomyopathy in a 34-year-old woman. She responds
well to medication and returns for regular follow-up appointments.

Which one of the following is the most appropriate action to extend care to the woman’s family?

A. Advise the woman of the genetic basis of hypertrophic cardiomyopathy.


B. Advise the woman that her immediate relatives should seek medical advice regarding their risk of
hypertrophic cardiomyopathy.
C. Contact the relatives regarding the risk of them having hypertrophic cardiomyopathy.
D. Contact the relatives’ doctors and recommend that the relatives be assessed regarding the
possibility of hypertrophic cardiomyopathy.
E. Confirm the familial basis of the diagnosis of hypertrophic cardiomyopathy by DNA testing.
140

Question 14
Presymptomatic genetic testing ("predictive genetic testing") may be justifiable on various medical,
psychological, or economic grounds. Which one of the following disorders provides the best
justification for presymptomatic genetic testing on medical grounds?

Therapeutic Approximate
Premorbid
Disorder interventions penetrance by
phenotype
available age 70 years
100%
A. Huntington disease no no
(chorea)
100%
Familial adenomatous
B. yes yes (colorectal
polyposis
cancer)
Adult-onset polycystic 60%
C. no yes
kidney disease (type I) (renal failure)
Presenilin-associated 60%
D. no no
dementia (dementia)
Hereditary non- 60%
E. yes no
polyposis colon cancer (cancer)
141

Question 15
A 25-year-old person has the karyotype shown below. What is the most likely phenotype of the
external genitalia at this age?

A. Normal male.
B. Normal female.
C. Penile hypoplasia.
D. Clitoral hypertrophy.
E. Testicular atrophy.
142

Question 16
A 55-year-old man develops choreiform movements. He has no personal or family history to
account for these symptoms. DNA studies identify a pathognomonic expansion in IT15, the gene
responsible for Huntington disease. The figure below summarises the results of DNA studies of the
IT15 repeat in the patient and his immediate family. Samples were collected from the patient (lane
1), the patient’s father (lane 2), the patient’s mother (lane 3), and the patient’s healthy sibling (lane
4). The range of sizes of the IT15 repeat in the normal population is shown.

What is the best explanation for the patient’s lack of a family history of Huntington disease?

A. Non-paternity.
B. Expansion of maternal allele.
C. Expansion of paternal allele.
D. Non-penetrance in father.
E. Non-penetrance in sibling.
143

Question 17
The concept that genetic counselling should be nondirective arises mostly from respect for which of
the following principles of medical ethics?

A. Justice.
B. Beneficence.
C. Autonomy.
D. Non-maleficence.
E. Dignity.

Question 18
A number of members of a family are affected with a rare syndrome of external ophthalmoplegia,
Parkinsonism, and early menopause. DNA studies identified multiple deletions of various sizes in
mitochondrial DNA in the affected people. The pedigree is shown below. What is the most likely
mode of inheritance of this syndrome?

A. Polygenic with sex-limited expression.


B. X-linked recessive with manifesting carriers.
C. Autosomal dominant with paternal imprinting.
D. Autosomal dominant without imprinting.
E. Mitochondrial with incomplete penetrance.
144

Question 19
Different mutations in the DMD (Duchenne muscular dystrophy) gene cause three clinically distinct
syndromes in males. The table below lists the three conditions, the degree of skeletal muscle
weakness, and a number of different abnormal genotypes involving the DMD gene.

Which of the following represents the most likely relationship between the genotypes and
phenotypes?

A. Option A.
B. Option B.
C. Option C.
D. Option D.
E. Option E.

Question 20
Presymptomatic genetic testing ("predictive genetic testing") may be justifiable on various medical,
psychological, or economic grounds. Which one of the following disorders provides the best
justification for presymptomatic genetic testing on medical grounds?
145

Question 21
A 47-year-old male has recently learnt that his brother has been diagnosed with Huntington’s disease
(HD). He is unaware of others in his family with HD, however his parents died in a car accident in
their mid-40s. He is examined and found to be neurologically normal. He does not want to be tested
for the HD mutation, however he wishes to know what the chance is that he may develop HD later
in life. The age-related penetrance of HD in a 47-year-old who has inherited the mutation responsible
is 50%,
rising to 100% by 70 years of age.

What is his future risk of developing features of Huntington’s disease?

A. 1 in 1 chance.
B. 1 in 2 chance.
C. 1 in 3 chance.
D. 1 in 4 chance.
E. No chance.

Question 22
A 19-year-old female presents with short stature, neck webbing, wide spaced nipples and primary
amenorrhea. Which of the following is her most likely karyotype result?

A. 48,XXXX
B. 46,XY
C. 47,XXX
D. 45,X
E. 47,XXY
146

Answers

1. B
2. B
3. A
4. D
5. D
6. B
7. D
8. E
9. B
10. D
11. D
12. A
13. B
14. E
15. E
16. C
17. C
18. D
19. B
20. E
21. C
22. D
147

Medical Sciences

Geriatrics

Question 1
In the elderly nursing home population, which one of the following contributes least to constipation?

A. Changes in gastrointestinal transit time.


B. Laxative abuse.
C. Polypharmacy.
D. Poor dietary habits.
E. Sedentary lifestyle.

Question 2
The most common cause of blindness in elderly Caucasians is:

A. Glaucoma.
B. Cataract.
C. Macular degeneration.
D. Diabetes mellitus.
E. Retinal ischaemia.

Question 3
For an 82-year-old nursing home resident who experiences recurrent falls, which one of the
following is least likely to prevent hip fracture?

A. Oestrogen replacement therapy.


B. Bisphosphonates.
C. Calcium supplements.
D. Balance and muscle strengthening exercises.
E. Hip protectors.

Question 4
In the Australasian population over the age of 65 years, the most frequent cause of disability
(defined as needing assistance or experiencing difficulties with self-care, mobility or
communication) is:

A. Dementia.
B. Arthritis.
C. Diabetes.
D. Heart failure.
E. Chronic obstructive pulmonary disease (COPD).
148

Question 5
The life expectancy (in years) for the average Caucasian Australasian male aged 65 is:

A. 5.
B. 7.
C. 10.
D. 15.
E. 25.

Question 6
Which intervention has been best demonstrated to extend maximal life span in animals?

A. Exercise.
B. Antioxidants.
C. Hormonal replacement.
D. Caloric restriction.
E. Immunisation.

Question 7
Which of the following is most characteristic of healthy ageing muscle?

A. Type 1 fiber atrophy.


B. Type 2 fiber atrophy.
C. Decreased capillary density.
D. Decreased oxidative capacity.
E. Intramuscular lipid accumulation.

Question 8
The most common identifiable cause of epilepsy in the elderly is:

A. Cerebrovascular disease.
B. Traumatic injury.
C. Alzheimer’s disease.
D. Tumours.
E. Alcohol.
149

Question 9
The most common cause of chronic incontinence in the elderly is:

A. Stress.
B. Overflow.
C. Urge.
D. Urinary infection.
E. Delirium.

Question 10
A 73-year-old man is referred for assessment of testamentary capacity. There is a past history of
excessive alcohol consumption and hypertension. Over the last six months, he has been more
forgetful and irritable with occasional socially inappropriate behaviour. His speech is fluent and
comprehension normal. There are no focal neurological findings.

Testamentary capacity can be best established by which of the following?

A. Clinical assessment.
B. Neuro-imaging.
C. Formal neuropsychological testing.
D. Interview of family members.
E. Mini-mental state examination.

Question 11
Old age is associated with diminished pharmacodynamic response to beta adrenergic antagonists
(beta blockers). Which of the following mechanisms best explains this?

A. Reduction in the hepatic clearance of beta blockers.


B. Decrease in renal clearance of active metabolites.
C. Decrease in density of beta-receptors.
D. Increase in blood levels of noradrenaline.
E. Decreased intracellular concentrations of cAMP (cyclic adenosine monophosphate).

Question 12
Which of the following is the main risk factor for the development of delirium in older people?

A. Surgery.
B. Dementia.
C. Polypharmacy.
D. Indwelling catheter.
E. Sensory impairment.
150

Question 13
Which class of drugs has been shown to be most closely associated with the risk of falling in older
people?

A. Beta blockers.
B. Selective serotonin re-uptake inhibitors.
C. Non steroidal anti-inflammatory drugs.
D. Calcium channel blockers.
E. Diuretics.

Question 14
Medication withdrawal is frequently undertaken in elderly patients with polypharmacy. Which of the
following medications can be stopped abruptly with minimal risk of a withdrawal syndrome in older
people?

A. Levodopa.
B. Metoprolol.
C. Oxazepam.
D. Sertraline.
E. Amlodipine.

Question 15
The main risk factor for the development of late-onset dementia in people over the age of 75 years is:

A. Homozygosity for apolipoprotein E4 (APOE4).


B. Mild cognitive impairment.
C. Advanced age.
D. Hypertension.
E. Cerebral atrophy on computed tomography (CT) scan.

Question 16
Which of the following conditions is the major contraindication for an older person to be involved in
an exercise programme?

A. Alzheimer’s dementia.
B. Unstable angina pectoris.
C. Atrial fibrillation.
D. Stroke.
E. Chronic obstructive pulmonary disease.
151

Question 17
What is the main factor responsible for systolic hypertension in the elderly?

A. Impaired renal perfusion.


B. High cardiac output.
C. Sympathetic activation.
D. Reduced arterial compliance.
E. High peripheral resistance.
152

Answers

1. A
2. C
3. C
4. B
5. D
6. D
7. B
8. A
9. C
10. A
11. C
12. B
13. B
14. E
15. C
16. B
17. D
153

Clinical Applications

Geriatrics

Question 1
In comparing tricyclic antidepressants to selective serotonin re-uptake inhibitors (SSRIs) as a
treatment for major depression in the elderly, in which one of the following effects do the drugs
differ most?

A. Gastrointestinal disturbance.
B. Dry mouth.
C. Frequency of falls.
D. Time to improvement.
E. Efficacy in treatment of depression.

Question 2
An 88-year-old man with a 10-year history of dementia is brought in by his 85-year-old wife. They
have been happily married for 55 years. The man has a history of falls, prostatism and postural
hypotension. The wife describes her husband’s delusions and accusations over the last six months
about her supposed infidelities with his brother who has been dead for 20 years. The wife is so
distressed by the accusations that she is considering placing him in a nursing home.

The most appropriate initial pharmacological treatment for this man is:

A. Donepezil.
B. Thioridazine.
C. Fluoxetine.
D. Selegiline.
E. Haloperidol.

Question 3
80-year-old man, who lives alone, is brought to you by his daughter who describes him as having a
several-month history of memory loss, language disturbance and urinary incontinence. He has also
been experiencing difficulty paying bills. There has been no change in his personality.

On examination, he is oriented in time and place. There is dysphasia, apraxia and agnosia. His gait
and peripheral neurological examination are normal.

The clinical presentation is most consistent with a diagnosis of:

A. Subdural haematoma.
B. Delirium.
C. Multi-infarct dementia.
D. Alzheimer’s disease.
E. Normal pressure hydrocephalus.
154

Question 4
An 80-year-old woman with no previous psychiatric history presents with a six-month history of
fatigue and loss of energy. She admits to a depressed mood and loss of interest in her usual hobbies.
These feelings are present on most days for most of the day. She has been waking early in the
morning and finds it difficult to return to sleep. She also experiences difficulty concentrating and
feelings of worthlessness. There is no suicidal ideation.

The most likely diagnosis is:

A. Dysthymia.
B. Minor depression.
C. Major depression.
D. Bipolar disorder.
E. Age-related mood change.

Question 5
An 85-year-old cognitively normal woman from a nursing home presents to the Accident and
Emergency Department because of a non-healing venous ulcer 5 cm above her medial malleolus. She
has no symptoms of urinary frequency, dysuria or incontinence. On examination, she is afebrile and
normotensive. Cardiovascular and respiratory examinations are normal. Her abdomen is soft with no
palpable bladder or inguinal lymphadenopathy. The ulcer measures 6 cm by 5 cm and has a sloughy
base but no surrounding cellulitis.

The following results are obtained:


haemoglobin 135 g/L [115-180]
white cell count 4 x 109/L [4-11]
platelet count 200 x 109/L [150-500]

mid-stream urine
microscopy:
leucocytes >100/µL
erythrocytes nil
casts nil
bacteria profuse
culture:
pure growth of Escherichia coli >105 cfu/mL

The most appropriate management of her mid-stream urine result is:

A. Treatment with an appropriate antibiotic.


B. Cranberry juice.
C. Urinary alkalinisation.
D. Treatment with long term antibiotic prophylaxis.
E. No treatment.
155

Question 6
An 85-year-old woman presents with two years of urinary urgency with recent daytime incontinence
but no nocturia. She also has involuntary loss of urine on coughing and laughing. There is no history
of other genito-urinary problems. There are no other significant symptoms. Past history includes
congestive heart failure, glaucoma, reflux oesophagitis and osteoporosis. Current medications are
frusemide, potassium, calcium, ranitidine and timolol eye drops.

Examination reveals mild bilateral pitting oedema but there are no other signs to suggest cardiac
failure.

Pelvic examination reveals a cystocele that descends 2 cm below the urethra with coughing which is
accompanied by the passage of several drops of urine. Rectal and vaginal examination are otherwise
normal. Post-void residual urine volume is 80 mL. Urinalysis and urine microscopy and culture are
normal.

What is the most appropriate next step in the management of this patient?

A. Prescribe oxybutynin hydrochloride.


B. Refer to a gynaecologist for repair of the cystocele.
C. Insertion of a vaginal ring.
D. Long-term prophylactic antibiotics.
E. Pelvic muscle exercises and bladder training techniques.

Question 7
An 83-year-old woman is brought to you by her son. Over the last 12 months she has become
increasingly forgetful and withdrawn and appears to have functionally declined with noticeable
weight loss. Otherwise she is in good general health and is taking no medications. Physical
examination is normal. Computed tomography (CT) scan of the brain is normal.

The most helpful next step in making a diagnosis is to:

A. Perform a magnetic resonance imaging (MRI) scan of the brain.


B. Perform electroencephalography.
C. Test cognitive and psychological function.
D. Measure folate and levels.
E. Perform thyroid function tests.

Question 8
For which of the following conditions is walking exercise most efficacious compared with
weightlifting?

A. Insulin resistance.
B. Depression.
C. Osteoarthritis of the knee.
D. Vascular claudication.
E. Falls.
156

Question 9
Which of the following interventions is most likely to prevent falls in an 80-year-old community-
dwelling woman?

A. Home hazard reduction.


B. Balance and strength training.
C. Antihypertensive medication reduction.
D. Walking daily.
E. A falls education program.

Question 10
An 82-year-old man is admitted with pneumonia and fever. Which one of the following is most
suggestive of the diagnosis of delirium?

A. Inability to stay focused on questions being asked.


B. Worsening behaviour at night.
C. Angry outbursts and claims that the staff are trying to harm him.
D. Mini-Mental Status Examination score of 18/30.
E. Depressed affect.

Question 11
An 86-year-old woman with moderate Alzheimer's disease, including difficulty with language, is
increasingly agitated and believes there are people living in her ceiling, despite ongoing reassurance
to the contrary. The patient lives with her son who seeks your advice regarding management of these
delusions.

Which of the following would be most beneficial?

A. Commence a benzodiazepine.
B. Find alternative accommodation.
C. Isolate and rest the patient in her room.
D. Address factors in the home that exacerbate the delusion.
E. Commence respite in a day care centre.

Question 12
An 82-year-old woman presents to the emergency department with an acute crush fracture of the
thoracic spine. She is discharged home with pain relief.

Which of the following factors is the best predictor of readmission in the next month?

A. Depressed mood.
B. Urinary incontinence.
C. Living alone.
D. Advanced age.
E. Functional impairment.
157

Question 13
An 85-year-old woman who fell in her home one week ago presents for follow-up examination.

At the time of her fall she experienced immediate pain in her buttocks and hips and was brought to
the emergency department, where all X-rays of the hips and pelvis were negative for fracture and she
was sent home.

A week later she continues to complain of severe pain and is unable to weight bear. On examination,
she has tenderness in the buttock area but a good range of motion in the hip joints.

Which of the following is the most likely cause of her pain?

A. Sciatica.
B. Lumbar stenosis.
C. Ischial bursitis.
D. Osteoarthritis of the hip.
E. Pelvic fracture.

Question 14
An 84-year-old female nursing home resident is ambulant but rarely goes outside. She is thin, eats
little and has reflux oesophagitis. Medical history includes mastectomy and radiation therapy for
breast cancer at age 60 years. There is a past history of deep venous thrombosis (DVT). Dual-energy
X-ray absorptiometry (DEXA) scan reveals a t score of -2.2 at the spine and -3.0 at the femoral neck.
Serum calcium is normal.

In addition to calcium supplementation, which of the following is the most appropriate initial therapy
for her osteoporosis?

A. Alendronate.
B. Calcitonin.
C. Vitamin D.
D. Oestrogen.
E. Raloxifene.

Question 15
An 82-year-old man presents with difficulty walking due to osteoarthritis in his right hip. When
prescribing a cane as a walking aide for him, the most appropriate instruction regarding use of the
cane is:

Hold the cane:

A. In the right hand and advance the cane with the right leg.
B. In the right hand and advance the cane with the left leg.
C. In the left hand and advance the cane with the right leg.
D. In the left hand and advance the cane with the left leg.
E. In the right hand and advance the cane after both feet have advanced.
158

Question 16
The daughter of an 80-year-old man phones you because her father seems unwell since breakfast. He
is described as leaning forward, drooling and producing large amounts of saliva. He is coughing but
not short of breath. His background medical history includes severe dementia and stroke disease. The
patient is unable to express what is wrong but points to his sternum. The daughter notices no new
weakness or change in speech.

Which one of the following is the most likely diagnosis?

A. New stroke.
B. Oesophageal foreign body.
C. Gastric ulcer with outlet obstruction.
D. Myocardial infarction.
E. Pneumonia.

Question 17
A 90-year-old male resident of a nursing home has severe dementia. He is bedbound, mute, doubly
incontinent and requires full nursing care. He has a large pressure area over one buttock. He has had
two recent admissions to hospital with aspiration pneumonia and has been placed on thickened fluids
to try to prevent aspiration. He does not have any close family members. Now he does not co-operate
with feeding and when food is placed in his mouth he does not swallow it. In the absence of any
advance directives, the most appropriate approach to the management of his hydration and nutrition
is:

A. Percutaneous endoscopic gastrostomy (PEG) tube feeding.


B. Nasogastric tube feeding.
C. Intravenous fluids.
D. Overnight subcutaneous fluids.
E. Mouth care.

Question 18
An 87-year-old man is admitted to hospital with a dense left hemiplegia and is totally bedbound and
immobile. Which of the following strategies is the most effective for reducing the risk of developing
a pressure ulcer in this patient?

A. Second-hourly turns.
B. Assessment of pressure area risk by Norton Risk Assessment Score.
C. Nutritional supplementation.
D. Indwelling urethral catheterisation.
E. Pressure relieving mattress.
159

Question 19
A 75-year-old woman is brought to a hospital emergency department by her son with whom she
lives. He describes a change in his mother’s mental state over the preceding week. She has been
wandering the house at night and has become uncharacteristically suspicious of her daughter-in-law,
accusing her of stealing her belongings. She also complained to her son that she had seen thieves in
her room at night. She has no prior psychiatric history.

The most likely diagnosis is:

A. Late onset schizophrenia.


B. Psychotic depression.
C. Delirium.
D. Dementia.
E. Temporal lobe epilepsy.

Question 20
Which of the following drugs is least likely to exacerbate urinary retention in an elderly man with
benign prostate hypertrophy?

A. Chlorpromazine.
B. Citalopram.
C. Amitriptyline.
D. Olanzapine.
E. Promethazine.

Question 21
An 81-year-old male has moderate dementia with mild behavioural disturbances, hypertension and
osteoporosis. He is living in a nursing home. His current medications are donepezil, olanzapine,
enalapril, vitamin D and calcium. A random fasting blood glucose is 11.5 mmol/L [<6 mmol/L]. The
next step in the management of this elevated blood glucose level should be:

A. Referral of patient and carer to a dietitian.


B. Commence short acting sulfonylurea.
C. Commence metformin.
D. Cease olanzapine.
E. Cease donepezil.
160

Question 22
A 79-year-old female living in her own home has had falls every week for the last six months and as
a result has fractured her radius and a lumbar vertebra. She has a past history of hypertension,
cataracts, depression and ischaemic heart disease and now mobilizes with a pick up frame. Which of
the following interventions will reduce her future risk of falling the most?

A. Vitamin D supplementation.
B. Withdrawal of antihypertensive medications.
C. Excision of cataracts.
D. Exercise programme.
E. Home visit and modifications by occupational therapist.

Question 23
A 75-year-old female complains of frequent urinary incontinence that occur mostly when bending
over or lifting heavy objects. She has been wearing incontinence pads for nearly one year. The initial
step in the management of her incontinence is:

A. Topic vaginal oestrogen.


B. Oral imipramine.
C. Oral duloxetine.
D. Pelvic muscle training.

Question 24
A 90-year-old female has osteoarthritis and early dementia. She is living at home with her daughter
who is her full time carer. Screening tests on her 90th birthday include an mid stream urine (MSU),
which showed bacteriuria (mixed growth) and pyuria.

Which of the following strategies is most appropriate?

A. Single dose of amoxycillin.


B. Five day course of trimethoprim.
C. Topical estrogen cream.
D. No intervention.
E. Repeat MSU in three months.

Question 25
The most common reason for reduced oral intake in people with endstage dementia is:

A. Impaired swallowing.
B. Deliberate self harm.
C. Physical inactivity.
D. Increased metabolic rate.
E. Dental disease.
161

Question 26
An 82-year-old female complains of a decline in short-term memory, specifically difficulty
remembering some names and telephone numbers. She lives alone in her own home and has been
independent in all activities of daily living and instrumental activities of daily living, since the death
of her husband one year previously. Her mini-mental state examination (MMSE) score is 27 out of
30, with short-term recall of only one out of three items.

The most likely diagnosis is:

A. Early vascular dementia.


B. Early Alzheimer’s dementia.
C. Mild cognitive impairment.
D. Grief reaction.
E. Normal aging.
162

Answers

1. B
2. E
3. D
4. C
5. E
6. E
7. C
8. D
9. B
10. A
11. D
12. E
13. E
14. C
15. C
16. B
17. E
18. E
19. C
20. B
21. D
22. D
23. D
24. D
25. C
26. C
163

Medical Sciences

Haematology

Question 1
Graft-versus-host disease is a major complication of allogeneic stem cell transplantation for
haematological malignancy in adults.

Which one of the following has the least influence on the risk of developing graft-versus-host
disease?

A. Age of recipient.
B. Donor-recipient ABO blood group incompatibility.
C. In vitro T cell depletion of the graft.
D. Type of HLA-matched donor (sibling versus unrelated).
E. Donor-recipient HLA mismatch.

Question 2
A 19-year-old woman presents with recent-onset of right leg swelling and pleuritic chest pain and is
found to have iliofemoral thrombosis on Doppler ultrasound examination. Results of a ventilation-
perfusion lung scan indicate a high probability for pulmonary emboli. She had been taking the oral
contraceptive pill for the last three years but has now ceased. She is a non-smoker. There is no
significant medical history and no known family history of venous thromboembolism.

Which one of the following investigations for an underlying hypercoagulable state is most likely to
be affected by the presence of the extensive thrombosis?

A. Antiphospholipid antibody screen.


B. Antithrombin level.
C. Factor V and prothrombin genotype assessment.
D. Activated partial thromboplastin time (APTT).
E. Full blood count including blood film.
164

Question 3
A 32-year-old woman, who has recently migrated from India, is breastfeeding her third child and
presents with lethargy and a history of recurrent chest infection. She has a vegetarian diet. Physical
examination shows her to be anaemic with no site of blood loss clinically evident and no other
abnormalities.

Investigations show:

Full blood count


haemoglobin 64 g/L [120-155]
mean corpuscular volume (MCV) 68 fL [80-95]
white cell count 7.4 x 109/L [3.5-9.5]
differential normal
platelet count 412 x 109/L [130-330]

Blood film is shown below.

With respect to the patient’s anaemia, which one of the following laboratory results has the most
diagnostic value?

A. A lower than normal serum iron level.


B. A lower than normal serum ferritin level.
C. A higher than normal serum transferrin level.
D. A negative urinary haemosiderin test.
E. A normal plasma haemoglobin A2 level.
165

Question 4
A patient undergoes a Schilling test because of vitamin B12 deficiency. The following results are
obtained:

With antibiotics/
Without intrinsic factor With intrinsic factor Without intrinsic
factor
Vitamin B12
low normal low
absorption

The most likely pathology is:

A. Small bowel bacterial overgrowth.


B. Pernicious anaemia.
C. Partial (Billroth II) gastrectomy.
D. Terminal ileal Crohn’s disease.
E. Short bowel syndrome.

Question 5
A 48-year-old previously well man is receiving a transfusion of four units of packed red cell
concentrate via a peripheral intravenous line after presenting with melaena secondary to a bleeding
duodenal ulcer. He has not been previously transfused. Admission biochemistry (including urea,
electrolytes and creatinine) is normal and pre-transfusion blood counts are normal apart from
anaemia (haemoglobin 64 g/L [135-170]). Less than five minutes after transfusion of the third unit of
blood is commenced, he complains of feeling very unwell and abruptly develops fever, chills, rigors
and profound hypotension.

In the absence of ABO, Rhesus or minor blood group donor-recipient incompatibility, the most likely
diagnosis is:

A. Bacterial contamination of the transfused blood.


B. Transfusion-associated graft-versus-host disease.
C. Electrolyte disturbance.
D. Citrate toxicity.
E. Reaction to plasticiser in infusion bag.

Question 6
Graft-versus-host disease is a major complication of allogeneic stem cell transplantation for
haematological malignancy in adults. Which one of the following has the least influence on the risk
of developing graft-versus-host disease?

A. Age of recipient.
B. Donor-recipient ABO blood group incompatibility.
C. In vitro T cell depletion of the graft.
D. Type of HLA-matched donor (sibling versus unrelated).
E. Donor-recipient HLA mismatch.
166

Question 7
A 32-year-old man presents with weight loss, splenomegaly 20 cm below the left costal margin and a
white cell count of 480.0 x 109/L [3.5-9.5]. Bone marrow examination confirms the diagnosis of
chronic myeloid leukaemia in chronic phase.

Which one of the following treatment options offers this man the best chance of long-term disease-
free survival?

A. Interferon.
B. Interferon and cytarabine.
C. Hydroxyurea.
D. Autologous bone marrow transplantation.
E. Allogeneic bone marrow transplantation.

Question 8
Which one of the following bone marrow cell populations has the highest proportion of its cells in
active cell cycle?

A. Pluripotential stem cell.


B. Myeloblast.
C. Myelocyte.
D. Metamyelocyte.
E. Reserve granulocyte pool.

Question 9
A 42-year-old previously healthy woman presents with an above-knee deep venous thrombosis of the
right leg complicated by probable pulmonary thrombo-embolism.

She has no clinical risk factors such as recent surgery or trauma.


In the investigation of her thrombo-embolic disease, which one of the following is the most likely
abnormality to be found?

A. Lupus anticoagulant.
B. Protein C deficiency.
C. Protein S deficiency.
D. Dysfibrinogenaemia.
E. Factor V Leiden.
167

Question 10
A 68-year-old man scheduled for herniorrhaphy presents for pre-anaesthetic assessment and is found
to have non-tender cervical lymphadenopathy.

Full blood examination shows:


haemoglobin 125 g/L [128-175]
mean cell volume 96 fL [80-97]
white cell count 19.5 x 109/L [3.9-12.7]
neutrophils 2.5 x 109/L [1.9-8.0]
lymphocytes 16.2 x 109/L [0.9-3.3]
monocytes 0.3 x 109/L [0.3-1.1]
eosinophils 0.4 x 109/L [0-0.5]
basophils 0.1 x 109/L [0-0.1]
platelet count 213 x 109/L [150-396]

The blood film is shown below:

With respect to the peripheral blood lymphoid cell immunophenotype, which of the following
surface antigens is most likely to be co-expressed with CD19 (cluster of differentiation 19)?

A. CD3.
B. CD5.
C. CD8.
D. CD10.
E. CD34.
168

Question 11
A 69-year-old woman commences a chronic transfusion program for anaemia secondary to
myelodysplasia. She experiences a non-haemolytic febrile transfusion reaction during each of two
red cell transfusions administered four weeks apart.

Which one of the following reasons best explains the benefit of using filtered blood to prevent
further reaction? Reduced exposure to:

A. Donor leucocytes.
B. Donor cytokines.
C. Bacterial antigen.
D. Cytomegalovirus (CMV).
E. Donor isohaemagglutinins.

Question 12
In a 50-year-old woman, which of the following isolated laboratory abnormalities would be
associated with the highest level of blood viscosity?

A. Serum immunoglobulin M (IgM) level of 40.0 g/L [0.5-3.0].


B. Peripheral blood leucocytosis of 25.0 x 109/L [3.9-12.7].
C. Haematocrit of 0.46 [0.32-0.42].
D. Serum immunoglobulin G (IgG) level of 40.0 g/L [7.5-15.6].
E. Peripheral blood thrombocytosis of 500 x 109/L [150-396].

Question 13
Infusion of cryoprecipitate is principally indicated for the replacement of:

A. Factor IX.
B. Fibrinogen.
C. Immunoglobulin A (IgA).
D. Anti-thrombin.
E. Protein C.
169

Question 14
A previously well 32-year-old man presents with progressive lethargy and no other symptoms.
Physical examination shows him to have an enlarged spleen 6 cm below the left costal margin. There
is no jaundice or lymphadenopathy and he is not clinically anaemic.

The full blood examination shows:


haemoglobin 125 g/L [128-175]
mean cell volume 95 fL [80-97]
white cell count 85.5 x 109/L [3.9-12.7]
platelet count 525 x 109/L [150-396]

The blood film is shown below:

The most likely marrow cytogenetic abnormality is:

A. Hyperdiploidy.
B. t(9;22).
C. t(15;17).
D. 5q minus.
E. Trisomy 21.

Question 15
Megaloblastosis is least likely to be seen in which one of the following?

A. Transcobalamin I deficiency.
B. Pernicious anaemia.
C. Strict veganism.
D. Coeliac disease.
E. Post-gastrectomy.
170

Question 16
Which intervention has been best demonstrated to extend maximal life span in animals?

A. Exercise.
B. Antioxidants.
C. Hormonal replacement.
D. Caloric restriction.
E. Immunisation.

Question 17
A 67-year-old woman develops symptomatic anaemia and jaundice nine days after subtotal
colectomy for adenocarcinoma of the colon, at the time of which she received a transfusion of 3 units
of red cells. She is on therapy (which includes sulfasalazine) for ulcerative colitis. Review of the
pre-operative antibody screen shows this to be negative for red cell alloantibodies.
Her current full blood examination includes the following:
haemoglobin 82 g/L [120-155]
mean cell volume 99 fL [80-97]
9
reticulocyte count 162 x 10 /L [8-104]

Her blood film is shown below:

The most likely cause of her anaemia is:

A. Iron deficiency.
B. Peri-operative blood loss.
C. Red cell aplasia.
D. Delayed haemolytic transfusion reaction.
E. Megaloblastosis.
171

Question 18
Which of the following findings is most specific for a diagnosis of pernicious anaemia?

A. A low serum vitamin B12 level.


B. An elevated fasting homocysteine level.
C. An elevated serum gastrin level.
D. A positive gastric parietal cell antibody.
E. A positive intrinsic factor antibody.

Question 19
During normal haemostasis, von Willebrand factor (vWF) plays a role in platelet
adhesion/aggregation. During this process, which of the following is the primary binding site of vWF
to platelets?

A. Collagen receptor.
B. Platelet glycoprotein Ib-IX-V.
C. Adenosine diphosphate receptor.
D. Platelet glycoprotein IIb-IIIa.
E. Ristocetin.

Question 20
Intravenous infusion of which of the following blood components is most likely to be associated with
a septic transfusion reaction?

A. Red cell concentrate.


B. Fresh frozen plasma.
C. Pooled gammaglobulin.
D. Platelet concentrate.
E. Cryoprecipitate.

Question 21
During normal haemostasis, von Willebrand factor (vWF) plays a role in platelet
adhesion/aggregation. During this process, which of the following is the primary binding site of
vWF to platelets?

A. Collagen receptor.
B. Platelet glycoprotein Ib-IX-V.
C. Adenosine diphosphate receptor.
D. Platelet glycoprotein IIb-IIIa.
E. Ristocetin.
172

Question 23
Which of the following is most characteristic of a patient with haemophilia A with 8% baseline
factor VIII activity?

A. Spontaneous cutaneous purpura.


B. Spontaneous gastrointestinal haemorrhage.
C. Spontaneous deep muscle bleeds.
D. Spontaneous haemarthroses.
E. No spontaneous bleeding.

Question 24
Which of the following is the primary mechanism of body iron regulation?

A. Shedding of duodenal enterocytes.


B. Regulation of enterocyte surface expression of transferrin receptor (TfR1).
C. Regulation of enterocyte ferritin content.
D. Renal haemosiderin excretion.
E. Alteration of Kuppfer cell iron metabolism gene expression.

Question 25
Which of the following findings is most specific for the diagnosis of pernicious anaemia?

A. Increased serum homocysteine.


B. Increased serum methylmalonate.
C. Positive gastric parietal cell antibodies.
D. Increased serum gastrin.
E. Positive intrinsic factor antibodies.

Question 26
Which of the following is the most common mechanism of a Febrile Non-Haemolytic Transfusion
reaction (FNHTR) to transfusion of packed red cells?

A. Bacterial contamination of donor product.


B. Immediate hypersensitivity to proteins of donor origin.
C. Interaction between donor antibodies with recipient granulocytes.
D. Interaction between donor granulocytes with recipient antibodies.
E. Interaction between donor red cells with recipient red cell antibodies.
173

Question 27
Which of the following is the greatest cause of mortality occurring more than 100 days following
allogeneic peripheral blood cell stem transplantation for acute leukaemia?

A. Hepatic veno-occlusive disease.


B. Graft-versus-host disease.
C. Transplantation-related lung injury.
D. Transplantation-related infection.
E. Recurrent leukaemia.

Question 28
Which of the following best describes the mechanism of action of the hirudin class of anticoagulant
drugs (lepirudin, bivalirudin)?

A. Direct thrombin inhibition.


B. Factor Xa inhibition.
C. Cycloxygenase antagonism.
D. Antithrombin-mediated thrombin inhibiton.
E. Inhibition of prothrombin g-carboxylation.

Question 29
Hepcidin and ferroportin regulate systemic iron absorption by inhibition of:

A. enterocyte brush border iron reduction.


B. enterocyte apical iron uptake.
C. iron oxidation by hephaestin.
D. enterocyte basolateral iron release.
E. iron binding to transferrin.

Question 30
Which of the following transfusion reactions is leucodepletion most effective in preventing?

A. Febrile non-haemolytic transfusion reaction.


B. Transfusion associated graft versus host disease.
C. Anaphylaxis.
D. Transfusion related acute lung injury.
E. Delayed haemolytic transfusion reaction.
174

Question 31
The following blood film showing acanthocytes and Howell-Jolly bodies is taken from a patient with
coeliac disease.

Which of the following is the most likely explanation for this appearance?

A. Malnutrition.
B. Hyposplenism.
C. Iron deficiency.
D. Hypothyroidism.
E. Folate deficiency.
175

Answers

1. B
2. B
3. B
4. B
5. A
6. B
7. E
8. B
9. E
10. B
11. A
12. A
13. B
14. B
15. A
16. D
17. D
18. E
19. B
20. D
21. B
22. D
23. E
24. A
25. E
26. D
27. E
28. A
29. D
30. A
31. B
176

Clinical Applications

Haematology

Question 1
A 42-year-old woman presents with several weeks of lethargy and is found to be clinically anaemic
and mildly icteric. There is no history or clinical evidence of overt blood loss. She has a normal
varied diet and is on no medication.

Her full blood count shows:


haemoglobin 67 g/L [120-155]
mean corpuscular volume (MCV) 129 fL [80-95]
white cell count 3.1 x 109/L [3.5-9.5]
neutrophils 1.4 x 109/L [1.5-6.0]
lymphocytes 0.9 x 109/L [0.7-3.2]
monocytes 0.5 x 109/L [0.2-0.6]
eosinophils 0.2 x 109/L [0-0.4]
basophils 0.1 x 109/L [0.1-0.2]
platelet count 98 x 109/L [130-330]
reticulocyte count 4 x 109/L [8-104]

The blood film is shown in the photograph below.

The most likely diagnosis is:

A. Pernicious anaemia.
B. Evans’ syndrome (autoimmune haemolytic anaemia and thrombocytopenia).
C. Aplastic anaemia.
D. Hypothyroidism.
E. Alcoholic liver disease.
177

Question 2
A 45-year-old woman presents with progressive lethargy over 10 days. There is no history
suggestive of infection. There has been no previous medication.

Initial investigations show:


haemoglobin 75 g/L [120-155]
mean corpuscular volume (MCV) 128 fL [80-95]
white cell count 10.5 x 109/L [3.5-9.5]
platelet count 356 x 109/L [130-330]
bilirubin 51 µmol/L [<15]
lactate dehydrogenase (LDH) 753 U/L [<250]
direct antiglobulin test C3b+++ (strongly positive)

Which one of the following is most likely to be present on the blood film?

A. Plasma cells.
B. Red cell autoagglutination.
C. Red cell fragmentation.
D. Hypersegmented neutrophils.
E. Target cells.
178

Question 3
A 65-year-old man with a history of osteoarthritis and mild chronic obstructive pulmonary disease
presents with several days of nausea and vomiting and is now confused, febrile and icteric. He has
extensive bruising and some oral mucosal bleeding. His medications are intermittent piroxicam and a
salbutamol inhaler.

His full blood count shows:


haemoglobin 85 g/L [120-155]
mean corpuscular volume (MCV) 101 fL [80-95]
white cell count 10.2 x 109/L [3.5-9.5]
neutrophils 8.5 x 109/L [1.5-6.0]
lymphocytes 1.2 x 109/L [0.7-3.2]
monocytes 0.4 x 109/L [0.2-0.6]
eosinophils 0.1 x 109/L [0-0.4]
platelet count 14 x 109/L [130-330]
reticulocyte count 125 x 109/L [8-104]

Blood film as shown in the photograph below.

Direct Coombs' test - negative

Coagulation studies:
prothrombin time-international normalised ratio (PT-INR) normal
activated partial thromboplastin time (APTT) normal
fibrinogen normal
D-dimers normal
Biochemistry:
creatinine 0.29 mmol/L [0.07-0.13]
urea 30.0 mmol/L [2.0-8.5]
bilirubin 56 µmol/L [<21]
lactate dehydrogenase (LDH) 950 U/L [<250]

The most likely diagnosis is:

A. Gram-negative sepsis.
B. Leptospirosis.
C. Disseminated intravascular coagulation.
D. Thrombotic thrombocytopenic purpura.
E. Piroxicam toxicity.
179

Question 4
A 65-year-old woman presents for inguinal hernia repair. She is otherwise well, apart from several
years of hypertension, currently controlled by captopril. There is no history of excessive infection.
Physical examination reveals borderline cervical lymphadenopathy.

A pre-operative full blood count shows:


haemoglobin 125 g/L [120-155]
mean corpuscular volume (MCV) 89 fL [80-95]
white cell count 16.7 x 109/L [3.5-9.5]
neutrophils 4.5 x 109/L [1.5-6.0]
lymphocytes 11.2 x 109/L [0.7-3.2]
monocytes 0.5 x 109/L [0.2-0.6]
eosinophils 0.4 x 109/L [0-0.4]
basophils 0.1 x 109/L [0.1-0.2]
platelet count 199 x 109/L [130-330]

The blood film is shown in the photograph below.

The most appropriate initial investigation for diagnosis is:

A. A computed tomography (CT) scan of the neck, chest, abdomen and pelvis.
B. Epstein-Barr virus serology.
C. Peripheral blood cytogenetic analysis.
D. Bone marrow examination.
E. Immunophenotypic analysis of peripheral blood mononuclear cells.
180

Question 5
A 58-year-old previously well man presents with a two-week history of spontaneous bruising and a
large, painful left thigh haematoma following minimal trauma. He is on no medication and his blood
count has been documented as normal 18 months earlier prior to elective surgery. There is no
previous history of bleeding with trauma or with surgery and no family history of bleeding disorder.
Physical examination is normal apart from scattered ecchymoses and the swollen, bruised left thigh.
The haematoma is confirmed on ultrasound examination.

Investigations show:

Full blood count


haemoglobin 119 g/L [135-170]
other parameters normal

Prothrombin time-international normalised ratio (PT-INR) 1.1 [1.0-1.3]


Activated partial thromboplastin time (APTT) 90 seconds [26-38]
APTT 1:1 mix, normal plasma:patient plasma 50 seconds [26-38]
Fibrinogen 2.7 g/L [2.0-4.0]
D-dimer 0.2 mg/L [<0.2]
platelet function tests (aggregometry) normal

The most likely diagnosis is:

A. Haemophilia A.
B. Haemophilia B.
C. Primary antiphospholipid antibody syndrome.
D. Von Willebrand’s disease.
E. Acquired factor VIIIc inhibitor.

Question 6
A couple comes to see you because they are wanting to start a family. There is a vague history of
thalassaemia in both families. Which one of the following combinations would cause the least
concern to this couple?

The woman is a carrier for: The man is a carrier for:


A -thalassaemia - thalassaemia
B -thalassaemia - thalassaemia
C. -thalassaemia -thalassaemia
D haemoglobin E -thalassaemia
E -thalassaemia -thalassaemia

Legend: = beta
= absent alpha chain production
= decreased alpha chain production
181

Question 7
A 24-year-old woman who has recently arrived in Australia from Vietnam, presents for evaluation of
abnormal menstrual bleeding. There are no abnormalities on examination.
Investigations show:
Full blood count
Haemoglobin 113 g/L [120-155]
red cell count 5.2 x 1012/L [4.1-5.2]
mean corpuscular volume (MCV) 71 fL [80-95]
mean corpuscular haemoglobin (MCH) 22.0 pg [27.0-32.5]
mean corpuscular haemoglobin concentration (MCHC) 310 g/L [325-360]
white cell count 6.6 x 109/L [3.5-9.5]
differential normal
platelet count 212 x 109/L [130-330]

Blood film shows red cell microcytosis and hypochromasia but is otherwise normal.
Haemoglobin (Hb) electrophoresis (cellulose acetate, pH 8.6):

HbA2 2.7% [1.8-3.5]


HbF 0.4% [0-2.0]
No abnormal bands

HbH preparation: HbH inclusions present

Serum biochemistry:
Iron 8 µmol/L [7-32]
transferrin 3.2 g/L [2.1-3.6]
ferritin 15 µg/L [7-280]

The most likely diagnosis is:

A. Homozygous alpha+ thalassaemia.


B. Early iron deficiency.
C. Congenital sideroblastic anaemia.
D. Sickle cell anaemia.
E. Heterozygous beta thalassaemia.
182

Question 8
A 78-year-old man presents with progressive lethargy for several months and the recent onset of
bruising, epistaxis, headaches and visual deterioration. Physical examination shows him to be
anaemic with scattered ecchymoses; he has reduced visual acuity bilaterally and fundoscopy reveals
retinal venous distension with numerous retinal haemorrhages.

Investigations show:

Full blood count


haemoglobin 72 g/L [135-170]
mean corpuscular volume (MCV) 101 fL [80-95]
white cell count 4.0 x 109/L [3.5-9.5]
neutrophils 1.2 x 109/L [1.5-6.0]
lymphocytes 1.9 x 109/L [0.7-3.2]
eosinophils 0.4 x 109/L [0-0.4]
monocytes 0.5 x 109/L [0.2-0.6]
platelet count 90 x 109/L [130-330]

Blood film shows mild macrocytosis with marked rouleaux formation, neutropenia and occasional
atypical lymphocytes.

Coagulation studies:
prothrombin time-international normalised ratio (PT-INR) 1.5 [1.0-1.3]
activated partial thromboplastin time (APTT) 45 seconds [26-38]

Biochemistry:
Urea 12.5 mmol/L [2.0-8.5]
Creatinine 0.15 mmol/L [0.07-0.13]
liver function tests and calcium level normal
protein electrophoresis
hypogammaglobulinaemia
IgM kappa paraprotein 60 g/L

The most appropriate initial therapy is:

A. Melphalan and prednis(ol)one.


B. Platelet transfusion.
C. Plasma exchange.
D. Renal dialysis.
E. Intravenous gamma globulin.
183

Question 9
A 62-year-old previously well woman is referred for evaluation of recurrent mouth ulceration and
several bacterial skin and urinary tract infections over 12 months, with one episode requiring
parenteral antibiotic therapy. She is on no medication apart from intermittent antibiotics. Physical
examination shows some mouth ulcers and pustular skin lesions. Full blood count (as follows) is
essentially unchanged compared with previous counts measured three times during the preceding few
weeks.

Investigations show:
Full blood count
haemoglobin 127 g/L [120-155]
mean corpuscular volume (MCV) 89 fL [80-95]
white cell count 2.9 x 109/L [3.5-9.5]
differential:
neutrophils 0.2 x 109/L [1.5-6.0]
lymphocytes 1.5 x 109/L [0.7-3.2]
monocytes 0.7 x 109/L [0.2-0.6]
eosinophils 0.4 x 109/L [0-0.4]
basophils 0.1 x 109/L [0.1-0.2]
platelet count 171 x 109/L [130-330]

The blood film is unremarkable except for severe neutropenia.

Antinuclear and rheumatoid factors are negative and a marrow examination is unremarkable except
for left-shifted granulopoiesis.

The most appropriate therapy is:

A. Granulocyte colony stimulating factor (G-CSF).


B. Lithium.
C. Intravenous gamma globulin.
D. Granulocyte transfusions from a family member.
E. Long-term prophylactic antibiotics.
184

Question 10
A 62-year-old woman underwent an elective cholecystectomy. Routine full blood examination
revealed:

haemoglobin 117 g/L [120-155]


white cell count 86.4 x 109/L [3.5-9.5]
neutrophils 1.75 x 109/L [1.50-7.50]
lymphocytes 81.2 x 109/L [ [1.5-4.0]
platelet count 165 x 109/L [130-330]

Physical examination revealed no lymphadenopathy and no hepatosplenomegaly. The woman was


asymptomatic. Flow cytometry revealed that the lymphocytosis marked as CD5+, CD19+ and
CD23+.

Which one of the following is the most appropriate management option?

A. Observation alone.
B. Prednis(ol)one therapy.
C. Chlorambucil therapy.
D. Combination chemotherapy.
E. Autologous bone marrow transplantation.

Question 11
A 45-year-old woman presents with progressive lethargy over 10 days. There is no history
suggestive of infection. There has been no previous medication.
Initial investigations show:

haemoglobin 75 g/L [120-155]


mean corpuscular volume (MCV) 128 fL [80-95]
white cell count 10.5 x 109/L [3.5-9.5]
platelet count 356 x 109/L [130-330]
bilirubin 51 µmol/L [<15]
lactate dehydrogenase 753 U/L [<250]
direct antiglobulin test C3b+++ (strongly positive)

Which one of the following is most likely to be present on the blood film?

A. Plasma cells.
B. Red cell autoagglutination.
C. Red cell fragmentation.
D. Hypersegmented neutrophils.
E. Target cells.
185

Question 12
A 74-year-old ex-smoking male with a history of ischaemic heart disease presents with a hip fracture
requiring surgical fixation within a few days. He is plethoric and there are no specific respiratory or
cardiac abnormalities on examination. Chest X-ray is normal and ECG shows left bundle branch
block.

His full blood count shows:


haemoglobin 219 g/L [120-155]
haematocrit 0.69 [0.40-0.49]
mean corpuscular volume (MCV) 78 fL [80-95]
red cell count 7.5 x 1012/L [4.0-5.7]
white cell count 13.4 x 109/L [3.5-9.5]
neutrophils 9.5 x 109/L [1.5-6.0]
lymphocytes 2.3 x 109/L [0.7-3.2]
monocytes 0.8 x 109/L [0.2-0.6]
eosinophils 0.6 x 109/L [0.0-0.4]
basophils 0.2 x 109/L [0.1-0.2]
platelet count 412 x 109/L [130-330]

The serum erythropoietin level will not be available for six weeks.
Which one of the following would be the most appropriate initial therapy?

A. Venesection.
B. Intravenous radioactive phosphorus.
C. Hydroxyurea.
D. Therapeutic anticoagulation.
E. Interferon
186

Question 13
A 26-year-old previously well woman on no medication presents with the recent-onset of easy
bruising. She has two small oral mucosal haemorrhages, scattered bruises and petechiae around the
ankles but no other abnormalities on examination.

Human immunodeficiency virus (HIV) antibody test is negative.

The full blood count shows:


Haemoglobin 119 g/L [120-155]
mean corpuscular volume (MCV) 83 fL [80-95]
white cell count 4.9 x 109/L [3.5-9.5]
differential: normal
platelet count 16 x 109/L [130-330] (confirmed on blood film)

Prothrombin time-international normalised ratio (PT-INR) and activated partial thromboplastin time
(APTT) are normal and bone marrow examination is also normal except for an increase in
megakaryocytes.

The most appropriate initial therapy is:

A. Intravenous gamma globulin.


B. Oral tranexamic acid.
C. Platelet transfusion.
D. Oral prednis(ol)one.
E. Oral danazol.

Question 14
A 24-year-old woman is referred after three months of warfarin therapy following presentation with
hemiplegia due to a thrombotic stroke, from which she has fully recovered. Investigations prior to
commencing warfarin revealed a positive antinuclear antibody (ANA) test at titre 1:40, speckled
pattern with negative anti-ENA (anti extractable nuclear antigen) and anti-DNA antibody tests. IgG
anticardiolipin antibody and lupus anticoagulant tests are strongly positive. Protein S and protein C
levels are normal as is Factor V DNA analysis.

Which one of the following represents optimal management?

A. Cessation of warfarin.
B. Cessation of warfarin and commencement of aspirin 100 mg/day indefinitely.
C. Continuation of warfarin for a further three months, then commencement of aspirin 100 mg/day
indefinitely.
D. Continuation of warfarin for a further nine months then commencement of aspirin 100 mg/day
indefinitely.
E. Continuation of warfarin indefinitely.
187

Question 15
A 47-year-old man with severe haemophilia A and a high titre factor VIII inhibitor requires hip
replacement surgery. The peri-operative replacement coagulation product of choice is:

A. Recombinant activated factor VII.


B. Recombinant factor VIII.
C. Recombinant factor IX.
D. Fresh frozen plasma.
E. Cryoprecipitate.

Question 16
An 80-year-old woman with longstanding atrial fibrillation and hypertension is referred for a second
opinion on further management. She has been on metoprolol and started warfarin a month ago. She is
asymptomatic.

On examination, she has an apex rate of 60/minute and blood pressure of 136/84 mmHg. She has no
signs of cardiac failure. An ECG confirms atrial fibrillation. A chest X-ray shows cardiomegaly with
a cardiothoracic ratio of 14.5/28 but clear lung fields. Echocardiography demonstrates left ventricular
hypertrophy and diastolic dysfunction. Systolic function is preserved with fractional shortening of
28%.

Atrial dimensions are normal.

Which of the following long-term management strategies is most appropriate?

Attempt elective direct current (DC) Anti-arrythmic


Thromboprophylaxis
cardioversion therapy
A. Yes Continue warfarin* Beta blocker
B. Yes Change to aspirin* Beta blocker
C. Yes Change to aspirin* Amiodarone
D. No Continue warfarin Beta blocker
E. No Continue warfarin Amiodarone
* If sinus rhythm present after cardioversion

Question 17
In adult Philadelphia-chromosome-positive chronic myeloid leukaemia (in chronic phase), treatment
with which of the following agents is associated with the greatest likelihood of achieving a complete
cytogenetic response?

A. Hydroxyurea.
B. Radioactive phosphorus.
C. Interferon.
D. Cytosine arabinoside.
E. Imatinib.
188

Question 18
A 72-year-old man with a history of chronic obstructive pulmonary disease (COPD) presents with an
intercurrent lower respiratory tract infection. Apart from plethora and pulmonary signs associated
with his respiratory disease, no other abnormalities are found.

Full blood examination shows:

haemoglobin 199 g/L [128 – 175]


haematocrit 0.59 [0.36 – 0.50]
mean cell volume 82fL [80 – 97]
white cell count 13.5 x 109/L [3.9 – 12.7]
neutrophils 10.1 x 109/L [1.9 – 8.0]
lymphocytes 0.9 x 109/L [0.9 – 3.3]
monocytes 1.9 x 109/L [0.3 – 1.1]
eosinophils 0.5 x 109/L [0 – 0.5]
basophils 0.1 x 109/L [0 – 0.1]
platelet count 405 x 109/L [150 – 396]
red cell mass 41mL/kg [28 – 35]
plasma volume 44mL/kg [40 – 50]
serum erythropoietin 34.0 IU/L [4.4 – 26.4]

The most likely cause of the elevated haematocrit is:

A. Dehydration.
B. Chronic hypoxaemia.
C. Polycythaemia vera.
D. Spurious polycythaemia.
E. Low affinity haemoglobin.

Question 19
Which of the following is most likely to prevent recurrent febrile non-haemolytic transfusion
reactions in a patient requiring regular transfusion?

A. Warm blood to 37°C.


B. Leucodepletion.
C. Gamma-irradiation.
D. Cytomegalovirus (CMV)-negative donors.
E. Slow transfusion time.
189

Question 20
A 28-year-old man presents with a fractured ankle requiring surgical fixation.

Full blood examination shows:


haemoglobin 115 g/L [128 - 175]
red cell count 5.6 x 1012/L [4.0 – 5.7]
mean cell count 62 fL [80 - 97]
white cell count 12.5 x 109/L [3.9 – 12.7]
(mild neutrophilia)
platelet count 390 x 109/L [150 - 396]
serum ferritin 95 µg/L [15 - 325]
Haemoglobin (Hb) studies show:
HbA2 5.2% [1.8 – 3.5]
HbF 1.2% [0 – 2.0]
HbH preparation No HbH bodies seen
Hb electrophoresis No abnormal Hb bands seen

The most likely explanation for his anaemia is:

A. ß-thalassaemia trait.
B. Chronic blood loss.
C. Anaemia of ‘chronic disease’.
D. Congenital sideroblastosis.
E. Sickle cell trait.
190

Question 21
A previously well 46-year-old woman presents acutely unwell with right iliac fossa pain and fever. A
laparotomy is planned.

The following pre-operative results are obtained:

haemoglobin 124 g/L [113-159]


white cell count 15.3 x 109/L [3.9 – 12.7]
platelet count 150 x 109/L [150 - 396]

activated partial thromboplastin time 56 seconds [26 - 38]


APTT mixing study
(1.1 mix, patient plasma with normal 48 seconds
plasma)
prothrombin time – international 1.2 [1.0 – 1.3]
normalised ration (PT-INR)
fibrinogen 4.5g/L [2.0 – 4.0]
D-dimer 0.2 [<0.2]
thrombin clotting time (TCT) 18 seconds [<24]

Which of the following is the most likely cause of the prolonged APTT?

A. Disseminated intravascular coagulation.


B. Heparin contamination.
C. Lupus inhibitor.
D. Factor XI deficiency.
E. Haemophilia carrier.
191

Question 22
A 16-year-old adolescent male presents with lethargy and lower respiratory tract infection. Physical
examination shows him to be febrile, icteric and pale in addition to respiratory findings.

Full blood examination shows:


haemoglobin 67 g/L [128-175]
mean cell volume 86 fL [80-97]
white cell count 13.0 x 109/L [3.9-12.7]
neutrophils 10.2 x 109/L [1.9-8.0]
lymphocytes 0.8 x 109/L [0.9-3.3]
monocytes 1.6 x 109/L [0.3-1.1]
eosinophils 0.3 x 109/L [0-0.5]
basophils 0.1 x 109/L [0-0.1]
platelet count 390 x 109/L [150-396]
reticulocyte count 213 x 109/L [9-116]

The blood film is shown below.

The most likely cause of the anaemia is:

A. Paroxysmal cold haemoglobinuria.


B. Glucose-6-phosphate dehydrogenase (G6PD) deficiency.
C. Red cell aplasia.
D. Cold agglutinin haemolysis.
E. Sickle cell anaemia.
192

Question 23
A previously well 37-year-old man presents with lethargy and easy bruising over several weeks.
Physical examination is unremarkable apart from occasional bruises.

His full blood examination shows:


haemoglobin 125 g/L [128-175]
mean cell volume (MCV) 82 fL [80-97]
9
white cell count 8.8 x 10 /L [3.9-12.7]
(normal white cell differential)
9
platelets 1060 x 10 /L [150-396]

His blood film is shown below.

The most likely diagnosis is:

A. Essential thrombocythaemia.
B. Occult haemorrhage.
C. Primary myelofibrosis.
D. Occult carcinoma.
E. Chronic myeloid leukaemia.
193

Question 24
A 65-year-old man presents to the emergency department with ischaemic chest pain. No
abnormalities are noted on examination apart from mild hypotension.

His full blood examination shows:


haemoglobin 125 g/L [128-175]
9
white cell count 10.2 x 10 /L [3.9-12.7]
(normal white cell differential)
9
platelets 180 x 10 /L [150-396]

Coagulation investigations show:


activated partial thromboplastin time (APTT) 100 seconds [26-38]
APTT 1:1 mix (patient: normal plasma) 52 seconds
prothrombin time-international normalised ratio (PT-INR) 1.2 [1.0-1.3]
thrombin clotting time (TCT) 200 seconds [<24]
fibrinogen 4.5 g/L [2.0-4.0]
reptilase time 18 seconds [<24]

The most likely cause of the coagulation test abnormalities is:

A. Disseminated intravascular coagulation.


B. Administration of low-molecular weight heparin.
C. Administration of thrombolytic agent.
D. Lupus inhibitor.
E. Administration of unfractionated heparin.

Question 25
A 36-year-old healthy male with no siblings is found incidentally to have mild anaemia.

Full blood examination shows:


haemoglobin 111 g/L [128-175]
mean cell volume (MCV) 62 fL [80-97]
white cell count and platelets normal
Iron studies:
serum iron 6 µmol/L [9-27]
serum transferrin 3.1 g/L [2.3-3.9]
serum transferrin saturation 23% [<50]
serum ferritin 29 µg/L [15-400]

β-thalassaemia trait is confirmed on routine haemoglobinopathy testing.

In addition to informing him of the diagnosis, which of the following is the most appropriate next step?

A. Globin gene analysis.


B. Provide information on partner screening.
C. Commence iron therapy.
D. Haemoglobinopathy screening for parents.
E. Gastrointestinal endoscopy.
194

Question 26
Which of the following clinical conditions is least likely to improve following plasmapheresis?

A. Myasthenia gravis.
B. Chronic inflammatory demyelinating polyneuropathy.
C. Thrombotic thrombocytopenic purpura (TTP).
D. Mesangiocapillary glomerulonephritis secondary to cryoglobulinaemia.
E. Multiple sclerosis.

Question 27
A 67-year-old man with a history of heavy smoking presents with new onset angina. On examination
he is found to be plethoric and hypertensive with no other abnormalities noted.

His full blood examination shows:


haemoglobin 205 g/L [128-175]
haematocrit 57% [36-50]
mean cell volume (MCV) 78 fL [80-97]
9
white cell count 11.6 x 10 /L [3.9-12.7]
9
neutrophils 9.50 x 10 /L [1.50-6.00]
9
lymphocytes 0.60 x 10 /L [0.70-3.15]
9
monocytes 0.30 x 10 /L [0.15-0.60]
9
eosinophils 0.80 x 10 /L [0.00-0.40]
9
basophils 0.40 x 10 /L [0.00-0.15]
9
platelets 499 x 10 /L [150-396]
red cell mass 39 mL/kg [28-35]
plasma volume (derived) 49 mL/kg [40-50]
serum erythropoietin 4.9 IU/L [4.8-21.9]

The most likely explanation for these findings is:

A. Carcinoma of the lung.


B. High affinity haemoglobin.
C. Dehydration.
D. Chronic obstructive pulmonary disease.
E. Polycythaemia vera.
195

Question 28
A 72-year-old man with a history of coronary artery bypass grafting is admitted with biventricular
failure. His full blood examination is normal on admission. In addition to therapy for cardiac failure,
he receives thromboprophylaxis with subcutaneous unfractionated heparin, 5000 units twice daily.
On day 8 of admission, he complains of left leg pain and an ultrasound shows extensive thrombosis
extending from the popliteal to the external iliac veins. His full blood examination now reveals
9
thrombocytopenia (platelets 35 x 10 /L [150-396]) and a test for heparin-induced thrombocytopenia
by enzyme-linked immunosorbent assay (ELISA) is positive. There is no current evidence of
bleeding.

In addition to stopping the heparin, which of the following is the most appropriate immediate
treatment option?

A. Observation.
B. Dalteparin.
C. Platelet transfusion.
D. Intravenous immunoglobulin.
E. Danaparoid.

Question 29
In a patient with a deep venous thrombosis, for which of the following factors occurring in isolation
at the time of the initial thrombotic event is there strongest indication for extending anticoagulation
beyond six months?

A. Heterozygosity for factor V Leiden mutation.


B. Presence of a lupus inhibitor/anticoagulant.
C. Absence of a provoking stimulus.
D. Presence of an elevated homocysteine level.
E. Heterozygosity for prothrombin G20210A gene mutation.
196

Question 30
A 21-year-old Filipino woman has been referred for investigation of anaemia. Her blood counts and
iron studies are as follows:

Haemoglobin 81 g/L [113-159]


Haematocrit 0.27 [0.32-0.42]
Red blood cells 4.58 x109/L [3.60-5.30]
White blood cells 4.60 x109/L [3.90-12.70]
Platelets 421 x109/L [150-396]
Mean corpuscular volume 59 fL [80-97]
Mean corpuscular haemoglobin 17.7 pg [28.0-34.0]
Mean corpuscular haemoglobin concentration 300 g/L [328-363]
Reticulocytes 142 x 109/L [8-104]
Serum ferritin 482 mcg/L [15-200]
HbA2 1.4% [1.6-3.5]
HbF 0.3% [0.2-1.0]

Which of the following is the most likely diagnosis?

A. Alpha+-thalassaemia trait (a-/aa).


B. Alpha0-thalassaemia trait (--/aa).
C. HbH disease (--/-a).
D. Beta0-thalassaemia trait.
E. Beta+-thalassaemia trait.
197

Question 31
A 78-year-old woman has a history of hypertension on amlodipine, and has been found to have a
persistently elevated platelet count for several months. She presents to the emergency department
with abdominal pain. Her full blood examination and serum biochemistry are as follows:

haemoglobin 104 g/L [128-175]


white blood cell 70.40 x 109/L [3.90-12.70]
neutrophils 62.80 x 109/L [1.90-8.00]
lymphocytes 2.46 x 109/L [0.90-3.30]
monocytes 0.67 x 109/L [0.30-1.10]
eosinophils 4.43 x 109/L [0.00-0.50]
basophils 0.05 x 109/L [0.00-0.20]
platelets 3524 x 109/L [150-396]
mean corpuscular volume 74 fL [80-97]

sodium 142 mmol/L [136-145]


potassium 7.2 mmol/L [3.5-5.1]
bicarbonate 16 mmol/L [21-32]
chloride 112 mmol/L [98-107]
urea 13.2 mmol/L [2.0-8.5]
creatinine 0.13 mmol/L [0.04-0.08]

Which of the following is the most likely cause of her elevated serum potassium?

A. Pseudohyperkalaemia.
B. Renal tubular acidosis.
C. Chronic renal impairment.
D. Haemolytic anaemia.
E. Lactic acidosis.

Question 32
A 47-year-old woman has coagulation studies performed as part of preoperative assessment prior to
elective hysterectomy for menorrhagia.

Prothrombin Time-International Normalised Ratio 1.0 [0.9-1.3]


Activated Partial Thromboplastin Time (APTT) patient 52.3 s [26-38]
Activated Partial Thromboplastin Time (APTT) control 30.0 s [26-38]
APTT correction (1:1 mix, Patient: Control) 49.6 s [26-38]
Thrombin Clotting Time 20 s [<24]
Fibrinogen 2.1 g/L [2.0-4.0]
Fibrinogen D-Dimer 0.50 [<0.35]

Which of the following is the most likely interpretation of these results?

A. Tranexamic acid therapy.


B. Disseminated intravascular coagulation.
C. Heparin exposure.
D. Von Willebrand disease.
E. Lupus anticoagulant.
198

Question 33
Which of the following clinical situations is the strongest contraindication to therapeutic
anticoagulant therapy?

A. Infective endocarditis.
B. Intracranial bleeding in the last ten days.
C. Severe hypertension (Systolic Blood Presssure (BP) >200 mmHg).
D. Recent major trauma.
E. Moderate thrombocytopenia; platelet count 20-50 x 109/L.

Question 34
A healthy 24-year-old primigravida is noted to have isolated thrombocytopenia (platelets 100
x109/L) on a full blood examination performed at 32 weeks gestation. She has no prior history of
thrombocytopenia, and is normotensive. Her urinalysis is unremarkable.

Which of the following is the most likely explanation for this abnormality?

A. Pre-eclampsia.
B. Gestational Thrombocytopenia.
C. Anti-phospholipid syndrome.
D. Thrombotic Thrombocytopenic Purpura.
E. Immune Thrombocytopenic Purpura

Question 35
A 68-year-old woman was found to have an elevated white cell count on routine full blood
examination. On examination she is well and has no palpable lymphadenopathy or organomegaly.
Lymphocyte surface markers show that the cells express CD5, CD19, CD20 (dim), CD23 and kappa
light chains (dim).

Full Blood Examination:


Haemoglobin 127 g/L [122-170]
White Cells 76.2 x109/L [4-11]
Platelets 120 x109/L [150-400]
Neutrophils 3.7 x109/L [2-4]
Lymphocytes 57.0 x109/L [1-3.5]
Monocytes 1.2 x109/L [0.4-1.0]
Eosinophils 0.3 x109/L [0-0.4]
Smear Cells 1 4.0 x109/L

Which of the following is the most appropriate management?

A. Rituximab.
B. Fludarabine.
C. Observation.
D. Chlorambucil.
E. Prednisolone.
199

Question 36
A 72-year-old woman who is taking warfarin for an embolic stroke secondary to atrial fibrillation
presents to the emergency department after a simple fall. On examination she has a few small bruises
on her legs but is otherwise well. Her pulse is 68 bpm and irregular and her blood pressure is 175/95
mmHg. Her full blood examination is normal however her serum creatinine is elevated at 170 mcg/L
[50-100 mcg/L] and her INR is elevated at 7.1.

In addition to regular monitoring, which of the following is the best management strategy for her
elevated international normalised ratio (INR)?

A. Withhold warfarin.
B. Withhold warfarin and administer Vitamin K1.
C. Withhold warfarin and administer Vitamin K1 and prothrombin complex concentrate.
D.Withhold warfarin and administer Vitamin K1, prothrombin complex concentrate with fresh
plasma.
E. Withhold warfarin and administer prothrombin complex concentrate with fresh plasma.

Question 37
A “reasonably well” (normal performance status) patient with non-haemorrhagic cerebral metastases
from breast carcinoma presents with spontaneous lower limb venous thrombosis and pulmonary
embolism. Which of the following is the most appropriate long term management regime?

A. Dalteparin.
B. Inferior vena caval filter.
C. Warfarin.
D. Graduated compression stockings.
E. Aspirin.

Question 38
A previously healthy 67-year-old man presents with spontaneous muscle bleeds. Coagulation tests
reveal the following:

Activated partial thromboplastin time (APTT) 81 sec [26 - 38 sec]


APTT correction (immediate mix) 37 sec [26 - 38 sec]
APTT correction (2 hour incubation) 76 sec [26 - 38 sec]
Thrombin Clotting Time 12 sec [10 - 12 sec]
Fibrinogen 3.2 g/L [2.0 - 4.0 g/L]

Which of the following best accounts for the coagulation test results?
A. Haemophilia A.
B. Von Willebrand disease.
C. Acquired factor VIII inhibitor.
D. Lupus anticoagulant.
E. Disseminated intravascular coagulation
200

Question 39
A 30-year-old female presents with a painful swollen left calf. Duplex study demonstrates a proximal
deep vein thrombus extending into the ileofemoral venous system. The following laboratory results
are obtained:

Prothrombin time (PT) 16 sec [11 – 18 sec]


Activated partial thromboplastin time (APTT) 43 sec [25 – 36 sec]
Dilute Russell Viper venom test Lupus anticoagulant detected
Anticardiolipin IgG negative
Anticardiolipin IgM negative
Anti-beta 2 glycoprotein 1 moderate positive

Treatment with anticoagulation with heparin followed by warfarin therapy is commenced. Repeat
testing at 12-weeks confirms a positive anti-beta 2 glycoprotein 1. What is the most appropriate
duration of anticoagulation?

A. 3 months.
B. 6 months.
C. 12 months.
D. 24 months.
E. Lifelong.

Question 40
A 29-year-old woman is referred for evaluation after she was found to be heterozygous for factor V
Leiden mutation on screening. This screening was initiated when her great aunt sustained a calf vein
thrombosis following hip arthroplasty. She is currently 16-weeks pregnant and has no personal
history of venous thromboembolic disease.

Which of the following should be recommended for the remainder of her pregnancy?

A. Low molecular weight heparin.


B. Aspirin.
C. No thromboprophylaxis.
D. Surveillance ultrasonography of the lower limbs.
E. Warfarin.
201

Question 41
A 46-year-old man with hypertension and previous end stage renal failure due to glomerulonephritis,
received a kidney transplant 15 years ago. His creatinine has been stable at 110-130 µmol/L [60 - 120
µmol/L] for the last five years. During this time his immunosuppression has been unchanged and
consists of azathioprine 100 mg daily and prednisolone 5 mg daily. His general practitioner is
concerned about his reported mean corpuscular volume (MCV) of 106 fL [80 – 96 fL].

Which of the following is the most likely cause?

A. Folate deficiency.
B. Azathioprine.
C. Myelodysplasia.
D. Haemolysis.
E. Hypothyroidism.
202

Answers

1. A
2. B
3. D
4. E
5. E
6. E
7. A
8. C
9. A
10. A
11. B
12. A
13. D
14. E
15. A
16. D
17. E
18. B
19. B
20. A
21. C
22. B
23. A
24. E
25. B
26. E
27. E
28. E
29. B
30. C
31. A
32. E
33. B
34. B
35. C
36. B
37. A
38. C
39. E
40. C
41. B
203

Medical Sciences

HIV and Infectious Diseases

Question 1
A number of patients in a ward have been found to be colonised with vancomycin-resistant
enterococci. What would be the best laboratory test to determine if transmission from one patient to
another has occurred?

A. Comparison of antibiotic susceptibility (antibiogram).


B. Polymerase chain reaction (PCR) for the van A and van B genes.
C. Enterococcal serology.
D. Pulse-field gel electrophoresis.
E. Speciation (e.g. faecalis vs. faecium).

Question 2
An 18-year-old woman presents with a two-week history of cervical lymphadenopathy, fever, fatigue
and myalgia. A Paul-Bunnell test (heterophile antibody) has been negative on two occasions, a week
apart. Full blood examination reveals a lymphocytosis with many atypical lymphocytes.

The most likely diagnosis is acute:

A. Epstein-Barr virus (EBV) infection.


B. Streptococcal pharyngitis.
C. Toxoplasmosis.
D. Cytomegalovirus (CMV) infection.
E. Human immunodeficiency virus (HIV) infection.

Question 3
Which one of the following antibiotics is least active against pathogenic anaerobic bacteria (e.g.
Bacteroides fragilis, Clostridium perfringens)?

A. Ticarcillin plus clavulanate.


B. Metronidazole.
C. Clindamycin.
D. Ceftriaxone.
E. Chloramphenicol.
204

Question 4
The mechanism of penicillin resistance in pneumococcal infections is most likely to be due to which
one of the following?

A. Plasmid-mediated beta lactamase production.


B. Chromosome-mediated beta lactamase production.
C. Plasmid-mediated penicillinase production.
D. Decreased affinity of penicillin-binding proteins.
E. Altered permeability to penicillin.

Question 5
A hospitalised patient with a rash on her face and a lace-like rash on her extremities has a confirmed
diagnosis of erythema infectiosum (parvovirus B19).

Which of the following infection controls would be the most appropriate?

A. Standard.
B. Contact.
C. Respiratory.
D. Single room.
E. Negative pressure room.

Question 6
Which one of the following is least likely to be associated with a reduced risk of mother-to-child
transmission of human immunodeficiency virus (HIV)?

A. Maternal plasma HIV RNA concentration (viral load) <1000 copies/mL.


B. Maternal CD4+ T cell count >500 cells/mm3.
C. Maternal antiretroviral therapy.
D. Avoidance of breastfeeding.
E. Elective Caesarean delivery.

Question 7
Which of the following antibiotics would be the most appropriate oral medication for the outpatient
treatment of a wound infection caused by Escherichia coli and Pseudomonas aeruginosa?

A. Moxifloxacin.
B. Gatifloxacin.
C. Ciprofloxacin.
D. Linezolid.
E. Amoxycillin/clavulanate.
205

Question 8
A 37-year-old man presents to the emergency department with symptoms of meningitis. Gram stain
of the cerebrospinal fluid reveals the presence of gram-negative diplococci. His 12-week pregnant
partner should receive which one of the following as prophylaxis?

A. Ciprofloxacin.
B. Ceftriaxone.
C. Penicillin.
D. Meningococcal vaccine.
E. Erythromycin.

Question 9
The most common cause of diarrhoea in an adult traveller to a developing country is:

A. Shigella species.
B. Salmonella species.
C. Campylobacter species.
D. Enterotoxigenic Escherichia coli.
E. Giardia lamblia.

Question 10
With which of the following viruses is cancer of the cervix most strongly associated?

A. Epstein-Barr virus (EBV).


B. Cytomegalovirus (CMV).
C. Human papilloma virus (HPV).
D. Human immunodeficiency virus (HIV).
E. Herpes simplex virus (HSV).

Question 11
Which of the following best describes the antiviral effect of zanamivir on the influenza virus?

A. Inhibition of M2 ion channel activity.


B. Inhibition of viral neuraminidase.
C. Competitive blocking of surface haemagglutinins.
D. Inhibition of reverse transcriptase.
E. Inhibition of DNA polymerase.
206

Question 12
In the treatment of hepatitis C infection with interferon and ribavirin, which one of the following is
the best predictor of sustained virological response?

A. Serum bilirubin.
B. Viral genotype.
C. Gender.
D. Viral load.
E. Portal fibrosis.

Question 13
Many human immunodeficiency virus (HIV) protease inhibitors are marketed in a combination that
includes a small dose of ritonavir eg. Lopinavir 133mg-ritonavir 233mg. Which of the following
provides the best rationale for the use of such combinations?

A. Enhancement of anti-HIV activity.


B. Improved patient compliance.
C. Improved pharmacokinetic profile.
D. Reduction of metabolic side effects.
E. Reduction in development of HIV drug resistance.

Question 14
A man with chronic renal failure is on the renal transplant waiting list. He is serologically
cytomegalovirus (CMV) negative. If he is to receive a transplant from a CMV positive donor, which
of the following is the most appropriate prophylactic regime for CMV disease?

A. Live attenuated CMV vaccination.


B. CMV hyperimmune globulin.
C. Oral acyclovir.
D. Intravenous ganciclovir.
E. Oral valganciclovir.

Question 15
A defect in elimination of which of the following is most likely to occur post-splenectomy?

A. Polysaccharide antigens.
B. T cell-dependent antigens.
C. Intracellular microbes.
D. Protein antigens.
E. Staphylococcus aureus.
207

Question 16
Defects in the terminal attack elements of the complement system (C6, C7, C8) are most strongly
associated with an increased risk of infection with which of the following pathogens?

A. Streptococcus pneumoniae.
B. Neisseria meningitidis.
C. Pneumocystis jiroveci.
D. Listeria monocytogenes.
E. Haemophilus influenzae.

Question 17
An 18-year-old male develops a rash and becomes critically ill. The rash is demonstrated below.

The most likely finding on blood cultures would be:

A. Gram negative rods.


B. Gram positive rods.
C. Gram negative diplococci.
D. Gram positive diplococci.
E. Gram positive cocci.

Question 18
The emergence of a pandemic strain of influenza is best explained by which of the following
mechanisms?

A. Antigenic drift in H and N proteins of influenza A.


B. Antigenic shift in H and N proteins of influenza A.
C. Point mutations in the M proteins of influenza A.
D. Development of neuraminidase resistance.
E. Recombination of influenza A and B haemagglutinins.
208

Question 19
Following kidney transplantation, which of the following is the most likely manifestation of BK
virus infection?

A. Aplastic anaemia.
B. Pulmonary infiltrates.
C. Nephropathy.
D. Progressive multifocal leucoencephalopathy.
E. Systemic vasculopathy.

Question 20
Resistance of gram-positive organisms to penicillin is most commonly mediated through which of
the following mechanisms?

A. Antibiotic efflux.
B. Beta-lactamase production.
C. Alteration of the ribosomal target.
D. Modification of penicillin binding proteins.
E. Decreased outer membrane permeability.

Question 21
Which of the following is the most common precipitating illness leading to the Guillain-Barre
syndrome?

A. Campylobacter jejuni gastroenteritis.


B. Herpes simplex infection.
C. Epstein-Barr virus infection.
D. Mycoplasma pneumonia.
E. Cytomegalovirus infection

Question 22
Recurrent infections with which of the following organisms is most characteristic of a disorder of
neutrophil phagocytic function (such as chronic granulomatous disease)?

A. Non-tuberculous mycobacteria.
B. Pseudomonas aeruginosa.
C. Nocardia species.
D. Pneumocystis jiroveci (carinii).
E. Staphylococcus aureus.
209

Question 23
The emergence of a pandemic strain of influenza is best explained by which of the following
mechanisms?

A. Antigenic drift in H and N proteins of influenza A.


B. Antigenic shift in H and N proteins of influenza A.
C. Point mutations in the M proteins of influenza A.
D. Development of neuraminidase resistance.
E. Recombination of influenza A and B haemagglutinins.

Question 24
Many HIV (human immunodeficiency virus) protease inhibitors are marketed in a combination that
includes a small dose of ritonavir eg. Lopinavir 133mg-ritonavir 233mg. Which of the following
provides the best rationale for the use of such combinations?

A. Enhancement of anti-HIV activity.


B. Improved patient compliance.
C. Improved pharmacokinetic profile.
D. Reduction of metabolic side effects.
E. Reduction in development of HIV drug resistance.

Question 25
Patients infected with strains of Staphylococcus aureus which secrete the Panton-Valentine
Leucocidin (PVL) toxin are most likely to present with which of the following?

A. Endocarditis.
B. Toxic shock syndrome.
C. Pyogenic skin infections.
D. Stevens-Johnson syndrome.
E. Intravenous catheter sepsis.

Question 26
Which of the following vaccines is contraindicated in immunosuppressed individuals?

A. Influenza.
B. Conjugated meningococcal.
C. Varicella/Zoster.
D. Conjugated pneumococcal.
E. Hepatitis A.
210

Question 27
Patients infected with strains of Staphylococcus aureus which secrete the Panton-Valentine
Leucocidin (PVL) toxin are most likely to present with which of the following?

A. Endocarditis.
B. Toxic shock syndrome.
C. Pyogenic skin infections.
D. Stevens-Johnson syndrome.
E. Intravenous catheter sepsis.

Question 28
Recurrent infections with which of the following organisms is most characteristic of a disorder of
neutrophil phagocytic function (such as chronic granulomatous disease)?

A. Non-tuberculous mycobacteria.
B. Pseudomonas aeruginosa.
C. Nocardia species.
D. Pneumocystis jiroveci (carinii).
E. Staphylococcus aureus.

Question 29
Other than bacterial infections, opportunistic infection with which of the following is most likely to
occur in a patient after induction chemotherapy for acute myeloid leukaemia (AML)?

A. Candida albicans.
B. Mycobacterium tuberculosis.
C. Cytomegalovirus.
D. Cryptosporidium.
E. Toxoplasma gondii.

Question 30
A 42-year-old woman presents to the Emergency Department with a febrile illness with no localizing
symptoms or signs. After 48-hours of incubation the laboratory reports the growth of Staphylococcus
epidermidis in the aerobic bottle of a blood culture set. What is the most likely explanation of this
result?

A. Skin contamination.
B. Native valve endocarditis.
C. Intrahepatic abscess.
D. Urinary tract infection.
E. Periodontic disease.
211

Question 31
Which of the following antibiotics is the commonest cause of Clostridium difficile colitis?

A. Clindamycin.
B. Metronidazole.
C. Gentamicin.
D. Amoxycillin.
E. Doxycycline.
212

Answers

1. D
2. D
3. D
4. D
5. A
6. B
7. C
8. B
9. D
10. C
11. B
12. B
13. C
14. E
15. A
16. B
17. C
18. B
19. C
20. B
21. A
22. E
23. B
24. C
25. C
26. C
27. C
28. E
29. A
30. A
31. D
213

Clinical Applications

HIV and Infectious Diseases

Question 1
Which one of the following is most likely to be the aetiological agent causing hospital-acquired
(nosocomial) pneumonia?

A. Aerobic gram-negative bacilli.


B. Gram-positive cocci.
C. Viruses.
D. Legionella species.
E. Anaerobic organisms.

Question 2
54-year-old diabetic woman with chronic renal impairment presents with peritonitis secondary to a
perforated diverticulum. Blood cultures grew Enterobacter species. The organism tests sensitive to
ceftriaxone. After laparotomy, lavage and commencement of intravenous ceftriaxone and
metronidazole, the patient makes no improvement. A computed tomography (CT) scan of the
abdomen is performed and reveals no collection. Blood culture repeated three days later again grows
the same Enterobacter species.

Which one of the following is the most appropriate next step?

A. Continuation of current therapy.


B. Laparotomy.
C. Cessation of ceftriaxone, continuation of metronidazole and addition of ampicillin and
gentamicin.
D. Cessation of current antibiotics and commencement of meropenem.
E. Addition of vancomycin.

Question 3
Which one of the following modes of transmission of hepatitis B virus (HBV) is most likely to result
in chronic carrier status in the non-immunised recipient?

A. A bite to a two-year-old by a chronic carrier.


B. Blood transfusion from a chronic carrier to a 60-year-old.
C. Horizontal transmission of HBV from a chronic carrier to his/her five-year-old sibling.
D. Needle-stick injury to a 21-year-old from a contaminated needle.
E. Vertical transmission of HBV from a pregnant carrier to her newborn baby.
214

Question 4
The optimal treatment for meningitis due to penicillin-resistant (minimum inhibitory concentration
(MIC) >1mg/L) Streptococcus pneumoniae is:

A. High-dose ceftriaxone.
B. High-dose penicillin.
C. Vancomycin and rifampicin.
D. Cotrimoxazole and high-dose penicillin.
E. Imipenem.

Question 5
A 35-year-old man returns from a holiday in Africa. He has been taking mefloquine antimalarial
prophylaxis until two weeks after his return to Australia. Eight weeks later he presents with fever and
rigors. A blood film examination demonstrates Plasmodium vivax in 1% of red cells.

Which one of the following is the most appropriate management?

A. A three-day course of chloroquine followed by a seven-day course of doxycycline.


B. A three-day course of doxycycline.
C. A single dose of mefloquine followed by a 14-day course of primaquine.
D. A three-day course of chloroquine followed by a 14-day course of primaquine.
E. A three-day course of intravenous quinine followed by a seven-day course of oral doxycycline.
215

Question 6
A 25-year-old man presents with dysuria and frequency after a trip to South-East Asia. A urethral
swab is obtained and a Gram stain is performed (shown in the photograph below).

Which one of the following would be the optimal antibiotic treatment for this man?

A. Ceftriaxone 250 mg (intramuscularly, as a single dose).


B. Ciprofloxacin 500 mg and azithromycin 1 g (both orally, as a single dose).
C. Amoxicillin 3 g, probenecid 1 g and azithromycin 1 g (all orally, as a single dose).
D. Amoxicillin 3 g, probenecid 1 g (both orally, as a single dose) and doxycycline 100 mg (orally, 12
hourly for 10 days).
E. Benzathine penicillin 1.8 g (intramuscularly, as a single dose).
216

Question 7
A chest X-ray and a Ziehl-Nielsen (ZN) stain of sputum from a patient who is about to commence
appropriate therapy are shown below. The patient is isolated with respiratory precautions.

Chest X-ray

ZN stain

How long should the patient stay in respiratory isolation?

A. Until the patient is smear-negative (ZN stain-negative).


B. Two to three weeks.
C. Until the patient is culture-negative.
D. Until therapy is commenced.
E. Until asymptomatic.
217

Question 8
A 57-year-old HIV (human immunodeficiency virus)-positive man presents with a six-month history
of gradual enlargement of the abnormality shown in the photographs below. Investigations reveal a
CD4+ T cell count of 30/µL and a plasma HIV RNA concentration (viral load) of 158,000
copies/mL. Current medications are four antiretroviral drugs (zidovudine, lamivudine, ritonavir and
saquinavir), antimicrobial prophylactic agents (cotrimoxazole, azithromycin) and amitriptyline.
Routine blood count and biochemistry screen are normal except for an elevated fasting blood sugar
level of 11.0 mmol/L [3.9-5.5].

Which one of the following represents the most likely pathogenesis for the abnormality shown in the
photographs?

A. Opportunistic infection.
B. Malignancy.
C. Cortisol excess.
D. Drug toxicity.
E. Autoimmune inflammation.
218

Question 9
A 31-year-old man who has been HIV (human immunodeficiency virus)-positive for some years and
who is on zidovudine, presents with acute left-sided chest pain and dyspnoea. He has smoked since
the age of 17, but denies having asthma. Physical examination shows a tall, thin man in mild
respiratory distress with evidence of weight loss. His pulse rate is 120/minute and his respiratory rate
is 28/minute. There are decreased breath sounds on the left side. His temperature is 38.6°C. Oxygen
saturation on room air is 90% and his chest X-ray reveals a 45% pneumothorax on the left side.

The most likely cause for the spontaneous pneumothorax in this patient is:

A. Smoking related disease.


B. HIV infection.
C. Streptococcus pneumoniae infection.
D. Pneumocystis carinii infection.
E. Anti-retroviral therapy.

Question 10
A 32-year-old man with known human immunodeficiency virus (HIV) infection returns for routine
assessment. Relevant history includes a documented seroconversion illness five years previously but
no other features. He has not received antiretroviral therapy.

Physical examination reveals generalised lymphadenopathy but no other abnormalities.


Investigations show normal full blood count and biochemistry screen. CD4+ T cell count is 560/µL
with an undetectable HIV RNA concentration (viral load).

Which one of the following therapeutic options is the most appropriate?

A. Cotrimoxazole prophylaxis.
B. Azithromycin prophylaxis.
C. Zidovudine (AZT) and lamivudine (3TC).
D. AZT, 3TC and a protease inhibitor.
E. No therapy.

Question 11
The effect of neuraminidase inhibitors (such as zanamivir) when given within 24 hours of onset of
symptoms on the course of influenza infection is best described as:

A. Complete suppression of influenza symptoms within a day of commencing treatment.


B. Reduction in the severity of influenza symptoms with no effect on duration of
illness.
C. Reduction in the severity of influenza symptoms and shortening of the duration of symptoms by
one to two days.
D. Reduction in the duration of illness by three to five days, with no effect on the
severity of symptoms.
E. No effect on the course of the illness itself, but a marked reduction in the infectivity of the
individual.
219

Question 12
A 45-year-old man with known human immunodeficiency virus (HIV) infection presents with
painful swallowing (odynophagia). He has not had any AIDS (acquired immune deficiency
syndrome)-defining illnesses. He takes no medication.

An upper endoscopy is performed. A photograph of the mid-oesophagus is shown below. Biopsies


are taken.

The most appropriate treatment at this stage is oral:

A. Aciclovir.
B. Fluconazole.
C. Nystatin.
D. Aciclovir and fluconazole.
E. Aciclovir and nystatin.
220

Question 13
A 35-year-old married indigenous woman presents with a four-day history of fever, mild headache
and severe polyarthritis involving the wrists, elbows and knees. Joint pain has been only slightly
relieved by naproxen 250 mg twice daily and paracetamol 1-2 g daily. Prior to the onset of arthritis,
she had a sore throat for several days but denies rash, vaginal discharge or history of sexually
transmitted diseases.

On examination, she appears unwell with a temperature of 39°C, an erythematous throat with
tonsillar enlargement and a few small tender cervical lymph nodes. There is tenderness and soft
tissue swelling of the wrists and knees. Cardiovascular examination is normal.

Which of the following investigations is most likely to identify the diagnosis?

A. Blood cultures for Staphylococcus aureus.


B. A high vaginal swab for Neisseria gonorrhoeae.
C. A throat swab for Streptococcus species.
D. A stool culture for Yersinia enterocolitica.
E. A synovial fluid culture for Neisseria meningitidis.
221

Question 14

Which of the following is the most likely organism causing the presentation shown in the
photographs below?

A. Neisseria meningitidis.
B. Staphylococcus aureus.
C. Mycoplasma pneumoniae.
D. Listeria monocytogenes.
E. Coxsackievirus.

Question 15
A 32-year-old man has a splenectomy following a motorcycle accident. Which of the following
organisms is most likely to cause overwhelming post-splenectomy infection in this man?

A. Streptococcus pneumoniae.
B. Neisseria meningitidis.
C. Staphylococcus aureus.
D. Escherichia coli.
E. Haemophilus influenzae.
222

Question 16
A 17-year-old adolescent male is referred for investigation of recurrent pyogenic skin lesions. He
gives an 18-month history of recurrent boils and cutaneous abscesses. There is no family history of
similar problems and the patient is otherwise in excellent health. Examination reveals evidence of
multiple healed lesions of the face and neck, axillae, groin and anterior chest wall. He has received
repeated courses of both oral and intravenous antibiotics, and on a number of occasions the lesions
have required surgical drainage. Culture of the abscess contents on one of these occasions yielded a
heavy growth of Staphylococcus aureus.

Which of the following is the most appropriate initial investigation?

A. Serum complement levels.


B. Culture of a nasal swab.
C. Serum immunoglobulin levels.
D. Neutrophil function studies.
E. Lymphocyte subsets.

Question 17
A 35-year-old man with a history of severe penicillin allergy presents with fever and a widespread
rash, part of which is shown in the photograph below.

Which of the following is the most appropriate immediate treatment?

A. Penicillin.
B. Ceftriaxone.
C. Erythromycin.
D. Ciprofloxacin.
E. Vancomycin.
223

Question 18
Angiostrongylus cantonensis, the rat lung worm, is acquired by eating the intermediate snail or slug
host or contaminated vegetables.

Which of the following clinical syndromes is most characteristic of this parasite?

A. Haematuria.
B. Rectal bleeding.
C. Ascending cholangitis.
D. Eosinophilic meningitis.
E. Visceral larva migrans.

Question 19
A 26-year-old man presents with a two-day history of fevers, rigors, headache, malaise, nausea and
vomiting, dry cough, mild arthralgia and backache. He reports no shortness of breath, diarrhoea,
neck stiffness or photophobia. He reports that he returned from a diving trip to the Solomon Islands
seven days ago. He had taken doxycycline regularly as malarial prophylaxis.

On day 3 after the onset of the illness he develops a rash over his trunk (shown below) and face. A
petechial rash was noted on his lower limbs.

Which of the following is the most likely diagnosis?

A. Malaria.
B. Dengue fever.
C. Typhoid fever.
D. Q fever.
E. Measles.
224

Question 20
In human immunodeficiency virus (HIV)-infected patients receiving combination antiretroviral
therapy, including nucleoside reverse transcriptase inhibitors, which of the following drug side-
effects is least likely to be due to mitochondrial toxicity?

A. Lactic acidosis.
B. Pancreatitis.
C. Stevens-Johnson syndrome.
D. Myopathy.
E. Peripheral neuropathy.

Question 21
Which of the following antibiotics would be the most appropriate choice for the treatment of a
vancomycin-resistant Enterococcus faecalis bloodstream infection?

A. Cefoxitin.
B. Linezolid.
C. Amikacin.
D. Tobramycin.
E. Rifampicin.

Question 22
A 30-year-old medical officer sustains a needle-stick injury while treating a trauma patient in the
emergency department. She had undergone a three-dose course of hepatitis B vaccine 10 years
earlier and one year following this course was documented to be hepatitis B surface antibody
(HbsAb)-positive. However, on repeat testing (done as a result of this incident), the medical officer is
now found to be HbsAb-negative. The source patient in the needle-stick injury is known to be an
intravenous drug user, and is shown to be hepatitis B surface antigen (HbsAg)-positive on baseline
serology. Human immunodeficiency virus (HIV) and hepatitis C serology are negative.

In addition to counseling, which of the following is the most appropriate immediate action following
this needle-stick injury?

A. Booster hepatitis B vaccination.


B. Hepatitis B immunoglobulin.
C. Hepatitis B vaccine and hepatitis B immunoglobulin.
D. No action required.
E. Post exposure prophylaxis for HIV.
225

Question 23
A 25-year-old man has recurrent haematuria. He had microscopic haematuria noted during an
admission with pneumonia six months earlier. Follow up urine microscopy was normal a month
later. He has developed macroscopic haematuria following a recent throat infection which lasted six
days. Examination is unremarkable.

Investigations show a normal full blood count, creatinine, electrolytes, calcium, liver function,
clotting screen and chest radiograph. A mid-stream specimen of urine shows 90 red blood cells per
high powered field but no casts or protein. Urine culture is sterile.

The most likely diagnosis is:

A. poststreptococcal glomerulonephritis.
B. IgA nephropathy.
C. Goodpasture’s syndrome.
D. Henoch-Schönlein purpura.
E. Wegener’s granulomatosis.

Question 24
A 56-year-old forestry worker develops a widespread rash involving the trunk and lower limbs two
weeks following a tick bite. He is generally unwell with myalgias, fevers, headache and anorexia.
The rash is papular, non-blanching and not itchy, and is shown below.

What is the most appropriate management?

A. Treat expectantly.
B. Doxycycline.
C. Ciprofloxacin.
D. Ceftriaxone.
E. Phenoxymethylpenicillin (penicillin V).
226

Question 25
A 60-year-old oyster-farmer presents with a six-week history of pain and swelling in his right hand.
He initially developed symptoms in the 3rd and 4th metacarpophalangeal (MCP) joints as well as the
dorsum of the wrist. He has occasionally cut his hands during his work and recalls having serous
drainage from a cut on his right hand two months previously.
rd
Examination confirms tenosynovial swelling over the right wrist and soft tissue swelling of the 3
th
and 4 MCP joints. There are no skin lesions or evidence of other joint pathology.
Blood count and chest X-ray are normal.

Following aspiration of fluid from the tenosynovial effusion, which of the following synovial fluid
studies would be most likely to provide the correct diagnosis?

A. Synovial fluid rheumatoid factor.


B. Culture for acid fast organisms.
C. Culture for fungi.
D. Polymerase chain reaction (PCR) examination for virus particles.
E. Gram stain and culture for bacteria.

Question 26
A 28-year-old woman becomes unwell the day after returning from a month-long holiday in
Thailand. She complains of a fever with severe headache, myalgia and sore eyes. On the fourth day
her temperature settles but two days later recurs and she then develops a generalised rash.
o
On examination she is pyrexial with a temperature of 37.9 C. Pulse is 80/minute, and blood pressure
120/80 mmHg. There is a diffuse macular rash over the chest and abdomen. The rest of the
examination is normal.

Blood tests show a normal haemoglobin, normal leukocyte count, and normal platelets. Blood film
examinations for malaria parasites are negative on three occasions.

Which one of the following is the most likely diagnosis?

A. Scrub typhus.
B. Typhoid.
C. Acute schistosomiasis.
D. Dengue.
E. Leptospirosis.
227

Question 27
A 39-year-old woman presents with signs and symptoms of pyelonephritis. Despite being treated for
five days with oral cephalexin (500 mg four times/day), a midstream urine culture is positive for
enterococci.

The most likely explanation for the lack of response to the prescribed antibiotic is:

A. Lack of tissue penetration.


B. Poor compliance.
C. Antibiotic resistance.
D. Oral rather than intravenous therapy.
E. Inadequate duration of therapy.

Question 28
A 45-year-old Australian-born woman with rheumatoid arthritis on long term prednisolone therapy
presents with a third nerve palsy, left sided cerebellar signs and altered consciousness. She has a
fever of 39°C and neck stiffness. Computed tomography (CT) scan of the brain is normal. Lumbar
puncture reveals an opening pressure of 20 cm [< 20 cm], white cell count of 80 x 106/L (80%
lymphocytes), protein 0.6 g/L [< 0.45], glucose 3.5 mmol/L (blood glucose 4.0 mmol/L) and no red
cells. Gram stain reveals no organisms and culture is pending. The most appropriate initial treatment
is:

A. Observation, pending culture results.


B. Aciclovir and ceftriaxone.
C. Ceftriaxone and ampicillin.
D. Isoniazid, rifampicin, ethambutol and pyrazinamide.
E. Vancomycin and penicillin.
228

Question 29
A 65-year-old man with a past history of diverticular disease presents with one week of fever and
right upper quadrant pain. He has never travelled overseas. His CT scan reveals a large fluid-filled
lesion as shown below.

In addition to drainage of the abscess, what is the most appropriate initial empiric antibiotic
treatment?

A. Ampicillin, gentamicin and metronidazole.


B. Cefazolin and metronidazole.
C. Vancomycin and metronidazole.
D. Meropenem.
E. Cefoxitin and gentamicin.

Question 30
A 40-year-old human immunodeficiency virus type 1 (HIV-1) -infected patient receiving highly
active anti-retroviral therapy (HAART) smokes 25 cigarettes a day and has a serum cholesterol level
of 6.0 mmol/L [2.5 – 5.2] and mild to moderate hypertension. Which of the following interventions
is most likely to reduce his cardiovascular risk?

A. Cessation of smoking.
B. Combined aerobic and strength exercise programme.
C. Change of anti-retroviral therapy.
D. Introduction of lipid-lowering agent.
E. Introduction of anti-hypertensive agent.
229

Question 31
A 62-year-old man is admitted to hospital with fevers, malaise and myalgias six weeks after a
laparoscopic cholecystectomy. On examination he has a temperature of 39°C, splinter haemorrhages
and a loud pansystolic murmur. He has a past history of mitral valve prolapse which was diagnosed
by echocardiography. Enterococcus faecalis has been identified in three sets of blood cultures. The
isolate is highly sensitive to penicillin. He has no known allergies. The most appropriate therapy is:

A. Ceftriaxone.
B. Vancomycin alone.
C. Ampicillin alone.
D. Ampicillin and gentamicin.
E. Cephalothin and gentamicin.

Question 32
A 36-year-old male with a history of recent injecting drug use presents with a week of fevers and
sweats and excruciating lower thoracic back pain. Blood cultures grow Staphylococcus aureus. What
would be the most useful next investigation to establish a diagnosis?

A. CT (computed tomography) scan of thoracolumbar spine.


B. MRI (magnetic resonance imaging) scan of thoracolumbar spine.
C. Lumbar puncture.
D. Radionuclide bone scan.
E. Labelled white cell scan.

Question 33
A physician becomes ill with nausea and vomiting four hours after attending a pharmaceutical
company sponsored dinner. Which of the following is the most likely cause of this?

A. Clostridium difficile.
B. Staphylococcus aureus.
C. Escherichia coli.
D. Vibrio parahemolyticus.
E. Campylobacter jejuni.
230

Question 34
A 35-year-old man is admitted to hospital with a red and tender left thigh, three days after sustaining
a superficial abrasion to the area while surfing. He gives a history of fevers and chills and says that
the pain in the leg is now increasing. On examination he appears unwell, is hypotensive, tachycardic
and febrile to 40°C. The left thigh is erythematous, tender to deep palpation with a 'boggy' feel over
the muscles. He is observed in the emergency department and the redness over the thigh progresses 2
cm in a two hour period. A computed tomography (CT) scan of the area shows inflammation of the
muscles and loss of fascial planes, but no abscess formation. Intravenous penicillin and clindamycin
are started. Which of the following additional interventions is most likely to result in survival and
limb preservation?

A. Surgical exploration and debridement.


B. Intravenous hydrocortisone.
C. Intravenous immunoglobulin.
D. Hyperbaric oxygen therapy.
E. Intravenous activated protein C.

Question 35
A 60-year-old man has been identified as HIV (human immunodeficiency virus) positive for at least
15 years. On antiretroviral therapy, his viral load is undetectable and his CD4 count is 220. He also
has vascular risk factors. His partner describes memory impairment of six months duration and a
change in personality. On mental state examination, he is irritable but is attentive and orientated. He
has an expressive dysphasia and some impairment of short-term memory. A CT scan of the brain
demonstrates mild cerebral atrophy but no evidence of opportunistic infection or cerebral
malignancy. Which of the following features in the history, examination and investigations is most
suggestive of a diagnosis other than HIV dementia as the cause of his cognitive impairment?

A. Short-term memory impairment.


B. Dysphasia.
C. Personality change.
D. CD4 count.
E. Undetectable viral load.
231

Question 36
A 21-year-old man with a history of injecting drug use is admitted to the Intensive Care Unit with a
severe bilateral pneumonia. He has a recent history of recurrent skin boils which have been difficult
to treat. He has received a number of courses of oral antibiotics for these lesions with little effect. He
is intubated and gram stain of the endotracheal aspirate shows Gram positive cocci. Sputum cultures
show a heavy growth of Staphylococcus aureus with sensitivities to follow. He is treated with
intravenous ceftriaxone and a macrolide.

What is the most appropriate additional treatment?

A. Flucloxacillin.
B. Rifampicin.
C. Meropenem.
D. Vancomycin.
E. Gentamicin.

Question 37
A 42-year-old woman with human immunodeficiency virus (HIV) infection presents to the
emergency department with a six week history of general malaise, fever, night sweats and loss of 5kg
weight. She was born in Papua New Guinea but came to Australia five years ago. She takes
zidovudine, lamivudine and nevirapine. Her viral load is undetectable and CD4 count is 0.41 x 109/L
[0.45-1.4 x 109/L]. Examination is unremarkable. Her Chest X-ray shows patchy consolidation in the
left upper zone as shown below.

What is the most likely cause of the radiological changes?

A. Pneumocystis pneumonia (PCP).


B. Mycobacterium avium complex (MAC).
C. Mycobacterium tuberculosis (MTB).
D. Burkholderia pseudomallei.
E. Cytomegalovirus (CMV).
232

Question 38
A 45-year-old human immunodeficiency virus-1 (HIV-1)-infected male who has been followed
without anti-retroviral therapy for 12 months is referred for assessment and further management. He
has no HIV-related symptoms or signs. Baseline evaluation and counselling is performed.

Which of the following findings provides the strongest indication to initiate anti-retroviral therapy
immediately?

A. CD4 cell count 300 per cubic millimeter.


B. Plasma HIV-1 viral load 100,000 copies/mL.
C. Decline of CD4 cell count from 350 to 300 per cubic millimetre over 12 months.
D. Evidence of drug-resistant HIV variants in HIV genotyping.
E. Active hepatitis C infection on serologic testing.

Question 39
A 45-year-old woman receiving total parenteral nutrition for short bowel syndrome through a
Hickman’s catheter presents with fever, sweats and rigors. Three sets of blood cultures grow Candida
albicans.

Which of the following is the most appropriate management?

A. Remove the catheter, no antibiotics required.


B. Remove the catheter, commence flucytosine.
C. Leave catheter in situ, commence voriconazole.
D. Leave catheter in situ, commence amphotericin B.
E. Remove the catheter, commence amphotericin B.

Question 40
A 62-year-old non-smoking woman is admitted with a 24-hour history of fever, rigors, cough and
breathlessness. A left lower lobe pneumonia is diagnosed on Chest X-ray and she is commenced on
intravenous ampicillin and oral clarithromycin. The next day she has improved clinically and
Streptococcus pneumoniae is isolated from blood cultures taken on admission. The isolate is found
tohave a minimal inhibitory concentration (MIC) for penicillin of 0.125mg/L (intermediate
resistance).

What is the most appropriate antibiotic for ongoing managemnet?

A. Vancomycin.
B. Moxifloxacin.
C. Ceftriaxone.
D. Ampicillin.
E. Cephalothin.
233

Question 41
A 72-year-old man admitted to hospital with a liver abscess was initially treated with intravenous
ceftriaxone alone. Cultures from a percutaneous aspirate specimen subsequently grow Streptococcus
milleri. On day seven he developed profuse, watery diarrhoea (eight bowel actions per day)
associated with fever and abdominal pain. A stool specimen has been taken. In addition to changing
to a narrow spectrum antibiotic, what is the most appropriate treatment while waiting for the result?

A. Intravenous clindamycin.
B. Oral vancomycin.
C. Oral metronidazole.
D. Oral ciprofloxacin.
E. Intravenous vancomycin.

Question 42
A 26-year-old man presents with a six month history of profound fatigue after a viral illness.
Extensive investigations have not found a primary cause for the fatigue. Which of the following
treatment options is most likely to improve his fatigue?

A. Educational material explaining chronic fatigue.


B. A supervised graded exercise program.
C. Selective serotonin re-uptake inhibitors.
D. Low dose prednisolone.
E. Supportive primary care.

Question 43
A 56-year-old farmer with a history of chronic mid and lower back pain is admitted with a two week
history of fevers, rigors, general malaise and excruciating mid-thoracic back pain. On examination he
has a temperature of 38.5 degrees Celsius and tenderness to palpation over the 6th and 7th thoracic
vertebrae. The rest of the examination is unremarkable. Full blood count shows a white cell count of
13.0 x 10^9/L [4.0–11.0] with a neutrophilia [10.2 x 10^9/L]. C reactive protein is 190 u/L [<5]) and
ESR 69 mm/hr [<20]. Blood cultures are pending.

Which of the following investigations is most likely to establish the diagnosis?

A. Radionuclide bone scan.


B. MRI of spine.
C. CT scan of spine.
D. Plain X-rays of spine.
E. Labelled white cell scan.
234

Question 44
A 26-year-old man recently returned from an overseas holiday, presents with a two day history of
joint pain. On examination his temperature is 38.1oC, he has two small pustular lesions on his left
hand, evidence of tenosynovitis of the left 4th flexor tendon and an effusions in the right knee.

Which diagnosis best explains this clinical pattern?

A. Reactive arthritis.
B. Parvovirus infection.
C. Gonococcal arthritis.
D. Staphylococcal arthritis.
E. Sub acute bacterial endocarditis.

Question 45
In preventing the recurrence of rheumatic fever in young patients with rheumatic heart disease,
whichis the most effective strategy?

A. Early treatment of throat infection.


B. Vaccination against common types of streptococcus.
C. Secondary prophylaxis with benzathine penicillin G.
D. Secondary prophylaxis with amoxycillin.
E. Topical treatment of impetigo in family members.

Question 46
A 50-year-old male has a two month history of frequent, small volume stools associated with
crampsand flatulence. This has occurred since returning from a holiday in South East Asia. The
empiric treatment most likely to be successful is:

A. Loperamide.
B. Tinidazole.
C. Albendazole.
D. Bismuth subsalicylate.
E. Norfloxacin.
235

Question 47
A 62-year-old non-smoking woman is admitted with a 24-hour history of fever, rigors, cough and
breathlessness. A left lower lobe pneumonia is diagnosed on chest X-ray and she is commenced on
intravenous ampicillin and oral clarithromycin. The next day she has improved clinically and
Streptococcus pneumoniae is isolated from blood cultures taken on admission. The isolate is found to
have a minimal inhibitory concentration (MIC) for penicillin of 0.125 mg/L (intermediate
resistance).

Which of the following is the most appropriate antibiotic for ongoing management?

A. Vancomycin.
B. Moxifloxacin.
C. Ceftriaxone.
D. Ampicillin.
E. Cephalothin.

Question 48
A 72-year-old woman with type 2 diabetes presents with a chronic ulcer on the dorsum of her left
foot. She has not been hospitalised in the past two years. The ulcer has been present for six months
and has been treated with daily dressings. In the last 48 hours the surrounding skin has become
inflamed, she has developed general malaise and her glycaemic control has worsened. Bone is visible
at the base of the ulcer. There is no history of antibiotic allergy. What is the most appropriate empiric
treatment pending the result of a swab?

A. Vancomycin and gentamicin.


B. Ticarcillin/Clavulanate.
C. Metronidazole.
D. Moxifloxacin.
E. Linezolid and ciprofloxacin.

Question 49
A 56-year-old man with a past history of bicuspid aortic valve develops bacteraemia with
Staphylococcus aureus and echocardiography shows a 1.0 cm vegetation on the aortic valve. He has
a known history of penicillin hypersensitivity – he reports a sudden onset of tongue and throat
swelling after receiving the drug when he was 20 years old.

What is the most appropriate intravenous antibiotic?

A. Ceftriaxone.
B. Meropenem.
C. Vancomycin.
D. Clindamycin.
E. Flucloxacillin.
236

Question 50
A 27-year-old medical student presents with three episodes of fevers, headache, chills and rigors and
loose stools over the past week. She returned from an elective placement in Papua New Guinea six
weeks previously. She took doxycycline for malaria prophylaxis. Examination reveals a fever of
40oC, but no other abnormal findings. Her chest X-ray is clear. The following test results were
obtained:

Haemoglobin (Hb) 115 g/L [115 – 135 g/L]


White cell count (WCC) within normal limits
Platelet count 120 x 109/L [150 – 400 x 109/L]
Thick and thin blood films no parasites detected
Liver function tests within normal limits

In addition to blood cultures, what is the most appropriate next step?

A. Repeat thick and thin blood film.


B. Dengue polymerase chain reaction (PCR).
C. Mantoux test.
D. Oral metronidazole.
E. Oral ciprofloxacin.

Question 51
A 20-year-old male with known human immunodeficiency virus (HIV) infection is admitted from
the community with a two day history of cough and sputum production. On examination he has a
temperature of 38oC and his chest X-ray shows patchy opacification of the right middle and lower
lobes.

The most likely organism responsible for his clinical features is:

A. Streptococcus pneumoniae.
B. Legionella pneumophila.
C. Mycobacterium avium-complex
D. Pneumocystis jiroveci (carinii).
E. Cytomegalovirus (CMV).
237

Answers
1. A 50. A
2. D 51. A
3. E
4. C
5. D
6. B
7. B
8. D
9. D
10. E
11. C
12. B
13. C
14. B
15. A
16. B
17. B
18. D
19. B
20. C
21. B
22. D
23. B
24. B
25. B
26. D
27. C
28. C
29 A
30. A
31. D
32. B
33. B
34. A
35. B
36. D
37. D
38. B
39. E
40. D
41. C
42. B
43. B
44. C
45. C
46. B
47. D
48. B
49. C
238

Medical Sciences

Basic Immunology

Question 1
A T lymphocyte encountering its specific antigen in the absence of a co-stimulatory signal via CD28
is least likely to:

A. Undergo apoptosis (programmed cell death).


B. Become anergic (unresponsive to appropriate stimulation).
C. Proliferate.
D. Produce low levels of helper T cell type 1 (Th1) cytokines.
E. Produce low levels of helper T cell type 2 (Th2) cytokines.

Question 2
Failure of signalling through the interleukin 7 receptor (IL-7 R), due to inherited deficiency of either
the common interleukin receptor • chain (•c) or Janus kinase 3 (JAK3 protein kinase), produces a
severe combined immune deficiency (SCID) state, the features of which are best explained by the
loss of expansion of which one of the following cell types?

A. Type 1 helper T lymphocytes (Th1 cells).


B. Type 2 helper T lymphocytes (Th2 cells).
C. B lymphocytes.
D. Natural killer (NK) cells.
E. Lymphoid progenitor cells.

Question 3
The large granular lymphocytes observed in the early cellular response to virus infected cells are
most likely to be:

A. B lymphocytes.
B. Naive T lymphocytes.
C. Memory CD4+ T lymphocytes.
D. Memory CD8+ T lymphocytes.
E. Natural killer (NK) cells.
239

Question 4
Cytokine regulation (especially involving interleukin 2) is least affected by which one of the
following immunosuppressive drugs?

A. Prednis(ol)one.
B. Mycophenolate.
C. Cyclosporin.
D. Tacrolimus (FK506).
E. Rapamycin.

Question 5
A type 2 helper T lymphocyte (Th2 cell) differs most from a type 1 helper T lymphocyte (Th1 cell)
in which one of the following?

A. Surface expression of CD4.


B. Surface expression of major histocompatibility gene complex (MHC) class II.
C. Production of interleukin 3 (IL-3).
D. Production of interleukin 5 (IL-5).
E. Responsiveness to antigen presented by macrophages.

Question 6
The low affinity antibodies produced following stimulation of B lymphocytes by a thymus
independent antigen, such as pneumococcal polysaccharide, belong predominantly to which one of
the following immunoglobulin classes/subclasses?

A. IgA.
B. IgG1.
C. IgG3.
D. IgM.
E. IgE.

Question 7
Dendritic cells are distinguished by an exceptional ability to carry out which one of the following
immune functions?

A. Phagocytosis.
B. Cytokine secretion.
C. Chemokine secretion.
D. Immunoglobulin secretion.
E. Antigen presentation.
240

Question 8
T lymphocytes expressing both CD4 and CD8 on the cell surface are most likely to be found in
which one of the following anatomical locations in healthy individuals?

A. Bone marrow.
B. Thymus.
C. Peripheral blood.
D. Lymph nodes.
E. Spleen.

Question 9
Which one of the following biologically active components of the complement cascade is most
directly responsible for the formation of transmembrane channels leading to cell lysis?

A. C3a.
B. C3b.
C. C4a.
D. C5a.
E. C5-9.

Question 10
At the onset of infection or inflammation, the level of which one of the following acute phase
proteins rises most rapidly and by the highest percentage over background?

A. Fibrinogen.
B. Haptoglobin.
C. Transferrin.
D. C-reactive protein.
E. Complement component 3 (C3).

Question 11
Diagnosis of a number of autoimmune diseases involves the detection of serum autoantibodies.
Which of the following criteria is most indicative of a pathogenic role for a disease-associated
autoantibody?

A. Demonstration of immunoglobulin at site of lesion.


B. Elution of immune complexes from site of lesion.
C. Reproduction of disease by passive transfer of patient immunoglobulin.
D. Clinical response to plasmapheresis.
E. Elution of complement from site of lesion.
241

Question 12
Which of the following conditions is most likely to be present in a patient with the nailfold lesions
shown below?

A. Polymyositis.
B. Systemic sclerosis.
C. Systemic lupus erythematosus (SLE).
D. Seropositive rheumatoid arthritis.
E. Multicentric reticulohistiocytosis.

Question 13
Urticaria, wheezing and hypotension may occur with each of the following agents. Which of the
following is most likely to have an immunoglobulin E (IgE)-mediated mechanism underlying this
reaction?

A. Radiographic contrast medium.


B. Captopril.
C. Acetylcysteine.
D. Vancomycin.
E. Penicillin.

Question 14
The cell population most responsible for driving the pathogenesis of sarcoidosis is:

A. Macrophage.
B. Fibroblast.
C. B lymphocyte.
D. Type 1 helper T lymphocyte (Th1 cell).
E. Type 2 helper T lymphocyte (Th2 cell).
242

Question 15
Which one of the following most differentiates between antigen-dependent and antigen-independent
B cell development?

A. B cell surface expression of surrogate light chain.


B. B cell surface expression of µ heavy chain.
C. B cell surface expression of CD79a (immunoglobulin α (Ig-α)).
D. Immunoglobulin gene rearrangement.
E. Somatic hypermutation.

Question 16
Which one of the following cells of the immune system is most reliant on its release of mediators
into the extracellular fluid surrounding the organism in order to carry out its role in eradication of
foreign organisms?

A. Neutrophils.
B. Eosinophils.
C. Macrophages.
D. Cytotoxic T lymphocytes.
E. Natural killer cells.

Question 17
Which of the following most accurately reflects the mechanism of complement consumption induced
by complement component 3 (C3) nephritic factors?

A. Activation of the classical complement pathway.


B. Stabilisation of the membrane attack complex, C5-9.
C. Amplification of C3 “tickover”.
D. Stabilisation of the C3 convertase, C3bBb.
E. Inhibition of the complement regulator, Factor I.

Question 18
Efalizumab is a humanized monoclonal antibody active against an adhesion molecule (CD11a) and is
used in the treatment of psoriasis. Which of the following is the best explanation for the therapeutic
effect of efalizumab?

A. Interference with extravasation of circulating T lymphocytes into skin.


B. Depletion of peripheral blood T lymphocyte numbers.
C. Inhibition of cytokine secretion by phagocytes.
D. Inhibition of chemokine secretion by epithelial cells.
E. Interference with danger signals released from inflamed skin.
243

Question 19
Which of the following is the strongest contraindication to the administration of influenza vaccine?

A. Egg anaphylaxis.
B. Previous local reaction to influenza immunization.
C. Recent pneumococcal immunization.
D. Pregnancy.
E. Immunodeficiency.

Question 20
A 67-year-old man has recently commenced haemodialysis for renal failure as a result of chronic
glomerulonephritis. He receives immunisation with three separate doses of hepatitis B vaccine
according to the recommended protocol. Six weeks after the third dose, his anti-hepatitis B surface
antigen antibody level is <10 mU/mL. Which one of the following is the most appropriate course of
action?

A. Reimmunise using Freund’s adjuvant.


B. Repeat anti-hepatitis B testing in 3 months.
C. No further hepatitis B immunization.
D. Give hepatitis B immunoglobulin.
E. Immunise with up to 3 further doses of hepatitis B vaccine.
244

Question 21
A 54-year-old woman presents with a six-month history of intensely itchy papules and vesicles on
elbows, knees and buttocks as shown above. She is systemically well other than mild diarrhoea.
Blood count is reported as normal. Serum ferritin is 5 µg/L [20-380] and red blood cell folate is 180
nmol/L [600-1600].

Which of the following investigations is most likely to confirm the diagnosis of the rash shown
above?

A. IgA endomysial antibodies.


B. Direct immunofluorescence of perilesional skin.
C. Light microscopy of unexcoriated vesicle.
D. Jejunal biopsy.
E. Response to gluten-free diet.

Question 22
An immediate generalised reaction to which of the following drugs is most likely to be mediated by
IgE (immunoglobulin E)?

A. Vancomycin.
B. Radiographic contrast media.
C. Aspirin.
D. Cephalothin.
E. Dextran.
245

Question 23
Which of the following best describes the toxicological mechanism of recurrent acute angioedema
related to the use of angiotensin converting enzyme (ACE) inhibitors?

A. IgE (immunoglobulin E) mediated hypersensitivity.


B. Delayed-type hypersensitivity.
C. Enhancement of complement activation.
D. Disruption of endothelial tight junctions.
E. Impairment of bradykinin catabolism.

Question 24
In which of the following does mannose binding lectin (MBL) have the most significant role?

A. Carbohydrate metabolism.
B. Fat metabolism.
C. Innate immunity.
D. Cardiovascular homeostasis.
E.Adaptive immunity

Question 25
In a patient with vasculitis whose serum contains perinuclear anti-neutrophil cytoplasmic antibodies
(p-ANCA), what is the most likely target antigen for this antibody?

A. Proteinase 3.
B. Myeloperoxidase.
C. Lactoferrin.
D. Cathepsin G.
E. Lyzosyme.

Question 26
Secretion of which of the following cytokines is most characteristic of T helper cells from patients
with cell-mediated immune disorders such as multiple sclerosis?

A. Interleukin - 4 (IL-4).
B. Interleukin - 10 (IL-10).
C. Interleukin - 12 (IL-12).
D. Interferon - gamma (IFN-g).
E. Interferon - beta (IFN-b).
246

Question 27
Desensitisation is most likely to be effective in individuals who have had which of the following
reactions to penicillin?

A. Haemolytic anaemia.
B. Laryngeal oedema.
C. Stevens-Johnson Syndrome.
D. Toxic epidermal necrolysis.
E. Immune complex reactions.

Question 28
A novel therapeutic agent is developed which increases numbers of CD4+ CD25+ regulatory T cells.
For which of the following conditions is there the most potential for benefit from treatment with this
agent?

A. Alzheimer’s disease.
B. HIV (human immunodeficiency virus) infection.
C. Idiopathic CD4 lymphopenia.
D. Hodgkin’s disease.
E. Multiple sclerosis.

Question 29
Which of the following manifestations of natural rubber latex (NRL) sensitivity is least likely to be
associated with circulating latex-specific Immunoglobulin E (IgE)?

A. Urticaria.
B. Occupational asthma.
C. Rhinoconjunctivitis.
D. Contact dermatitis.
E. Angioedema.

Question 30
The following proteins are all components of amyloid deposits. Which is universally present?

A. Transthyretin.
B. Serum amyloid P (SAP).
C. β2-microglobulin.
D. Immunoglobulin light chain.
E. Serum amyloid A (SAA).
247

Question 31
A certain autosomal recessive disorder affects 1 in 1600 people; the carrier frequency is 5%. A DNA
assay can identify the mutation in 80% of carriers; the false-positive rate of this assay is zero. What
is the best estimate of the positive predictive value (PPV) and negative predictive value (NPV) of
this assay in screening the population for carriers?
248

Answers

1. C
2. E
3. E
4. B
5. D
6. D
7. E
8. B
9. E
10. D
11. C
12. B
13. E
14. D
15. E
16. B
17. D
18. A
19. A
20. E
21. B
22. D
23. E
24. C
25. B
26. D
27. B
28. E
29. D
30. B
31. C
249

Clinical Applications

Clinical Immunology

Question 1
A 42-year-old woman with diffuse proliferative glomerulonephritis secondary to systemic lupus
erythematosus (SLE) presents with the rash shown in the two photographs below, three months after
commencement of therapy with high dose prednis(ol)one, 1 mg/kg/day and cyclophosphamide, 1
g/m2/month by intravenous administration.

Reduced function of which one of the following immunological effector mechanisms is likely to be
the main contributor to the severity of the rash?

A. Complement.
B. Neutrophils.
C. Specific antibody.
D. Natural killer (NK) cells.
E. Specific cell-mediated immunity.
250

Question 2
A 32-year-old woman with mitral valve disease was given an injection of penicillin immediately
prior to a general anaesthetic for extraction of impacted wisdom teeth. She developed hypotension
and exhibited difficulty with ventilation.

Immediately following resuscitation, which one of the following tests would be most helpful to
confirm a diagnosis of anaphylaxis?

A. Penicillin radioallergosorbent test (RAST).


B. Skin testing for the anaesthetic agents used.
C. Serum tryptase.
D. Complement component (C1) inhibitor level.
E. Serum histamine.

Question 3
A 27-year-old woman presents with an allergic reaction to a bee sting. Which one of the following
clinical features, listed in order of severity, would be the minimum requirement sufficient to advise
bee venom immunotherapy?

A. Large local reaction.


B. Localised urticaria.
C. Generalised urticaria.
D. Distant angio-oedema.
E. Anaphylaxis.

Question 4
Which one of the following is the most common cause of initial-onset of angio-oedema in patients
over the age of 60?

A. Food allergy.
B. Hereditary angio-oedema.
C. Acquired C1 inhibitor deficiency.
D. Penicillin allergy.
E. Angiotensin-converting enzyme (ACE) inhibitor therapy.
251

Question 5
An 87-year-old man presents with extensive intact blisters over his trunk and all limbs but with
sparing of his face and mucous membranes. Direct immunofluorescence on biopsy of perilesional
skin shows a linear deposition of immunoglobulin G (IgG) and complement component 3 (C3) at the
dermal- epidermal junction.

Which of the following would be the most appropriate initial treatment?

A. Azathioprine 2 mg/kg/day.
B. Oral prednis(ol)one 2 mg/kg/day.
C. Topical high-potency corticosteroids.
D. Plasmapheresis, five times over two weeks.
E. Intravenous cyclophosphamide 0.7 g/m2 body surface area.

Question 6
A 45-year-old woman presents with a three-year history of recurrent rash, characterised as palpable
purpura over both legs. Some of these lesions have ulcerated.

Laboratory results include:

serum:
creatinine 0.10 mmol/L [0.06-0.11]
alanine transaminase (ALT) 51 U/L [15-50]
aspartate transaminase (AST) 64 U/L [15-45]
hepatitis C antibody positive
cryoglobulins positive with 8% cryoprecipitate
urinalysis normal

Which of the following is the most appropriate management?

A. Plasmapheresis and corticosteroids.


B. Corticosteroids and interferon α.
C. Interferon α and ribavirin.
D. Cyclophosphamide and corticosteroids.
E. Cyclophosphamide and ribavirin.
252

Question 7
A previously well 58-year-old woman presents with a two-year history of marked sicca symptoms,
intermittent bilateral parotid swelling and an abnormal Schirmer’s test. Investigations reveal a
positive antinuclear antibody (ANA) test (titre >1:2560), speckled pattern with positive Ro
(Sjögren’s syndrome A (SS-A)) and La (Sjögren’s syndrome B (SS-B)) antibody tests and negative
anti-DNA antibodies. Rheumatoid factor is strongly positive and marked polyclonal gammopathy is
noted on serum protein electrophoresis (EPG). Full blood count is normal. Serum biochemistry is
normal except for a serum bicarbonate of 15 mmol/L [24-31].

In the long-term follow-up, which of the following features is least likely to occur?

A. Glomerulonephritis.
B. Raynaud’s phenomenon.
C. Arthritis.
D. Nephrocalcinosis.
E. Lymphoma.

Question 8
A 25-year-old female presents with recurrent facial angioedema. The following laboratory results are
obtained:

Complement C3 0.36 g/L [0.79-1.80]


Complement C4 0.06 g/L [0.13-0.48]
Total haemolytic complement (CH100) 210 CH100 units [390-1015]

Which of the following is the most likely explanation for these findings?

A. Hereditary C1 (complement 1) inhibitor deficiency.


B. Delayed transfer of serum to laboratory.
C. C4 null allele.
D. Acquired C1 inhibitor deficiency.
E. ACE-inhibitor therapy.

Question 9
A patient is at high risk for repeat radiographic contrast media reactions. Which of the following is
least likely to be effective?

A. Desensitisation with contrast media.


B. Change to use of non-ionic contrast media.
C. Premedication with oral diphenhydramine.
D. Premedication with oral prednis(ol)one.
E. Premedication with intramuscular adrenaline.
253

Question 10
A 21-year-old female presents with anaphylactic shock. Her blood pressure initially responds to a
dose of adrenaline however it falls again 30 minutes later. What would be the most important next
intervention?

A. Subcutaneous adrenaline.
B. Intramuscular adrenaline.
C. Intravenous fluid challenge.
D. Histamine 1 and 2 receptor antagonists.
E. Intravenous corticosteroid.

Question 11
Skin prick testing for specific IgE (immunoglobulin E) is most appropriate for investigation of which
of the following conditions?

A. Severe systemic reaction to gadolinium-based contrast medium.


B. Chronic urticaria.
C. Acute urticaria/angioedema following aspirin administration.
D. Large local reaction to bee sting.
E. Anaphylaxis following peanut ingestion.

Question 12
A 55-year-old male with a history of controlled hypertension treated with metoprolol presents with
anaphylaxis characterised by hypotension, laryngeal oedema and generalised urticaria occurring 20
minutes after eating a seafood banquet. He remains hypotensive (systolic BP < 90mmHg) despite
repeated intramuscular boluses of adrenaline to a total dose of 1.3mg, intravenous infusion of
crystalloid 1000mL, and intravenous injection of ranitidine 50mg and promethazine 25mg.

Which of the following is the most appropriate next step in management?

A. Further intravenous infusion of crystalloid 1000mL.


B. Intravenous injection of hydrocortisone100mg.
C. Further intramuscular adrenaline 0.5mg.
D. Intravenous injection of 0.3mg adrenaline.
E. Intravenous injection of glucagon 5mg.
254

Question 13
A 45-year-old male develops a blistering skin rash as shown below during treatment with
cotrimoxazole.

A diagnosis of drug allergy is suspected.

Which of the following is the most appropriate next step?

A. Skin testing with trimethoprim.


B. Skin testing with sulphamethoxazole.
C. Oral challenge with co-trimoxazole.
D. RAST (radioallergosorbent test) for co-trimoxazole.
E. No further investigations necessary.

Question 14
A 20-year-old female presents with a one month history of malaise, nausea, abdominal pain, itch and
polyarthralgia. She gives no history of recent use of prescribed or illicit drugs. Examination reveals
jaundice, multiple bruises, and tender hepatomegaly. Laboratory results are consistent with an acute
hepatocellular injury, with a polyclonal increase in IgG (immunoglobulin G) of 32g/L (6.1-15.5).
Serological tests for hepatitis A, B, and C viruses are negative. Histopathological findings on liver
biopsy are of a periportal mixed mononuclear cell infiltrate of plasma cells, lymphocytes and
eosinophils consistent with an autoimmune hepatitis.

Which of the following autoantibodies, if present, is most predictive of a poor clinical and
biochemical response to therapy?

A. Anti-nuclear antibody.
B. Anti-smooth muscle antibody (F-actin–specific).
C. Anti-mitochondrial antibody.
D. Anti-liver-kidney microsomal antibody.
E. Atypical perinuclear anti-neutrophil cytoplasmic antibody.
255

Question 15
Among patients with phospholipid antibodies, which of the following factors most increases the risk
of developing new thrombotic events?

A. History of recurrent foetal loss.


B. Prior stroke.
C. Increasing antibody titre.
D. Recent discontinuation of warfarin therapy.
E. Cigarette smoking.

Question 16
A 25-year-old female with a history of allergic rhinitis and mild atopic dermatitis is currently
pregnant
with her second child. Her two-year-old son has infantile eczema and peanut allergy confirmed on
skin prick testing. The patient requests advice about dietary interventions during this pregnancy to
reduce the risk of the second child developing peanut sensitisation.

Which of the following is the most appropriate advice?

A. Avoidance of peanuts and peanut products.


B. Avoidance of all nuts.
C. Omega-3 fatty acid supplementation.
D. Elimination diet (low allergen, additive and preservative).
E. No specific dietary intervention

Question 17
A 21-year-old male is referred for management of severe rhinosinusitis. He describes symptoms of
chronic nasal congestion, post-nasal drip and headache. Symptoms are acutely worse after aspirin
ingestion which also results in development of dyspnoea and wheeze. Examination reveals bilateral
nasal polyposis with nasal airway obstruction. Skin prick testing for common aeroallergens is
negative. In addition to advising avoidance of aspirin and non-steroidal anti-inflammatory agents,
which of the following is the most appropriate first line of therapy for management of rhinosinusitis
in this patient?

A. Referral for surgical polypectomy.


B. Intranasal corticosteroid spray.
C. Oral histamine-1 (H1) – blocker.
D. Aspirin desensitisation.
E. Oral montelukast.
256

Question 18
A 15-year-old male presents with acute onset of symptoms of upper airway obstruction. He has a
history of hereditary angioneurotic oedema. Which of the following is the most effective therapy?

A. Histamine-1 (H1) blockers.


B. Intravenous glucocorticoids.
C. Fresh frozen plasma.
D. Intramuscular adrenaline.
E. C1 inhibitor concentrate.
257

Answers

1. E
2. C
3. E
4. E
5. C
6. C
7. A
8. B
9. A
10. B
11. E
12. E
13. E
14. D
15. D
16. B
17. E
18. E
258

Medical Sciences

Neurology

Question 1
Which one of the following is the strongest independent risk factor for stroke?

A. Hypertension.
B. Cigarette smoking.
C. Physical inactivity.
D. Age.
E. Hypercholesterolaemia.

Question 2
The visual evoked responses (VER) shown were recorded in a 20 year old woman who
presented with a two day history of visual loss in the left eye.

The most likely diagnosis is:

A. Retinal detachment.
B. Optic neuritis.
C. Retinal artery occlusion.
D. Conversion disorder.
E. Retinal migraine.
259

Question 3
In which one of the following conditions are nerve conduction studies and electromyography (EMG)
least likely to give useful diagnostic information?

A. Carpal tunnel syndrome.


B. Deep branch of ulnar nerve lesion.
C. Radial nerve palsy.
D. Sciatica.
E. Polymyositis.
D. Systolic blood pressure less than 160 mmHg.
E. Elevated diastolic blood pressure

Question 4
An abnormality in which one of the following investigations is of most value in confirming
suspected motor neuron disease?

A. Sural nerve biopsy.


B. Magnetic resonance imaging (MRI) of the brain and cervical cord.
C. Electromyography (EMG).
D. Nerve conduction studies.
E. Cerebrospinal fluid protein concentration.

Question 5
A 21 year old student presents with a painless right wrist drop. The most likely site of pathology is
the:

A. Right radial nerve.


B. Right C7 nerve root.
C. Right extensor digitorum communis muscle.
D. Left internal capsule.
E. Right brachial plexus.

Question 6
In a patient with oropharyngeal dysphagia following a stroke, which one of the following would be
the strongest indicator of increased risk of aspiration?

A. Facial weakness with drooling.


B. Refusal to eat.
C. Moist vocal quality and weak cough.
D. Bilateral absent gag reflex.
E. Dysarthria.
260

Question 7
The most useful investigation for the diagnosis of multiple sclerosis is:

A. Cerebrospinal fluid protein concentration.


B. Visual evoked responses.
C. Magnetic resonance imaging (MRI) scan of the brain.
D. Serum oligoclonal bands.
E. Cerebrospinal fluid microscopy.

Question 8
Vertigo which occurs with neck extension is most likely due to:

A. Cervical spondylosis.
B. Benign paroxysmal positional vertigo.
C. Kinking of the vertebral artery.
D. Endolymphatic hydrops.
E. Chiari 1 malformation.

Question 9
Which one of the following is the strongest independent risk factor for stroke?

A. Hypertension.
B. Cigarette smoking.
C. Physical inactivity.
D. Age.
E. Hypercholesterolaemia.

Question 10
A 22 year old woman has been on a stable dose of anticonvulsant for three years. She has recently
experienced difficulties in seeing at night. A formal ophthalmological assessment demonstrates
visual field constriction.

For which one of the following anticonvulsants is it important to monitor visual fields?

A. Carbamazepine.
B. Gabapentin.
C. Lamotrigine.
D. Valproate.
E. Vigabatrin.
261

Question 11
In complex partial seizures of temporal lobe type, the most common finding at surgery for the
epilepsy is:

A. Mesial temporal sclerosis.


B. Neuronal migration disorder.
C. Old infarction.
D. Normal brain.
E. Cavernous angioma.

Question 12
In a patient with oropharyngeal dysphagia following a stroke, which one of the following would be
the strongest indicator of increased risk of aspiration?

A. Facial weakness with drooling.


B. Refusal to eat.
C. Moist vocal quality and weak cough.
D. Bilateral absent gag reflex.
E. Dysarthria.

Question 13
A 25 year old man dislocates his right shoulder while playing football. This is reduced in the
emergency department and he is discharged after observation. After removal of his sling he has
problems with abduction of the arm. Examination shows weakness of abduction at the shoulder and
sensory loss on the lateral aspect of his upper arm.

The nerve involved is the:


A. Accessory nerve.
B. Axillary nerve.
C. Musculocutaneous nerve.
D. Radial nerve.
E. Suprascapular nerve.

Question 14
Which one of the following histopathological features is least likely to be seen in cranial (temporal)
arteritis?

A. Thickening of the intima.


B. Fragmentation of the internal elastic lamina.
C. Panarteritis.
D. Presence of giant cells.
E. Aneurysmal dilatation.
262

Question 15
A patient presents 12 hours after a thunderclap headache. A contrast cranial computed tomography
(CT) scan is normal. To determine if there is aneurysmal bleeding, the most appropriate next
investigation is:

A. Cranial magnetic resonance imaging (MRI) with diffusion weighted imaging.


B. Cerebrospinal fluid (CSF) spectroscopy for bilirubin.
C. CSF spectroscopy for crenated red blood cells.
D. Four vessel cerebral angiogram.
E. Cranial magnetic resonance imaging/angiography (MRI/MRA).

Question 16
Multiple sclerosis is a disease primarily of the:

A. Oligodendrocyte.
B. Schwann cell.
C. Astrocyte.
D. Axon.
E. Neuronal cell body.

Question 17
In patients dying of Sudden Unexpected Death in Epilepsy (SUDEP), the most likely
mechanism of death is:

A. Aneurysmal rupture.
B. Central hypoventilation.
C. Ventricular fibrillation.
D. Neurogenic pulmonary oedema.
E. Laryngospasm.

Question 18
A 56 year old man is noted to have a right foot drop three days following a right total hip
replacement. On examination, there is weakness of right ankle dorsiflexion and toe extension (grade
4/5). Other muscle groups are normal. The knee jerks are symmetrical with an absent right ankle
jerk. Sensation is reduced on the sole and dorsum of the right foot.

The most likely diagnosis is:

A. Femoral neuropathy.
B. Sciatic neuropathy.
C. L5 radiculopathy.
D. Tibial neuropathy.
E. Peroneal neuropathy.
263

Question 19
A 55 year old woman presents with right shoulder pain and a cough. Her chest Xray is shown below.
A bronchoscopy confirms a malignant process. The most likely neurological complication of a
tumour in this location would be involvement of the:

A. Axillary nerve.
B. Stellate ganglion.
C. Parasympathetic chain.
D. Recurrent laryngeal nerve.
E. Coeliac plexus.

Question 20
In patients presenting with subacute lower extremity symptoms, the presence of pyramidal lower
extremity weakness, a mid-thoracic sensory level to pin-prick but relatively preserved posterior
columns is most consistent with:

A. Subacute combined degeneration of the cord.


B. Brown-Séquard syndrome.
C. Syringomyelia.
D. A central cord syndrome.
E. An anterior spinal artery syndrome.
264

Question 21
Which one of the following best explains the mechanism of trigeminal neuralgia?

A. Herpes virus ganglionitis.


B. Microvascular compression.
C. Central demyelination.
D. Vascular steal phenomena.
E. Trigeminal neuroma.

Question 22
In a patient with unilateral facial weakness, which one of the following findings on physical
examination most strongly suggests a peripheral cause for the facial weakness?

A. Frontalis weakness.
B. Associated lateral rectus weakness.
C. Loss of taste.
D. Facial numbness.
E. Nystagmus.

Question 23
A 57 year old man with a long history of poorly controlled hypertension and smoking presents with
sudden onset of right sided weakness. On examination, his speech is slurred but content and
comprehension are normal. There is a right facial weakness, with the right side of the soft palate
elevating poorly. He has a right hemiparesis, power grade 2-3/5, in a pyramidal distribution with
normal sensation to all modalities. Right sided reflexes are enhanced with an extensor right plantar
response. The most likely cause of this stroke syndrome is left:

A. Middle cerebral artery occlusion.


B. Posterior inferior cerebellar artery occlusion.
C. hemispheric cortically based haemorrhage.
D. middle cerebral penetrating artery occlusion.
E. internal carotid artery occlusion.

Question 24
Diabetic neuropathic arthropathy is most likely to affect which of the following areas?

A. Knee.
B. Mid-foot.
C. 1st metatarsophalangeal joint.
D. Elbow.
E. Ankle.
265

Question 25
A patient presents with weakness of the right hand and face, but sparing the leg. The most
likely anatomic location of the stroke is:

A. Cerebral cortex.
B. External capsule.
C. Internal capsule.
D. Pons.
E. Medulla.

Question 26
A 66-year-old man presents with numbness involving the right thumb and index finger, and the first
dorsal webspace and dorsoradial forearm. There is no associated neck pain. On examination, power
is normal. The biceps and triceps reflexes are normal and symmetrical. The brachioradialis reflex is
reduced on the right.

The most likely diagnosis is:

A. Median neuropathy.
B. C6 radiculopathy.
C. Radial neuropathy.
D. Brachial plexopathy.
E. Cortical stroke.

Question 27
Memory loss in Alzheimers disease most closely correlates with changes in which brain region?
A. Cingulate gyrus.
B. Thalamus.
C. Frontal lobe.
D. Amygdala.
E. Hippocampus.
266

Question 28
An 18-year-old gives a history of recurrent morning myoclonus and absences after sleep deprivation.

The most likely epilepsy syndrome is:

A. Juvenile myoclonic epilepsy.


B. Benign myoclonic epilepsy.
C. Petit mal epilepsy.
D. Benign Rolandic epilepsy.
E. Absence epilepsy.

Question 29
The first manifestation of illness in a patient with new variant Creutzfeldt-Jacob Disease (nvCJD) is
most likely to be which of the following?

A. Psychiatric.
B. Myoclonus.
C. Ataxia.
D. Slurred speech.
E. Epilepsy.

Question 30
The aura in migraine is most likely to be due to:

A. Neurogenic inflammation.
B. Neuronal dysfunction.
C. Cerebral vasoconstriction.
D. Cerebral vasodilatation.
E. Central sensitisation.

Question 31
The most common pathology underlying temporal lobe epilepsy is:

A. Temporal neuronal migration abnormality.


B. Temporal dysembryoplastic neuroepithelial tumour (DNET).
C. Mesial temporal sclerosis.
D. Temporal lobe dysplasia.
E. Temporal demyelination.
267

Question 32
The computed tomography (CT) scan below shows an abnormality.

In addition to hemiparesis, which of the following clinical signs would you expect to be associated
with this abnormality?

A. Acalculia and finger agnosia.


B. Hemianopia and neglect.
C. Quadrantanopia and neglect.
D. Receptive aphasia.
E. Global aphasia.

Question 33
A 52-year-old woman presents with loss of feeling in the outer aspect of her left foot, which is made
worse by standing. She has some difficulty standing on tiptoe on the left and has an absent left ankle
jerk. This is most likely:

A. An S1 radiculopathy.
B. Common peroneal nerve compression.
C. An L5 radiculopathy.
D. Posterior tibial nerve compression.
E. Thoracic myelopathy
268

Question 34
A 38-year-old man presents complaining that he is no longer able to read his newspaper while
commuting on the train. On examination, he has an abnormality of gaze on looking to the right; his
right eye develops nystagmus and his left eye is slow to come across to the nasal side. This problem
is most likely due to a structural lesion in which area?

A. Medulla.
B. Cerebellum.
C. Right semi-circular canals.
D. Midbrain.
E. Right lateral geniculate body.

Question 35
In a patient with extrapyramidal symptoms, which of the following is more suggestive of Lewy body
disease than Parkinson’s disease?

A. Rest tremor.
B. Rigidity.
C. Visual hallucinations.
D. Response to dopaminergic agonists.
E. Bradykinesia.

Question 36
When an individual denies their blindness (Anton’s syndrome), it is most likely to be due to:

A. Bilateral middle cerebral artery infarction.


B. Infarction of the optic chiasm following pituitary apoplexy.
C. Bilateral optic neuritis.
D. Bilateral posterior cerebral artery infarction.
E. Mania in bipolar illness.
269

Answers

1. D
2. B
3. D
4. C
5. A
6. C
7. C
8. B
9. D
10. E
11. A
12. C
13. B
14. E
15. B
16. A
17. B
18. B
19. B
20. E
21. B
22. C
23. D
24. B
25. A
26. B
27. E
28. A
29. A
30. B
31. C
32. B
33. A
34. D
35. C
36. D
270

Clinical Applications

Neurology

Question 1
Prednis(ol)one is not a recognised treatment for which one of the following conditions?

A. Myasthenia gravis.
B. Temporal arteritis.
C. Polymyositis.
D. Neurosarcoidosis.
E. Acute demyelinating polyneuropathy (Guillain Barré syndrome).

Question 2
A 52 year old retired school teacher with a 20 year history of type 2 (non-insulin-dependent) diabetes
mellitus and a 10 year history of hypertension presents with a two day history of progressive right
sided ptosis and diplopia. Examination reveals signs consistent with an incomplete right third nerve
palsy with pupil sparing. The erythrocyte sedimentation rate
(ESR) is 45 mm/h [0-15].

The most likely diagnosis is:

A. Diabetic right third nerve palsy.


B. Mucormycosis of the right orbit.
C. Right posterior communicating artery aneurysm.
D. Giant cell arteritis involving the right third nerve.
E. Right mid-brain infarct.

Question 3
Dementia is least likely to be a feature in which one of the following conditions?

A. Huntingtonʹs disease.
B. Progressive supranuclear palsy.
C. Motor neuron disease.
D. Multiple sclerosis.
E. Parkinson’s disease.
271

Question 4
Which one of the following features is least characteristic of transient global amnesia?

A. Absence of focal neurological signs.


B. Normal conscious state.
C. Abnormal cerebral computed tomography (CT) scan.
D. Symptoms lasting four to six hours.
E. Normal baseline EEG.

Question 5
A 52 year old man presents with a three month history of general ill health, fevers and weight loss.
Examination reveals hypertension (blood pressure of 150/110 mmHg), left wrist drop and weakness
of dorsiflexion of the left foot.

Which one of the following investigations is most likely to be diagnostic?

A. Anti-ganglioside M1 (Anti GM1) antibodies.


B. C-reactive protein level.
C. Magnetic resonance imaging (MRI) scan of the lumbar spine.
D. Lumbar puncture.
E. Left sural nerve biopsy.

Question 6
In a patient with relapsing-remitting multiple sclerosis, which one of the following is most likely to
delay disease progression?

A Corticosteroids.
B. Azathioprine.
C. Cyclophosphamide.
D. Interferon beta.
E. Cyclosporin.
272

Question 7
Which one of the following clinical features is most characteristic of the condition shown in the
cervical magnetic resonance imaging (MRI) scan?

A. Spasticity of the upper limbs.


B. Dissociated sensory loss of the upper limbs.
C. Normal reflexes in the upper limbs.
D. Gait ataxia.
E. Extensor plantar responses.

Question 8
A 25 year old woman presents with a six week history of headaches and intermittent blurring of
vision. Examination reveals obesity and bilateral papilloedema. A computed tomography (CT) scan
of the brain with contrast is normal.

The most appropriate next investigation is:

A. Cerebral magnetic resonance venography.


B. Cerebral magnetic resonance imaging (MRI) scan.
C. Lumbar puncture.
D. Cerebral angiography.
E. Computed tomography (CT) scan of the orbits.
273

Question 9
A 39 year old woman presents with a three month history of numbness, pins and needles and pain in
the right hand involving all fingers which wakes her from sleep at night. Nerve conduction studies
show marked slowing of the right median motor distal latency, absent median finger sensory
potentials, reduced amplitude of the median ascending action potential and normal conduction of the
ulna nerve.

The most appropriate next step in management is:

A. Nerve conduction studies of the lower limbs.


B. Xray of the right wrist and right elbow.
C. Right carpal tunnel decompression.
D. Surgical exploration of the right proximal median nerve.
E. Use of splint and diuretics.

Question 10
A 68 year old woman with longstanding rheumatoid arthritis presents with a four month history of
worsening occipital headache. She has advanced peripheral joint damage and a restricted range of
movement of the cervical spine. There is mild proximal and distal lower limb weakness with hyper
reflexia in both the upper and lower limbs. The plantar responses are flexor.

Investigations show:
haemoglobin 108 g/L [120-160]
white cell count normal
platelet count normal
erythrocyte sedimentation rate (ESR) 105 mm/h [0-20]
Liver function tests are normal except for a serum alkaline phosphatase level of 146 U/L [30-115].

What is the next most appropriate investigation?

A. Lateral view Xray of the cervical spine with flexion and extension views.
B. Cervical magnetic resonance imaging (MRI) scan.
C. Temporal artery biopsy.
D. Upper limb electromyography.
E. Bone scan.

Question 11
Dementia is least likely to be a feature in which one of the following conditions?

A. Huntingtonʹs disease.
B. Progressive supranuclear palsy.
C. Motor neuron disease.
D. Multiple sclerosis.
E. Parkinson’s disease.
274

Question 12
A 56 year old man presents with a left hemispheric transient ischaemic attack. Carotid angiography
reveals 80% stenosis of the left internal carotid artery.

The most appropriate management is:

A. Carotid angioplasty.
B. Carotid endarterectomy.
C. Warfarin therapy.
D. Tissue-plasminogen activator (t-PA) therapy.
E. Treatment with aspirin and dipyridamole.

Question 13
Prednis(ol)one is not a recognised treatment for which one of the following conditions?

A. Myasthenia gravis.
B. Temporal arteritis.
C. Polymyositis.
D. Neurosarcoidosis.
E. Acute demyelinating polyneuropathy (Guillain Barré syndrome).

Question 14
A 16 year old girl presents with a three month history of blank spells. The 16 lead
electroencephalography (EEG) shows normal background rhythm and right temporal sharp
wave activity.

The drug of choice in the treatment of this condition is:

A. Carbamazepine.
B. Ethosuximide.
C. Phenytoin.
D. Lamotrigine.
E. Vigabatrin.
275

Question 15
A 30 year old woman presents with a six week history of numbness of the right arm and leg,
weakness of the left leg and mild hesitancy of urination. She takes thyroxine for hypothyroidism.
Cranial nerve examination is normal. In the left upper and lower limbs, tone is mildly increased,
power is reduced to 4+/5 and reflexes are increased. The left plantar response is equivocal. Pinprick
and temperature sensation are reduced on the right side below the C3 level. The cervical magnetic
resonance imaging (MRI) scan is shown below.

Which is the most appropriate next investigation?

A. Median and tibial nerve somatosensory evoked potentials.


B. Cranial MRI scan.
C. Biopsy.
D. Spinal angiography.
E. Visual evoked potentials.
276

Question 16
A 24 year old man, with a history of idiopathic epilepsy from the age of 15 years, presents with poor
seizure control. He is employed on a casual basis as a barman, and drinks 5-6 standard drinks per day
on average. He also smokes 20 cigarettes per day and smokes marijuana occasionally. He has been
on phenytoin monotherapy since June 1999 after suffering gastrointestinal adverse effects from
carbamazepine. His phenytoin concentrations have been very variable, as shown in the following
Table. The therapeutic range for the laboratory is 10-20 mg/L.

DATE PRESCRIBED CLINICAL SITUATION PHENYTOIN


DOSE CONCENTRATION
(MG/DAY)
16 JULY 1999 360 Routine test; seizure-free for past 16.7
three months
24 SEPT 1999 360 Accident and Emergency Department; 4.5
two seizures in past two days; dose
increased
28 SEPT 1999 400 In-patient; no seizures since 25 18.3
September
31 OCT 1999 400 Out-patient visit; nausea and vertigo 26.6
for past two weeks; seizure-free since
discharge on 1 October; dose reduced
5 DEC 1999 330 Follow-up test; well and seizure-free 13.2
15 JAN 2000 330 Accident and Emergency Department; 3.8
five seizures over the past week

Which one of the following is the most likely cause of the low concentrations measured in
the Accident and Emergency Department on 24 September 1999 and 15 January 2000?

A. Concomitant use of illicit drugs which induce hepatic enzymes.


B. Intermittent excessive alcohol intake.
C. Intermittent heavy tobacco smoking.
D. Concomitant use of antacids, reducing gastrointestinal absorption.
E. Poor compliance.

Question 17
A 23 year old female secretary presents with a 12 month history of tremor of the hands, minimal at
rest and increasing with intention. Her thyroid function tests are normal and she is on no medication.

Which one of the following is least characteristic of the tremor in this person?

A. A family history of similar tremor.


B. Improvement with alcohol.
C. Response to beta blockers.
D. Response to levodopa (L dopa).
E. Exacerbation with anxiety.
277

Question 18
A 24 year old man developed quadriparesis within five days of the onset of an acute demyelinating
polyneuropathy, which occurred two weeks after an episode of gastroenteritis. Stool cultures were
positive for Campylobacter jejuni.

This sequence of events is most likely to be the result of which one of the following?

A. Chance occurrence.
B. Side effects of antibiotic therapy for the gastroenteritis.
C. Peripheral nerve damage by C. jejuni toxins.
D. Molecular mimicry between antigens of C. jejuni and peripheral nerves.
E. HIV (human immunodeficiency virus) induced immune deficiency.

Question 19
A 57 year old man with a long history of poorly controlled hypertension and smoking presents with
sudden onset of right sided sensory change and mild change of speech. On examination, his speech is
slurred but content and comprehension are normal. He has a mild right hemisensory change to pin-
prick, with normal power and symmetrical reflexes.

His magnetic resonance imaging (MRI) scans are shown below (T2 weighted axial image (A)
and diffusion weighted image (B).

The most likely cause of the stroke syndrome is:

A. Left middle cerebral artery thrombosis.


B. Left basal ganglia haemorrhage.
C. Amyloid angiopathy.
D. Left middle cerebral penetrating artery occlusion.
E. Acute demyelination.
278

Question 20
An 88 year old man is brought to the emergency department following sudden onset of headache and
left hemiparesis. He has a history of recurrent falls and treated polymyalgia rheumatica. He has no
history of hypertension. Over the last year, he has generally been well except for some cognitive
impairment and transient left-sided weakness. A computed tomography (CT) scan reveals a
subcortical intracerebral haemorrhage in the right posterior frontal lobe.

Which of the following is the most likely cause of this haemorrhage?

A. Cerebral amyloid angiopathy.


B. Hypertension.
C. Giant cell arteritis.
D. Arteriovenous malformation.
E. Trauma.

Question 21
A 42 year old man presents with a two year history of increasing right facial numbness. He has a
history of intermittent unsteadiness, mild hearing loss and vertigo but has otherwise been well.
Cranial magnetic resonance imaging (MRI) (T1 weighted following gadolinium contrast) is shown
below.

The most likely diagnosis is:

A. Multiple sclerosis.
B. Neurofibromatosis type 2.
C. Cerebellar haemangioblastoma.
D. Meningioma.
E. Pontine glioma.
279

Question 22
A 22 year old woman with primary generalised epilepsy presents for advice regarding a planned
pregnancy. At age 16 years, she presented with a seizure following sleep deprivation and alcohol
excess. She was commenced on phenytoin, but after two further seizures in similar circumstances she
was changed to sodium valproate and has been seizure-free for three years. She seeks pre-conception
advice regarding ongoing anticonvulsant therapy.

To have the best probability of a successful pregnancy, the patient should be advised to
minimise provoking factors, take folate and:

A. Continue sodium valproate therapy.


B. Stop anticonvulsant therapy.
C. Change to carbamazepine therapy.
D. Change to topiramate therapy.
E. Change to clonazepam therapy.

Question 23
A 43 year old woman presents with a two day history of mid thoracic back pain and leg weakness.
On examination, cranial nerve and upper extremities are normal. Lower extremity examination
reveals normal tone, with brisk reflexes (left greater than right) and an upgoing left plantar response.
Power is reduced with grade 4/5 power in hip flexion and ankle dorsiflexion. Sensory examination
reveals reduced sensation to pin-prick and temperature involving the right leg to the level of the
umbilicus.

Her magnetic resonance imaging (MRI) scans are shown below.

The most likely diagnosis is:

A. Transverse myelitis.
B. Spinal cord compression.
C. Anterior spinal artery thrombosis.
D. Spinal cord arteriovenous malformation.
E. Syringomyelia.
280

Question 24
Therapeutic blockade of alpha 4 (α4) integrins is currently under clinical trial for several diseases.
On the basis of known pathogenesis, patients with which one of the following conditions are most
likely to receive benefit from this treatment?

A. Migraine.
B. Stroke.
C. Multiple sclerosis.
D. Alzheimer’s disease.
E. Epilepsy.

Question 25
A 68 year old man presents with an altered conscious state. Three days prior to presentation he had
an episode of ataxia and vertigo, with residual imbalance. On the day prior to presentation he
developed an acute horizontal diplopia, which persists. He is brought to hospital after a sudden
deterioration in his level of consciousness.

The most likely diagnosis is:

A. Pontine haemorrhage.
B. Paraneoplastic syndrome.
C. Acute disseminated encephalomyelitis (ADEM).
D. Multiple cardioembolic strokes.
E. Basilar artery thrombosis.
281

Question 26
A 45 year old man with a long history of asthma presents to hospital with acute shortness of breath,
requiring intubation and ventilation for respiratory support. A chest Xray reveals right upper lobe
consolidation. He is treated with intravenous methylprednis(ol)one, metronidazole, and ceftriaxone
with his clinical course complicated by renal and cardiac failure requiring inotropic support and a
short period of dialysis. Attempts to wean ventilation support are unsuccessful. Nerve conduction
studies reveal the following:

Needle examination reveals fibrillation potentials with small motor unit potentials in all
Muscles examined.

The most likely diagnosis is:


A. critical illness myopathy.
B. critical illness neuropathy.
C. acute polymyositis.
D. acute inflammatory demyelinating polyneuropathy.
E. steroid myopathy.
282

Question 27
A 57 year old man with hypercholesterolaemia and hypertension treated with simvastatin 80 mg/day
and hydrocholorothiazide 25 mg/day develops myalgia and atypical chest pain. A stress test and
coronary angiogram are normal. Diltiazem (controlled delivery) 180 mg/day is added for persisting
hypertension. Four weeks later he presents with shoulder and pelvic girdle pain and stiffness of
seven to ten days duration and more recent ‘bloody discolouration of the urine’. Examination reveals
mild proximal weakness, normal neck flexor and extensor power, no difficulty with swallowing and
no rash.

Laboratory tests reveal:


potassium 2.7 mmol/L [3.4-5.0]
creatinine 80 µmol/L [60-110]
bilirubin 24 µmol/L [0-25]
alkaline phosphatase (ALP) 48 U/L [38-126]
alanine transaminase (ALT) 112 U/L [14-51]
aspartate transaminase (AST) 259 U/L [15-45]
gamma glutamyltranspeptidase (GGT) 38 U/L [0-30]
albumin 32 g/L [33-48]
total protein 65 g/L [61-79]
creatine kinase (CK) 30 568 U/L [<170]

The most likely diagnosis is:

A. Hypokalaemic myopathy.
B. Hypothyroid myopathy.
C. Polymyositis.
D. Simvastatin myopathy.
E. Diltiazem myopathy.

Question 28
A 70 year old male with type 2 (non-insulin-dependent) diabetes mellitus and hypertension
experiences severe retrosternal pain while playing soccer. He arrives at the hospital emergency
department thirty minutes later. His blood pressure is 105/70 mmHg. His ECG shows extensive
anterior ST elevation. He is given aspirin and tenecteplase followed by a heparin infusion, with rapid
resolution of the ST elevation. His pain settles rapidly with morphine and metoclopramide. Soon
after the heparin infusion is commenced, he begins vomiting. His heart rate is 50/minute and blood
pressure 185/100 mmHg. He is drowsy, but has no focal neurological signs. The most likely cause of
his deterioration is:

A. Intracranial haemorrhage.
B. Painless myocardial ischaemia.
C. Allergy to tenecteplase.
D. Hypertensive encephalopathy.
E. Reaction to morphine.
283

Question 29
A patient is diagnosed with brain metastases from large cell lung cancer. He had been started on
steroids at diagnosis and completed his radiotherapy three weeks ago. He now complains of
worsening weakness of his limbs. His leg strength had been normal at the time of diagnosis of brain
metastases. Physical examination reveals weakness of hip flexion with intact reflexes and down
going plantar responses. Sensory examination is normal.

The most likely diagnosis is:

A. Tumour progression.
B. Eaton-Lambert syndrome.
C. Hypercalcaemia.
D. Steroid myopathy.
E. Paraneoplastic myopathy.

Question 30
A 73 year old man presents with a 12 month history of left leg pain radiating from the buttock to the
heel. There is no associated back pain. Neurological examination reveals straight leg raising of 60
degrees on the left with a positive dorsiflexion test. Power is normal with a reduced left ankle jerk.

The remainder of the neurological examination is normal. His


lumbar magnetic resonance imaging (MRI) scan at the L5/S1 region is shown above.

The most likely diagnosis is:

A. L5/S1 disc protrusion.


B. Left L5/S1 facetal pain.
C. Schwannoma involving the left S1 nerve root.
D. Malignant inflitration of the left S1 nerve root.
E. Piriformis syndrome.
284

Question 31
A 67 year old man, with a history of smoking 20 cigarettes per day, presents with acute onset of
painless loss of vision in the right eye, as if a curtain has suddenly descended over the right eye.
After five minutes, vision returned quite rapidly. There are no associated neurological symptoms.

Cranial magnetic resonance imaging (MRI) is normal. His carotid angiogram is shown above.

Which one of the following interventions will give the greatest reduction in his risk of strokeor death
in the next year?

A. Aspirin.
B. Aspirin/dipyridamole.
C. Clopidogrel.
D. Early revascularisation (< 6 weeks).
E. Late revascularisation (> 6 weeks).

Question 32
A 45 year old female is recovering from septic shock due to faecal peritonitis. Repeated attempts to
wean her from mechanical ventilation have failed due to hypercapnic respiratory failure.
Neurological examination reveals diffuse weakness with decreased reflexes and distal sensory loss.
Creatine kinase levels have been normal. Nerve conduction studies show a diffuse, symmetric, distal
axonal sensorimotor neuropathy. Which of the following is the most likely explanation for the
inability to wean from mechanical ventilation?

A. Critical illness polyneuropathy.


B. Acute inflammatory demyelinating polyneuropathy.
C. Prolonged action of neuromuscular blocking agents.
D. Epidural abscess.
E. Critical illness myopathy.
285

Question 33
A 22-year-old woman is brought to the Emergency Department after collapsing. She recalls feeling
“spaced out” and nauseated for 20-30 seconds prior to collapsing. Her friends state that she fell to the
floor and had some brief twitching movements of the face and limbs. She regained consciousness
quickly and was coherent by the time the ambulance arrived.

This episode is most likely to be a:

A. Stokes-Adams attack.
B. Subarachnoid haemorrhage (SAH).
C. Complex partial seizure.
D. Primary generalized seizure.
E. Vasovagal syncope.

Question 34
In an 80 year old woman with low back pain, which of the following is most suggestive of a
diagnosis of spinal canal stenosis?

A. Increased pain with prolonged weight bearing.


B. Lower-extremity numbness with prolonged weight bearing.
C. Loss of lumbar lordosis.
D. Advanced degenerative changes seen on plain radiograph.
E. Loss of vibratory sense in the feet.

Question 35
A 68 year old man presents with a dementia syndrome. The presence of which of the following
clinical problems is most indicative of Lewy body dementia?

A. Visual hallucinations.
B. Myoclonus.
C. Depression.
D. Incontinence.
E. Delusional beliefs.
286

Question 36
A 66-year-old man presents with a history of slowing of most movements over the last six months,
with a bilateral resting tremor. On examination there is evidence of retropulsion. Examination of his
eye movements reveals impaired vertical saccades. He gives a history of frequent falls.

These features are most characteristic of which of the following?

A. Multiple systems atrophy.


B. Parkinson’s disease.
C. Lewy body disease.
D. Progressive supranuclear palsy.
E. Corticobasal degeneration.

Question 37
A 24-year-old man awakens with right shoulder/trapezius discomfort. The pain persists for two
weeks, and he is then aware of weakness of right shoulder movements. On examination he has a full
range of movement of his cervical spine without pain. He is unable to actively abduct his right arm
past 45°, and has Grade 0/5 power of right deltoid and infraspinatus. Upper extremity power is
otherwise normal, with normal biceps, triceps and brachioradialis reflexes. Sensory examination is
normal. A cervical magnetic resonance imaging (MRI) scan and right shoulder ultrasound are
normal.

The most likely diagnosis is:

A. Herpes zoster radiculopathy.


B. C5 radiculopathy.
C. Rotator cuff injury.
D. Inflammatory brachial plexopathy.
E. Suprascapular neuropathy.

Question 38
A 66-year-old alcoholic man collapses and is brought to the Emergency Department. He is found to
have fixed dilated pupils, and is somnolent with no upgaze. Reflexes are brisk with bilaterally
upgoing plantar responses. A cranial computed tomography (CT) scan performed after one hour is
normal. An electroencephalography (EEG) reveals intermittent generalized anterior predominant
slowing, with frontal intermittent rhythic delta activity (FIRDA) without focal of epileptiform
features. Neurological findings remain unchanged over a period of 24 hours. The most likely
diagnosis is:

A. Middle cerebral artery stroke with cerebral herniation ("coning").


B. Bilateral middle cerebral artery stroke.
C. Ponto-mesencephalic ("top of the basilar") stroke.
D. Post-ictal encephalopathy.
E. Wernicke’s encephalopathy.
287

Question 39
A 63-year-old man presents with a six-month history of gradual onset of right hand weakness. On
examination there is wasting of the intrinsic muscles of the right hand. Power is normal proximally,
with weakness of right finger extensors, right finger abduction and adduction, and right flexor
pollicis longus (all Grade 3/5). There is milder weakness of right forearm pronation and right wrist
extensors (Grade 4/5). The right triceps and finger jerks are brisk relative to the left sided reflexes.
The biceps and brachioradialis reflexes are symmetrical. Sensory examination is normal. A cervical
magnetic resonance imaging (MRI) scan is normal.

The most likely diagnosis is:


A. Multiple sclerosis.
B. Motor neurone disease.
C. Brachial plexopathy.
D. Thoracic outlet syndrome.
E. Mononeuritis multiplex.

Question 40
A 63-year-old man presents with aching legs, and an uncontrollable feeling that he needs to move his
legs when he goes to bed. The most likely diagnosis is:

A. Periodic leg movements of sleep.


B. Fibromyalgia.
C. Peripheral neuropathy.
D. Nocturnal cramp syndrome.
E. Restless legs syndrome.

Question 41
A 57-year-old man with a long history of poorly controlled hypertension and smoking presents with
sudden onset of right-sided weakness. On examination, his speech is slurred but content and
comprehension are normal. There is a right facial weakness, with the right side of the soft palate
elevating poorly. He has a right hemiparesis, power grade 2-3/5, in a pyramidal distribution with
normal sensation to all modalities. Right-sided reflexes are enhanced with an extensor right plantar
response.

The most likely cause of this stroke syndrome is left:

A. Middle cerebral artery occlusion.


B. Posterior inferior cerebellar artery occlusion.
C. Hemispheric cortically based haemorrhage.
D. Middle cerebral penetrating artery occlusion.
E. Internal carotid artery occlusion.
288

Question 42
A 70-year-old female with a history of cognitive impairment has the computed tomography
(CT) scan of the brain shown below.

This scan is most suggestive of which of the following


diagnoses?

A. Alzheimer’s disease.
B. Vascular dementia.
C. Dementia with Lewy bodies.
D. Frontotemporal dementia.
E. Normal pressure hydrocephalus.

Question 43
A 75-year-old woman presents with a twelve-month history of deteriorating behaviour and memory
and increasing social withdrawal. She has been tearful and emotional with poor quality sleep. A
diagnosis of depression has been made by her general practitioner, and she has been commenced on
Sertraline 50mg / day with a limited response over the last eight weeks. Neurological examination
reveals normal tone and no pyramidal findings. She scores 21/30 on a minimental state examination.
Thyroid stimulating hormone, full blood count, urea and electrolytes, calcium and liver function tests
are normal. Cranial CT Scan shows age related involutional change. Her EEG demonstrates diffuse
symmetrical slowing over both fronto-temporal head regions with an excess of delta activity.

The most likely diagnosis is:

A. Undertreated Major depression.


B. Diffuse Lewy body disease with associated depression.
C. Alzheimers type dementia with associated depression.
D. Creutzfeldt Jakob disease with associated depression.
E. Depression with hysterical conversion disorder.
289

Question 44
A 33-year-old female presents with headache followed by reduced level of consciousness. Her CT
scan is shown below. She has a systolic blood pressure (SBP) of 180mmHg and a Glasgow Coma
Score of 8/15. Which of the following is the most appropriate initial management?

A. Hypothermia.
B. Control SBP to below 120mmHg.
C. Nimodipine infusion.
D. Recombinant activated factor VII.
E. Orotracheal intubation for airway protection

Question 45
A 60-year-old man, previously well, presents with a two day history of word finding difficulty,
vagueness and irritability. After a generalized tonic clonic seizure he is brought to the emergency
department. Neurological examination reveals a confused and irritable man, who is uncooperative
with the clinical examination. Reflexes are symmetrically brisk with bilaterally downgoing plantar
responses. His CT scan is normal. His cranial MRI scan is shown below.

The most likely diagnosis is:

A. Herpes simplex encephalitis.


B. Intracerebral haemorrhage.
C. Middle cerebral artery stroke.
D. Paraneoplastic limbic encephalitis.
E. Cerebral vasculitis.
290

Question 46
An 82-year-old male has had Parkinsons disease for ten years. His current medications include
levodopa/carbidopa and pergolide. For the last 18 months he has been in high level care (nursing
home) because of his immobility but can still walk five-ten metres with a forearm support frame. His
mini-mental state examination (MMSE) is 26/30. For the last three-six months he has had increasing
problems with hallucinations of people coming into his room and has repeatedly accused his family
members of stealing money from his bank account.

The most appropriate first step in the management of his hallucinations and paranoia is:

A. Commence risperidone.
B. Commence clozapine.
C. Cease pergolide.
D. Reduce dose of levodopa/carbidopa.
E. Commence trimethoprim.

Question 47
A 23-year-old man presents with numbness involving the right fifth digit and ulnar border of the
hand and distal forearm. On examination there is no evidence of muscle wasting. There is weakness
of right finger abduction and adduction (Grade 4/5), abductor pollics brevis (Grade 4/5), flexor
pollicis longus (Grade 4/5) and finger extensor (Grade 4/5). Other muscles groups are normal.
Sensation is reduced along the ulnar border of the forearm and hand involving the 5th digit. Reflexes
are symmetrical and preserved. The remainder of the neurological examination is normal.

The most likely diagnosis is:

A. Ulnar neuropathy at the elbow.


B. C7 radiculopathy.
C. Lower trunk brachial plexopathy.
D. C8 radiculopathy.
E. Radial nerve palsy.

Question 48
The predominant cause of stroke after cardiac surgery is:

A. Air embolism.
B. Aortic atherosclerotic embolism.
C. Intra-cranial haemorrhage.
D. Hypotension.
E. Carotid stenosis.
291

Question 49
A 60-year-old man, with a past history of hypertension, type 2 diabetes and a 20 pack year history of
smoking, presents with sudden onset of headache and a mild left sided hemiparesis. His cranial CT
scan is shown below.

The most likely diagnosis is:

A. Cerebral metastasis.
B. Hypertensive haemorrhage.
C. Amyloid angiopathy.
D. Arteriovenous haemorrhage.
E. Giant cerebral aneurysm.
292

Question 50
A 58-year-old woman presents with a long history of low back pain without radicular features. Over
the last three months, her low back pain has been associated with a feeling of weakness in both legs
on standing for five minutes or after walking 50 metres. There is no disturbance of bladder or bowel
function. General and neurological examination are normal.
Her lumbar MRI scan (sagittal T2) is shown below.

Her symptoms are most likely due to:

A. Spinal cord compression.


B. S1 radiculopathy.
C. Lumbar spinal stenosis.
D. Facet joint cyst.
E. Infective discitis.

Question 51
A 63-year-old man presents with a six-month history of gradual onset of right hand weakness. On
examination there is wasting of the intrinsic muscles of the right hand. Power is normal proximally,
with weakness of right finger extensors, right finger abduction and adduction, and right flexor
pollicis longus (all Grade 3/5). There is milder weakness of right forearm pronation and right wrist
extensors (Grade 4/5). The right triceps and finger jerks are brisk relative to the left sided reflexes.
The biceps and brachioradialis reflexes are symmetrical. Sensory examination is normal. A cervical
magnetic resonance imaging (MRI) scan is normal.

The most likely diagnosis is:


A. Multiple sclerosis.
B. Motor neurone disease.
C. Brachial plexopathy.
D. Thoracic outlet syndrome.
E. Mononeuritis multiplex.
293

Question 52
An 80-year-old man had two transient ischaemic attacks 18 months ago. These consisted of right
sided weakness and aphasia lasting 60 minutes with full recovery. A Doppler ultrasound examination
of the carotids reveals 80% stenosis at the bifurcation of the left carotid artery. The main reason not
to
advise carotid endartectomy is:

A. Advanced age.
B. Degree of carotid artery stenosis.
C. Side of carotid artery stenosis.
D. Duration since transient ischaemic attack.
E. Full neurological recovery following transient ischaemic attack.

Question 53
A 32-year-old man presents with a four day history of lower back pain and a two day history of
difficulty rising from a chair. These symptoms were preceded by a diarrhoeal illness which has now
resolved. On examination there is weakness of hip flexors bilaterally and he has trouble arising from
a squat. He has normal upper limb reflexes and absent lower limb reflexes. Bladder and bowel
function are normal.

The most appropriate next test in establishing a diagnosis would be:

A. Nerve conduction studies.


B. Needle electromyography.
C. Lumbar puncture.
D. Lumbar computed tomography (CT) scan.
E. Visual evoked potentials.
294

Question 54
A 24-year-old man is admitted to intensive care after being knocked off his bicycle by a car. He is
intubated and sedated. His family tell you he has had epilepsy with generalised seizures since age 14.
His only regular medication is phenytoin 300 mg daily and his last seizure was one year previously.
After resuscitation the results of his blood tests include the following:

Sodium (Na) 132 mmol/L [136 - 145 mmol/L]


Potassium (K) 4.6 mmol/L [3.5 - 5.0 mmol/L]
Creatinine 70 µmol/L [<110 µmol/L]
Albumin 24 g/L [36-45 g/L]
Haemoglobin (Hb) 96 g/L [130 - 175 g/L]
Total Phenytoin 6 mg/L [10 - 20 mg/L]
Free Phenytoin 1.5 mg/L [1 - 2 mg/L]

To treat his epilepsy, the best approach is to:

A. Continue his usual dose of phenytoin.


B. Decrease the dose of phenytoin.
C. Prescribe an additional loading dose of phenytoin and continue his usual maintenance dose.
D. Prescribe an additional loading dose of phenytoin and increase the maintenance dose.
E. Peplace phenytoin with sodium valproate.

Question 55
The clinical presentation of motor neurone disease that is most likely to be associated with an
aggressive course and a short life expectancy is:

A. Bulbar weakness with dysarthria.


B. Foot drop.
C. Wasting of the small muscles of the hand.
D. Widespread fasciculations in all four limbs.
E. Progressive spasticity with hyper-reflexia.
295

Question 56
A 28-year-old woman presents with a three month history of numbness of the hands and feet. On
examination, upper and lower extremity reflexes are symmetrically brisk with bilaterally downgoing
plantar responses. Upper extremity power is normal with a normal sensory examination. There is
mild weakness of ankle dorsiflexion bilaterally with reduced sensation to pin prick to the level of the
ankles. Joint position sense is normal. Vibration sense is impaired to the level of the tibial tuberosity
bilaterally. Her MRI scan is shown below.

The most likely diagnosis is:

A. Syringomyelia.
B. Spinal demyelination.
C. Spinal cord ependymoma.
D. Subacute combined degeneration of the cord.
E. Chronic inflammatory demyelinating polyneuropathy.
296

Question 57
A 64-year-old previously well smoker develops a cough and is found to have a lung mass. An
endobronchial biopsy reveals small cell lung carcinoma. He has lost 7 kg in weight, and is now 65
kg, but he still has a good performance status.

His liver function tests reveal:


Alkaline phosphatase 189 U/L [50 – 130 U/L]
Alanine transferase 70 U/L [<45 U/L]
Bilirubin 37 µmol/L [<20 µmol/L]

A computed tomography (CT) scan confirms multiple liver secondaries. In the last two days he has
developed some leg weakness, but is still able to walk. A magnetic resonance imaging (MRI) scan of
his spine is shown below.

The most appropriate next step in his management is:

A. Systemic chemotherapy.
B. External beam radiotherapy.
C. Best supportive care.
D. Decompression laminectomy.
E. Strontium.
297

Answers

1. E 48. B
2. A 49. B
3. C 50. C
4. C 51. B
5. E 52. D
6. D 53. C
7. B 54. A
8. C 55. A
9. C 56. B
10. A 57. A
11. C
12. B
13. E
14. A
15. B
16. E
17. D
18. D
19. D
20. A
21. B
22. B
23. B
24. C
25. E
26. A
27. D
28. A
29. D
30. A
31. D
32. A
33. E
34. B
35. A
36. D
37. D
38. C
39. B
40. E
41. D
42. D
43. C
44. E
45. A
46. C
47. C
298

Medical Sciences

Oncology

Question 1
The most important rationale for using adjuvant chemotherapy soon after surgery for cancer (rather
than deferring chemotherapy until the cancer has recurred) is that after surgery:

A. Cancer volume is smaller.


B. There is no resistance to cytotoxic agents.
C. There are fewer cells in cell cycle.
D. Drug penetration into the tissues is reduced.
E. Cancer growth is slower.

Question 2
The principal aim of a phase 1 trial of a cytotoxic agent is to:

A. Maintain the patient’s hope that treatment is possible.


B. Determine the best schedule of administration.
C. Define tumour response rate.
D. Measure progression-free survival.
E. Establish the maximum tolerated dose (MTD).

Question 3
Which one of the following is least toxic to bone marrow?

A. Carboplatin.
B. Etoposide.
C. Cyclophosphamide.
D. Doxorubicin.
E. Vincristine.

Question 4
Of the following cancer treatments, the one most likely to cause delayed (>24 hours after
chemotherapy) nausea is:

A. Cisplatin.
B. 5-fluorouracil (5-FU).
C. Vincristine.
D. Methotrexate.
E. Capecitabine.
299

Question 5
A 38-year-old woman presents with breast cancer and multiple liver metastases two years after
resection of a left breast tumour. Which of the following is most likely to affect treatment decisions?

A. Tumour grade.
B. Presence of BRCA1 mutation.
C. Oestrogen receptor status.
D. HER-2 overexpression.
E. Number of involved lymph nodes.

Question 6
Long-term hormone replacement therapy (>5 years) in postmenopausal women results in the greatest
increase in absolute risk of cancer of the:

A. Lung.
B. Ovary.
C. Colon.
D. Breast.
E. Uterus.

Question 7
Which one of the following is the most important consideration in optimising treatment outcome
from combination chemotherapy?

A. Use drugs partially effective as single agents.


B. Use drugs with similar toxicity profiles.
C. Use drugs with similar schedules.
D. Use drugs with similar treatment free intervals.
E. Minimise interaction between drugs.

Question 8
A 40-year-old woman treated for Hodgkin’s disease in her early twenties with mantle radiotherapy is
at highest risk of which of the following cancers?

A. Breast.
B. Lung.
C. Thyroid.
D. Lymphoma.
E. Sarcoma.
300

Question 9
Which one of the following patterns of electrolyte changes is most consistent with tumour lysis?

Potassium Phosphate Calcium


A. ↓ ↓ ↓
B. ↓ ↑ ↑
C. ↑ ↓ ↑
D. ↑ ↑ ↓
E. ↑ ↑ ↑

Question 10
Of the following tumour markers, which has the highest specificity for the cancer indicated?
A. Beta-human chorionic gonadotrophin (β-hCG) and testicular cancer.
B. CA 125 and ovarian cancer.
C. CA 19-9 and pancreatic cancer.
D. CA 15-3 and breast cancer.
E. Carcinoembryonic antigen (CEA) and colon cancer

Question 11
In addition to ondansetron and dexamethasone, the most effective strategy to reduce delayed
chemotherapy nausea for someone receiving highly emetogenic chemotherapy is to add:
A. Tropisetron.
B. Haloperidol.
C. Lorazepam.
D. Aprepitant.
E. Droperidol.

Question 12
For which of the following cancers is screening most strongly associated with improved survival?
A. Lung.
B. Prostate.
C. Ovarian.
D. Colon.
E. Melanoma.
301

Question 13
A 40-year-old woman treated for Hodgkin’s disease in her early twenties with mantle radiotherapy is
at highest risk of which of the following cancers?

A. Breast.
B. Lung.
C. Thyroid.
D. Lymphoma.
E. Sarcoma.

Question 14
Tumour markers are often used indiscriminately to monitor therapy. For which of the following
tumour markers and associated tumours does normalisation with therapy best predict overall
survival?

A. Ca 15-3 in breast carcinoma.


B. Carcinoembryonic antigen in bowel carcinoma.
C. Lactate dehydrogenase in lymphomas.
D. Alpha fetoprotein in testicular carcinoma.
E. Ca 19-9 in pancreatic carcinoma.

Question 15
Adjuvant chemotherapy is given in many solid tumours. What is the main reason for doing this?

A. To shrink the primary tumour.


B. To improve survival by eradicating micrometastases.
C. To delay appearance of the secondary tumours.
D. To delay recurrence of primary tumour.
E. To reduce toxicity of the treatment regimen

Question 16
The main reason for bone loss in patients treated for metastatic prostate cancer with the luteinizing
hormone releasing hormone (LHRH) agonist goserelin and the antiandrogen flutamide is:

A. Oestrogen deficiency due to goserelin.


B. Androgen deficiency due to goserelin.
C. Oestrogen deficiency due to flutamide.
D. Androgen blockade due to flutamide.
E. Multiple bony metastases.
302

Question 17
Human papilloma virus (HPV) is a well-recognised causal factor for cancer of the cervix. For which
of the following other cancers is there the strongest association with HPV?

A. Uterine.
B. Anal.
C. Rectal.
D. Oral.
E. Vulval.

Question 18
Which of the following tumours is the most chemo-resistant?

A. Breast carcinoma.
B. Renal cell carcinoma.
C. Testicular carcinoma.
D. Colon carcinoma.
E. Small cell lung carcinoma
303

Answers

1. A
2. E
3. E
4. A
5. D
6. D
7. A
8. A
9. D
10. A
11. D
12. D
13. A
14. D
15. B
16. A
17. B
18. B
304

Clinical Applications

Oncology

Question 1
In a post-menopausal patient, which one of the following is the best initial hormone therapy for
metastatic breast cancer?

A. Androgen.
B. Anti-oestrogen.
C. Aromatase inhibitor.
D. Oestrogen.
E. Progesterone.

Question 2
A 71-year-old woman, who lives alone, presents to the accident and emergency department With A
history of increasing back pain over two months. The X-ray obtained is shown below.

What is the most likely diagnosis?

A. Myeloma.
B. Osteoporosis.
C. Breast cancer.
D. Lymphoma.
E. Paget’s disease
305

Question 3
The best prognostic indicator in operable breast cancer is:

A. Oestrogen receptor (ER) status.


B. Her-2 / neu gene over-expression.
C. Tumour size.
D. Nodal status.
E. Tumour grade.

Question 4
Concerning carcinoma of unknown primary, which one of the following clinical situations is least
likely to have a specific therapy?

A. Adenocarcinoma in an axillary lymph node in a 55-year-old woman.


B. Squamous cell carcinoma in a mid cervical lymph node in a 60-year-old male smoker.
C. Adenocarcinoma in the liver in a 60-year-old man.
D. Poorly differentiated carcinoma in the mediastinum in a 38-year-old man.
E. Adenocarcinoma in sclerotic bony metastases in a 70-year-old man.

Question 5
A patient has severe pain from locally advanced pancreatic cancer. The pain is eventually well
controlled by 80 mg/day of subcutaneous morphine infusion in hospital. It is intended to discharge
the patient on oral sustained-release morphine.

Which one of the following is likely to be the most appropriate dose?

A. 40 mg bd.
B. 80 mg bd.
C. 120 mg bd.
D. 160 mg bd.
E. 200 mg bd.

Question 6
Abnormalities in which one of the following genes or proteins is most closely associated with the
development of drug resistance during chemotherapy?

A. P-glycoprotein (MDRI).
B. p53.
C. BCL2.
D. Cytochrome P450.
E. Thiopurine S-methyltransferase.
306

Question 7
A 42-year-old woman has recently had a mastectomy after being diagnosed with breast carcinoma.
Her paternal grandmother had ovarian carcinoma in her early fifties. She has a 32-year-old well
sister. Apart from advising her on appropriate adjuvant therapy, the next most important issue you
should also discuss with her is:

A. Prophylactic oophorectomy.
B. Radiation ovarian ablation.
C. Prophylactic contralateral mastectomy.
D. Screening her sister for a genetic mutation.
E. Screening her for a genetic mutation.

Question 8
In a randomised controlled trial of a cancer screening program, the most important indicator of the
program’s effectiveness is demonstration in the screened subpopulation of improved:

A. Case detection.
B. Detection of cases at an earlier stage.
C. Cancer-specific survival.
D. Cancer mortality.
E. Cancer progression-free survival.

Question 9
A 35-year-old woman seeks your advice about her risk of developing breast cancer.

Which one of the following would place her at greatest risk of developing breast cancer?

A. Menarche less than 12 years.


B. Birth of first child after the age of 25.
C. Oral contraceptive use for more than 10 years.
D. Sister and aunt diagnosed with breast cancer.
E. Excision of a benign breast lump.
307

Question 10
A 76-year-old man with a history of prostate cancer treated with radiotherapy a year ago, presents
with increasing bone pain and tenderness involving the right chest wall. His prostate specific antigen
is 766 ng/mL [<6]. His bone scan is shown below.

The most effective therapy would be:

A. Corticosteroids.
B. Androgen deprivation.
C. Radiotherapy.
D. Chemotherapy.
E. Best supportive care.

Question 11
The least likely reason for shorter cancer-specific survival among the elderly is the finding that
elderly patients:

A. Present with more aggressive cancers.


B. Undergo fewer investigations.
C. Receive less treatment.
D. Have co-morbidities which limit their tolerance of treatment.
E. Undergo less cancer screening.
308

Question 12
A 63-year-old woman with metastatic breast cancer with skeletal metastases presents with dry cough
and shortness of breath. Six weeks ago she completed palliative radiotherapy to her T5 vertebra
because of painful metastatic involvement. She is currently receiving tamoxifen. On examination she
is afebrile and her chest auscultation reveals fine crepitations in the left upper zone. Her chest X-ray
is shown below.

The most appropriate next step in her management is to:

A. Cease tamoxifen and commence aromatase inhibitor therapy.


B. Cease tamoxifen and commence cytotoxic chemotherapy.
C. Add antifungal treatment.
D. Add intravenous antibiotics.
E. Add corticosteroids.

Question 13
A 22-year-old man is evaluated post resection of a stage I nonseminomatous testicular germ cell
tumour with no vascular invasion (low risk).

The most appropriate next step in his management is:

A. Regular surveillance.
B. Carboplatin chemotherapy.
C. Para-aortic radiotherapy.
D. Hormonal therapy.
E. Testicular radiotherapy.
309

Question 14
A young woman with advanced ovarian cancer presents with nausea, vomiting and abdominal pain.
Her abdominal X-rays are shown below.

Which of the following treatments is most likely to improve her symptoms?

A. Ondansetron.
B. Domperidone.
C. Octreotide.
D. Hyoscine.
E. Prochlorperazine.

Question 15
A 47-year-old woman with advanced cancer is admitted at 3 a.m. with a four-day history of
weakness, nausea, vomiting and constipation. Her serum calcium is 3.8 mmol/L [2.2-2.6] and
albumin is 32 g/L [31-44].

The most appropriate initial step in her management is:

A. Intravenous rehydration.
B. Diuretics.
C. Steroids.
D. Bisphosphonates.
E. Calcitonin.

Question 16
In assessment of cancer patients, the Karnofsky performance scale best correlates with:

A. Treatment dose.
B. Survival.
C. Analgesic requirement.
D. Rehabilitation needs.
E. Cancer volume.
310

Question 17
In addition to a 5HT3-receptor antagonist, the most effective agent used to prevent acute
chemotherapy-induced nausea with a highly emetogenic drug (i.e. cisplatin) is:

A. Lorazepam.
B. Droperidol.
C. Prochlorperazine.
D. Metoclopramide.
E. Dexamethasone.
311

Question 18
The photographs below are of the immunohistochemistry staining of breast tumour in a patient
presenting with metastatic disease. The antigens used are human epidermal growth factor receptor 2
protein (Her2) (Figure 1), and oestrogen receptor (ER) and progesterone receptor (PR) (Figure 2).

Figure 1. Immunohistochemistry for Her2.

Figure 2. Immunohistochemistry for ER and PR.

A patient with this result is most appropriately treated with:

A. Trastuzumab (herceptin) but not letrozole.


B. Trastuzumab and letrozole.
C. Letrozole but not trastuzumab.
D. Trastuzumab and tamoxifen.
E. Tamoxifen but not trastuzumab.
312

Question 19
Among the following anti-cancer agents, the one most likely to cause acute respiratory distress is:

A. Bleomycin.
B. All-trans-retinoic acid.
C. Busulphan.
D. Gemcitabine.
E. Cyclophosphamide.

Question 20
A 50-year-old man presents with shortness of breath, facial swelling and chest tightness. The
computed tomography (CT) scan of his chest is shown below.

The malignancy most likely responsible for this presentation is:

A. Lymphoma.
B. Thymoma.
C. Lung cancer.
D. Germ cell tumour.
E. Melanoma.
313

Question 21
A patient with lung cancer receives chemotherapy with carboplatin and etoposide. He is given
ondansetron and dexamethasone pre-chemotherapy and dexamethasone and metoclopramide
postchemotherapy.
On day two post-chemotherapy he complains of stiffness around his shoulders and feeling jittery and
irritable. The drug most likely to be responsible for these symptoms is:
A. Carboplatin.
B. Etoposide.
C. Ondansetron.
D. Dexamethasone.
E. Metoclopramide.

Question 22
In which of the following is radiotherapy least likely to be of clinical benefit to the patient?
A. Bleeding malignant gastric ulcer.
B. Brain metastases with raised intracranial pressure.
C. Painful bone metastases from prostate cancer.
D. Intrahepatic metastases causing cholestasis.
E. Non small cell lung cancer causing bronchial obstruction.

Question 23
Of the following metastatic cancers, the one associated with the highest long term survival after
treatment is:
A. Breast cancer with bone metastases.
B. Ovarian cancer with malignant ascites.
C. Germ cell cancer with lung metastases.
D. Prostate cancer with bone metastases.
E. Lung cancer with brain metastases.

Question 24
A “reasonably well” (normal performance status) patient with non-haemorrhagic cerebral metastases
from breast carcinoma presents with spontaneous lower limb venous thrombosis and pulmonary
embolism. Which of the following is the most appropriate long term management regime?
A. Dalteparin.
B. Inferior vena caval filter.
C. Warfarin.
D. Graduated compression stockings.
E. Aspirin.
314

Question 25
The most appropriate clinical scenario for the use of non-steroidal anti-inflammatory agents for
management of cancer pain is which of the following?
A. Liver capsule pain in a patient with metastatic cancer.
B. Headache due to malignant meningitis.
C. Small bowel obstruction pain in a patient with ovarian cancer.
D. Bone pain due to metastatic lung cancer.
E. Neuropathic pain due to malignant nerve root infiltration.

Question 26
A 50-year-old male has chronic liver disease secondary to Hepatitis B infection. Previous endoscopy
has revealed large oesophageal varices. Blood results are:

Albumin 32 g/L [38-55]


Prothrombin Time 14 seconds [8-12]
Platelet count 70 x 109/L [150-450]

On routine review, his alpha fetoprotein is 300 ng/mL [0-15].

His abdominal computed tomography (CT) scan is shown below.

A. Chemoembolisation.
B. Liver transplantation.
C. Cryotherapy.
D. Local resection.
E. Radiofrequency ablation.
315

Question 27
A 55-year-old woman presents with vague abdominal pain. Her CT scan is shown below.

A biopsy of the lesion shows spindle shaped cells. The most likely diagnosis is:

A. Adenocarcinoma of esophagus.
B. Gastrointestinal stromal tumour.
C. Linitis plastica.
D. Adrenal tumour.
E. Renal cell tumour (hypernephroma).

Question 28
The most common side effect of antiangiogenesis agent anticancer treatments is:

A. Hypotension.
B. Hypertension.
C. Bleeding.
D. Clotting.
E. Cerebrovascular accidents.
316

Question 29
A 62-year-old woman with history of breast cancer treated with surgery, chemotherapy and
radiotherapy three years earlier presents with the following abnormality.

The most likely diagnosis is:

A. Cellulitis.
B. Radiation recall.
C. tumour recurrence.
D. inflammatory breast cancer.
E. Paget’s disease.

Question 30
Which of the following situations is most likely to result in a false negative FDG-PET
(Fluorodeoxyglucose-Positron Emission Tomography) scan when restaging a patient with aggressive
lymphoma post-chemotherapy?

A. Necrosis at site of previous involvement.


B. Fibrosis at site of previous involvement.
C. Brown fat.
D. Uncontrolled diabetes.
E. Recent surgery.
317

Question 31
A 50-year-old woman undergoes breast conserving surgery for a breast cancer found on self
examination. Histology shows an invasive ductal carcinoma, moderately differentiated, 22mm in
size, oestrogen receptor positive, progesterone receptor negative and HER-2 positive. Sentinel node
biopsy is negative.

What further treatment will have the greatest impact on her survival probability?

A. Chemotherapy and tamoxifen.


B. Trastuzamab alone.
C. Aromatase inhibitor.
D. Chemotherapy and trastuzamab.
E. Tamoxifen followed by an aromatase inhibitor.

Question 32
A woman with metastatic breast cancer and painful bone secondaries develops nausea and vomiting.
She is found to have a bowel obstruction and is admitted and made nil orally. She usually takes MS
Contin 70 mg bd, and 2 mls of morphine mixture 10 mg/ml for breakthrough pain, usually twice
each day. The bowel obstruction has not exacerbated her pain.

The best estimate of the equivalent parenteral four hourly dose of morphine is:
A. 2 mg.
B. 10 mg.
C. 5 mg.
D. 50 mg.
E. 20 mg.

Question 33
A 65-year-old male is reviewed following surgical resection of non-small cell lung cancer. Pathology
reveals involvement of local hilar lymph nodes, in addition to the primary lung cancer. What is the
most appropriate further management?

A. Radiotherapy.
B. Chemotherapy.
C. Regular radiological follow-up.
D. Combined chemo-radiation therapy.
E. Prophylactic cranial irradiation.
318

Question 34
A 69-year-old lady presents with chest pain following a motor vehicle accident and is found on
thoracic computed tomography (CT) scan to have multiple sclerotic lesions involving ribs and
thoracic vertebrae. A technetium-99m-methylene diphosphonate (MDP) bone scan reveals multiple
areas of radiotracer uptake throughout the skeleton. Physical examination and mammography are
normal.

Which of the following is most likely to give a diagnosis?

A. Bone marrow biopsy.


B. Assay of tumour markers.
C. Serum protein electrophoresis.
D. Magnetic resonance imaging (MRI) scan of thorax.
E. Biopsy of vertebral lesion.

Question 35
You are caring for a 55-year-old man on haemodialysis. He is on the transplant waiting list and asks
about his risk of developing cancer if he has a transplant. You note he is of Anglo-Saxon descent.
Which of the following cancers is he most likely to develop over the ensuing ten years post-
transplantation?

A. Carcinoma of the prostate.


B. Squamous cell carcinoma of the skin.
C. Non-Hodgkin’s lymphoma.
D. Chronic myeloid leukaemia.
E. Carcinoma of the colon.
319

Question 36
A 24-year-old man presented with a left testicular mass and a serum αfetoprotein (AFP) of 560 µg/L
[<10 µg/L] with a normal serum human chorionic gonadotrophin (HCG). An orchidectomy revealed
embryonal testicular carcinoma. He received four cycles of chemotherapy with cisplatin, etoposide
and bleomycin (BEP regime). His serum αfetoprotein is now normal. His post-completion
chemotherapy abdominal computed tomography (CT) scan is shown below.

The most appropriate next step in his management is:

A. Autologous stem cell transplant.


B. External beam radiotherapy.
C. Ongoing observation.
D. Continue BEP chemotherapy.
E. Retroperitoneal lymph node dissection.

Question 37
Compared with tamoxifen, aromatase inhibitors are more likely to be associated with an increased
risk
of which of the following?

A. Endometrial cancers.
B. Osteoporosis.
C. Hot flushes.
D. Thrombo-embolic disease.
E. Local recurrence.
320

Question 38
A 37-year-old male is found to have some persistently enlarged lymph nodes (<2 cm diameter) in the
left posterior triangle. A biopsy reveals follicular, Grade 1, B-cell non-Hodgkin lymphoma. Staging
procedures including computed tomography (CT) scan, positron emission tomography (PET) scan
and bone marrow biopsy confirm the disease is confined to a single lymph node region in the neck.

Which of the following management strategies is most appropriate?

A. Watch and wait.


B. Oral chlorambucil.
C. Radiotherapy.
D. Multiagent chemotherapy.
E. Combined modality therapy.

Question 39
An 81-year-old female has had surgery for early stage breast cancer (node negative) and the tumour
is ER+ (oestrogen receptor positive) and HER+ (herceptin receptor positive). She also has congestive
cardiac failure for which she takes frusemide and candesartan. Which of the following treatment
approaches would most improve her life expectancy?

A. Trastuzumab.
B. Tamoxifen.
C. Letrozole (aromatase inhibitor).
D. Cyclophosphamide, methotextrate, 5-fluorouracil (CMF).
E. No treatment.

Question 40
A 57-year-old woman who had been previously well, presented with altered bowel habit and was
found to have a descending colon carcinoma. She subsequently had a left hemicolectomy.
Pathologically, there was no evidence of perforation, the adenocarcinoma was confined to the bowel
wall, but one of five nodes contained tumour.

What is the optimal treatment for her once she recovers from her surgery?

A. 5 Fluorouracil-based chemotherapy.
B. Close observation.
C. Further node dissection to obtain more nodes.
D. 5 Fluorouracil-based chemotherapy with radiotherapy.
E. Radiotherapy.
321

Question 41
A 54-year-old woman is receiving chemotherapy with oxaliplatin, 5 fluorouracil (5FU) and folinic
acid as adjuvant therapy for newly diagnosed node positive colon cancer. During the continuous
infusion of 5FU, she experiences retrosternal chest pain. She is not known to have a history of
coronary artery disease, and has not had chest pain previously. What is the most likely cause of her
chest pain?

A. Coronary artery spasm.


B. Oesophageal spasm.
C. Anaphylaxis.
D. Aortic dissection.
E. Mucositis.
322

Answers

1. B
2. C
3. D
4. C
5. C
6. A
7. E
8. D
9. D
10 B
11. A
12. E
13. A
14. C
15. A
16. B
17. E
18. A
19. B
20. C
21. E
22. D
23. C
24. A
25. D
26. B
27. B
28. B
29. C
30. C
31. D
32. B
33. B
34. E
35. B
36. E
37. B
38. C
39. C
40. A
41. A
323

Medical Sciences

Pharmacology

Question 1
The diagram below shows cumulative quantal dose-response curves for a hypothetical drug (drug X).
The Y-axis represents the percentage of the population studied who: for curve (a) achieved the
desired therapeutic effect at a given dose; and for curve (b) experienced the most important toxic
effect at a given dose.

Based on this information, the best estimate of the therapeutic index of drug X is approximately:

A. 0.
B. 0.3.
C. 0.7.
D. 1.
E. 3.

Question 2
At 72 hours following a significant paracetamol overdose, which one of the following tests will
allow the best prediction of death or need for liver transplantation?

A. Serum alanine transaminase (ALT).


B. Serum bilirubin.
C. Prothrombin time-international normalised ratio (PT-INR).
D. Serum gamma glutamyltranspeptidase (GGT).
E. Blood ammonia.
324

Question 3
A bioavailability study has been performed to compare a proposed new generic capsule, in a strength
of 50 mg, with a marketed tablet preparation in a strength of 100 mg. The active ingredient is a drug
with linear kinetics. A cross-over design was used, with each subject receiving, in random order and
with an adequate wash-out period, a single oral dose of one tablet on one occasion and one capsule
on the other. The mean results for the area under the plasma concentration versus time curve (AUC)
are 300 mg.h/L for the capsule, and 500 mg.h/L for the tablet.

The best estimate of the relative bioavailability of the capsule with respect to the tablet is:

A. 0.3.
B. 0.6.
C. 1.2.
D. 1.7.
E. 3.3.

Question 4
Allopurinol precipitates azathioprine toxicity by which one of the following mechanisms?

A. Displacement of azathioprine from protein-binding sites.


B. Impairment of the renal excretion of azathioprine.
C. Promotion of gastrointestinal absorption of azathioprine.
D. Prolongation of the half-life of the biologically active azathioprine metabolite.
E. Inhibition of hydrolysis of azathioprine.
325

Question 5
The diagram below shows cumulative quantal concentration-response curves for a hypothetical drug
(drug X) which has linear kinetics. The Y-axis represents the percentage of the population studied
who: for curve (a) achieved the desired therapeutic effect at a given plasma concentration; and for
curve (b) experienced the most important toxic effect at a given plasma concentration.

If a patient with no evidence of toxicity who has not achieved the desired therapeutic response has a
measured concentration of 40 mg/L, the best response would be to:

A. Check compliance with no change in dose.


B. Increase the dose by 25%.
C. Increase the dose by 50%.
D. Increase the dose by 75%.
E. Increase the dose by 100%.
326

Question 6
The diagram below shows the relationship between the plasma concentration of a drug and the effect
of that drug over time in a single individual given one dose of the drug. The direction of change over
time is shown by the arrows. The relationship is described by an anti-clockwise hysteresis loop. The
numbers on the loop indicate the number of hours after the dose was given.

The best explanation of this phenomenon is that:

A. The drug is binding irreversibly to its target receptor.


B. The drug is active itself but has no active metabolites.
C. There is a delay in the absorption of the drug from the gastrointestinal tract.
D. There is down-regulation of the drug’s target receptor over time.
E. There is a delay in the distribution of the drug from plasma to its site of action.

Question 7
The following pharmacokinetic data (normalised to a 70 kg individual) are provided for five drugs.

Drug Volume of distribution (L) Plasma half-life (h)


Amiloride 1190 21
Amiodarone 4620 600
Dapsone 70 22
Tolbutamide 7 6
Trimethoprim 126 11

Which one of the following drugs is most likely to have the slowest rate of clearance from plasma?

A. Amiloride.
B. Amiodarone.
C. Dapsone.
D. Tolbutamide.
E. Trimethoprim
327

Question 8
The following pharmacokinetic data (normalised to a 70 kg individual) are provided for five drugs.

Volume of
Clearance
Drug distribution
(mL/minute)
(L)
Naproxen 9.1 11.2
Phenobarbitone 4.3 37.8
Piroxicam 2.5 10.5
Valproate 7.7 15.4
Warfarin 3.2 9.8

Which one of these drugs is most likely to have the longest elimination half-life?

A. Naproxen.
B. Phenobarbitone.
C. Piroxicam.
D. Valproate.
E. Warfarin.

Question 9
Which one of the following pharmacokinetic parameters is most important in the optimal prescribing
of beta-lactam antibiotics?

A. The peak concentration/mean inhibitory concentration (MIC) ratio.


B. Area under the plasma concentration versus time curve (AUC).
C. The AUC/MIC ratio.
D. Time above the MIC.
E. Both AUC/MIC ratio and peak concentration/MIC ratio.
328

Question 10
Drug X is given intravenously, using two different regimens, to the same patient on two different
occasions. On both occasions, there is no measurable amount of the drug in plasma prior to its
administration. On both occasions, the volume of distribution of X is 0.85 L/kg and the plasma half-
life is six hours.

Regimen A: continuous intravenous infusion of 10 mg/kg/h,


with no loading dose.
Regimen B: a loading dose of 50 mg/kg given rapidly intravenously,
then a continuous intravenous infusion of 10 mg/kg/h.

Using Regimen A, the plasma concentration of X after 48 hours was 100 mg/L.

Using Regimen B, the plasma concentration of X after 48 hours would most closely
approximate:

A. 50 mg/L.
B. 75 mg/L.
C. 100 mg/L.
D. 125 mg/L.
E. 150 mg/L.

Question 11
A 62-year-old man on weekly oral methotrexate for psoriatic arthritis inadvertently receives a course
of oral trimethoprim-sulfamethoxazole therapy for urinary infection. He becomes ill with mucositis,
fever and bruising and is found to have moderately severe pancytopenia.
Folinic acid (in the form of calcium folinate) is administered intravenously as an antidote for
presumed methotrexate toxicity.
The principal mode of action of folinic acid is:

A. Enhanced renal excretion of methotrexate.


B. Reduced intracellular accumulation of methotrexate.
C. Conversion of methotrexate to an inactive metabolite.
D. Activation of the enzyme dihydrofolate reductase.
E. Provision of reduced folates for nucleic acid synthesis.

Question 12
Allopurinol precipitates azathioprine toxicity by which one of the following mechanisms?

A. Displacement of azathioprine from protein-binding sites.


B. Impairment of the renal excretion of azathioprine.
C. Promotion of gastrointestinal absorption of azathioprine.
D. Prolongation of the half-life of the biologically active azathioprine metabolite.
E. Inhibition of hydrolysis of azathioprine.
329

Question 13
After commencement of once daily maintenance dosing of a drug with an elimination half-life of 48
hours, the concentration of the drug in plasma will continue to accumulate:

A. For 2 days.
B. For 4 days.
C. For 10 days.
D. For 16 days.
E. Indefinitely.

Question 14
Non-steroidal anti-inflammatory drugs (NSAIDs) which selectively inhibit cyclooxygenase 2 (COX-
2 inhibitors) will produce the greatest reduction in which one of the following adverse effects when
compared with non-selective NSAIDs?

A. Renal failure.
B. Hypertension.
C. Gastric ulceration.
D. Rashes.
E. Headache.

Question 15
Drug X has linear pharmacokinetics and a narrow therapeutic index and is cleared by both renal
filtration (80%) and hepatic metabolism (20%). The standard daily dose required by an individual
with normal renal and hepatic function to achieve a therapeutic response is 100 mg.
Which one of the following is the best estimate of the daily maintenance dose that would be required
in an elderly person with a creatinine clearance one quarter of normal?

A. 20 mg.
B. 25 mg.
C. 30 mg.
D. 35 mg.
E. 40 mg.

Question 16
Which one of the following drugs is most likely to interact with lithium to cause lithium toxicity in a
previously stable patient on a stable dose of lithium?

A. Amiodarone.
B. Cimetidine.
C. Digoxin.
D. Bendrofluazide.
E. Probenecid.
330

Question 17
The concentration-response relationships for five different drugs (Drugs A-E) are shown in the graph
below.

Which one of the drugs has the highest potency?

A. Drug A.
B. Drug B.
C. Drug C.
D. Drug D.
E. Drug E.

Question 18
With which one of the following non-steroidal anti-inflammatory drugs is the complication
haemorrhagic cystitis most commonly associated?

A. Piroxicam.
B. Naproxen.
C. Ketoprofen.
D. Sulindac.
E. Tiaprofenic acid.
331

Question 19
Which one of the following is least likely to cause a reduction in the extent of protein binding of
phenytoin?

A. Pregnancy.
B. Chronic renal failure.
C. Chronic liver failure.
D. Warfarin therapy.
E. Carbamazepine therapy.

Question 20
The following pharmacokinetic data (normalised to a 70 kg individual) are provided for five drugs.

Drug Volume of distribution (L) Plasma half-life (h)


Amiloride 1190 21
Amiodarone 4620 600
Dapsone 70 22
Tolbutamide 7 6
Trimethoprim 126 11

Which one of the following drugs is most likely to have the slowest rate of clearance from plasma?

A. Amiloride.
B. Amiodarone.
C. Dapsone.
D. Tolbutamide.
E. Trimethoprim.
332

Question 21
Which of the following drugs newly prescribed to a patient with stable and therapeutic lithium levels
is most likely to produce an increase in his serum lithium?

A. Fluoxetine.
B. Theophylline.
C. Captopril.
D. Hydrochlorothiazide.
E. Aspirin.

Question 22
The plasma concentration of a drug at steady state is mostly determined by the drug's:

A. Volume of distribution (VD).


B. Clearance (CL).
C. Elimination rate constant (kel).
D. Extent of protein binding.
E. Elimination half-life (t1/2).

Question 23
Chronic use of thiazide diuretics is least likely to lead to which of the following?

A. Hypercalciuria.
B. Hyperglycaemia.
C. Hyperuricaemia.
D. Metabolic alkalosis.
E. Hypomagnesaemia.

Question 24
Which of the following vasodilators acts predominantly by reducing preload?

A. Hydralazine.
B. Nifedipine.
C. Nitroglycerine.
D. Captopril.
E. Prazosin.
333

Question 25
A 23-year-old woman with bipolar disorder is on lithium 500 mg twice daily. She complains of
polyuria and nocturia four to five times a night. Her blood sugar level is normal.

Which of the following drugs is most likely to decrease her polyuria?

A. Amiloride.
B. Frusemide.
C. Carbamazepine.
D. Ibuprofen.
E. Antidiuretic hormone.

Question 26
The delay in the onset of the anticoagulant effects of warfarin is most dependent on the:

A. Loading dose of warfarin.


B. Rate of catabolism of clotting factors.
C. Rate of absorption of warfarin.
D. Clearance rate of the S-isomer of warfarin.
E. Clearance rate of the R-isomer of warfarin.

Question 27
Which one of the following medications is most likely to cause foetal abnormalities if taken in the
first trimester of pregnancy?

A. Enalapril.
B. Gentamicin.
C. Ibuprofen.
D. Tetracycline.
E. Warfarin.

Question 28
A 24-year-old man with a previous diagnosis of schizophrenia has been receiving depot haloperidol
monthly for four years. He presents with the gradual onset of choreoathetoid movements of his upper
limbs.
The most likely explanation for this presentation is:

A. Dystonia.
B. Drug-induced Parkinson’s disease.
C. Tardive dyskinesia.
D. Amphetamine abuse.
E. Akathisia.
334

Question 29
Therapeutic blockade of alpha 4 (α4) integrins is currently under clinical trial for several diseases.
On the basis of known pathogenesis, patients with which one of the following conditions are most
likely to receive benefit from this treatment?

A. Migraine.
B. Stroke.
C. Multiple sclerosis.
D. Alzheimer’s disease.
E. Epilepsy.

Question 30
Which electrolyte abnormality in a 65-year-old man is most likely to be due to treatment with a
selective serotonin reuptake inhibitor (SSRI)?
A. Hyperkalaemia.
B. Hyperglycaemia.
C. Hyponatraemia.
D. Hypocalcaemia.
E. Acidosis.

Question 31
A 75-year-old man is placed on oxybutynin (an anticholinergic medication) for an overactive
bladder. The most common side effect of this medication he will experience is:

A. Gastroesophageal reflux.
B. Blurred vision.
C. Urinary retention.
D. Dry mouth.
E. Constipation.

Question 32
A 16-year-old girl is admitted in a very confused state after a school dance. Her heart rate is
130/minute and blood pressure is 110/70 mmHg. She is hypervigilant, flushed and has dilated pupils.
She has a temperature of 38.5°C, and her skin and mucous membranes are dry. She is moving all
limbs and her reflexes and tone are normal.
Her friends imply this may be the result of a drug or toxin ingestion. Which of the following drugs or
toxins best explains this presentation?

A. Gamma hydroxybutyrate (GHB).


B. Methylenedioxymethamphetamine (MDMA) (Ecstasy).
C. Pheniramine.
D. Nitrous oxide.
E. Psilocybin (Magic Mushrooms).
335

Question 33
Which one of the following is the most appropriate medication for the acute treatment of alcohol
withdrawal?

A. Diazepam.
B. Phenytoin.
C. Midazolam.
D. Naltrexone.
E. Acamprosate.

Question 34
A 33-year-old woman presents with a three-week history of a constant generalised headache. The
headache has been partially responsive to paracetamol (500mg) with codeine (30mg), two tablets six
times per day, but the duration of effect for each dose is only a few hours. The headache persists
through the night and disturbs her sleep. There are no associated neurological symptoms. Her general
and neurological examination including fundoscopy are normal and blood pressure is 130/80 mmHg.
A cranial computed tomography (CT) scan and lumbar puncture are normal.
The most appropriate management is cessation of the simple analgesics and:

A. slow release tramadol.


B. slow release morphine.
C. botulinum toxin to the craniofacial muscles.
D. amitriptyline at night.
E. regular chlorpromazine.

Question 35
Which of the following anticonvulsant drugs is most likely to cause weight gain in young women?

A. Topiramate.
B. Lamotrigine.
C. Sodium valproate.
D. Carbamazepine.
E. Phenytoin.
336

Question 36
Oral medications, Drugs A and B, have the following properties in healthy individuals.

Drug A Drug B
Absorption 100% 100%
Bioavailability 2% 90%
Volume of distribution 350 L 50 L
Renal Clearance 2 L/hr 1 L/hr
Hepatic clearance 10 L/hr 2 L/hr
Protein Binding 75% 25%

Compared to healthy individuals, patients with cirrhosis have a much higher concentration of drug A
than drug B. The best explanation for this is the difference in:

A. Bioavailability.
B. Volume of distribution.
C. Renal clearance.
D. Hepatic clearance.
E. Protein binding.

Question 37
Of the following antiplatelet actions, which best describes the mechanism of clopidogrel?

A. Phosphodiesterase inhibition.
B. COX1 acetylation.
C. ADP receptor blocking.
D. GP IIb/IIIa blockade.
E. Factor Xa inhibition.

Question 38
Hepatic drug clearance is reduced in old age. Which of the following ageing changes is currently
believed to have the biggest impact on hepatic drug metabolism?

A. Reduced expression of CYP (cytochrome P450) genes.


B. Oxidative (post-translational) injury to drug metabolizing enzymes.
C. Impaired protein binding and reduced serum albumin.
D. Diminished hepatic volume.
E. Diminished hepatic blood flow.
337

Question 39
Drugs A and B have the following properties in healthy individuals:

Drug A Drug B
Volume of distribution 50 L 350 L
Renal Clearance 0.2 L/hr 7 L/hr
Hepatic Clearance 4 L/hr Nil
Protein Binding Nil 25%
Molecular Weight (Daltons) 343 390

In such healthy individuals, haemodialysis shortens the half-life of drug A much more than it
shortens the half-life of drug B. This difference is best explained by which of the following
characteristics of drug B?

A. Higher volume of distribution.


B. Much higher renal clearance.
C. Higher plasma protein binding.
D. Higher molecular weight.
E. Lower hepatic clearance.

Question 40
The renal clearance of drugs that are weak bases and weak acids is affected by changes in the urine
pH. The most accurate representation of how the pH changes clearance is:

Urine made more alkaline: Urine made more acidic:


Basic Drug Acid drug Basic Drug Acid Drug
A. ⇓ ⇑ ⇓ ⇑
B. ⇓ ⇓ ⇑ ⇑
C. ⇑ ⇑ ⇓ ⇓
D. ⇓ ⇑ ⇑ ⇓
E. ⇑ ⇓ ⇓ ⇑

⇑ increased clearance ⇓ decreased clearance

Question 41
Cholinesterase inhibitors are prescribed for Alzheimer's disease. Which of the following is a typical
dose-dependent (type A) adverse drug reaction expected with this group of drugs?

A. Bradycardia.
B. Urinary retention.
C. Constipation.
D. Dry mucous membranes.
E. Extrapyramidal movements.
338

Question 42
Which analgesic is most likely to lead to a serious drug interaction if combined with an MAO
(monoamine oxidase) inhibitor (e.g. moclobemide)?

A. Morphine.
B. Indomethacin.
C. Tramadol.
D. Celecoxib.
E. Fentanyl.

Question 43
A 50 kg person is given gentamicin 320 mg daily IV. The serum concentration measured one hour
post the infusion is 20 mg/L and the serum concentration six hours post infusion is 5 mg/L. Using
this data, what is the best estimate of the serum gentamicin clearance in this person?

A. 37 mg/hour.
B. 12.5 mg/hour.
C. 1 L/hour.
D. 3.5 L/hour.
E. 3 mg/L/hour.

Question 44
Cholinesterase inhibitors are widely used to improve cognitive function in Alzheimers disease.
Which of the following urinary symptoms would be the most likely type A (dose-dependent) adverse
drug reaction?

A. Increased urinary frequency.


B. Reduced urine output.
C. Increased urine output.
D. Urinary retention.
E. Stress incontinence.

Question 45
Which of the following most increases the risk of hepatotoxicity of paracetamol when taken in
overdose?

A. Cigarette smoking.
B. Old age.
C. Chronic Alcohol ingestion.
D. Acute Alcohol ingestion.
E. Hepatitis B Virus (HBV) infection.
339

Question 46
All of the following drugs have clinically significant variation in response due to genetic
polymorphisms. For which of the following drugs is the variation in effect most related to
pharmacodynamic (as opposed to pharmacokinetic) response?

A. Warfarin.
B. Carvedilol.
C. Azathioprine.
D. Phenytoin.
E. Isoniazid.

Question 47
Genetic variation in the thiopurine methyltransferase gene is most likely to lead to increased toxicity
following administration of:

A. Oxaliplatin.
B. Allopurinol.
C. Azathioprine.
D. Cyclophosphamide.
E. 5-fluorouracil.

Question 48
In older patients, beta blockers have decreased effect. Which of the following mechanisms best
explains this change in effect?

A. Decreased density of beta adrenergic receptors.


B. Decreased endogenous noradrenaline.
C. Decreased liver metabolism.
D. Decreased renal excretion.
E. Reduced intracellular cyclic adenosine monophosphate (cAMP).

Question 49
The main purpose for performing a phase I clinical trial of a drug in patients with metastatic cancer is
to:

A. Determine the drug anti-tumour activity.


B. Establish a dosage schedule for the drug.
C. Assess survival benefits.
D. Assess symptom palliation.
E. Determine the drug side effects.
340

Question 50
Levodopa is combined with carbidopa for the treatment of Parkinson’s disease. Carbidopa is added
to:

A. Slow absorption of levodopa.


B. Reduce nausea caused by levodopa.
C. Reduce the required dose of levodopa.
D. Block chemoreceptor-trigger zone receptors.
E. Reduce the breakdown of levodopa by catechol-O-methyl transferase.

Question 51
Nicotine users experience physiological dependence and titrate their dose primarily to:

A. Achieve a morning peak.


B. Achieve a consistent daily dose.
C. Achieve a dose interval of <2 half lives.
D. Avoid withdrawal symptoms.
E. Avoid mild toxicity.

Question 52
Approximately 5 – 10 percent of Caucasians have reduced capacity to metabolise drugs via
cytochrome P450 2D6 (CYP 2D6). This is most likely to be clinically relevant in treatment with
standard doses of which of the following drugs?

A. Cyclosporin (renal toxicity).


B. Codeine (failure of analgesia).
C. Hydralazine (drug-induced lupus).
D. Metoprolol (severe bradycardia).
E. Paracetamol (hepatic toxicity).

Question 53
Linezolid is most commonly associated with which of the following adverse reactions?

A. Interstitial nephritis.
B. Bullous skin eruptions.
C. Myelosuppression.
D. Cholestatic jaundice.
E. Myaesthenia gravis.
341

Question 54
Which of the following most increases the risk of hepatotoxicity of paracetamol when taken in
overdose?

A. Cigarette smoking.
B. Old age.
C. Chronic alcohol ingestion.
D. Acute alcohol ingestion.
E. Hepatitis B virus (HBV) infection.

Question 55
Poisoning with which of the following causes a shift in the Hb-oxygen dissociation curve to the left?

A. Carbon monoxide.
B. Cyanide.
C. Gamma-hydroxy-butyrate (GHB).
D. Methanol.
E. Aspirin.

Question 56
Which of the following factors is most predictive of the development of cyclophosphamide-induced
gonadal failure in pre-menopausal females, with systemic necrotising vasculitis, who are undergoing
cyclophosphamide therapy?

A. Gonadal activity during therapy.


B. Older age.
C. Cumulative dose.
D. Pulsed intravenous cyclophosphamide delivery.
E. Concurrent therapy with MESNA.
342

Question 57
The bioavailability of a rectal preparation of an anticonvulsant is assessed in a crossover study. The
results are as follows.

Which of the following is the best estimate of the rectal bioavailability of this drug?

A. 12.5%
B. 20%
C. 30%
D. 66%
E. 90%
343

Answers

1. E 30. C
2. C 31. D
3. C 32. C
4. D 33. A
5. B 34. D
6. D 35. C
7. E 36. A
8. E 37. C
9. D 38. E
10. C 39. A
11. E 40. D
12. D 41. A
13. C 42. C
14. C 43. D
15. E 44. A
16. D 45. C
17. A 46. B
18. E 47. C
19. E 48. A
20. D 49. B
21. D 50. B
22. B 51. D
23. A 52. B
24. C 53. C
25. A 54. C
26. B 55. A
27. E 56. B
28. C 57. C
29. C
344

Clinical Applications

Pharmacology

Question 1
Digoxin is most likely to have an adverse pharmacodynamic (as opposed to pharmacokinetic)
interaction with which one of the following drugs?

A. Gentamicin.
B. Lithium.
C. Diltiazem.
D. Indomethacin.
E. Amiodarone.

Question 2
The anticoagulant effect of warfarin is most likely to be enhanced by which one of the following?

A. Carbamazepine.
B. Erythromycin.
C. Oral contraceptive.
D. Phenytoin.
E. Vitamin K.

Question 3
In a woman receiving long-term phenytoin therapy for epilepsy, pregnancy is likely to result in a
reduction in the total concentration of phenytoin measured in plasma.

The pharmacokinetic alteration most important for the interpretation of plasma concentrations during
pregnancy is that there is:

A. reduced absorption of phenytoin from the gastrointestinal tract.


B. a dilutional effect resulting from an increase in plasma volume.
C. reduced protein-binding resulting in an increased free fraction of phenytoin.
D. increased hepatic clearance of phenytoin.
E. increased renal clearance of the metabolites of phenytoin.
345

Question 4
In a patient with an acute overdose or poisoning, haemodialysis is of most benefit in reducing the
toxicity of which one of the following drugs/substances?

A. Ethylene glycol.
B. Tricyclic antidepressants.
C. Digoxin.
D. Barbiturates.
E. Paraquat.

Question 5
An 18-year-old woman presents with a five-hour history of vomiting, air hunger, sweating,
palpitations, tinnitus, lethargy and confusion, progressing to depressed consciousness. Her sister says
that she has been depressed recently over a relationship breakdown. Physical examination reveals
dehydration, tachycardia and pulmonary congestion but is otherwise normal. Urinalysis is positive
for ketones. ECG shows sinus tachycardia with no other abnormalities.

Biochemical results indicate:


hypernatraemia
increased anion gap
metabolic acidosis
normal lactate
mild hypoglycaemia.

The cause of this presentation is most likely to be due to an overdose of which one of the following?

A. Salicylate.
B. Paracetamol.
C. Ethylene glycol.
D. Tricyclic antidepressants.
E. Iron.
346

Question 6
A 45-year-old woman collapses in the supermarket and the rhythm strip taken by the paramedical
ambulance staff shows initial torsades de pointes ventricular tachycardia with a rate of 200/minute.
After reversion to sinus rhythm, it is observed that she has a long QTc interval. Her complete drug
history is uncertain, although it is known that she was prescribed a course of erythromycin over the
previous week to treat a respiratory tract infection.

Which one of the following drugs is most likely to have interacted with erythromycin to cause her
current problem?

A. Quinidine.
B. Loratadine.
C. Cimetidine.
D. Sotalol.
E. Terfenadine.

Question 7
A patient who shows an unexpectedly small analgesic response to codeine phosphate is most likely
to have which one of the following phenotypes of polymorphic drug metabolising enzymes?

A. Slow acetylator (reduced activity of N-acetyl transferase).


B. Atypical plasma pseudocholinesterase.
C. Poor metaboliser (reduced activity of cytochrome P450 2D6).
D. Impaired hydroxylation of mephenytoin.
E. Rapid metaboliser (multiple copies of the gene encoding cytochrome P450 2D6).

Question 8
The most important reason for the higher incidence of adverse drug reactions in the elderly compared
to young people is:

A. Pharmacokinetic changes.
B. Pharmacodynamic changes.
C. Reduction in the efficiency of homeostatic mechanisms.
D. The presence of co-existing pathologies.
E. Polypharmacy.

Question 9
Which one of the following is least characteristic of cyclosporin nephrotoxicity?

A. Interstitial fibrosis.
B. Arterial endothelial injury.
C. Reduced uric acid secretion.
D. Impaired potassium secretion.
E. Glomerulopathy.
347

Question 10
A 75-year-old patient with chronic atrial fibrillation has been stable on warfarin 4 mg/day
(international normalised ratio (INR) ranges from 2.1 to 2.5). The patient takes thyroxine 100 µg/day
for hypothyroidism.

Which one of the following actions is most likely to potentiate the risk of bleeding?

A. Reducing the dose of thyroxine.


B. Increasing dietary vegetable intake.
C. Treatment with cholestyramine.
D. Treatment with omeprazole.
E. Treatment with amiodarone.

Question 11
Which of the following antibiotics most commonly leads to oesophageal ulceration?

A. Amoxycillin.
B. Ciprofloxacin.
C. Clindamycin.
D. Doxycycline.
E. Erythromycin.

Question 12
‘Punchdrunk’ is a hypothetical new drug for alcohol withdrawal. The pharmacokinetic section of the
product information states:

“Punchdrunk is rapidly absorbed after oral administration but undergoes extensive hepatic
metabolism before reaching the systemic circulation. Peak plasma levels are reached within 90
minutes for the oral capsules. The bioavailability of oral capsules is low and variable (2-10% after
two capsules in healthy adults) and may increase at higher doses. Punchdrunk has been shown to be
approximately 65% bound to plasma proteins. The volume of distribution has been shown to be of
the order of 9 L/kg in healthy adults and increases to about 13 L/kg in the healthy elderly. The half-
life during the elimination phase is approximately four hours in young healthy volunteers.
Punchdrunk is metabolised extensively in the body since less than 1% of the dose is excreted
unchanged in the urine. Five urinary metabolites have been found to account for 80% of an oral dose.
The metabolites each have between 10-20% of the potency of the parent drug.”

Which clinical condition would lead to the largest increase in peak drug concentration with regular
doses?

A. Nephrotic syndrome.
B. Portal vein thrombosis.
C. Acute infectious hepatitis.
D. Chronic renal failure.
E. End stage renal disease (on haemodialysis).
348

Question 13
An unconscious 50-year-old man is brought to the emergency department four hours after an
intentional overdose of approximately fifty 50 mg amitriptyline tablets. He has had a brief
convulsion in the ambulance on the way to hospital. An ECG shows a sinus tachycardia with
widened QRS complexes of 0.13 seconds. Monitoring shows runs (lasting up to five seconds) of a
broad complex tachycardia.

He is intubated and transferred to intensive care.

The most important initial management is:

A. Amiodarone therapy.
B. Phenytoin therapy.
C. Bicarbonate therapy.
D. Lignocaine therapy.
E. Haemodialysis.

Question 14
A 24-year-old woman is in the orthopaedic ward following surgery for a fractured femur two days
previously. She develops symptoms of agitation, abdominal cramps and diarrhoea.

On examination she is febrile and sweaty, with stimulus-sensitive myoclonus and mild confusion.
Her medical history includes depression currently treated with sertraline 200 mg/day. Post-operative
medications include tramadol SR 200 mg twice daily, paracetamol 1 g six-hourly, naproxen 500 mg
twice daily and temazepam 10 mg nocte.

The most likely diagnosis is:

A. Fat embolism syndrome.


B. Serotonin syndrome.
C. Malignant hyperpyrexia syndrome.
D. Malignant neuroleptic syndrome.
E. Alcohol withdrawal.

Question 15
To prevent adverse drug effects, which of the following antimicrobials requires the greatest dose
reduction in severe renal impairment?

A. Ticarcillin.
B. Aciclovir.
C. Cefaclor.
D. Norfloxacin.
E. Meropenem.
349

Question 16
A 52-year-old woman has a long history of bipolar affective disorder and is treated with lithium 750
mg/day. She has been well for four years but has recently been prescribed frusemide 40 mg/day for
treatment of mild peripheral oedema. She develops ataxia, tremor and diarrhoea. Her lithium level is
1.9 mmol/L (therapeutic range 0.4-1.0 mmol/L).

The most appropriate management is to cease:

A. Lithium and repeat lithium level in one week.


B. Lithium and frusemide and repeat lithium level in 24 hours.
C. Frusemide and repeat lithium level in 24 hours.
D. Lithium and frusemide and repeat lithium level in one week.
E. Frusemide and repeat lithium level in one week.

Question 17
Which of the following anticonvulsant drugs is most likely to cause a clinically important drug
interaction with lamotrigine?

A. Clonazepam.
B. Gabapentin.
C. Vigabatrin.
D. Carbamazepine.
E. Sodium valproate.

Question 18
A study is comparing the bioavailability of a new oral preparation of a drug (100 mg tablet) with the
marketed intravenous preparation (50 mg in 2 mLs). In this crossover study, healthy volunteers were
given in random order one 100 mg tablet or 50 mg intravenously (IV). The drug has linear kinetics
within the range of concentrations studied. The mean results are as follows:

100 mg tablet 50 mg IV
Maximum concentration (C max) 5 12
(mg/L)
Time to peak concentration (Tmax) 2 0.25
(hours)
Area under the Curve (AUC) 45 25
(mg/L.h)
Elimination half life (t1/2) (hours) 4 3.8

Which of the following is the best estimate of the oral bioavailability of the tablet?
A. 12.5%.
B. 40 %.
C. 50%.
D. 90%.
E. 96%.
350

Question 19
The relationship of the left ventricular pressure to volume throughout a single cardiac cycle
(Pressure-Volume Loop) is recorded on two occasions in an intact anaesthetised animal, before
(control) and during an intravenous infusion of a pharmacological agent (treatment). The results are
shown below.

Which of the following agents is most likely to have been infused?

A. Noradrenaline.
B. Glyceryl trinitrate.
C. Metoprolol.
D. Isoprenaline.
E. Metaraminol.
351

Question 20
A patient is taking Drug A for a chronic condition. Drug B is added for a period of time and then
stopped. The half-life of both drugs is between 6 and 12 hours. The following graph shows the total
and free concentrations of Drug A: before Drug B is added, over the time Drug B is added, and after
Drug B is ceased.

The most likely mechanism for the interaction is:

A. Protein binding displacement.


B. Induction of metabolism.
C. Inhibition of metabolism.
D. Binding in gut.
E. Decreased hepatic blood flow.
352

Question 21
Drugs A and B have the following properties in healthy individuals:

Drug A Drug B
Volume of distribution 50 L 350 L
Renal Clearance 0.2 L/hr 7 L/hr
Hepatic Clearance 4 L/hr Nil
Protein Binding Nil 25%
Molecular Weight (Daltons) 343 390

In such healthy individuals, haemodialysis shortens the half-life of drug A much more than it
shortens the half-life of drug B. This difference is best explained by which of the following
characteristics of drug B?

A. Higher volume of distribution.


B. Much higher renal clearance.
C. Higher plasma protein binding.
D. Higher molecular weight.
E. Lower hepatic clearance.

Question 22
Which of the following is the most important reason for not recommending gentamicin for the
treatment of coliform central nervous system (CNS) infections?

A. It is not active in an acidic environment.


B. It is not active in a low oxygen tension environment.
C. It has poor CNS penetration.
D. It may precipitate seizures.
E. Ototoxicity risk is accentuated.

Question 23
Knowledge of which of the following genotypes is most likely to be useful in preventing serious
adverse drug events?

A. Thiopurine methyltransferase (azathioprine).


B. CYP3A4 (cyclosporin).
C. Dihydrofolate reductase (methotrexate).
D. Type II glucocorticoid receptor (prednis(ol)one).
E. Plasma cholinesterase (cyclophosphamide).
353

Question 24
A 42-year-old man in the intensive care unit following a motor vehicle accident develops
Staphylococcus aureus bacteraemia. He is commenced on vancomycin. Following the first infusion
he develops a rash involving the face and upper trunk. His blood pressure falls to 100/70 mmHg.
What is the most appropriate next step?

A. Stop vancomycin, commence linezolid.


B. Stop vancomycin, commence dalfopristin/quinupristin.
C. Commence rapid desensitisation for vancomycin.
D. Continue vancomycin but slow the rate of infusion.
E. Continue vancomycin but premedicate with hydrocortisone.

Question 25
A 20-year-old female presents 48 hours after ingesting 16g of paracetamol. There are no signs of
hepatic encephalopathy. Blood tests on admission include alanine transaminase (ALT) of 8000 IU/L
[15 – 45 IU/L]. Treatment with N-acetyl cysteine is commenced. After 16 hours she has developed
mild abdominal tenderness and her international normalised ratio (INR) is 2.5.

In addition to liaising with a liver transplant unit, the most appropriate management is:

A. Continue N-acetyl cysteine.


B. Give vitamin K.
C. Commence treatment with extracorporeal liver-assist device.
D. Parenteral antioxidants.
E. Intravenous prostacyclin.

Question 26
A 68-year-old female presents with headache, nausea and vomiting. She is noted to have retinal
haemorrhages and papilloedema. Her blood pressure is 250/150 mmHg, creatinine 200 µmol/L[50 -
120 µmol/L] and urea 28 mmol/L [<8.0 mmol/L]. The computed tomography (CT) scan of her brain
shows abnormalities in the parieto-occipital regions.

Which is the most appropriate initial treatment?

A. Captopril 12.5 mg three times daily orally.


B. Metoprolol 5 mg intravenously.
C. Sodium nitroprusside 0.25 microgram/kg/min intravenously.
D. Nifedipine 10 mg sublingually.
E. Glyceryl trinitrate 10 mg/24h topically.
354

Question 27
An 81-year-old female has atrial fibrillation and osteoarthritis, treated with warfarin, digoxin and
paracetamol. She recently complained of depressive symptoms and was commenced on the selective
serotonin reuptake inhibitor, fluoxetine. Two weeks later she is brought to casualty with an
extradural haematoma and her international normalised ratio (INR) is 6.9.
The most likely mechanism by which fluoxetine contributed to the haemorrhage is by its effect on:

A. Cytochrome P-450.
B. Glucuronidation.
C. Platelets.
D. Vitamin K.
E. Compliance.

Question 28
A previously well 30-year-old woman presents 24 hours after ingestion of around 50 tablets which
have not been identified. She initially had minimal symptoms for a few hours and then developed
severe vomiting and bloody diarrhoea. This has now settled but she is now hypotensive, tachycardic,
tachypnoeic, has a metabolic acidosis, poor urine output and raised liver transaminases (alanine
transaminase (ALT) 350 U/L [<35 U/L]).

The poisoning most consistent with this presentation is:

A. Paracetamol.
B. Theophylline.
C. Amitriptyline.
D. Iron.
E. Ibuprofen.

Question 29
A 60-year-old female with hypertension and epilepsy is referred to outpatients for management. She
smokes 20 cigarettes/day. What is the most appropriate intervention to achieve abstinence from
tobacco smoking?

A. Nicotine replacement therapy.


B. Referral to a telephone counselling line.
C. Clear medical advice to stop smoking.
D. Referral for acupuncture.
E. Prescription of bupropion.
355

Question 30
A 67-year-old diabetic man with atrial fibrillation is taking warfarin for primary prophylaxis and is
scheduled for inguinal hernia repair. His echocardiogram shows no structural abnormality. His
international normalised ratio (INR) is 2.9. Which of the following preoperative anticoagulant
management strategies is most appropriate?

A. Withhold warfarin therapy for four days before surgery and administer low-molecular-weight
heparin (LMWH) for these four days, ceasing 24 hours prior to surgery.
B. Withhold warfarin therapy four days before surgery and administer vitamin K1 on the evening
prior to surgery if his INR> 1.5.
C. Withhold warfarin for two days prior to surgery.
D. Continue warfarin up until the time of surgery then administer prothrombin complex
concentrate on the day of surgery.
E. Withhold warfarin therapy for four days before surgery. Commence intravenous infusion of
unfractionated heparin two days prior to surgery, ceasing four hours prior to surgery.

Question 31
Use of which of the following drug classes is most likely to lead to the onset of type 2 diabetes?

A. Atypical antipsychotic drugs.


B. Beta-blockers.
C. Thiazides.
D. Monoamine oxidase inhibitors.
E. Tricyclic antidepressants.
356

Answers

1. C 17. E
2. B 18. D
3. C 19. A
4. A 20. A
5. A 21. A
6. E 22. C
7. C 23. A
8. E 24. D
9. E 25. A
10. E 26. C
11. D 27. A
12. B 28. D
13. C 29. A
14. B 30. B
15. B 31. A
16. B
357

Medical Sciences

Renal Medicine

Question 1
Regulation of water reabsorption in the collecting duct of the nephron is least dependent upon which
one of the following molecules?

A. Mineralocorticoid receptor.
B. Aquaporin 2.
C. Adenylyl cyclase.
D. Antidiuretic hormone receptor.
E. G proteins.

Question 2
A 25-year-old woman presents to the emergency department with renal colic and macroscopic
haematuria. She reports having had several previous episodes treated at another hospital.

The least likely diagnosis is:

A. IgA disease.
B. Warfarin use.
C. Narcotic abuse.
D. Goodpasture’s syndrome.
E. Polycystic kidney disease.

Question 3
A patient with pancreatic insufficiency refuses to take enzyme replacement and later presents with
renal calculi.

Increased excretion of which one of the following is the most likely cause of calculus formation?

A. Urate.
B. Phosphate.
C. Bicarbonate.
D. Oxalate.
E. Cysteine.
358

Question 4
A 35-year-old migrant to Australia is concerned that he may have polycystic kidney disease as his
mother was recently diagnosed with the condition. Which of the following has the highest negative
predictive value for polycystic kidney disease?

A. Direct genetic mutation analysis.


B. Absence of proteinuria.
C. Genetic linkage analysis.
D. Normal renal ultrasound.
E. Normal creatinine clearance.

Question 5
In renal transplant biopsies, which one of the following features of acute rejection has the worst
prognostic implication?

A. Glomerular T cell infiltrate.


B. Dense interstitial mononuclear infiltrate.
C. Increased expression of HLA-DR on graft cells.
D. Vascular injury.
E. Tubulitis due to CD8+ T cells.

Question 6
Which feature of glomerular pathology is least consistent with the diagnosis of Wegener's
granulomatosis?

A. Focal necrosis.
B. Crescents.
C. IgG deposition.
D. T cell infiltrate.
E. Macrophage infiltrate.

Question 7
A rise in creatinine after the initiation of antihypertensive treatment for chronic hypertension is
primarily due to which of the following mechanisms?

A. Changes in efferent arteriolar myointimal constriction.


B. Consistent reductions in heart rate.
C. Shift in autoregulatory response.
D. Increase in intra-glomerular pressure.
E. Altered sodium concentration in the distal tubule.
359

Question 8
In patients with autosomal dominant polycystic kidney disease (PKD) due to a mutation in the PKD1
gene, which one of the following processes best accounts for the formation of renal cysts?

A. Somatic mutation of normal PKD1 allele.


B. Non-clonal expansion of tubular epithelial cell types.
C. Apoptosis of tubular epithelial cell types.
D. Somatic recombination between PKD1 and PKD2 genes.
E. Downregulation of the c-myc gene.

Question 9
Endothelial cell pathology on renal biopsy is most characteristic of which one of the following
diagnoses?

A. Pre-eclampsia.
B. Haemolytic uraemic syndrome.
C. Lupus nephritis.
D. Immunoglobulin A (IgA) disease.
E. Wegener's granulomatosis.

Question 10
Which pathologic process on renal biopsy is most characteristic of acute vascular renal transplant
rejection?

A. Eosinophil infiltration.
B. Macrophage infiltration.
C. T lymphocyte infiltration.
D. Interstitial haemorrhage.
E. Mesangial proliferation.

Question 11
In a patient with IgA nephropathy who has a serum creatinine in the normal range, which of the
following is the best predictor of developing end-stage renal failure?

A. Urine red cell count.


B. Serum IgA level.
C. Creatinine clearance.
D. Level of proteinuria.
E. Blood pressure.
360

Question 12
Which of the following is the most effective therapy for hyperphosphataemia associated with chronic
renal failure?

A. Low phosphate diet.


B. Oral phosphate binders.
C. Parathyroidectomy.
D. Vitamin D analogues.
E. Low protein diet.

Question 13
A 24-year-old is noted to have persistent microscopic haematuria following an insurance
examination. Which one of the following factors provides the strongest indication for a renal biopsy?

A. 90 glomerular red cells/µl in the urine [<10].


B. Blood pressure 145/80 mmHg.
C. Proteinuria 0.9 g/day [<0.12].
D. Serum creatinine 105 µmol/L [60-120].
E. Male gender.

Question 14
A 60-year-old man presents with nephrotic syndrome. He undergoes a renal biopsy. What is the
most likely histological finding?

A. Minimal change nephritis.


B. Focal and segmental glomerulosclerosis.
C. IgA (immunoglobulin A) nephropathy.
D. Membranous glomerulonephritis.
E. Amyloidosis.

Question 15
A man with chronic renal failure is on the renal transplant waiting list. He is serologically
cytomegalovirus (CMV) negative. If he is to receive a transplant from a CMV positive donor, which
of the following is the most appropriate prophylactic regime for CMV disease?

A. Live attenuated CMV vaccination.


B. CMV hyperimmune globulin.
C. Oral acyclovir.
D. Intravenous ganciclovir.
E. Oral valganciclovir.
361

Question 16
Hypomagnesemia is a common biochemical abnormality after renal transplantation and may cause
serious cardiac arrhythmias. In this setting, it is most commonly due to:

A. Persistent hyperparathyroidism.
B. Vitamin D deficiency.
C. Mycophenolate.
D. Cyclosporin.
E. Prednisolone.

Question 17
A 60-year-old man presents with nephrotic syndrome. He undergoes a renal biopsy. What is the most
likely histological finding?

A. Minimal change nephritis.


B. Focal and segmental glomerulosclerosis.
C. IgA (immunoglobulin A) nephropathy.
D. Membranous glomerulonephritis.
E. Amyloidosis.

Question 18
A 30-year-old woman has known Sjogren’s syndrome and a past history of a single renal calculus.
She presents with a metabolic acidosis, with hyperchloraemia. The plasma anion gap is normal.
There is no bicarbonate in the urine and the urine pH is 5.7. What is the most likely cause of her
presentation?

A. Proximal renal tubular acidosis.


B. Osteomalacia.
C. Chronic diarrhoea.
D. Distal renal tubular acidosis.
E. Type 4 renal tubular acidosis.

Question 19
Which of the following measures is most effective in preventing contrast nephropathy in at-risk
patients undergoing imaging procedures involving iodine based contrast agents?

A. The use of a low osmolar contrast agent.


B. The use of a non-ionic contrast agent.
C. Hydration with normal saline prior to the procedure.
D. Pre-treatment with N-acetyl cysteine.
E. Pre-treatment with sodium bicarbonate.
362

Answers

1. A
2. D
3. D
4. D
5. D
6. C
7. C
8. B
9. A
10. D
11. D
12. B
13. C
14. D
15. E
16. D
17. A
18. D
19. C
363

Clinical Applications

Renal Medicine

Question 1
A 45-year-old woman has had six episodes of nephrotic syndrome over the last five years. The initial
episodes responded promptly and completely to oral prednis(ol)one, but recent episodes have
responded more slowly. At the completion of her last course of prednis(ol)one two weeks ago, urine
protein was 500 mg/day.

The trichrome-stained renal biopsy is shown below.

Which one of the following is the most likely diagnosis?

A. Minimal change glomerulonephritis.


B. Amyloidosis.
C. Focal sclerosing glomerulonephritis.
D. IgA nephropathy.
E. Membranous glomerulonephritis.
364

Question 2
A 23-year-old woman presents with generalised swelling. Over the last three days she has felt tired, has
experienced bilateral loin pain and has noticed darkening of her urine. She has had a skin infection on her
knee which has been slow to resolve. On examination, she has mild generalised oedema, a blood pressure of
170/115 mmHg and tenderness in the loins. The skin lesion is still encrusted and erythematous. Urinalysis
shows ++proteinuria and ++blood. Other examination is unremarkable.

Investigations show:
Full blood count normal
Serum biochemistry:
sodium 135 mmol/L [137-146]
potassium 5.1 mmol/L [3.5-5.0]
chloride 102 mmol/L [98-108]
bicarbonate 21 mmol/L [26-30]
urea 44.0 mmol/L [3.5-8.5]
creatinine 0.72 mmol/L [0.06-0.12]
albumin 35 g/L [35-45]
total protein 60 g/L [55-90]
Microurine shows:
red blood cells >100 per high power field [0-1]
white blood cells 10-50 per high power field [0-1]
red cell casts present [0]

An urgent renal biopsy is performed, and is shown below. All glomeruli in the biopsy have similar changes to
those shown. Immunofluorescence shows IgG and C3 (complement component 3) along glomerular walls.

Serological investigations show:


antinuclear antibody (ANA) <1:40 [<1:40]
antineutrophil cytoplasmic antibody (ANCA) negative
antistreptolysin-O titre (ASOT) not elevated
antiDNAse-B titre 200 U [<30]
cryoglobulins positive
complement component 3 (C3) 0.30 g/L [0.55-1.20]
complement component 4 (C4) 0.18 g/L [0.20-0.50]
throat culture negative
Apart from control of blood pressure, fluid and electrolyte management and acute dialysis if indicated, the
most appropriate therapy for this renal disease is:

A. Prednis(ol)one.
B. Prednis(ol)one and an alkylating agent.
C. Prednis(ol)one, an alkylating agent and plasmapheresis.
D. Long-term dialysis.
E. No additional therapy, observation only.
365

Question 3
A 31-year-old female Cambodian refugee is found to have nephrotic syndrome.

Investigations reveal:
urinary protein 7.00 g/day [<0.25]
serum albumin 23 g/L [35-50]
serum creatinine 0.07 mmol/L [0.03-0.11]

A renal biopsy shows membranous nephropathy.

Which one of the following tests is least likely to find the cause?

A. Mantoux test.
B. Antinuclear antibody.
C. Malaria thick film.
D. Hepatitis B serology.
E. Syphilis serology.

Question 4
A 55-year-old male wheat farmer has a five-year history of recurrent renal calculi, occurring two to
five times every year. He has required surgery or lithotripsy treatment on five occasions. He has no
other significant history of illness and his physical examination is normal. All stones that have been
analysed are composed of calcium oxalate. Repeated testing has revealed normal serum calcium,
phosphate, alkaline phosphatase and parathyroid hormone levels. His serum electrolytes, urea,
creatinine and uric acid are normal. Analysis of 24-hour urinary collection reveals hypercalciuria and
hyperuricosuria.

The therapy least likely to reduce his rate of renal calculus formation is:

A. A low calcium diet.


B. A high fluid intake.
C. Alkalinisation of urine.
D. Allopurinol.
E. A thiazide diuretic.

Question 5
Which one of the following is least characteristic of cyclosporin nephrotoxicity?

A. Interstitial fibrosis.
B. Arterial endothelial injury.
C. Reduced uric acid secretion.
D. Impaired potassium secretion.
E. Glomerulopathy.
366

Question 6
You are asked to see a 68-year-old man with mild, stable, intermittent claudication and hypertension.
He also has symptoms of prostatism. In the past year, his blood pressure control has deteriorated
despite increasing antihypertensive medication. One month ago, his local physician initiated therapy
with an angiotensin-converting enzyme (ACE) inhibitor.

When you see the patient, his blood pressure is 150/90 mmHg. However, his serum creatinine has
increased from 0.13 to 0.41 mmol/L [0.05-0.11] in the month following commencement of blood
pressure therapy.

The most likely cause of the patient’s renal deterioration is:

A. atheroembolism to the renal arteries.


B. rapidly progressive arteriolar nephrosclerosis.
C. obstructive uropathy.
D. bilateral renal artery stenosis.
E. improvement in blood pressure control.

Question 7
A 56-year-old man presents with a three-day history of left-sided loin pain, nausea and vomiting.
Past operations include a right nephrectomy for renal calculi. On examination he is slightly
dehydrated and his blood pressure is 130/80 mmHg with no postural drop. There are no abdominal
masses but tenderness in the left loin is detected. Rectal examination is normal. Urinary
catheterisation reveals an empty bladder. Over the last hour urine output has been 20 mL.

Blood tests reveal:

full blood examination normal


sodium 141 mmol/L [136-145]
potassium 5.0 mmol/L [3.5-5.0]
chloride 108 mmol/L [98-106]
bicarbonate 18 mmol/L [27-32]
urea nitrogen 39.0 mmol/L [3.6-7.0]
creatinine 0.62 mmol/L [0.04-0.13]
glucose 6.8 mmol/L [4.2-5.8]

Liver function tests, amylase and lipase are normal.

An urgent renal ultrasound shows a dilated calyceal system in the left kidney and a large calculus at
the pelvi-ureteric junction.

The most important next therapy is:

A. Urgent haemodialysis.
B. Oral resonium.
C. Percutaneous nephrostomy.
D. Retrograde pyelography and stent.
E. Lithotripsy.
367

Question 8
A 30-year-old woman has known Sjogren’s syndrome and a past history of a single renal calculus.
She presents with a metabolic acidosis, with hyperchloraemia. The plasma anion gap is normal.
There is no bicarbonate in the urine and the urine pH is 5.7. What is the most likely cause of her
presentation?

A. Proximal renal tubular acidosis.


B. Osteomalacia.
C. Chronic diarrhoea.
D. Distal renal tubular acidosis.
E. Type 4 renal tubular acidosis.

Question 9
A 23-year-old woman, during her first pregnancy, has a blood pressure of 145/90 mmHg (on
repeated measurements) at 32 weeks’ gestation. Her previous blood pressures have been 125/75
mmHg or less. Her serum electrolytes, urea and creatinine are normal but her serum uric acid is 0.38
mmol/L [0.09-0.36]. Urinalysis shows proteinuria +++, but no evidence of blood or glucose. Liver
function tests and full blood count are normal.

Which one of the following is the most appropriate initial treatment?

A. Bed rest.
B. Chlorothiazide.
C. Irbesartan.
D. Enalapril.
E. Methyldopa.

Question 10
A 57-year-old man has known chronic kidney disease due to mesangio-capillary glomerulonephritis.
He presents now, relatively asymptomatic but with the following parameters:

His anaemia is most likely due to:

A. Worsening of his renal function.


B. Haemolysis associated with his glomerulonephritis.
C. Haematuria associated with his glomerulonephritis.
D. Gastro-intestinal bleeding.
E. Folate deficiency.
368

Question 11
Persistent reduction of serum complement component 3 (C3) is most strongly associated with which
one of the following forms of glomerulonephritis?

A. Mesangiocapillary.
B. Diffuse proliferative systemic lupus erythematosus (SLE).
C. Idiopathic membranous.
D. Post-streptococcal.
E. Mesangial IgA disease.

Question 12
A patient develops fever, abnormal liver function tests and diarrhoea on day 60 post-renal
transplantation.

Which of the following would be the most likely cause?

A. Acute renal rejection.


B. Graft-versus-host disease.
C. Tacrolimus toxicity.
D. Mycophenolate toxicity.
E. Cytomegalovirus (CMV) disease.

Question 13
In a patient with rheumatoid arthritis who has a rising serum creatinine level, continued use of
methotrexate, without dose reduction, is most likely to cause which of the following?

A. Marrow suppression.
B. Hepatotoxicity.
C. Accelerated renal failure.
D. Hypersensitivity pneumonitis.
E. Hypertension.
369

Question 14
A 35-year-old man who is receiving erythropoietin (EPO) therapy for anaemia associated with end
stage renal disease presents with lethargy. There is no obvious source of bleeding.

Blood investigations show:


haemoglobin 65 g/L [110-160]
mean corpuscular volume (MCV) 109 fL [83-97]
reticulocyte count 15 x 109/L [50-100]

The most likely cause of his anaemia is:

A. Red cell aplasia.


B. Reduced iron stores.
C. Non-neutralising anti-EPO antibodies.
D. Aluminium toxicity.
E. Myelodysplasia.

Question 15
A 47-year-old man on long-term haemodialysis presents with a fever (39.5°C) and chills. He has had
multiple thrombosed fistulas and a thrombosed right axillary vein. He had a left subclavian catheter
inserted under ultrasound guidance three days ago.

On examination he looks well, with a blood pressure of 170/80 mmHg and an exit site of the
vascular access catheter with the appearance shown below.

Which of the following management strategies would be most appropriate in his case?

A. Intravenous vancomycin and observation.


B. Immediate haemodialysis.
C. Intravenous cephalexin and observation.
D. Intravenous vancomycin and catheter removal.
E. Replace vascular catheter with a re-wire technique.
370

Question 16
A 29-year-old woman has been on haemodialysis for 14 years. She has no other comorbidities. Her
examination shows an elevated jugular venous pressure (JVP) (3 cm) and a displaced apex beat, her
heart sounds are dual and there are no crepitations in her lung bases. Her blood pressure is 150/90
mmHg. Her current medications include darbepoetin for anaemia, calcium supplementation and
calcitriol.

Serum electrolytes show:


sodium 145 mmol/L [135-145]
chloride 100 mmol/L [89-99]
potassium 5.3 mmol/L [3.5-5.0]
phosphate 2.7 mmol/L [0.3-1.2]
calcium 2.8 mmol/L [2.2-2.7]

Her urea is 18 mmol/L [3-8] and haemoglobin is 105 g/L [128-175].

Which of the above abnormalities is most indicative of risk for future cardiovascular disease?

A. Increased urea concentration.


B. Hyperkalaemia.
C. Anaemia.
D. Hypercalcaemia.
E. Hyperphosphataemia.
371

Question 17
A 59-year-old lady presents for reassessment of her increasing blood pressure. Her height is 162 cm
and weight is 52 kg. She has a 15-year history of hypertension and recent blood pressure recordings
have shown a marked increase. She has had a recent episode of dysarthria and clumsy right hand
which lasted 24 hours.

On examination her blood pressure is 180/80 mmHg lying and 180/100 mmHg sitting; her heart
sounds are dual and there is a 1/6 mid-systolic murmur at the left sternal edge. Her jugular venous
pressure (JVP) is not elevated. Her chest is clear and her abdominal examination is normal with no
vascular bruits noted. Her eye examination shows grade 2 changes of retinal arterial vessels. Her
right hand strength and speech are normal.

Her recent serum biochemistry shows:


sodium 144 mmol/L [135-145]
potassium 3.5 mmol/L [3.5-5.0]
creatinine 0.104 mmol/L [0.060-0.090]
chloride 101 mmol/L [89-98]
bicarbonate 29 mmol/L [24-35]

A magnetic resonance angiogram (MRA) is shown below.

A reduction in which one of the following is most likely to be achieved from an intervention to
correct the demonstrated abnormality?

A. Antihypertensive requirement.
B. Rate of renal artery thrombosis.
C. Blood pressure.
D. Rate of decline of glomerular filtration rate.
E. Episodes of pulmonary oedema.
372

Question 18
In deciding when to initiate renal replacement therapy in a patient with progressive chronic renal
failure, which of the following is the strongest indication for commencing dialysis?

A. Peripheral neuropathy.
B. Serum urea 50 mmol/L [<6.5].
C. Serum potassium 6.1 mmol/L [3.5-5.5].
D. Pericarditis.
E. Peripheral oedema.

Question 19
A 35-year-old man who is receiving erythropoietin (EPO) therapy for anaemia associated with end
stage renal disease presents with lethargy. There is no obvious source of bleeding.

Blood investigations show:


haemoglobin 65 g/L [110-160]
mean corpuscular volume (MCV) 100 fL [83-97]
9
reticulocyte count 15 x 10 /L [50-100]

The most likely cause of his anaemia is:

A. Red cell aplasia.


B. Reduced iron stores.
C. Non-neutralising anti-EPO antibodies.
D. Aluminium toxicity.
E. Myelodysplasia.

Question 20
A 38-year-old man with abdominal pain has the following results:

sodium 142 mmol/L [138-145]


potassium 2.9 mmol/L [3.5-5.2]
chloride 112 mmol/L [96-109]
corrected calcium 2.40 mmol/L [2.15-2.57]
creatinine 0.11 mmol/L [0.5-0.12]
bicarbonate 15 mmol/L [24-28]
albumin 42 g/L [38-48]

What is the best explanation for these results?

A. Salicylate overdose.
B. Renal tubular acidosis.
C. Ketoacidosis.
D. Severe vomiting.
E. Lactic acidosis.
373

Question 21
A 45-year-old woman presents with a three-year history of recurrent rash, characterised as palpable
purpura over both legs. Some of these lesions have ulcerated.

Laboratory results include:

Serum:
creatinine 0.10 mmol/L [0.06-0.11]
alanine transaminase (ALT) 51 U/L [15-50]
aspartate transaminase (AST) 64 U/L [15-45]
hepatitis C antibody positive
cryoglobulins positive with 8% cryoprecipitate

Urinalysis is normal.

Which of the following is the most appropriate management?

A. Plasmapheresis and corticosteroids.


B. Corticosteroids and pegylated interferon α.
C. Pegylated interferon α and ribavirin.
D. Cyclophosphamide and corticosteroids.
E. Cyclophosphamide and ribavirin.

Question 22
A 65-year-old man developed Wegener’s granulomatosis eight years previously. At that time he had
upper and lower respiratory tract and renal involvement. He achieved remission with prednis(ol)one
and daily cyclophosphamide which was continued over 18 months. Seven months previously he
experienced a relapse with arthritis, sinusitis, an active renal sediment and a rise in serum creatinine
from 90 to 115 µmol/L. The relapse was treated successfully with prednis(ol)one and weekly
methotrexate. Prednis(ol)one was tapered and withdrawn. Currently, he feels well and examination is
normal.

Laboratory tests show normal blood count, erythrocyte sedimentation rate (ESR) 25 mm/hr [0-20],
and normal creatinine. Urine examination shows no protein, 10-100 red cells/high power field, and
there are no dysmorphic cells.

Which of the following is the next most appropriate course of action?

A. Renal biopsy.
B. Replace methotrexate with cyclophosphamide.
C. Cystoscopy.
D. Serum antineutrophil cytoplasmic antibodies (ANCA) testing.
E. Reintroduce prednis(ol)one.
374

Question 23
A 65-year-old woman presents to the hospital for the first time with vague symptoms of fatigue. The
following routine biochemistry is noted:

potassium 5.5 mmol/L [3.5-5.5]


urea 48 mmol/L [<6.5]
creatinine 1010 µmol/L [<120]
calcium 2.1 mmol/L [2.1-2.5]
phosphate 2.3 mmol/L [0.8-1.4]

She is hypertensive with a sitting blood pressure of 165/95 mmHg. There is no clinical oedema.
Apart from the fatigue she is asymptomatic, although some scratch marks are noted. She has 8.8cm
kidneys bilaterally on ultrasound, without obstruction. She has a past history of cholecystectomy and
perforated appendix.

What is the most appropriate next step in her management?

A. Urgent haemodialysis.
B. Urgent peritoneal dialysis.
C. Elective haemodialysis.
D. Elective peritoneal dialysis.
E. Urgent continuous venovenous haemodiafiltration.

Question 24
A 65-year-old woman, established on haemodialysis for nine months, is maintained on erythropoietin
4,000 units twice a week intravenously. Previously her haemoglobin (Hb) had remained steady in
the range of 105 – 115 gm/L. She now presents with a Hb of 89 gm/L on routine testing. There have
been no changes to her erythropoietin dose or other medications. Her mean corpuscular volume
(MCV) is 84 fL [78-98], her serum iron is 13 µmol/l [13-35], iron saturation is 18% [15-46], serum
ferritin is 630 µg/l [20-300] and her reticulocyte count is 30 x 109/L [12-100]. Which of the
following is the most likely cause for her erythropoietin resistance?

A. Inflammation.
B. Pure red cell aplasia.
C. Vitamin B12 deficiency.
D. Iron deficiency.
E. Hyperparathyroidism.
375

Question 25
A 65-year-old man presents with a two week history of fever and arthralgia and is found to have a
serum creatinine of 750 µmol/L [<110 µmol/L]. Which of the following best supports a diagnosis of
crescentic glomerulonephritis?

A. 24 hour urinary protein excretion of 2.3 gm/day [<0.2 gm/day].


B. The presence of macroscopic haematuria.
C. Urinary albumin/creatinine ratio of 35 mg/mmol [<3.5 mg/mmol]
D. The presence of urinary red cell casts.
E. The presence of dysmorphic urinary red cells >100/µl [<10/µl]

Question 26
A 70-year-old woman has been on haemodialysis for 11 years. She complains of painful shoulders
and knees and wrists and cannot raise her arms beyond 90°. She has noted a gradual deterioration in
her arthralgias over a period of 12 months. A radiograph of her wrist is shown below. Which of the
following is most likely to explain her symptoms?

A. Pseudogout.
B. Proximal myopathy.
C. Dialysis related amyloidosis.
D. Gout.
E. Osteoarthritis.
376

Question 27
A 67-year-old diabetic female is referred for the first time. She has significant renal impairment with
the following results:

Serum creatinine 180 µmol/L [<110 µmol/L]


Estimated glomerular filtration rate (eGFR) 34 ml/min [90 – 150 ml/min].
Haemoglobin (Hb) 105 gm/L [115 - 135 gm/L]
Proteinuria 0.38 gm/day [<0.12 gm/day]
Blood pressure 150/90 mmHg
HbA1c 7.5%

Which of the following interventions is likely to have the greatest impact on slowing the progression
of her renal impairment?

A. Blood pressure reduction.


B. Tight glycaemic control.
C. Erythropoietin therapy.
D. Control of serum phosphate.
E. Low protein diet.

Question 28
A 62-year-old man with type 2 diabetes and hypertension is on long term aspirin, quinapril,
metoprolol and insulin. He presents with two hours of central chest pain. A diagnosis of non-ST
elevation myocardial infarction is made. He proceeds to have coronary angiography which
demonstrates diffuse three-vessel coronary artery disease. He is discharged home with the
introduction of a long acting nitrate to await elective coronary artery surgery. He represents two
weeks later with malaise and abdominal pain. His serum creatinine has increased to 0.45mmol/L
from a serum creatinine of 0.14mmol/L on his previous admission.

Which of the following is the most likely cause of the deterioration in renal function?

A. Aortic dissection.
B. Renovascular disease.
C. Angiotensin converting enzyme inhibitor nephropathy.
D. Cholesterol embolisation.
E. Contrast nephropathy.

Question 29
When considering the long-term outcome of renal transplantation, other than rejection, which of the
following has the greatest adverse effect on graft survival?

A. Diabetes in the recipient.


B. Donor age over 65 years.
C. Recipient age over 65 years.
D. Polycystic kidney disease in the recipient.
E. Post-transplant hypertension.
377

Question 30
A 26-year-old man is referred for assessment of proteinuria. He has a six-month history of recurrent
episodes of lower extremity petechial rash (shown below), arthralgia, Raynaud’s phenomenon and
fever. He has an eight-year history of intravenous drug use. Physical examination reveals
erythematous raised papules on both lower extremities and dependent oedema.

Investigations reveal:
Serum creatinine 110 umol/L [< 120]
Albumin 28 gm/L [35-50]
C3 1.05 gm/L [0.80-1.50]
C4 <0.01 gm/L [0.16 – 0.55]
Antinuclear antibody negative
Antineutrophil cytoplasmic antibody (ANCA) negative

Midstream urine reveals 50 x 106 glomerular red blood cells per litre and proteinuria is quantitated at
5.5 gm/day [< 0.12].

A renal biopsy is performed. What is the most likely histological diagnosis on biopsy?

A. Membranous glomerulonephritis.
B. Idiopathic mesangiocapillary glomerulonephritis.
C. Cryoglobulinaemic glomerulonephritis.
D. Chronic post-infectious glomerulonephritis.
E. Henoch-Schonlein Purpura with IgA (immunoglobulin A) nephropathy.
378

Question 31
A 23-year-old male has had his third episode of passing a renal calculus. Each episode has been
preceded by renal colic, twice on the left and once on the right. A calculus has been analysed in the
biochemistry laboratory and found to contain predominantly calcium oxalate. Typical calculi are
shown in the image below.

Which of the following is most likely to reduce his chance of further stones?

A. Institution of a low calcium diet.


B. Avoiding oxalate rich foods (such as asparagus and rhubarb).
C. Drinking 3-4 litres of fluid per day.
D. Administration of oral citrate.
E. Use of a loop diuretic such as frusemide.

Question 32
A 23-year-old man has end-stage renal failure secondary to immunogobulin A (IgA) nephropathy.
He is managed with three times per week haemodialysis. When reviewed in clinic you discover he
has not been taking any of his oral medications and has the following serum results:

Calcium (corrected): 1.96 mmol/l [2.20 – 2.55]


Phosphate 2.2 mmol/l [0.8 – 1.5]
Parathyroid hormone 45 pmol/l [1.0 – 7.0]

The most appropriate initial treatment of these abnormalities is:

A. A low phosphate diet.


B. Low phosphate dialysis.
C. Oral calcitriol.
D. Oral phosphate binders.
E. High calcium dialysis.
379

Question 33
Apart from end-stage renal failure, which of the following occurs most frequently in patients with
adult polycystic kidney disease?

A. Mitral valve prolapse.


B. Renal infection.
C. Ischaemic heart disease.
D. Pancreatic cysts.
E. Cerebral aneurysm rupture.

Question 34
A 28-year-old female has developed end-stage renal failure secondary to reflux nephropathy.
Which of the following options is most likely to provide best long term survival for the patient?

A. Nocturnal home haemodialysis.


B. Peritoneal dialysis.
C. Living related transplant from her healthy 62-year-old father.
D. Cadaveric transplant from a 30-year-old donor.
E. Living unrelated transplant from her 30-year-old husband.

Question 35
A week after a flu like illness, a 22-year-old presents with a haemorrhagic rash on the lower
extremities (as shown in the figure below), and generalised arthralgia. The serum creatinine is
elevated (125 µmol/L, [<110 µmol/L]) and there is blood and protein in the urine.

The most likely finding on renal biopsy is:

A. Glomerular microangiopathy (haemolytic-uraemic syndrome).


B. Mesangiocapillary glomerulonephritis.
C. Henoch-Schönlein nephropathy.
D. Antineutrophil cytoplasmic antibody (ANCA) positive vasculitis.
E. Lupus nephropathy.
380

Question 36
Dialysis patients have a high mortality, with average survival in the order of 40-50% at five years.
For a new patient commencing dialysis, the most likely cause of death will be:

A. Infection.
B. Malignancy.
C. Electrolyte abnormality.
D. Cardiovascular event.
E. Withdrawal from dialysis

Question 37
A 64-year-old man presents with renal impairment (serum creatinine 300 umol/l), haematuria and
proteinuria. He is anti-neutrophil cytoplasmic antibody (ANCA) positive and a renal biopsy reveals a
focal necrotising glomerulonephritis consistent with vasculitis. He is commenced on prednisolone
and cyclophosphamide orally. Six weeks later he represents with macroscopic haematuria. He is
afebrile and his creatinine is 225 umol/l.

The most appropriate next management step is:

A. IV antibiotics.
B. Cystoscopy.
C. IV methylprednisolone.
D. Repeat renal biopsy.
E. Change cyclophosphamide to mycophenolate.

Question 38
Which of the following is most likely to increase serum phosphate in chronic renal failure?

A. Calcium trisilicate.
B. Calcitriol.
C. Cinacalcet.
D. Calcitonin.
E. Alendronate.
381

Question 39
A 67-year old diabetic woman has end-stage renal failure managed by continuous ambulatory
peritoneal dialysis. She presents with abdominal pain, low grade fever and cloudy dialysate. On
examination she has mild generalised abdominal tenderness and guarding but does not appear
particularly unwell. She is admitted to the ward and started on intra-peritoneal cephalexin. The
subsequent day, her dialysate culture grows enterococci, E coli and Klebsiella species.

The most appropriate management step is:

A. Exploratory laparotomy.
B. Add intraperitoneal Ampicillin.
C. Removal of the Tenckhoff catheter.
D. Intraperitoneal gentamicin.
E. Change to intravenous antibiotics.

Question 40
A 26-year-old male is involved in a motor vehicle accident, following which he undergoes extensive
surgery including a laparotomy for suspected ruptured bowel and decompression surgery on his legs
for compartment syndrome. He receives multiple antibiotics including gentamicin and ampicillin,
and parenteral feeding. Ten days later, after discharge to the general ward, he develops mild oliguria
but a rapidly rising serum creatinine. Obstruction is excluded on a renal ultrasound examination.

The most likely cause of his acute renal failure is:


A. Penicillin related interstitial nephritis.
B. Rhabdomyolysis.
C. Hypovolaemia related to his injuries.
D. Aminoglycoside-related acute tubular necrosis.
E. Intratubular calcium deposition.

Question 41
A 45-year-old male receives a kidney transplant from his brother. Ten days after the operation, the
transplant function appears to worsen, with serum creatinine rising from 140 µmol/L to 190 µmol/L
[60 - 120 µmol/L].

What is the most likely cause of the rise in creatinine?.

A. Ureteric obstruction.
B. Calcineurin inhibitor toxicity.
C. Polyoma (BK) virus nephropathy.
D. Acute rejection.
E. Acute tubular necrosis (ATN).
382

Question 42
Cardiovascular disease is the major cause of death in dialysis patients. Elevation of which of the
following clinical parameters carries the highest mortality risk?.

A. Serum phosphate.
B. Serum cholesterol.
C. Serum homocysteine.
D. Serum parathyroid hormone.
E. Blood pressure.
383

Answers

1. C
2. E
3. A
4. A
5. E
6. D
7. C
8. D
9 E
10. D
11. A
12. E
13. A
14. A
15. D
16. E
17. B
18. D
19. A
20. B
21. C
22. C
23. C
24. A
25. D
26. C
27. A
28. D
29. A
30. C
31. C
32. D
33. C
34. C
35. C
36. D
37. B
38. B
39. A
40. D
41. D
42. A
384

Medical Sciences

Respiratory

Question 1
In a patient with cryptogenic fibrosing alveolitis (idiopathic pulmonary fibrosis), the finding on
bronchoalveolar lavage of an elevated proportion of which one of the following cell types is
associated with a better response to therapy?

A. Eosinophils.
B. Lymphocytes.
C. Macrophages.
D. Mast cells.
E. Neutrophils.

Question 2
A 67-year-old woman with severe chronic obstructive pulmonary disease undergoes an all-night
respiratory sleep study. The study reveals no evidence of obstructive sleep apnoea but there are three
periods of repeated falls in oxyhaemoglobin saturation (SaO2).

Which one of the following is the most likely explanation for the falls in SaO2?

A. SWS (slow wave sleep)-related upper airway narrowing.


B. REMS (rapid eye movement sleep)-related increase in nasal resistance.
C. REMS-related increase in ventilation-perfusion mismatching.
D. SWS-related reduction in cardiac output.
E. REMS-related reduction in respiratory drive.

Question 3
Which one of the following mechanisms best explains the mode of action of nasal continuous
positive airway pressure treatment in obstructive sleep apnoea?

A. Augmentation of genioglossal muscle tone.


B. An increase in functional residual capacity.
C. Reduction in oedema of palatal tissues.
D. Pneumatic splinting of the upper airway.
E. Prevention of epiglottic closure during inspiration.
385

Question 4
Which feature of glomerular pathology is least consistent with the diagnosis of Wegener's
granulomatosis?

A. Focal necrosis.
B. Crescents.
C. IgG deposition.
D. T cell infiltrate.
E. Macrophage infiltrate.

Question 5
The alveolar-arterial gradient for oxygen tension in arterial blood (PA-aO2) can be up to 15 mmHg
in healthy young adults and as high as 30 mmHg in elderly people.

Which one of the following is considered to be the predominant physiological mechanism for a
normal PA-aO2?

A. Variation in distribution of ventilation.


B. Variation in diffusion for oxygen.
C. Right to left shunt.
D. Ventilation/perfusion (V/Q) inequality.
E. Variation in distribution of pulmonary blood flow.

Question 6
In a patient with cryptogenic fibrosing alveolitis (idiopathic pulmonary fibrosis), the finding on
bronchoalveolar lavage of an elevated proportion of which one of the following cell types is
associated with a better response to therapy?

A. Eosinophils.
B. Lymphocytes.
C. Macrophages.
D. Mast cells.
E. Neutrophils.
386

Question 7
In a patient with a severe exacerbation of asthma, the use of a peak-flow meter may underestimate
the severity of airflow limitation compared to the use of spirometry and measurement of the forced
expiratory volume in 1 second (FEV1).

Which one of the following is mainly responsible for this reduction in the accuracy of the peak-flow
meter?

A. The hyperinflation of severe asthma.


B. The effort-independent component of the forced expiratory manoeuvre.
C. The effort-dependent component of the forced expiratory manoeuvre.
D. Narrowing of small airways.
E. Narrowing of large airways.

Question 8
A 67-year-old woman with severe chronic obstructive pulmonary disease undergoes an all-night
respiratory sleep study. The study reveals no evidence of obstructive sleep apnoea but there are three
periods of repeated falls in oxyhaemoglobin saturation (SaO2).

Which one of the following is the most likely explanation for the falls in SaO2?

A. SWS (slow wave sleep)-related upper airway narrowing.


B. REMS (rapid eye movement sleep)-related increase in nasal resistance.
C. REMS-related increase in ventilation-perfusion mismatching.
D. SWS-related reduction in cardiac output.
E. REMS-related reduction in respiratory drive.
387

Question 9
A 40-year-old woman presents with progressive breathlessness on exertion. She is a non-smoker.
Physical examination and chest X-ray reveal no significant abnormality. Pulmonary function tests
show the following results:

Measurement Result Predicted % Predicted


Forced expiratory volume in 1 second
2.8 L 2.9 L 98%
(FEV1)
Forced vital capacity (FVC) 3.2 L 3.3 L 96%
FEV1/FVC ratio 88% 86%
Total lung capacity (TLC) 5.4 L 5.2 L 103%
Residual volume (RV) 1.72 L 1.7 L 101%
Gas diffusion (DLCO) 14.6 mL/min/mmHg 26 mL/min/mmHg 56%
Forced mid-expiratory flow (FEF25-75) 3.2 L/sec 3.5 L/sec 91%

Which one of the following is the most likely explanation for this woman's pulmonary function test
results?

A. Pulmonary vascular disease.


B. Pulmonary fibrosis.
C. Small airway disease.
D. Inadequate effort during the tests.
E. Hyperventilation syndrome

Question 10
A previously well 25-year-old man has three episodes of acute pancreatitis. Genetic testing reveals
compound heterozygosity at the cystic fibrosis gene.

Which other manifestation of cystic fibrosis is he most likely to have or develop in the future?

A. Hyponatraemia.
B. Bronchiectasis.
C. Nasal polyps.
D. Infertility.
E. Cirrhosis.
388

Question 11
A flow-volume loop is shown below.

Which of the following is the most likely diagnosis?

A. Chronic obstructive pulmonary disease (COPD).


B. Vocal cord paralysis.
C. Tracheal tumour.
D. Pulmonary fibrosis.
E. Supraglottic stenosis.

Question 12
Which of the following drugs is most likely to cause urinary retention, when administered to a 74-
year-old man?

A. Inhaled ipratropium.
B. Sertraline.
C. Glibenclamide.
D. Propranolol.
E. Amitriptyline.
389

Question 13
A 55-year-old man has the following lung function test results:

forced expiratory volume in 1 second (FEV1) 2.0 L (61% predicted)


forced vital capacity (FVC) 2.6 L (68% predicted)
FEV1/FVC 77% (85% predicted)
total lung capacity (TLC) 72% predicted
residual volume (RV) 75% predicted
diffusing capacity for carbon monoxide (DLCO) 64% predicted
DLCO corrected for alveolar volume (KCO) 110% predicted

His body mass index (BMI) is 31 kg/m2 [18-25].

Which of the following is the best explanation for these results?

A. Previous lobectomy.
B. Obesity.
C. Pulmonary fibrosis.
D. Asthma.
E. Pulmonary haemorrhage.

Question 14
A 66-year-old man with a 40 pack-year smoking history has the following spirometry:

forced expiratory volume in 1 second (FEV1) 0.65 L (24% predicted)


forced vital capacity (FVC) 2.10 L (60% predicted)
FEV1/FVC 31%

His body mass index (BMI) is 30 kg/m2 [18-25].

Which of the following is the most likely explanation for the observed reduction in FVC?

A. Extrapulmonary restriction due to obesity.


B. Extrapulmonary restriction due to altered operating length of the diaphragm.
C. Intrapulmonary restriction due to reduced lung compliance.
D. Gas trapping due to increased airway closure.
E. Gas trapping due to reduced expiratory time.
390

Question 15
Inhaled beta-agonists reduce breathlessness and increase exercise tolerance in patients with chronic
obstructive pulmonary disease (COPD).

Which of the following is the most likely mechanism for these effects?

A. Increased forced expiratory volume in 1 second (FEV1).


B. Improved ventilation-perfusion (VA/Q) matching.
C. Reduced dynamic hyperinflation.
D. Increased cardiac output
E. Decreased pulmonary arterial pressure.

Question 16
A 58-year-old male is referred with breathlessness on exercise. He has smoked 20 cigarettes/day
since the age of 13. He completed his electrician's apprenticeship at a naval dockyard, and worked in
the construction industry until sustaining a back injury and retiring at the age of 50 years. A chest X-
ray is reported as abnormal and a thoracic computed tomography (CT) scan (shown below) is
performed.

The CT scan appearances are most characteristic of which one of the following conditions?

A. Emphysema.
B. Asbestos-related pleural disease.
C. Pulmonary fibrosis.
D. Silicosis.
E. Asbestosis.
391

Question 17
A 47-year-old male presents with haemoptysis on a background of a 47 pack year smoking history.
A chest X-ray shows an irregular mass lesion in the right lower lobe, and sputum examination
reveals large undifferentiated malignant cells.

Which of the following is the most sensitive method of determining if the mediastinal lymph nodes
are involved with the disease?

A. Gallium scan.
B. Standard thoracic computed tomographic (CT) scanning.
C. High resolution computed tomographic (CT) scanning.
D. Magnetic resonance imaging (MRI).
E. Positron emission tomography (PET).
392

Question 18
A 57-year-old male presents with progressive breathlessness on exercise over five years. His lung
function tests are shown below.

Key:
PRE-BRONCH Pre-bronchodilator
POST-BROCH Post-bronchodilator
FVC Forced vital capacity
FEV Forced expiratory volume in one second
1
SVC Slow vital capacity
IC Inspiratory capacity
FRC Functional residual capacity
RV Residual volume
TLC Total lung capacity
(N2) measured by nitrogen washout
(Pleth) measured by body plethysmography
DLCO(cor) diffusing capacity for carbon monoxide

What is the most likely diagnosis from the lung function tests?

A. Chronic bronchitis.
B. Pulmonary fibrosis.
C. Emphysema.
D. Bronchiectasis.
E. Asthma.
393

Question 19
Bosentan is one of a number of new pharmacological therapies for pulmonary hypertension. Which
of the following best describes the predominant mechanism of action of bosentan?

A. Thromboxane-receptor antagonism.
B. Inhibition of platelet aggregation.
C. Phosphodiesterase inhibition.
D. Endothelin-receptor antagonism.
E. Reversal of hypoxic vasoconstriction.

Question 20
A 56-year-old male presents with a three year history of progressive breathlessness on exercise with
a dry unproductive cough. He works in a bank and has a ten pack year smoking history. His lung
function is shown below.

Key:
BMI Body mass index
FVC Forced vital capacity
FEV1 Forced expiratory volume in one second
FEF25-75 Forced expiratory flow rates over 25-75% of FVC
SVC Slow vital capacity
IC Inspiratory capacity
ERV Expiratory reserve volume
RV Residual volume
TLC Total lung capacity
(Pleth) measured by body plethysmography
DLCO(cor) diffusing capacity for carbon monoxide corrected for Hb
VA alveolar volume

What is the most likely diagnosis?

A. Chronic bronchitis.
B. Interstitial fibrosis.
C. Emphysema.
D. Kyphoscoliosis.
E. Pulmonary embolism.
394

Question 21
A 48-year-old male presents with a six-month history of daytime somnolence. He worked as a
shipbuilder and had previous asbestos exposure from replacing boiler lagging. He has a 15 pack year
smoking history. His body mass index (BMI) is 39 kg/m2 [18-26], but otherwise examination is
normal. His lung function tests and arterial blood gas are shown below.

Key:
PRE-BRONCH Pre-bronchodilator
POST-BROCH Post-bronchodilator
FVC Forced vital capacity
FEV1 Forced expiratory volume in one second
SVC Slow vital capacity
IC Inspiratory capacity
FRC Functional residual capacity
RV Residual volume
TLC Total lung capacity
(Pleth) measured by body plethysmography
DLCO(cor) diffusing capacity for carbon monoxide

The most likely diagnosis is:

A. Idiopathic pulmonary fibrosis.


B. Obesity-hypoventilation syndrome.
C. Asbestosis.
D. Chronic obstructive pulmonary disease.
E. Post-polio syndrome.
395

Question 22
In idiopathic pulmonary arterial hypertension, an excess of which of the following mediators is most
likely to be involved in the pathogenesis?

A. Nitric Oxide.
B. Vasointestinal Peptide.
C. Serotonin.
D. Endothelin 1.
E. Prostaglandin I2.

Question 23
Airline travel is undertaken by many patients with chronic cardio-respiratory disease.

When travelling with standard international commercial airlines currently, the partial pressure of
oxygen inflight approximates which sea-level FIO2 [Fraction of inspired oxygen]?

A. 5%.
B. 10%.
C. 15%.
D. 21%.
E. 25%.

Question 24
The most common cause of treatment failure in pulmonary tuberculosis is:

A. Co-existing chronic airways limitation.


B. Presence of cavitatory disease.
C. Poor adherence to therapy.
D. HIV (human immunodeficiency virus) co-infection.
E. Isoniazid resistance.

Question 25
High-altitude pulmonary oedema is a potentially fatal condition that can occur in 1-2% of individuals
travelling to altitudes above 3000m.

Which of the following physiological responses to altitude is the most likely cause of high-altitude
pulmonary oedema?

A. Profound respiratory alkalosis (pH > 7.6).


B. Cheyne-Stokes respiration during sleep.
C. Hypoxic pulmonary vasoconstriction.
D. Hypoxia-induced left ventricular dysfunction.
E. Hypoxia-induced systemic hypertension.
396

Question 26
A 45-year-old female with a ten-pack year history of cigarette smoking and a body mass index of 39
kg/m2, presents with a 12-month history of shortness of breath on exercise. An exercise test was
performed, with no evidence of myocardial ischaemia.

What is the most likely cause of the breathlessness and oxygen desaturation on exercise?

A. Chronic obstructive pulmonary disease.


B. Pulmonary vascular disease.
C. Morbid obesity.
D. Chronic asthma.
E. Beta-blockade.

Question 27
The principle mechanism of action by which inhaled nitric oxide can improve oxygenation in severe
hypoxemic respiratory failure is:

A. Vasoconstriction of pulmonary vessels in non-ventilated lung.


B. Decreased pulmonary vascular resistance.
C. Vasodilatation of pulmonary vessels in ventilated lung.
D. Bronchodilatation.
E. Increase in cardiac output.
397

Question 28
A 33-year-old male presents to hospital with recurrent episodes of breathlessness and wheeze.
Flowvolume loops are shown during (A) and between (B) episodes.

The most likely cause of the episodes are:

A. Hyperventilation syndrome.
B. Paroxysmal vocal cord dysfunction.
C. Acute asthma.
D. Tracheobronchomalacia.
E. Respiratory bronchiolitis.
398

Question 29
Arterial blood gases are shown below for an individual breathing room air and then 100% oxygen:

Actual Predicted Actual


Blood Gases
Inspired O2 (%) 21.0 100.0
pH 7.42 7.40 7.42
PaCO2 (mmHg) 40.0 38.0 - 42.0 41.0
PaO2 (mmHg) *50.0 100.0 *63.0
SaO2 Measured (%) 85.5 91.2
Base Excess 1.6 2.1
HCO3 (mEq/L) 25.9 23.0 - 27.0 26.6
Hb (gm/dL) 20.3 12 - 18 20.1
COHb (%) 1.1 <1.5% 1.3

Legend
Hb (Haemoglobin)
COHb (Carboxyhaemoglobin)

The best explanation for these results is:

A. Interstitial pulmonary fibrosis.


B. Chronic obstructive pulmonary disease.
C. Pulmonary arteriovenous malformation.
D. Pulmonary embolism.
E. Methaemoglobinaemia.

Question 30
In the normal lung, the greatest contribution to increased airway resistance is:

A. Dynamic compression of the airways.


B. The peripheral small airways.
C. Increased total lung volume.
D. Bronchial smooth muscle constriction.
E. The medium-sized bronchi.
399

Question 31
In a healthy adult, the predominant limitation to exercise capacity is:

A. Lactic acid production.


B. Alveolar ventilation.
C. Cardiac output.
D. Ventilation-perfusion inequality.
E. Oxygen diffusion.

Question 32
In the patient with acute respiratory distress syndrome (ARDS), which of the following conditions is
the greatest contra-indication to a low tidal volume, low airway pressure ventilation strategy?

A. Ischaemic heart disease.


B. Flail chest.
C. Pulmonary embolism.
D. Closed head injury.
E. Asthma.

Question 33
A 19-year-old male complains of intermittent breathlessness and wheezing. Which of the following
lung function abnormalities is most specific for a diagnosis of asthma?

A. A ratio of the forced expiratory volume in one second to forced vital capacity (FEV1/FVC) of
less than 0.7.
B. An increase in the forced expiratory volume in one second (FEV1) of 15% following inhalation
of 400 mcg salbutamol.
C. A ratio of residual volume to total lung capacity (RV/TLC) greater than 120% of predicted.
D. A concentration of inhaled methacholine causing a 25% fall in the forced expiratory volume in
one second (FEV1) of 1 mg/ml.
E. An increase in the carbon monoxide diffusing capacity to 125% of predicted.
400

Answers

1. B
2. E
3. D
4. C
5. D
6. B
7. C
8. E
9. A
10. D
11. C
12. E
13. A
14. D
15. C
16. B
17. E
18. C
19. D
20. B
21. B
22. D
23. C
24. C
25. C
26. B
27. C
28. B
29. C
30. E
31. C
32. D
33. D
401

Clinical Applications

Respiratory

Question 1
A 45-year-old woman has a long history of recurrent breathlessness, cough and sputum plugs. Her
chest X-ray is shown below. Sputum cultures are negative. Serum IgE is 1350 U/L [0-250].

Which one of the following is the most likely diagnosis?

A. Legionnaire’s disease.
B. Eosinophilic pneumonia.
C. Allergic bronchopulmonary fungal disease.
D. Recurrent aspiration pneumonia.
E. Cystic fibrosis.
402

Question 2
A 34-year-old man presents with fever, malaise, a red indurated rash on his lower leg and marked
painful swelling of both ankles. His chest X-ray is shown below.

The most likely diagnosis is:

A. Sarcoidosi.
B. Tuberculosis.
C. Lymphoma.
D. Reiter's syndrome.
E. Systemic lupus erythematosus.
403

Question 3
A 62-year-old man is admitted to the intensive care unit with community-acquired pneumonia and
severe respiratory failure for which he is intubated and given mechanical ventilation.

He becomes progressively hypoxaemic, hypotensive and oliguric.


Assessment at this stage shows:
arterial blood pressure 93/55 mmHg
arterial blood gases on 60% inspired oxygen
PaO2 53 mmHg
PaCO2 34 mmHg
PH 7.31
oxygen saturation 83%
haemoglobin concentration 105 g/L [125-180]
pulmonary capillary wedge pressure 5 mmHg [3-12]
cardiac output 3.0 L/min [predicted cardiac output 4.5]

The most appropriate intervention to achieve an increase in oxygen delivery to his tissues is:

A. The addition of positive end-expiratory pressure (PEEP).


B. An increase in the inspired O2 (FiO2).
C. A volume infusion.
D. A transfusion of packed red cells.
E. An increase in minute ventilation.

Question 4
A 66-year-old man with a 48-pack-year history of smoking presents for assessment of disabling
dyspnoea and reduced exercise tolerance. Physical examination reveals poor chest expansion with
increased percussion note and vesicular but reduced breath sounds. There is no evidence of cardiac
failure. His pulmonary function tests show a forced expiratory volume in 1 second (FEV1) of 0.90 L
(predicted 2.95 L) and a forced vital capacity (FVC) of 2.20 L (predicted 3.65 L). After
bronchodilator, FEV1 increased to 1.1 L and FVC to 3.0 L. Residual volume was 185% predicted,
carbon monoxide diffusing capacity was 50% predicted and his resting SaO2 was 95% on room air.

Which one of the following is least likely to improve after completion of a pulmonary rehabilitation
program?

A. Exercise tolerance.
B. Exertional dyspnoea.
C. Lung function.
D. Overall quality of life.
E. Emotional function.
404

Question 5
A 45-year-old man presents with acute respiratory distress and is noted to have oral candidiasis.
Chest X-ray shows bilateral alveolar infiltrates. Arterial PaO2 on air is 55 mmHg.

A silver-stained preparation of his sputum is shown below.

Which one of the following is the most appropriate initial therapy?

A. Intravenous itraconazole.
B. Intravenous itraconazole and oral prednis(ol)one.
C. Intravenous amphotericin.
D. Intravenous cotrimoxazole.
E. Intravenous cotrimoxazole and oral prednis(ol)one.

Question 6
A 43-year-old woman presents with increasing breathlessness and wheeze. Inspiratory and expiratory
flow-volume curves show normal expiration but flow limitation during inspiration producing an
"inspiratory plateau".

The flow-volume curves are most likely to be caused by:

A. A fixed extra-thoracic obstruction.


B. A variable extra-thoracic obstruction.
C. A fixed intra-thoracic obstruction.
E. Widespread intra-pulmonary obstruction.
405

Question 7
A 58-year-old woman presents one week after a flight from London. She gives a two-day history of
worsening breathlessness which was first apparent while dusting the house. No abnormalities are
found on physical examination.

Spirometry results and chest X-ray are shown below. Her ECG is normal.

Arterial blood gas results on breathing room air are:


pH 7.46 [7.35-7.45]
bicarbonate 21 mmol/L [24-30]
PaO2 80 mmHg [80-100]
PaCO2 28 mmHg [36-44]

Which one of the following is the most appropriate next investigation?

A. Ventilation/perfusion lung scan.


B. Echocardiogram.
C. Pulmonary function tests.
D. Non-contrast high resolution computed tomography (CT) scan of thorax.
E. Histamine challenge test.
406

Question 8
A 17-year-old farm worker presents with a three-week history of cough, yellow sputum,
breathlessness and right pleuritic chest pain. His medical history is unremarkable and he has no
allergies. On examination he is not cyanosed and he manifests the following vital signs.

Temperature 37.7°C
Pulse rate 96/minute
Blood pressure 130/80 mmHg
Respiratory rate 22/minute
Blood tests reveal:
haemoglobin 117 g/L [130-180]
white cell count 18.5 x 109/L [4.0-11.0]
differential:
Neutrophils 16.4 x 109/L [1.8-7.7]
Electrolytes normal
Renal function normal
Liver function normal

His chest X-ray is shown below.

What is the most appropriate initial treatment?

A. Oral albendazole.
B. Intravenous penicillin and metronidazole.
C. Intercostal catheter.
D. Intravenous ticarcillin/clavulanic acid, gentamicin and erythromycin.
E. Oral rifampicin, isoniazid, ethambutol and pyrazinamide.
407

Question 9
A 50-year-old man with chronic asthma and a history of fleeting lung infiltrates presents with
worsening symptoms of asthma, low grade fever, purpuric lesions on both legs and a left foot drop.

Total white cell count is 18.0 x 109/L [4.5-11.0] of which 3.60 x 109/L [0.05-0.25] are eosinophils.
Serum IgE is 1250 U/L [0-250].

There was a poor response to oral prednis(ol)one. However, his condition improved considerably
when cyclophosphamide was added.

Which one of the following is most likely to be the diagnosis in this patient?

A. Allergic bronchopulmonary aspergillosis.


B. Chronic eosinophilic pneumonia.
C. Löffler's syndrome.
D. Extrinsic allergic alveolitis.
E. Churg-Strauss syndrome.
408

Question 10
A 55-year-old man presents to the emergency department with fever and haemoptysis. His chest X-
ray is shown below. No significant bacteria are seen on Gram stain of a sputum specimen. Treatment
is commenced with intravenous antibiotics. Blood and sputum have been sent for standard culture.

Which one of the following is the most appropriate next investigation?

A. Legionella urinary antigen test.


B. Bronchoscopy and transbronchial lung biopsy.
C. Ziehl-Nielsen stain of sputum specimen.
D. Silver stain of induced sputum specimen.
E. Computed tomography (CT) scan of the chest.
409

Question 11
A 49-year-old firefighter is brought to the emergency department after passing out while working at
an extensive factory fire. He has a history of type 2 (non-insulin-dependent) diabetes mellitus. Apart
from some hyperventilation, there are no abnormalities on physical examination. Pulse oximetry on
room air reveals an oxygen saturation of 99%. Arterial blood gases on room air are as follows:

PaO2 97 mmHg [85-100]


PaCO2 29 mmHg [36-44]
pH 7.28 [7.36-7.44]
bicarbonate 3 mmol/L [24-30]

Which one of the following is the most likely explanation for this patient's condition?

A. Non-cardiogenic pulmonary oedema.


B. Carbon monoxide poisoning.
C. Ketoacidosis.
D. Acute renal failure.
E. Anxiety-related hyperventilation.

Question 12
A 35-year-old man presents to the emergency department with haemoptysis and dyspnoea. He has
been unwell for some time with fatigue and general weakness. Physical examination is normal other
than marked pallor and inspiratory crackles in both lower zones.

Initial investigations show:


Haemoglobin 71 g/L 130-180]
platelet count normal
creatinine 0.25 mmol/L [0.06-0.12]
urinalysis microscopic haematuria with red cell casts
chest X-ray normal cardiac size and bilateral alveolar filling defects
antineutrophil cytoplasmic antibody (ANCA) negative
antiglomerular basement membrane antibody (anti-GBM) negative

In addition to the administration of oxygen and blood transfusion, which one of the following
investigations would be most appropriate at this point?

A. Thoracoscopic lung biopsy.


B. Sinus biopsy.
C. Bronchoscopy.
D. Percutaneous renal biopsy.
E. Serum assay for antinuclear antibodies (ANA) and DNA antibodies.
410

Question 13
A 72-year-old man presents to hospital having woken from sleep with paroxysmal nocturnal
dyspnoea. This is his second presentation to hospital with this problem. He is known to have
ischaemic heart disease and an echocardiogram performed during his last admission revealed a left
ventricular ejection fraction of 25%. The respiratory parameters from a polysomnographic sleep
study are shown below.

Nasal P: affected by artefact so disregard


Therm: Nasal airflow
Thor: Thoracic respiratory inductance plethysmography bands
Abdo: Abdominal inductance plethymography bands
Sum: summed signal from Thor and Abdo
Leg: leg movement sensor
Db: decibels

The most likely explanation for the change is:

A. Cheyne-Stokes respiration (periodic breathing).


B. Obstructive sleep apnoea.
C. Periodic leg movement disorder.
D. Upper airway resistance syndrome.
E. Obesity hypoventilation syndrome.
411

Question 14
A 61-year-old woman is reviewed for ongoing management of chronic obstructive pulmonary
disease (COPD). She has smoked 30 cigarettes/day for 40 years. She stopped smoking three years
ago. She is breathless on moderate exertion.

Examination reveals a pulse oximetry of 94% on room air, hyperinflation of the chest and soft
normal breath sounds. There is no evidence of right heart failure. Her lung function is consistent with
COPD. Her forced expiratory volume in 1 second (FEV1) is 51% of predicted with a 6% increase
following bronchodilator. Her full blood count is normal. Her medications include regular inhaled
ipratropium, salbutamol as required and a regular inhaled corticosteroid (budesonide 800 mcg bd).

Which one of the following is the most likely effect of regular inhaled corticosteroids in this woman?

A. Reduced deterioration in FEV1.


B. Reduced exacerbations of COPD.
C. Induced diabetes mellitus.
D. Increased risk of cataracts.
E. Reduced breathlessness on exertion.
412

Question 15
A 50-year-old man presents with fever and pulmonary opacities. His chest X-ray is shown below. He
recently had grommets inserted for chronic middle ear disease.

The investigation most likely to aid diagnosis is:

A. Mantoux test.
B. Fine needle aspiration of a lung lesion.
C. Antineutrophil cytoplasmic antibody (ANCA).
D. Computed tomography (CT) scan of the chest.
E. Test for IgG antibodies to Aspergillus antigens.

Question 16
A 35-year-old woman presents for review after a presentation to her local emergency department
with a severe episode of breathlessness and wheeze. She had taken two aspirin tablets about one hour
before the episode. She has a history of chronic sinusitis and nasal polyps.

She improved with treatment of salbutamol and oral corticosteroids followed by inhaled fluticasone.
At the time of follow-up she remains symptomatic despite increasing doses of inhaled
corticosteroids.

Which one of the following agents has the greatest potential to improve this patient's asthma control?

A. Montelukast.
B. Ipratropium.
C. Sodium cromoglycate.
D. Salmeterol.
E. Nedocromil.
413

Question 17
A 65-year-old man presents to outpatients complaining of breathlessness on exercise, which has been
progressive over the last four years. He has a 40 pack-year smoking history and has had daily cough
with clear sputum production over the past 10 years.

Lung function testing shows:

forced expiratory volume in 1 second (FEV1) 0.70 L (30% predicted)


forced vital capacity (FVC) 3.77 L (90% predicted)
PaO2 61 mmHg [75-90]

Which of the following interventions is most likely to improve his survival?

A. Inhaled anticholinergics.
B. Long-term oxygen therapy.
C. Inhaled corticosteroids.
D. Smoking cessation.
E. Lung volume reduction surgery.

Question 18
A 55-year-old woman develops pleuritic chest pain three days after laparoscopic cholecystectomy. A
chest X-ray is normal. A ventilation-perfusion (VA/Q) lung scan is reported as ‘low-probability’ and
lower limb venous ultrasonography is negative.

Which of the following is the most appropriate next step?

A. No further testing.
B. Computed tomography (CT) pulmonary angiogram.
C. Arterial blood gas analysis.
D. Repeat VA/Q lung scan.
E. D-dimer assay.

Question 19
A 45-year-old man presents with increasing breathlessness on exertion and poor sleep quality.
Physical examination shows paradoxical breathing.

Which of the following is the most appropriate investigation to confirm a diagnosis of bilateral
diaphragm paralysis?

A. Erect and supine vital capacity (VC).


B. Arterial blood gas analysis.
C. Sleep study.
D. Diaphragm fluoroscopy (sniff test).
E. Maximum inspiratory pressure.
414

Question 20
For which of the following conditions is non-invasive positive pressure ventilation (NPPV) least
likely to be of benefit?

A. Chronic obstructive pulmonary disease (COPD) with acute hypercapnic hypoxic respiratory
failure.
B. Obesity-hypoventilation syndrome with chronic hypercapnic respiratory failure.
C. Pneumonia in a stem cell allograft patient with acute hypoxic respiratory failure.
D. Community-acquired pneumonia with acute hypercapnic hypoxic respiratory failure.
E. Duchenne muscular dystrophy (DMD) with chronic hypercapnic respiratory failure.

Question 21
A 35-year-old woman from the United Kingdom is referred for investigation of an abnormal chest X-
ray (shown below) taken for immigration screening. She is asymptomatic.

Which one of the following is the most likely diagnosis?

A. Sarcoidosis.
B. Tuberculosis.
C. Hodgkin’s lymphoma.
D. Non-Hodgkin’s lymphoma.
E. Small cell lung cancer.
415

Question 22
A 76-year-old woman with a 45 pack-year smoking history presents with a six-month history of
worsening dry cough. High-resolution computed tomography (CT) scans of the thorax are shown
below.

Based on these scans, which of the following is the most likely cause of her cough?

A. Chronic bronchitis.
B. Emphysema.
C. Interstitial pulmonary fibrosis.
D. Bronchiolitis obliterans.
E. Bronchiectasis.
416

Question 23
A 21-year-old overseas student from India presents with malaise, mild shortness of breath and low-
grade fevers over four weeks. Her chest X-ray is shown below.

Which one of the following investigations is most likely to provide the diagnosis?

A. Pleural fluid aspiration.


B. Mantoux testing.
C. Sputum culture.
D. Bronchoscopy.
E. Pleural biopsy.
417

Question 24
A 30-year-old man presents with dry cough and shortness of breath. His chest X-ray is shown below.

The most likely diagnosis is:

A. Pneumonia.
B. Pericardial effusion.
C. Thymoma.
D. Non-small cell lung cancer.
E. Lymphoma.

Question 25
A 60-year-old man is admitted with severe congestive cardiac failure. He has no other significant
medical history, has normal renal function and no risk factors for bleeding.

Which of the following interventions has been shown to be most effective in reducing his chance of a
thromboembolic event during his hospital stay?

A. Compression stockings.
B. Dipyridamole.
C. Enoxaparin.
D. Low-dose warfarin.
E. Aspirin.
418

Question 26
A 67-year-old man with chronic obstructive pulmonary disease (COPD) has severe exertional
dyspnoea and is receiving maximal inhaled bronchodilator therapy. He no longer smokes.

Lung function tests are as follows:


forced expiratory volume in one second (FEV1) 0.54 L (18% predicted)
forced vital capacity (FVC) 2.87 L (75% predicted)
FEV1/FVC 19%
total lung capacity (TLC) 7.96 L (121% predicted)
residual volume (RV) 5.09 L (245% predicted)
diffusing capacity for carbon monoxide (DLCO) 6.1mL/min/mmHg
(19% predicted)
Arterial blood gases on room air are as follows:
PaO2 65 mmHg
PaCO2 52 mmHg
pH 7.36

Which of the following is the most appropriate management to reduce this man’s exertional
dyspnoea?

A. Inhaled corticosteroids.
B. Pulmonary rehabilitation program.
C. Lung volume reduction surgery.
D. Supplemental oxygen.
E. Lung transplantation.

Question 27
A 68-year-old female is referred to outpatients with a four-year history of progressive shortness of
breath on exercise. She smoked from 14 to 60 years of age, averaging 20 cigarettes/day over this
period.

Spirometry shows:
forced expiratory volume in 1 second (FEV) 1.0L (38% predicted)
forced vital capacity (FVC) 2.5L (78% predicted)

Which of the following agents is most likely to improve her quality of life?

A. Ipratropium bromide.
B. Tiotropium bromide.
C. Salmeterol.
D. Salbutamol.
E. Theophylline.
419

Question 28
A 45-year-old male is referred to outpatients complaining of a two-year history of early morning
cough and clear sputum production. He started smoking at the age of 12, smoking on average 25
cigarettes/day since the age of 18. He has controlled epilepsy.

Which of the following interventions is most likely to result in sustained smoking cessation?

A. Brief advice on smoking cessation.


B. Individual counselling.
C. Nicotine replacement therapy.
D. Group therapy.
E. Acupuncture.

Question 29
A 75-year-old woman with known chronic obstructive pulmonary disease (COPD) is admitted to
hospital with a two-day history of worsening breathlessness, cough and sputum production. On
admission her arterial blood gases breathing air show:

pH 7.31 [7.35-7.45]
PaCO2 62 mmHg [35-45]
PaO2 42 mmHg [>75]

Which of the following interventions is most likely to reduce her risk of intubation and ventilation?

A. Antibiotics.
B. Non-invasive continuous positive airway pressure.
C. Oral corticosteroids.
D. Non-invasive positive pressure ventilation.
E. Nebulised bronchodilators.

Question 30
In the long-term management of asthma, which of the following is most effective in improving
disease
outcomes?

A. Providing a written action plan.


B. Peak flow monitoring.
C. Regular medical follow-up.
D. Providing asthma education.
E. Referral to a self-help group.
420

Question 31
A 25-year-old male comes for assessment of fitness to dive. Which of the following is the strongest
contraindication to diving?

A. Mild bronchial hyperresponsiveness.


B. Previous spontaneous pneumothorax.
C. Sarcoidosis.
D. Coronary artery bypass grafting.
E. Upper respiratory tract infection.

Question 32
A 65-year-old male is referred with a three-year history of progressive breathlessness on exercise.
He has smoked 20 cigarettes per day for 45 years.

Spirometry is as follows:
FEV1 (forced expiratory volume in one second) 1.0 L [3.02 L]
FVC (forced vital capacity) 3.0 L [3.97 L]

Arterial blood gases on air are as follows:


pH = 7.40
PaO2 = 61 mm Hg
PaCO2 = 43 mm Hg.

Which of the following interventions is most likely to improve his survival?

A. Smoking cessation.
B. Inhaled corticosteroids.
C. Inhaled long-acting beta-agonists.
D. Long term oxygen therapy.
E. Inhaled long-acting anticholinergics.

Question 33
A 55-year-old male smoker is referred to outpatients complaining of breathlessness. Which of the
following is the most effective intervention for him to achieve smoking abstinence?

A. Cognitive behavioural therapy.


B. Acupuncture.
C. Nicotine replacement therapy.
D. Hypnotherapy.
E. Medical advice to quit.
421

Question 34
Which of the following features of cystic fibrosis is most likely to be present in a 20-year-old male
patient?

A. Cirrhosis.
B. Infertility.
C. Diabetes.
D. Vitamin E deficiency
E. Arthropathy.

Question 35
A 50-year-old man with a 60 pack year history of smoking presents with a two-month history of
cough and weight loss. His performance status is normal (ECOG (Eastern Cooperative Oncology
Group) = 0). Investigations show him to have a T2N1M0 non-small cell carcinoma with a forced
expiratory volume in one second (FEV1) 80% predicted. Which of the following will provide the
highest survival?

A. Surgical resection.
B. Radical radiotherapy.
C. Combined chemoradiation.
D. Downstaging with chemotherapy and surgical resection.
E. Surgical resection and adjuvant chemotherapy.
422

Question 36
A 32-year-old male develops hemoptysis and fever two days after hospital admission with dyspnea.
The computed tomogram chest scans taken on admission, and after development of hemoptysis are
shown below. The most likely diagnosis is which of the following?

A. Pulmonary infarction.
B. Community acquired pneumonia.
C. Left ventricular failure.
D. Parapneumonic effusion.
E. Pulmonary haemorrhage.

Question 37

Non-invasive positive pressure ventilation would be most appropriate for which of the following
patients?

A. Traumatic acute lung injury with hypoxia.


B. Post extubation respiratory failure with acute hypercapnia.
C. Cystic fibrosis with acute hypercapnia.
D. Acute cardiogenic pulmonary oedema with hypoxia.
E. Chronic obstructive pulmonary disease with acute hypercapnia.
423

Question 38
A 64-year-old builder is referred for assessment of a chronic cough. There is no personal or family
history of asthma. He has smoked 20 cigarettes per day since the age of 18 years. There are no
symptoms to suggest gastro-oesophageal reflux or post-nasal drip. His practitioner had performed a
chest X-ray and thoracic computed tomography (CT) scan (shown below).

The appearances of the CT scan are most consistent with:

A. Bronchiectasis.
B. Idiopathic pulmonary fibrosis.
C. Asbestos-related pleural plaques.
D. Asbestosis.
E. Healed pleural tuberculosis.
424

Question 39
Which of the following is the strongest indication for thrombolysis in a patient with pulmonary
embolism?

A. Pre-existing history of pulmonary hypertension.


B. Tachycardia (pulse >120/minute).
C. Hypotension (systolic blood pressure <90mmHg).
D. Right ventricular enlargement on echocardiogram.
E. Large area of embolism on ventilation perfusion lung scan.

Question 40
A 65-year-old male is referred with progressive shortness of breath on exercise. Lung function
testing shows a parenchymal restrictive abnormality and a high-resolution thoracic computed
tomographic scan reveals basal and peripheral fibrosis. He has worked all his life in the banking
industry and has no significant medication history.

What is the most appropriate initial management?

A. High dose oral corticosteroids.


B. Moderate dose oral corticosteroids and azathioprine.
C. Referral for an open lung biopsy.
D. Lung function monitoring.
E. N-acetyl cysteine.

Question 41
A 30-year-old female presents to an Emergency Department with acute breathlessness. The forced
expiratory volume in one second (FEV1) is 3.00 (predicted 4.00) litres and forced vital capacity
(FVC) is 3.75 (predicted 5.00) litres.

Arterial blood gases show:

pH = 7.47
PaO2 = 95
PaCO2 = 32
HCO3 = 36 mEq/litre.

The most likely cause of the results is:

A. Pulmonary thromboembolism.
B. Bulimia.
C. Acute asthma.
D. Salicylate overdosage.
E. Hyperventilation syndrome.
425

Question 42
A 72-year-old man presents to hospital having woken from sleep with paroxysmal nocturnal
dyspnoea. This is his second presentation to hospital with this problem. He is known to have
ischaemic heart disease and an echocardiogram performed during his last admission revealed a left
ventricular ejection fraction of 25%. The respiratory parameters from a polysomnographic sleep
study are shown below.

Nasal P: affected by artefact so disregard


Therm: Nasal airflow
Thor: Thoracic respiratory inductance plethysmography bands
Abdo: Abdominal inductance plethymography bands
Sum: summed signal from Thor and Abdo
Leg: leg movement sensor
Db: decibels

The most likely explanation for the changes are:

A. Cheyne-Stokes respiration (periodic breathing).


B. Obstructive sleep apnoea.
C. Periodic leg movement disorder.
D. Upper airway resistance syndrome.
E. Obesity hypoventilation syndrome.

Question 43
A 30-year-old previously fit childcare worker presents with a four-week history of a non-productive
cough. Clinical examination is normal. The most likely diagnosis is:

A. Gastro-oesophageal reflux disease.


B. Hypersensitivity pneumonitis.
C. Post nasal drip syndrome.
D. Pertussis infection.
E. Asthma.
426

Question 44
An 82-year-old ex-smoker with chronic obstructive pulmonary disease has increasing ankle edema
and breathlessness. Arterial blood gas analysis shows a PO2 of 57mmHg on breathing air. His chest
X-ray is shown below.

The treatment that is most likely to improve his survival is:

A. Long-term oxygen therapy.


B. Inhaled long acting beta agonists.
C. Inhaled corticosteroids.
D. Angiotensin converting enzyme inhibitor.
E. Angiotensin receptor blocker.

Question 45
In a 70-year-old male with chronic obstructive pulmonary disease (COPD) which of the following
carries the greatest risk of death over the next 12 months?

A. Forced expiratory volume in one second (FEV1) 60% predicted.


B. Body mass index (BMI) <20 kg/m2.
C. PaO2 of 58 mmHg.
D. Hospitalisation for an acute exacerbation of the disease.
E. PaCO2 of 47 mmHg.
427

Question 46
A 48-year-old female presents to an Emergency Department complaining of acute breathlessness
following a four-hour car journey. Her breathlessness resolves in the Emergency Department.
Physical examination is normal, the SaO2 is 95% breathing room air and a chest X-ray and ECG are
normal. A sensitive d-dimer is negative.

What is the most appropriate management approach?

A. Computed tomography (CT) pulmonary angiogram.


B. Venous Doppler ultrasound of lower limbs.
C. Ventilation-perfusion lung scan.
D. Arterial blood gas analysis.
E. Reassurance and discharge.
428

Answers

1. C 24. E
2. A 25. C
3. C 26. B
4. C 27. B
5. E 28. C
6. B 29. D
7. A 30. A
8. B 31. B
9. E 32. A
10. C 33. C
11. B 34. B
12. D 35. E
13. A 36. A
14. D 37. E
15. C 38. C
16. A 39. C
17. D 40. D
18. B 41. E
19. A 42. A
20. D 43. D
21. D 44. A
22. C 45. D
23. E 46. E
429

Medical Sciences

Rheumatology

Question 1
The COX-2 isoform of cyclo-oxygenase is most strongly expressed in which one of the following
tissues?

A. Platelets.
B. Established scar tissue.
C. Synovium from osteoarthritic joint.
D. Gastric mucosa.
E. Rheumatoid synovium.

Question 2

Which one of the following is the most common association of the abnormality shown in the X-ray
above?

A. Haemochromatosis.
B. Hypomagnesaemia.
C. Old age.
D. Hyperparathyroidism.
E. Osteoporosis.
430

Question 3
Deficiency of which one of the following enzymes is most likely to cause severe hyperuricaemia?

A. Xanthine oxidase.
B. Adenosine deaminase.
C. Phosphoribosyl pyrophosphate (PRPP) synthetase.
D. Hypoxanthine-guanine phosphoribosyl transferase (HGPRTase).
E. Hexosaminidase.

Question 4
A 62-year-old man on weekly oral methotrexate for psoriatic arthritis inadvertently receives a course
of oral trimethoprim-sulfamethoxazole therapy for urinary infection. He becomes ill with mucositis,
fever and bruising and is found to have moderately severe pancytopenia.

Folinic acid (in the form of calcium folinate) is administered intravenously as an antidote for
presumed methotrexate toxicity.

The principal mode of action of folinic acid is:

A. Enhanced renal excretion of methotrexate.


B. Reduced intracellular accumulation of methotrexate.
C. Conversion of methotrexate to an inactive metabolite.
D. Activation of the enzyme dihydrofolate reductase.
E. Provision of reduced folates for nucleic acid synthesis.
431

Question 5
A lump was removed from the forearm of a patient. The histology shows a central area of fibrinoid
necrosis surrounded by palisading histiocytes and a chronic inflammatory cell infiltrate.

These features fit best with which one of the following?

A. Sarcoidosis.
B. Tuberculosis.
C. Infected sebaceous cyst.
D. Rheumatoid nodule.
E. Gouty tophus.
432

Question 6
Knee pain experienced principally when walking downstairs is most likely to reflect pathology in
which of the following knee structures?

A. Medial compartment.
B. Lateral compartment.
C. Retropatellar compartment.
D. Suprapatellar bursa.
E. Lateral meniscus.

Question 7
In a woman with antiphospholipid syndrome and recurrent pregnancy loss, which of the following
treatment approaches provides the greatest chance of her achieving a successful pregnancy?

A. Low-dose corticosteroids.
B. Low-dose corticosteroids and aspirin.
C. Aspirin and low molecular weight heparin.
D. Intravenous immunoglobulin.
E. Warfarin.

Question 8
The sister of a patient with systemic lupus erythematosus (SLE) is most likely to be concordant for
this disease when they share which of the following states?

A. Homozygous deficiency of C1q.


B. Monozygosity with normal C1q concentration.
C. Positivity for antinuclear antibodies.
D. HLA DR3.
E. Rhesus (Rh) blood group.
433

Question 9
In rheumatoid arthritis, the slow-acting or so-called 'disease-modifying' antirheumatic drugs have
been shown convincingly to retard joint damage. Although in osteoarthritis it remains to be proven
that any intervention alters the course of the disease, which of the following has the strongest
experimental support in the clinical situation as a therapy able to retard cartilage loss?

A. Ketoprofen.
B. Glucosamine sulphate.
C. Rofecoxib.
D. Intra-articular corticosteroids.
E. Intra-articular hyaluran.

Question 10
Headache, blindness and jaw claudication secondary to end-organ ischaemia are common clinical
manifestations of temporal arteritis (giant cell arteritis). Which of the following most accurately
describes the common pathophysiologic mechanism of end-organ ischaemia in this condition?

A. Luminal occlusion secondary to concentric intimal hyperplasia.


B. Thrombotic occlusion of medium arteries.
C. Spasm of medium and large arteries.
D. Aneurysm formation, haemorrhage and rupture.
E. Arterial dissection, haemorrhage and rupture.
434

Question 11
A 54-year-old man presents with neck pain. Examination reveals a limited range of movement of the
neck.

Which feature on the X-ray shown above best explains the limited range of movement?

A. Zygoapophyseal joint fusion.


B. Diffuse disc degeneration.
C. Upper cervical spine instability.
D. Calcification of the posterior longitudinal ligament.
E. Diffuse idiopathic skeletal hyperostosis.

Question 12
Which of the following dietary factors most increases the risk of gout?

A. Red meat consumption.


B. Beer consumption.
C. Dairy products consumption.
D. Purine rich vegetables consumption.
E. Fish consumption.
435

Question 13
Diabetic neuropathic arthropathy is most likely to affect which of the following areas?

A. Knee.
B. Mid-foot.
C. 1st metatarsophalangeal joint.
D. Elbow.
E. Ankle.

Question 14
A 52-year-old woman has been given a diagnosis of chronic fatigue syndrome (CFS) by her general
practitioner. She meets Centres for Disease Control (CDC) criteria for this diagnosis.

Which approach is most likely to be beneficial?

A. Referral to a CFS support group.


B. A graded exercise programme.
C. A selective serotonin reuptake inhibitor (SSRI) antidepressant.
D. Intravenous immunoglobulin.
E. Rest.

Question 15

Which of the following factors is most predictive of future erosions in a patient with rheumatoid
arthritis?

A. Sustained raised tender joint count.


B. Sustained prolonged early morning stiffness.
C. Sustained low serum C4 level.
D. Sustained raised C-reactive protein (CRP) level.
E. Expression of human lymphocyte antigen (HLA) - DR 3.

Question 16
Which of the following cells is most important in the pathogenesis of vasculitis in temporal arteritis?

A. Multinucleate giant cell.


B. Macrophage.
C. CD8+ T cell.
D. CD4+ T cell.
E. Dendritic cell.
436

Question 17
Which of the following cells is most likely to cause bony erosions in patients with rheumatoid
arthritis?

A. T-Lymphocyte.
B. B-Lymphocyte.
C. Osteoclast.
D. Osteoblast.
E. Plasma cells

Question 18
Arthritis in which of the following joints is most suggestive of haemachromatosis?

A. Knee.
B. Ankle.
C. Distal interphalangeal.
D. Metacarpophalangeal.
E. Metatarsophalangeal

Question 19
Which of the following tests is most specific for a diagnosis of rheumatoid arthritis?

A. Anti-cyclic citrullinated peptide antibody.


B. Rheumatoid factor.
C. Antinuclear antibody.
D. Anti-Ro antibody.
E. Anti-neutrophil cytoplasmic antibody

Question 20
Which of the following is the most likely initial site of involvement in primary osteoarthritis of the
knee?

A. Medial tibiofemoral cartilage.


B. Lateral tibiofemoral cartilage.
C. Patellofemoral cartilage.
D. Intercondylar cartilage.
E. All of the above.
437

Question 21
Where is pain most likely to be felt in a patient with osteoarthritis of the hip?

A. Groin.
B. Buttock.
C. Lateral thigh.
D. Knee.
E. Medial thigh.

Question 22
The computed tomography (CT) scan below shows an abnormality.

In addition to hemiparesis, which of the following clinical signs would you expect to be associated
with this abnormality?

A. Acalculia and finger agnosia.


B. Hemianopia and neglect.
C. Quadrantanopia and neglect.
D. Receptive aphasia.
E. Global aphasia.
438

Question 23
Which of the following is the most likely initial site of involvement in primary osteoarthritis of the
knee?

A. Medial tibiofemoral cartilage.


B. Lateral tibiofemoral cartilage.
C. Patellofemoral cartilage.
D. Intercondylar cartilage.
E. All of the above.
439

Answers

1. E
2. C
3. D
4. E
5. D
6. C
7. C
8. A
9. B
10. A
11. A
12. A
13. B
14. B
15. D
16. D
17. C
18. D
19. A
20. C
21. A
22. C
23. A
440

Clinical Applications

Rheumatology

Question 1
A 39-year-old woman presented with the development over two weeks of the lesions shown in
photograph A

Direct immunofluorescence performed on a biopsy of the skin adjacent to the lesions revealed the
changes shown in photograph B which has been stained for complement component 3 (C3).

Which one of the following is most likely to be responsible for the immunopathogenesis of the
lesions shown in photograph A?

A. Immunoglobulins.
B. CD4+ T lymphocytes.
C. Dendritic cells
D. Monocytes.
E. Mast cells.
441

Question 2
A 25-year-old man presents with a history of three episodes of low back pain over six months, each
lasting for ten days. The pain was worse later in the day, eased by rest, and was not associated with
morning stiffness in the back.

Investigations show:
full blood examination within normal limits
erythrocyte sedimentation rate (ESR) 19 mm/h [2-20]
HLA-B27 positive

X-ray of lumbosacral spine and sacro-iliac joints normal

The most likely diagnosis is:

A. Non-specific low back pain.


B. Spondylolysis.
C. Discitis.
D. Ankylosing spondylitis.
E. Scheuermann’s osteochondritis.

Question 3
A 90-year-old woman complains of an aching, burning pain in the right lateral hip area, which
radiates down the lateral thigh to the knee. The pain worsens when she crosses her affected leg over
the other leg and when she walks. It restricts her from lying on her right side at night.

The most likely cause of her symptoms is:

A. Osteoarthritis of the hip.


B. Avascular necrosis of the hip.
C. Trochanteric bursitis.
D. Meralgia paraesthetica.
E. L3 nerve root compression.
442

Question 4
A 22-year-old woman presents with an eight-week history of erythematous rash on the arms and
malar region, mild arthralgia and malaise.

Investigations show:
haemoglobin 112 g/L [115-160]
white cell count 3.8 x 109/L [4.0-11.0]
neutrophils 1.8 x 109/L [2.0-7.5]
platelet count 201 x 109/L [150-400]
erythrocyte sedimentation rate (ESR) 56 mm/h [2-20]
antinuclear antibody (ANA) 1/1600, homogeneous pattern
DNA binding 32 U/mL [<7]
urine microscopy within normal limits
serum creatinine 0.07 mmol/L [0.03-0.10]

The most appropriate initial treatment is:

A. Paracetamol.
B. Naproxen.
C. Hydroxychloroquine.
D. Prednis(ol)one 15 mg/day.
E. Pulse cyclophosphamide.

Question 5
A 38-year-old woman presents with pain over the radial side of her right wrist radiating down into
the thumb. There is no swelling but she is locally tender just distal to the radial styloid. The pain is
reproduced by ulnar deviation of the wrist with the thumb folded across the palm.

The most likely diagnosis is:

A. Osteoarthritis of the first carpometacarpal joint.


B. Avascular necrosis of the scaphoid.
C. Rheumatoid arthritis.
D. Calcium pyrophosphate deposition disease.
E. De Quervain’s tenosynovitis.
443

Question 6
A 43-year-old man with rheumatoid arthritis, inadequately controlled on ibuprofen and methotrexate
(15 mg/week), is commenced on cyclosporin 3 mg/kg/day. His baseline serum creatinine is 0.05
mmol/L [0.05-0.09] and his blood pressure is 130/80 mmHg.

Six weeks later, his serum creatinine is 0.09 mmol/L and his blood pressure is 150/95 mmHg.

The most appropriate next step in management is to:

A. Add nifedipine.
B. Add diltiazem.
C. Cease ibuprofen.
D. cease methotrexate.
E. cease cyclosporin.

Question 7
A 58-year-old man presents with a two-month history of knee and wrist pain. He has mild local
tenderness with no soft tissue swelling or joint effusion. The physical examination is otherwise
unremarkable.

An X-ray of his knee is shown below.

Which one of the following is the most appropriate next investigation?

A. Chest X-ray.
B. Joint aspiration.
C. Isotope bone scan.
D. Serum rheumatoid factor.
E. Serum calcium and parathyroid hormone.
444

Question 8
A 56-year-old woman with longstanding rheumatoid arthritis presents with a three-month history of
increasing groin pain radiating to the knees. Her magnetic resonance imaging (MRI) scan is shown
below

The most likely cause of the pain is:

A. Synovitis.
B. Osteoarthritis.
C. Avascular necrosis.
D. Pigmented villonodular synovitis.
E. Psoas abscess.
445

Question 9
A 72-year-old man with longstanding diabetes presents with a three-month history of discomfort and
swelling in the midtarsal region.

Investigations show:
full blood examination within normal limits
erythrocyte sedimentation rate (ESR) 22 mm/h [2-14]
urea, electrolytes and creatinine normal
serum uric acid 0.49 mmol/L [0.20-0.47]
glycosylated haemoglobin 7.9% [<6.1]
rheumatoid factor 22 U/mL [0-20]

The X-ray of his foot is shown below.

The most likely cause of the foot pain is:

A. Osteoarthritis.
B. Gout.
C. Septic arthritis.
D. Neuropathic arthropathy.
E. Rheumatoid arthritis.
446

Question 10
A 44-year-old man presents with a four-day history of acute right knee pain. He has a 10-year history
of episodic pain and swelling of his toes, ankles and wrists. Examination reveals a warm, swollen
right knee with an effusion. There is soft tissue swelling in both wrists, the left elbow and the right
ankle and forefoot.

A previous X-ray of his foot is shown below.

Investigations show:
haemoglobin 150 g/L [130-180]
white cell count 12 x 109/L [4-11]
erythrocyte sedimentation rate (ESR) 90 mm/h [1-10]
C-reactive protein 186 mg/L [0-8]

Which one of the following is the most likely diagnosis?

A. Rheumatoid arthritis.
B. Gout.
C. Chondrocalcinosis.
D. Seronegative arthritis.
E. Septic arthritis.
447

Question 11
A 62-year-old woman with a 15-year history of rheumatoid arthritis presents with a four-month
history of intermittent occipital headaches. She has mildly active synovitis. Neurological
examination is normal.

The next most appropriate investigation is:

A. Computed tomography (CT) scan of the brain.


B. Skull X-ray.
C. Magnetic resonance imaging (MRI) scan of the brain.
D. Lateral flexion X-ray of cervical spine.
E. Computed tomography (CT) scan of the neck.

Question 12
A 64-year-old woman has painful swelling, pallor and blue discolouration of her fingers when cold.

An X-ray of one of her hands is shown below.

Which one of the following autoantibodies is most likely to be present?

A. Anti-RNP.
B. Anti-Pm-Scl.
C. Anti-centromere.
D. Anti-topoisomerase 1 (Scl 70).
E. Anti-filaggarin..
448

Question 13
A 25-year-old man presents with a five-year history of low back pain and stiffness. Symptoms are
generally worse in the morning with some improvement following activity. Over the past three years
he has also had episodes of Achilles tendonitis. Examination reveals marked limitation of lumbar
spine movements in all planes.

Investigations show:
haemoglobin 35 g/L [130-180]
erythrocyte sedimentation rate (ESR) 8 mm/h [0-10]
C-reactive protein 4 mg/L [<8]
rheumatoid factor negative

X-rays of the patient’s pelvis and lumbar spine are shown above.

What is the most appropriate initial management?

A. Paracetamol 1 g qid.
B. Indomethacin 25 mg tds.
C. Indomethacin 25 mg tds and a physical therapy programme.
D. A physical therapy programme and hydrotherapy.
E. Sulphasalazine 1 g bd and hydrotherapy.
449

Question 14
A patient being investigated for recurrent bilateral parotid swelling and xerostomia complains of
generalised muscle weakness. Blood pressure, blood count, liver function tests and creatine kinase
level are all normal. The serum potassium is 2.1 mmol/L [3.5-5.2] and the serum bicarbonate is 16
mmol/L [22-30].

Which one of the following is most likely to be diagnostic?

A. Thyroid function tests.


B. Antibodies to nuclear and extractable nuclear antigens (ANA and ENA).
C. Serum angiotensin-converting enzyme (ACE).
D. Acetylcholine receptor (AChR) antibodies.
E. Chest X-ray.

Question 15
A 25-year-old man presents with a history of three episodes of low back pain over six m onths, each
lasting for ten days. The pain is worse later in the day, eased by rest, and is not associated with
morning back stiffness.

Investigations show:
full blood examination within normal limits
erythrocyte sedimentation rate (ESR) 19 mm/h [2-20]
HLA-B27 positive

X-ray of lumbosacral spine and sacro-iliac joints normal

The most likely diagnosis is:

A. Non-specific low back pain.


B. Spondylolysis.
C. Discitis.
D. Ankylosing spondylitis.
E. Scheuermann’s osteochondritis.
450

Question 16
A 64-year-old man with a history of recurrent gout and a recent past history of pulmonary embolism
for which he is taking warfarin (prothrombin time-international normalised ratio (PT-INR) 2.5 [1.0-
1.3]), presents with acute arthritis in the left mid-foot which is consistent with his previous attacks of
crystal synovitis. Recently, allopurinol was added to his other drug therapy and the dose has been
slowly increased to 200 mg/day. His serum urate on admission is 0.49 mmol/L [0.18-0.45] and his
creatinine is within the range of 0.16-0.18 mmol/L [0.05-0.13].

Which one of the following management options would be most appropriate for the treatment of the
synovitis?

A. Cease the allopurinol and add colchicine 0.5 mg tds.


B. Continue allopurinol at the current dose and add indomethacin 50 mg 6th hourly.
C. Increase the allopurinol to 300 mg/day and add prednis(ol)one 15 mg/day.
D. Reduce the allopurinol to 100 mg/day and add celecoxib 200 mg bd.
E. Continue allopurinol at the current dose and add prednis(ol)one 20 mg/day.

Question 17
A 54-year-old man with a previous history of gout presents with an acutely swollen painful left knee
for 24 hours. On examination he is febrile (38.2oC) and has a large effusion of the left knee and
limited range of motion due to pain. Aspiration of the joint reveals:

White cell count 50,000 cells/ml


Microscopy No crystals
Gram stain negative

The best initial treatment is:


A. Flucloxacillin.
B. Indomethacin.
C. Prednisolone.
D. Allopurinol.
E. Colchicine.
451

Question 18
An 80-year-old woman presents following a fall. She describes pain on shoulder movement
unilaterally. On examination she has marked pain on active abduction of the shoulder between 60-
120 degrees but minimal pain on passive abduction.

Blood investigations show:


haemoglobin 11.0 g/L [11.5-13.5]
erythrocyte sedimentation rate (ESR) 35 mm/h [0-7]

The most likely diagnosis is:

A. Polymyalgia rheumatica.
B. Osteoarthritis of the shoulder.
C. Humeral head subluxation.
D. Rotator cuff injury.
E. Fractured clavicle.

Question 19
A 21-year-old woman presents with a two-year history of colour changes in her hands from white to
blue to pink following exposure to cold.

Which of the following laboratory results would indicate the highest risk of future development of a
Raynaud’s phenomenon-associated secondary disease?

A. Mild anaemia.
B. Elevated C-reactive protein (CRP).
C. Positive rheumatoid factor.
D. Positive speckled pattern antinuclear antibody test (ANA).
E. Positive anticentromere antibody test.
452

Question 20
A frail 88-year-old woman is admitted to hospital within a few hours of having developed sudden,
severe and disabling pain in the lower back and right buttock. She has previously been well aside
from well-controlled hypertension and the resection of a sigmoid colon cancer 15 years ago.

Examination is difficult because of pain. There is considerable but poorly localised tenderness
around the sacral area and buttock, pain on movement of either hip and no obvious neurological
deficit.

X-rays of the lumbo-sacral spine, pelvis and hips reveal degenerative changes in the lumbosacral
spine of a degree expected in a woman of her age.

A bone scan, performed on the afternoon of her admission, is shown below.

The most likely diagnosis is:

A. Prolapsed intervertebral disc causing sciatic pain.


B. Stress fracture of the sacrum or pelvis.
C. Metastatic bone disease.
D. Musculo-ligamentous low back strain.
E. Referred pain from pelvic recurrence of colonic neoplasm.
453

Question 21
A 67-year-old man presents with a six-month history of increasing bilateral leg pain, exacerbated by
walking more than 100-150 m and often forcing him to stop and sit to relieve the pain. He has a long
history of hypertension and hyperlipidaemia and he suffered a myocardial infarction six months
previously.

Examination reveals bruits over both carotids and the femoral arteries but all pedal pulses are readily
palpable. Ankle reflexes are absent. Proprioception is absent at the toes but sensation is otherwise
intact.

Which one of the following investigations is most likely to establish the cause of this man’s leg pain?

A. Arterial Doppler studies of the lower limbs.


B. Nerve conduction studies of the lower limbs.
C. Lower limb arteriography.
D. A bone scan with SPECT (single photon emission computed tomography) of the lumbosacral
spine.
E. A computed tomography (CT) scan of the lumbar spine.

Question 22
A 19-year-old woman presents with intermittent knee pain following a netball injury. An X-ray of
her knee is shown below.

The most likely diagnosis is:

A. Aneurysmal bone cyst.


B. Plasmacytoma.
C. Ewing sarcoma.
D. Osteomyelitis.
E. Osteogenic sarcoma.
454

Question 23
An 83-year-old man has osteoarthritis of the knee. Pain limits his ability to walk more than 300 m.
Paracetamol gives minimal relief. Examination of the knee reveals a varus deformity and a warm
effusion. He is commenced on naproxen 500 mg twice daily. Four weeks later his knee has not
improved and his serum creatinine has risen from 140 mmol/L to 350 mmol/L [60-120].

In addition to cessation of naproxen, which of the following would be the most appropriate next step
in the management of his osteoarthritis of the knee?

A. Sulindac.
B. Rofecoxib.
C. Intra-articular corticosteroids.
D. Intra-articular hyaluronic acid.
E. Total joint replacement.

Question 24
A 60-year-old Polynesian man presents with a 12-month history of pain and swelling in the left third
proximal interphalangeal joint. X-rays of the hand and affected finger are shown below.

The X-rays are most consistent with which of the following n b m g?

A. Gout.
B. Tuberculous arthritis.
C. Chronic septic arthritis.
D. Hyperparathyroidism.
E. Psoriatic arthritis.
455

Question 25
A 45-year-old woman is referred following relocation from interstate. Her accompanying medical
records list her major diagnosis as systemic lupus erythematosus (SLE). You elicit a history of three
to four years of polyarthritis, Raynaud’s phenomenon, myalgia and proximal muscular weakness and

Recently, increasing shortness of breath on exertion. Examination reveals a blood pressure of 120/70
mmHg, a single prominent second heart sound, diffuse puffiness of the digits, mild proximal
weakness and mild synovitis of the wrists and small joints of the hands.

Laboratory studies reveal:


haemoglobin 117 g/L [115-165]
creatinine 0.112 mmol/L 0.060-0.110]
urinalysis protein +
no cells or casts on microurine examination
antinuclear antibodies positive 1:5120 [<1:40]
anti-double stranded DNA negative
anti-Sm (Smith) negative
anti-Ro (SS-A) negative
anti-La (SS-B) negative
anti-U1 RNP (ribonucleoprotein) positive

Chest X-ray is normal.

Pulmonary function tests show:


forced vital capacity (FVC) 80% predicted
total lung capacity (TLC) 80% predicted
diffusion capacity for carbon monoxide
corrected for alveolar volume (KCO) 50% predicted

Which of the following would be the most likely cause of death in this woman?

A. Diffuse proliferative glomerulonephritis.


B. Myocarditis.
C. Renovascular hypertension.
D. Pulmonary hypertension.
E. Cerebral vasculitis.

Question 26
A 72-year-old man presents with a four-week history of mid-lumbar back pain. This pain began
spontaneously and is much worse at night. He has mild treated hypertension but is otherwise well.

The most appropriate initial investigation is:

A. A plain X-ray of the lumbar spine.


B. A radionuclide bone scan.
C. Erythrocyte sedimentation rate (ESR).
D. Computed tomography (CT) scan of the lumbar spine.
E. Magnetic resonance imaging (MRI) of the lumbar spine.
456

Question 27
A 52-year-old female presents with a 12-month history of painful swollen fingers. X-rays of her
hands are shown below.

For which of the following conditions are these findings most characteristic?

A. Rheumatoid arthritis.
B. Systemic lupus erythematosus.
C. Psoriatic arthritis.
D. Gout.
E. Scleroderma.

Question 28
A 67-year-old man presents with a two-day history of painful and swollen right first
metatarsophalangeal joint. He has a previous history of ischaemic heart disease, renal impairment
(creatinine 0.24 mmol/L [0.05-0.09]) and mild type 2 (non-insulin-dependent) diabetes mellitus
treated with diet. On examination his temperature is 37.3°C and the affected joint is red and
exquisitely tender.

The most appropriate initial treatment is:

A. Fucloxacillin.
B. Indomethacin.
C. Celecoxib.
D. Prednis(ol)one.
E. Allopurinol.
457

Question 29
A 75-year-old woman has bilateral knee osteoarthritis, diabetes and ischaemic heart disease.
Functionally she can walk two kilometres with pain in her back usually causing her to stop. She
presents with a four-week history of severe pain in the right knee on weight bearing. This pain is
minimally improved with non-steroidal anti-inflammatory drugs, paracetamol and intra-articular
corticosteroid injection, and there is no history of trauma or falls. On examination there is no heat but
a small effusion, the knee is tender along the medial joint line and ligaments appear normal. Her X-
ray report suggests moderate changes consistent with osteoarthritis.

Which of the following investigations is most likely to provide the diagnosis?

A. Gallium scan.
B. Bone scan.
C. Computed tomography (CT) scan.
D. Magnetic resonance imaging (MRI) scan.
E. Arthroscopy.

Question 30
A 38-year-old woman presents with a six-week history of increasing shortness of breath while
playing tennis and a two-week history of increasing muscle weakness. She reports mild symmetrical
polyarthralgia of the hands, wrists and knees for six months and Raynaud’s phenomenon during the
recent winter. Examination reveals proximal muscle weakness and a hyperkeratotic rash with
fissuring on the palmar and lateral surfaces of the fingers on both hands. Investigations reveal a
serum creatine kinase (CK) level of 890 U/L [16-139] and an interstitial pulmonary infiltrate on high
resolution computed tomography (CT) scan of the chest.

Autoantibody tests show the following results:


anti-nuclear antibody (ANA) test titre 1:2560, speckled pattern
anti-ENA (extractable nuclear antigens) antibodies Jo-1 positive
anti-DNA antibodies negative

The most likely diagnosis is:

A. Dermatomyositis.
B. Progressive systemic sclerosis.
C. Polymyositis (anti-synthetase syndrome).
D. Systemic lupus erythematosus.
E. Sjögren’s syndrome.
458

Question 31
A 65-year-old woman presents with six months of worsening left knee pain. The knee pain is worse
during and after walking and improved by rest. On examination she has a body mass index (BMI) of
2
36 kg/m [18-25]. Her left knee is not swollen and has a painful but full range of movement.

Her X-ray is as shown below.

The most appropriate initial management, in addition to simple analgesia with paracetamol, is:

A. Commence non-steroidal anti-inflammatory medications.


B. Inject the left knee with a depot corticosteroid.
C. Refer the patient for quadriceps muscle strengthening exercises.
D. refer to a dietician for a weight-loss program.
E. Refer to an orthopedic surgeon for arthroscopy.
459

Question 32
A 25-year-old female presents with symmetrical small joint polyarthralgia associated with a
photosensitive facial rash, and low grade fever. Examination reveals small joint tenderness in the
hands without synovitis, and an erythematous, slightly scaly malar rash. The examination is
otherwise unremarkable, and urinalysis is normal. The following laboratory results are obtained.
C-reactive protein (CRP) 18 mg/L [0-11]
Antinuclear antibody (ANA) positive, titre: 1/2560; pattern: homogeneous
Anti-double stranded DNA (anti-dsDNA) 17 IU/mL [0-5]
Rheumatoid factor <20 IU/mL [0-20]

In addition to non-steroidal anti-inflammatory agents, which of the following is the most appropriate
next step in management?

A. Azathioprine.
B. Topical corticosteroids.
C. Methotrexate.
D. Oral corticosteroids.
E. Hydroxychloroquine.

Question 33
A 52-year-old woman presents with dry eyes which have progressively worsened over the past three
years. On examination her Schirmer test is 1mm in 5 minutes [5mm in 5 minutes] and she has
positive Rose Bengal staining of both corneas. Her erythrocyte sedimentation rate is 98 mm/hr
[<21mm/hr], antinuclear antibody is positive with a speckled pattern and a titre of 1:640 and Anti-Ro
and –La antibodies are detected.
The most appropriate initial treatment is:

A. Hydroxychloroquine.
B. Low dose oral prednisolone.
C. Methotrexate.
D. Hypomellose tear substitute.
E. Prednisolone eye drops.

Question 34
A 28-year-old woman presents with episodes of colour change in her distal fingers when they are
exposed to the cold. The episodes occur most days and are characterised by white numb finger tips
which become blue and then red. She describes similar episodes that have occurred for many years.

She is otherwise well. Examination is unremarkable. Investigations show a positive anti-nuclear


antibody of a speckled pattern with a titre of 1:80. The most likely diagnosis is:

A. Limited scleroderma.
B. Diffuse scleroderma.
C. Primary Raynaud's phenomenon.
D. Systemic lupus erythematosus.
E. Carcinoid syndrome.
460

Question 35
A 49-year-old female presents with a five week history of symmetrical polyarthralgia. The following
laboratory results are obtained:

Antinuclear antibody (ANA) positive, titre: 1/80, pattern: speckled


Antibodies to extractable nuclear antigens (ENAs) negative
Anti-double stranded DNA antibody (anti-dsDNA) 7 IU/mL [0-5]
Anti-cyclic citrullinated peptide (anti-CCP) positive
Anti-filaggrin antibody positive
Rheumatoid factor (RF) < 20 [0-20]

Which of the following is the most likely diagnosis?

A. Rheumatoid arthritis.
B. Systemic lupus erythematosus.
C. Mixed connective tissue disease.
D. Viral polyarthritis.
E. Osteoarthritis.

Question 36
A 26-year-old man presents with a swollen, painful left ankle and right wrist ten days after being
treated with doxycycline for chlamydial urethritis. Which of the following is the most appropriate for
the initial treatment of his joint symptoms?

A. Azithromycin.
B. Prednisolone.
C. Sulfasalazine.
D. Indomethacin.
E. Hydroxychloroquine

Question 37
A 26-year-old man presents with a six month history of lower back and buttock pain with prolonged
stiffness in the morning. Physical examination reveals a Schober test of 4 cm on flexion [>5cm].

Which of the following tests is most likely to support a diagnosis of ankylosing spondylitis?

A. X-ray sacroiliac joints.


B. HLA B27 genotyping.
C. Erythrocyte sedimentation rate.
D. Rheumatoid factor.
E. Radionuclide bone scan.
461

Question 38
A 76-year-old woman presents with a swollen, painful left knee. The knee has become increasingly
painful over two weeks and she is now unable to walk on it. She does not remember injuring the
knee and is otherwise well. On examination she is afebrile and has a large left knee effusion with
reduced range of movement.

An X-ray of the knee is shown above.

What is the most likely diagnosis?

A. Osteoarthritis.
B. Gout.
C. Pseudogout.
D. Haemarthrosis.
E. Septic arthritis.
462

Question 39
A 54-year-old female presents with a three month history of symmetrical polyarthralgia. The
following laboratory results are obtained:

Antinuclear antibody (ANA) positive, titre: 1/160, pattern: speckled


titre 1/640, pattern: cytoplasmic
Antibodies to extractable nuclear antigens (ENAs) positive anti-SS-A (Ro)
Antibodies to ribosomal p protein positive
Anti-double stranded DNA antibody (anti-dsDNA) 4 IU/mL [0-5]
Anti-cyclic citrullinated peptide (anti-CCP) negative

Rheumatoid factor (RF) 45 IU/mL [0-20]

Which of the following is the most likely diagnosis?

A. Rheumatoid arthritis.
B. Systemic lupus erythematosus.
C. Sjogren’s syndrome.
D. Mixed connective tissue disease.
E. Systemic sclerosis.

Question 40
In untreated patients with hyperuricaemia, which of the following medications is most likely to
precipitate an attack of gout?

A. Thyroxine.
B. Allopurinol.
C. Frusemide.
D. Hydrochlorothiazide.
E. Pyrazinamide.
463

Question 41
A 67-year-old man with longstanding rheumatoid arthritis treated with methotrexate, prednisolone
and celecoxib presents unwell with fatigue and anorexia. On examination his arthritis is quiescent
but he has two mouth ulcers. His full blood examination is shown
below.

Hb 102 g/L [115-150]


MCV 101.2 fL [80-96]
MCH 29.1 pg [27-33]
MCHC 350 g/L [320-360]
WCC 1.4x109 [4-11]
Neutrophils 0.5x109 [2-5]
Plt 90x109 [150-400]
ESR 67 mm per hour [<22]

The most appropriate drug to administer is:


A. Folic acid.
B. Filgrastim.
C. Folinic acid.
D. Cholestyramine.
E. Prednisolone.

Question 42
When aspirating a small effusion in the knee joint, where should the needle be inserted (see below)?

A. 1 cm medial to the junction of the upper and middle third of the patella.
B. 3 cm medial to the lower pole of the patella.
C. 1 cm medial to the mid point of the patella tendon.
D. 1 cm lateral to the lower pole of the patella.
E. 3 cm lateral to the junction of the upper and middle third of the patella.
464

Question 43
A 67-year-old man with longstanding rheumatoid arthritis treated with methotrexate, prednisolone
and celecoxib presents unwell with fatigue and anorexia. On examination his arthritis is quiescent
but he has two mouth ulcers. His full blood examination is shown below.

Haemoglobin (Hb) 102 g/L [115 - 150 g/L]


Mean corpuscular volume (MCV) 101.2 fL [80.0 - 96.0 fL]
Mean corpuscular haemoglobin (MCH) 29.1 pg [27.0 - 33.0 pg]
Mean corpuscular haemoglobin concentration (MCHC) 350 g/L [320 - 360 g/L]
White cell count (WCC) 1.4 x 109/L [4 - 11 x 109/L]
Neutrophils 0.5 x 109/L [2 - 5 x 109/L]
Platelets 90 x 109/L [150-400 x 109/L]
Erythrocyte sedimentation rate (ESR) 67 mm/hour [<22 mm/hour]

The most appropriate drug to administer is:

A. Folic acid.
B. Filgrastim (Granulocyte Colony Stimulating Factor (GCSF)).
C. Folinic acid.
D. Cholestyramine.
E. Prednisolone.
465

Question 44
A 43-year-old man with human immunodeficiency virus (HIV), first diagnosed 20 years previously,
presents with a gradual onset of right-sided hip pain. He is on tenofovir, emtracitabine and efavirenz.
He has had no acquired immunodeficiency syndrome (AIDS) defining illnesses in the past. On
examination he has evidence of facial lipoatrophy and there is limitation of the range of movement of
the right hip because of pain. All other joints are normal. There are no skin rashes. An X-ray of his
hip is shown below.

Laboratory results show:


CD4 count 0.41 x 109/L [0.45 – 1.1 x 109/L]
HIV viral load undetectable
Full blood count (FBC) within normal limits
Uric acid 0.55 mmol/L [0.18 – 0.48 mmol/L]
Antinuclear antibodies not detected
Rheumatoid factor not detected

What is the most likely diagnosis?

A. Avascular necrosis.
B. Psoriatic arthritis.
C. Osteoarthritis.
D. Rheumatoid arthritis.
E. Gout.
466

Question 45
In a 75-year-old woman presenting with a unilateral temporal headache, which of the following
clinical features is most specific for a diagnosis of temporal arteritis?

A. Temporal artery tenderness.


B. Blurred vision.
C. Jaw (masseter) claudication.
D. Pain and stiffness around the shoulders and hips.
E. Fever.

Question 46
A 27-year-old man presents with an 18 month history of low back pain and stiffness, a swollen left
knee and heel pain. Examination demonstrates reduced lumbar flexion and chest expansion. He is
HLA-B27 positive and has a C-reactive protein (CRP) of 25 mg/L [<8 mg/L].

In addition to physiotherapy what is the most appropriate initial pharmacological therapy?

A. Prednisolone.
B. Naproxen.
C. Methotrexate.
D. Etanercept.
E. Salazopyrin.

Question 47
A 35-year-old man with a longstanding history of attacks of acute gout is reviewed three months
after commencing allopurinol 100 mg daily. Which factor is most relevant in determining whether
the dose of the allopurinol should be increased on this visit?

A. Frequency of attacks of gout.


B. Uric acid level.
C. Presence of tophi.
D. Active knee inflammation.
E. C-reactive protein (CRP) level.
467

1. A
2. A
3. C
4. C
5. E
6. E
7. A
8. C
9. D
10. B
11. D
12. C
13. C
14. B
15. A
16. E
17. A
18. D
19. E
20. B
21. E
22. E
23. C
24. A
25. D
26. A
27. E
28. D
29. D
30. C
31. C
32. E
33. C
34. C
35. A
36. D
37. B
38. C
39. B
40. B
41. C
42. A
43. C
44. A
45. C
46. B
47. B

You might also like